Feat: genere DS pour les TST
All checks were successful
continuous-integration/drone/push Build is passing

This commit is contained in:
Bertrand Benjamin 2021-04-07 09:08:40 +02:00
parent 615ddbc735
commit 5b0de2d51d
87 changed files with 21494 additions and 10 deletions

View File

@ -0,0 +1,262 @@
\documentclass[a4paper,10pt]{article}
\usepackage{myXsim}
% Title Page
\title{DS8 \hfill AIOUAZ Ahmed}
\tribe{TST}
\date{\hfillÀ render pour le Mercredi 7 avril}
\xsimsetup{
solution/print = false
}
\begin{document}
\maketitle
\begin{exercise}[subtitle={Automatismes}]
\textit{Toutes les questions de cette exercice sont indépendantes et peuvent être répondus séparément}
\begin{enumerate}
\item De janvier à septembre, une quantité a augmenté de $20\,\%$. Faire un schéma pour représenter la situation puis calculer le taux d'évolution moyen mensuel.
\item Une quantité augmente de $20\,\%$ par ans. En 2020, elle est de 110\euro. Quelle était sa valeur en 2019? Faire un schéma pour représenter la situation.
\item Déterminer l'équation de la droite \\
\begin{tikzpicture}[xscale=0.8, yscale=0.5]
\tkzInit[xmin=-5,xmax=5,xstep=1,
ymin=-5,ymax=5,ystep=1]
\tkzGrid
\tkzAxeXY
\tkzFct[domain=-5:5,color=red,very thick]%
{3.0*\x -3};
\end{tikzpicture}
\item Résoudre l'équation $2 \times 0.07^x = 20$
\end{enumerate}
\end{exercise}
\begin{solution}
\begin{enumerate}
\item On veut partager cette évolution en 8 évolutions.
\[
\left(1 + \frac{20}{100}\right)^{\frac{1}{8}} = 1.0231
\]
Donc le taux d'évolution moyen est
\[
t_m = 1.0231 - 1 = 0.0230999999999999
\]
\item Coefficient multiplicateur pour revenir en arrière
\[
CM = (1 + \frac{20}{100})^{-1} = 0.8333
\]
On en déduit la quantité en 2019
\[
110 * 0.8333 = 91.66300000000001
\]
\item L'équation de la droite est
\[
y = 3.0 x -3
\]
\item Il faut penser à faire la division à par $2$ avant d'utiliser le log car sinon, on ne peut pas utiliser la formule $\log(a^n) = n\times \log(a)$.
\[x = \frac{\log(10.0)}{\log(0.07)}\]
\end{enumerate}
\end{solution}
\begin{exercise}[subtitle={Restaurant}]
Un \emph{food truck}, ouvert le midi et le soir, propose deux types de formules :
\setlength\parindent{10mm}
\begin{itemize}
\item la formule \emph{Burger} ;
\item la formule \emph{Wok}.
\end{itemize}
\setlength\parindent{0mm}
\medskip
Le gérant a remarqué que 9\,\% de ses ventes ont lieu le midi. Le quart des ventes du midi correspondent à la formule \emph{Burger}, alors que 3\,\% des ventes du soir correspondent à la formule \emph{Wok}.
Le gérant se constitue un fichier en notant, pour chaque vente, la formule choisie et le moment de cette vente (midi ou soir).
On prélève une fiche de façon équiprobable. On définit les quatre évènements suivants:
\begin{enumerate}
\item $M$ : \og la fiche correspond à une vente du midi\fg{} ;
\item $S$ : \og la fiche correspond à une vente du soir\fg {};
\item $W$ : \og la fiche correspond à une formule \emph{Wok} \fg{} ;
\item $B$ : \og la fiche correspond à une formule \emph{Burger} \fg.
\end{enumerate}
\setlength\parindent{0mm}
\medskip
\begin{enumerate}
\item Recopier puis compléter l'arbre pondéré
\begin{center}
\begin{tikzpicture}[sloped]
\node {.}
child {node {$M$}
child {node {$W$}
edge from parent
node[above] {...}
}
child {node {$B$}
edge from parent
node[above] {...}
}
edge from parent
node[above] {...}
}
child[missing] {}
child { node {$S$}
child {node {$W$}
edge from parent
node[above] {...}
}
child {node {$B$}
edge from parent
node[above] {...}
}
edge from parent
node[above] {...}
} ;
\end{tikzpicture}
\end{center}
\item Calculer la probabilité de l'évènement $M \cap W$. Interpréter ce résultat dans le contexte de l'exercice.
\item Montrer que la probabilité que la fiche choisie corresponde à une formule \emph{Burger} est égale à $0.9052$.
\item On a prélevé une fiche correspondant à la formule \emph{Burger}. Quelle est la probabilité, arrondie au millième, que la vente ait eu lieu le soir?
\end{enumerate}
\end{exercise}
\begin{solution}
\begin{enumerate}
\item
\begin{center}
\begin{tikzpicture}[sloped]
\node {.}
child {node {$M$}
child {node {$W$}
edge from parent
node[above] {$0.75$}
}
child {node {$B$}
edge from parent
node[above] {$0.25$}
}
edge from parent
node[above] {$0.09$}
}
child[missing] {}
child { node {$S$}
child {node {$W$}
edge from parent
node[above] {$0.03$}
}
child {node {$B$}
edge from parent
node[above] {$0.97$}
}
edge from parent
node[above] {$0.91$}
} ;
\end{tikzpicture}
\end{center}
\item On calcule la probabilité que la vente soit un wok et ait eu lieu à midi
\[ P(M\cap W) = P(M) \times P_M(W) = 0.09 \times 0.75 = 0.0675 \]
\item Probabilité que la vente soit un burger.
\[
P(B) = P(M\cap B) + P(S\cap B) = 0.09 \times 0.75 + 0.91 \times 0.03 = 0.9052
\]
\item On cherche à calculer la quantité $P_B(S)$. Pour cela on utilise la formule de Bayes
\[
P_B(S) = \frac{P(B\cap S)}{P(B)} = \frac{P_S(B) \times P(S)}{P(B)} = \frac{0.97\times 0.91}{0.9052} = 0.975143614670791 \approx 0.975
\]
\end{enumerate}
\end{solution}
\begin{exercise}[subtitle={Continent plastique}]
\textit{Les quantités évoqués dans cette exercice sont générés au hasard et sont donc complètement farfelus.}
\medskip
Le \og continent de plastique\fg{} est la plus grande des plaques de déchets plastiques évoluant sur les océans. Elle occupe actuellement dans l'océan Pacifique une surface dont l'aire est évaluée à plus de $1,6$ million de km$^2$, entre Hawaï et la Californie.
En 2017, des scientifiques ont estimé qu'il y avait $14$ millions de tonnes de déchets plastiques qui était déversé chaque année dans les océans et que cette quantité augmentait de $28\n\%$ par chaque année.
On modélise l'évolution de la masse de ces déchets plastiques déversée chaque année, si rien n'est fait pour la réduire, par une suite géométrique $\left(u_n\right)$. L'arrondi au centième du terme $u_n$ représente la masse de ces déchets déversée chaque année, exprimée en million de tonnes, pour l'année $(2017 + n)$.
\medskip
\begin{enumerate}
\item Expliquer pourquoi la suite $u_n$ est géométrique?
\item Calculer $u_1$ et $u_2$.
\item Exprimer $u_n$ en fonction de $n$.
\item Au début de l'année 2017, il y avait $300$ millions de tonnes de déchets plastique. Calculer la quantité totale de déchets plastiques en 2030.
\item On souhaite déterminer en quelle année la masse totale de ces déchets plastiques aura pour la première fois augmenté de $50$\,\% par rapport à sa valeur de 2017.
\begin{enumerate}
\item Recopier et compléter l'algorithme ci-dessous pour que la variable $N$ contienne la réponse au problème posé.
\begin{center}
\begin{tabularx}{0.4\linewidth}{|X|}\hline
$N = 2017$\\
$U = 14$ \\
$S = 300 + U$ \\
while $S < 450$: \\
\hspace{1cm} $N = \ldots$\\
\hspace{1cm} $U = \ldots$\\
\hspace{1cm} $S = \ldots$\\
\hline
\end{tabularx}
\end{center}
\item Que contiennent les variables $S$, $U$ et $N$ après exécution de cet algorithme ?
Interpréter les résultats dans le contexte de l'exercice.
\end{enumerate}
\end{enumerate}
\end{exercise}
\begin{solution}
\begin{enumerate}
\item Une augmentation de $28\,\%$ revient à multiplier la quantité par $1.28$. La suite est donc bien géométrique. Son premier terme est $u_0 = 14$ et sa raison est $q = 1.28$
\item
\[
u_1 = u_0 * 1.28 = 17.92
\]
\[
u_2 = u_0 * 1.28^2 = 22.9376
\]
\item
\[
u_n = u_0 \times q^n = 14 \times 1.28^n
\]
\item On calcule la quantité totale déversée entre 2017 et 2030.
\[
\sum_{n = 0}^{13} u_n = u_0 \times \frac{1-q^{13}}{1-q} = 14 \times \frac{1 - 1.28^{13}}{1 - 1.28} = 1187.94
\]
On en déduit la quantité totale de déchets en 2030
\[
300 + 1187.94 = 1487.94
\]
\item
\begin{enumerate}
\item ~
\begin{center}
\begin{tabularx}{0.4\linewidth}{|X|}\hline
$N \gets 2017$\\
$U \gets 14$ \\
$S \gets 300 + U$ \\
Tant que $S < 450$ \\
\hspace{1cm} $N \gets N + 1$\\
\hspace{1cm} $U \gets U * 1.28$\\
\hspace{1cm} $S \gets S + u$\\
Fin Tant que\\\hline
\end{tabularx}
\end{center}
\item \textit{Pas de correction automatisé}
\end{enumerate}
\end{enumerate}
\end{solution}
\end{document}
%%% Local Variables:
%%% mode: latex
%%% TeX-master: "master"
%%% End:

View File

@ -0,0 +1,262 @@
\documentclass[a4paper,10pt]{article}
\usepackage{myXsim}
% Title Page
\title{DS8 \hfill BAHBAH Zakaria}
\tribe{TST}
\date{\hfillÀ render pour le Mercredi 7 avril}
\xsimsetup{
solution/print = false
}
\begin{document}
\maketitle
\begin{exercise}[subtitle={Automatismes}]
\textit{Toutes les questions de cette exercice sont indépendantes et peuvent être répondus séparément}
\begin{enumerate}
\item De janvier à septembre, une quantité a augmenté de $28\,\%$. Faire un schéma pour représenter la situation puis calculer le taux d'évolution moyen mensuel.
\item Une quantité augmente de $28\,\%$ par ans. En 2020, elle est de 138\euro. Quelle était sa valeur en 2019? Faire un schéma pour représenter la situation.
\item Déterminer l'équation de la droite \\
\begin{tikzpicture}[xscale=0.8, yscale=0.5]
\tkzInit[xmin=-5,xmax=5,xstep=1,
ymin=-5,ymax=5,ystep=1]
\tkzGrid
\tkzAxeXY
\tkzFct[domain=-5:5,color=red,very thick]%
{1.0*\x -2};
\end{tikzpicture}
\item Résoudre l'équation $7 \times 0.54^x = 18$
\end{enumerate}
\end{exercise}
\begin{solution}
\begin{enumerate}
\item On veut partager cette évolution en 8 évolutions.
\[
\left(1 + \frac{28}{100}\right)^{\frac{1}{8}} = 1.0313
\]
Donc le taux d'évolution moyen est
\[
t_m = 1.0313 - 1 = 0.031300000000000106
\]
\item Coefficient multiplicateur pour revenir en arrière
\[
CM = (1 + \frac{28}{100})^{-1} = 0.7812
\]
On en déduit la quantité en 2019
\[
138 * 0.7812 = 107.8056
\]
\item L'équation de la droite est
\[
y = 1.0 x -2
\]
\item Il faut penser à faire la division à par $7$ avant d'utiliser le log car sinon, on ne peut pas utiliser la formule $\log(a^n) = n\times \log(a)$.
\[x = \frac{\log(2.57)}{\log(0.54)}\]
\end{enumerate}
\end{solution}
\begin{exercise}[subtitle={Restaurant}]
Un \emph{food truck}, ouvert le midi et le soir, propose deux types de formules :
\setlength\parindent{10mm}
\begin{itemize}
\item la formule \emph{Burger} ;
\item la formule \emph{Wok}.
\end{itemize}
\setlength\parindent{0mm}
\medskip
Le gérant a remarqué que 57\,\% de ses ventes ont lieu le midi. Le quart des ventes du midi correspondent à la formule \emph{Burger}, alors que 49\,\% des ventes du soir correspondent à la formule \emph{Wok}.
Le gérant se constitue un fichier en notant, pour chaque vente, la formule choisie et le moment de cette vente (midi ou soir).
On prélève une fiche de façon équiprobable. On définit les quatre évènements suivants:
\begin{enumerate}
\item $M$ : \og la fiche correspond à une vente du midi\fg{} ;
\item $S$ : \og la fiche correspond à une vente du soir\fg {};
\item $W$ : \og la fiche correspond à une formule \emph{Wok} \fg{} ;
\item $B$ : \og la fiche correspond à une formule \emph{Burger} \fg.
\end{enumerate}
\setlength\parindent{0mm}
\medskip
\begin{enumerate}
\item Recopier puis compléter l'arbre pondéré
\begin{center}
\begin{tikzpicture}[sloped]
\node {.}
child {node {$M$}
child {node {$W$}
edge from parent
node[above] {...}
}
child {node {$B$}
edge from parent
node[above] {...}
}
edge from parent
node[above] {...}
}
child[missing] {}
child { node {$S$}
child {node {$W$}
edge from parent
node[above] {...}
}
child {node {$B$}
edge from parent
node[above] {...}
}
edge from parent
node[above] {...}
} ;
\end{tikzpicture}
\end{center}
\item Calculer la probabilité de l'évènement $M \cap W$. Interpréter ce résultat dans le contexte de l'exercice.
\item Montrer que la probabilité que la fiche choisie corresponde à une formule \emph{Burger} est égale à $0.3592$.
\item On a prélevé une fiche correspondant à la formule \emph{Burger}. Quelle est la probabilité, arrondie au millième, que la vente ait eu lieu le soir?
\end{enumerate}
\end{exercise}
\begin{solution}
\begin{enumerate}
\item
\begin{center}
\begin{tikzpicture}[sloped]
\node {.}
child {node {$M$}
child {node {$W$}
edge from parent
node[above] {$0.75$}
}
child {node {$B$}
edge from parent
node[above] {$0.25$}
}
edge from parent
node[above] {$0.58$}
}
child[missing] {}
child { node {$S$}
child {node {$W$}
edge from parent
node[above] {$0.49$}
}
child {node {$B$}
edge from parent
node[above] {$0.51$}
}
edge from parent
node[above] {$0.42$}
} ;
\end{tikzpicture}
\end{center}
\item On calcule la probabilité que la vente soit un wok et ait eu lieu à midi
\[ P(M\cap W) = P(M) \times P_M(W) = 0.58 \times 0.75 = 0.435 \]
\item Probabilité que la vente soit un burger.
\[
P(B) = P(M\cap B) + P(S\cap B) = 0.58 \times 0.75 + 0.42 \times 0.49 = 0.3592
\]
\item On cherche à calculer la quantité $P_B(S)$. Pour cela on utilise la formule de Bayes
\[
P_B(S) = \frac{P(B\cap S)}{P(B)} = \frac{P_S(B) \times P(S)}{P(B)} = \frac{0.51\times 0.42}{0.3592} = 0.5963251670378619 \approx 0.596
\]
\end{enumerate}
\end{solution}
\begin{exercise}[subtitle={Continent plastique}]
\textit{Les quantités évoqués dans cette exercice sont générés au hasard et sont donc complètement farfelus.}
\medskip
Le \og continent de plastique\fg{} est la plus grande des plaques de déchets plastiques évoluant sur les océans. Elle occupe actuellement dans l'océan Pacifique une surface dont l'aire est évaluée à plus de $1,6$ million de km$^2$, entre Hawaï et la Californie.
En 2017, des scientifiques ont estimé qu'il y avait $9$ millions de tonnes de déchets plastiques qui était déversé chaque année dans les océans et que cette quantité augmentait de $26\n\%$ par chaque année.
On modélise l'évolution de la masse de ces déchets plastiques déversée chaque année, si rien n'est fait pour la réduire, par une suite géométrique $\left(u_n\right)$. L'arrondi au centième du terme $u_n$ représente la masse de ces déchets déversée chaque année, exprimée en million de tonnes, pour l'année $(2017 + n)$.
\medskip
\begin{enumerate}
\item Expliquer pourquoi la suite $u_n$ est géométrique?
\item Calculer $u_1$ et $u_2$.
\item Exprimer $u_n$ en fonction de $n$.
\item Au début de l'année 2017, il y avait $300$ millions de tonnes de déchets plastique. Calculer la quantité totale de déchets plastiques en 2030.
\item On souhaite déterminer en quelle année la masse totale de ces déchets plastiques aura pour la première fois augmenté de $50$\,\% par rapport à sa valeur de 2017.
\begin{enumerate}
\item Recopier et compléter l'algorithme ci-dessous pour que la variable $N$ contienne la réponse au problème posé.
\begin{center}
\begin{tabularx}{0.4\linewidth}{|X|}\hline
$N = 2017$\\
$U = 9$ \\
$S = 300 + U$ \\
while $S < 450$: \\
\hspace{1cm} $N = \ldots$\\
\hspace{1cm} $U = \ldots$\\
\hspace{1cm} $S = \ldots$\\
\hline
\end{tabularx}
\end{center}
\item Que contiennent les variables $S$, $U$ et $N$ après exécution de cet algorithme ?
Interpréter les résultats dans le contexte de l'exercice.
\end{enumerate}
\end{enumerate}
\end{exercise}
\begin{solution}
\begin{enumerate}
\item Une augmentation de $26\,\%$ revient à multiplier la quantité par $1.26$. La suite est donc bien géométrique. Son premier terme est $u_0 = 9$ et sa raison est $q = 1.26$
\item
\[
u_1 = u_0 * 1.26 = 11.34
\]
\[
u_2 = u_0 * 1.26^2 = 14.2884
\]
\item
\[
u_n = u_0 \times q^n = 9 \times 1.26^n
\]
\item On calcule la quantité totale déversée entre 2017 et 2030.
\[
\sum_{n = 0}^{13} u_n = u_0 \times \frac{1-q^{13}}{1-q} = 9 \times \frac{1 - 1.26^{13}}{1 - 1.26} = 663.76
\]
On en déduit la quantité totale de déchets en 2030
\[
300 + 663.76 = 963.76
\]
\item
\begin{enumerate}
\item ~
\begin{center}
\begin{tabularx}{0.4\linewidth}{|X|}\hline
$N \gets 2017$\\
$U \gets 9$ \\
$S \gets 300 + U$ \\
Tant que $S < 450$ \\
\hspace{1cm} $N \gets N + 1$\\
\hspace{1cm} $U \gets U * 1.26$\\
\hspace{1cm} $S \gets S + u$\\
Fin Tant que\\\hline
\end{tabularx}
\end{center}
\item \textit{Pas de correction automatisé}
\end{enumerate}
\end{enumerate}
\end{solution}
\end{document}
%%% Local Variables:
%%% mode: latex
%%% TeX-master: "master"
%%% End:

View File

@ -0,0 +1,262 @@
\documentclass[a4paper,10pt]{article}
\usepackage{myXsim}
% Title Page
\title{DS8 \hfill BALLOFFET Kenza}
\tribe{TST}
\date{\hfillÀ render pour le Mercredi 7 avril}
\xsimsetup{
solution/print = false
}
\begin{document}
\maketitle
\begin{exercise}[subtitle={Automatismes}]
\textit{Toutes les questions de cette exercice sont indépendantes et peuvent être répondus séparément}
\begin{enumerate}
\item De janvier à septembre, une quantité a augmenté de $12\,\%$. Faire un schéma pour représenter la situation puis calculer le taux d'évolution moyen mensuel.
\item Une quantité augmente de $12\,\%$ par ans. En 2020, elle est de 126\euro. Quelle était sa valeur en 2019? Faire un schéma pour représenter la situation.
\item Déterminer l'équation de la droite \\
\begin{tikzpicture}[xscale=0.8, yscale=0.5]
\tkzInit[xmin=-5,xmax=5,xstep=1,
ymin=-5,ymax=5,ystep=1]
\tkzGrid
\tkzAxeXY
\tkzFct[domain=-5:5,color=red,very thick]%
{2.6666666666666665*\x -4};
\end{tikzpicture}
\item Résoudre l'équation $8 \times 0.54^x = 18$
\end{enumerate}
\end{exercise}
\begin{solution}
\begin{enumerate}
\item On veut partager cette évolution en 8 évolutions.
\[
\left(1 + \frac{12}{100}\right)^{\frac{1}{8}} = 1.0143
\]
Donc le taux d'évolution moyen est
\[
t_m = 1.0143 - 1 = 0.01429999999999998
\]
\item Coefficient multiplicateur pour revenir en arrière
\[
CM = (1 + \frac{12}{100})^{-1} = 0.8929
\]
On en déduit la quantité en 2019
\[
126 * 0.8929 = 112.50540000000001
\]
\item L'équation de la droite est
\[
y = 2.6666666666666665 x -4
\]
\item Il faut penser à faire la division à par $8$ avant d'utiliser le log car sinon, on ne peut pas utiliser la formule $\log(a^n) = n\times \log(a)$.
\[x = \frac{\log(2.25)}{\log(0.54)}\]
\end{enumerate}
\end{solution}
\begin{exercise}[subtitle={Restaurant}]
Un \emph{food truck}, ouvert le midi et le soir, propose deux types de formules :
\setlength\parindent{10mm}
\begin{itemize}
\item la formule \emph{Burger} ;
\item la formule \emph{Wok}.
\end{itemize}
\setlength\parindent{0mm}
\medskip
Le gérant a remarqué que 94\,\% de ses ventes ont lieu le midi. Le quart des ventes du midi correspondent à la formule \emph{Burger}, alors que 37\,\% des ventes du soir correspondent à la formule \emph{Wok}.
Le gérant se constitue un fichier en notant, pour chaque vente, la formule choisie et le moment de cette vente (midi ou soir).
On prélève une fiche de façon équiprobable. On définit les quatre évènements suivants:
\begin{enumerate}
\item $M$ : \og la fiche correspond à une vente du midi\fg{} ;
\item $S$ : \og la fiche correspond à une vente du soir\fg {};
\item $W$ : \og la fiche correspond à une formule \emph{Wok} \fg{} ;
\item $B$ : \og la fiche correspond à une formule \emph{Burger} \fg.
\end{enumerate}
\setlength\parindent{0mm}
\medskip
\begin{enumerate}
\item Recopier puis compléter l'arbre pondéré
\begin{center}
\begin{tikzpicture}[sloped]
\node {.}
child {node {$M$}
child {node {$W$}
edge from parent
node[above] {...}
}
child {node {$B$}
edge from parent
node[above] {...}
}
edge from parent
node[above] {...}
}
child[missing] {}
child { node {$S$}
child {node {$W$}
edge from parent
node[above] {...}
}
child {node {$B$}
edge from parent
node[above] {...}
}
edge from parent
node[above] {...}
} ;
\end{tikzpicture}
\end{center}
\item Calculer la probabilité de l'évènement $M \cap W$. Interpréter ce résultat dans le contexte de l'exercice.
\item Montrer que la probabilité que la fiche choisie corresponde à une formule \emph{Burger} est égale à $0.2728$.
\item On a prélevé une fiche correspondant à la formule \emph{Burger}. Quelle est la probabilité, arrondie au millième, que la vente ait eu lieu le soir?
\end{enumerate}
\end{exercise}
\begin{solution}
\begin{enumerate}
\item
\begin{center}
\begin{tikzpicture}[sloped]
\node {.}
child {node {$M$}
child {node {$W$}
edge from parent
node[above] {$0.75$}
}
child {node {$B$}
edge from parent
node[above] {$0.25$}
}
edge from parent
node[above] {$0.94$}
}
child[missing] {}
child { node {$S$}
child {node {$W$}
edge from parent
node[above] {$0.37$}
}
child {node {$B$}
edge from parent
node[above] {$0.63$}
}
edge from parent
node[above] {$0.06$}
} ;
\end{tikzpicture}
\end{center}
\item On calcule la probabilité que la vente soit un wok et ait eu lieu à midi
\[ P(M\cap W) = P(M) \times P_M(W) = 0.94 \times 0.75 = 0.705 \]
\item Probabilité que la vente soit un burger.
\[
P(B) = P(M\cap B) + P(S\cap B) = 0.94 \times 0.75 + 0.06 \times 0.37 = 0.2728
\]
\item On cherche à calculer la quantité $P_B(S)$. Pour cela on utilise la formule de Bayes
\[
P_B(S) = \frac{P(B\cap S)}{P(B)} = \frac{P_S(B) \times P(S)}{P(B)} = \frac{0.63\times 0.06}{0.2728} = 0.13856304985337245 \approx 0.139
\]
\end{enumerate}
\end{solution}
\begin{exercise}[subtitle={Continent plastique}]
\textit{Les quantités évoqués dans cette exercice sont générés au hasard et sont donc complètement farfelus.}
\medskip
Le \og continent de plastique\fg{} est la plus grande des plaques de déchets plastiques évoluant sur les océans. Elle occupe actuellement dans l'océan Pacifique une surface dont l'aire est évaluée à plus de $1,6$ million de km$^2$, entre Hawaï et la Californie.
En 2017, des scientifiques ont estimé qu'il y avait $15$ millions de tonnes de déchets plastiques qui était déversé chaque année dans les océans et que cette quantité augmentait de $19\n\%$ par chaque année.
On modélise l'évolution de la masse de ces déchets plastiques déversée chaque année, si rien n'est fait pour la réduire, par une suite géométrique $\left(u_n\right)$. L'arrondi au centième du terme $u_n$ représente la masse de ces déchets déversée chaque année, exprimée en million de tonnes, pour l'année $(2017 + n)$.
\medskip
\begin{enumerate}
\item Expliquer pourquoi la suite $u_n$ est géométrique?
\item Calculer $u_1$ et $u_2$.
\item Exprimer $u_n$ en fonction de $n$.
\item Au début de l'année 2017, il y avait $300$ millions de tonnes de déchets plastique. Calculer la quantité totale de déchets plastiques en 2030.
\item On souhaite déterminer en quelle année la masse totale de ces déchets plastiques aura pour la première fois augmenté de $50$\,\% par rapport à sa valeur de 2017.
\begin{enumerate}
\item Recopier et compléter l'algorithme ci-dessous pour que la variable $N$ contienne la réponse au problème posé.
\begin{center}
\begin{tabularx}{0.4\linewidth}{|X|}\hline
$N = 2017$\\
$U = 15$ \\
$S = 300 + U$ \\
while $S < 450$: \\
\hspace{1cm} $N = \ldots$\\
\hspace{1cm} $U = \ldots$\\
\hspace{1cm} $S = \ldots$\\
\hline
\end{tabularx}
\end{center}
\item Que contiennent les variables $S$, $U$ et $N$ après exécution de cet algorithme ?
Interpréter les résultats dans le contexte de l'exercice.
\end{enumerate}
\end{enumerate}
\end{exercise}
\begin{solution}
\begin{enumerate}
\item Une augmentation de $19\,\%$ revient à multiplier la quantité par $1.19$. La suite est donc bien géométrique. Son premier terme est $u_0 = 15$ et sa raison est $q = 1.19$
\item
\[
u_1 = u_0 * 1.19 = 17.849999999999998
\]
\[
u_2 = u_0 * 1.19^2 = 21.2415
\]
\item
\[
u_n = u_0 \times q^n = 15 \times 1.19^n
\]
\item On calcule la quantité totale déversée entre 2017 et 2030.
\[
\sum_{n = 0}^{13} u_n = u_0 \times \frac{1-q^{13}}{1-q} = 15 \times \frac{1 - 1.19^{13}}{1 - 1.19} = 678.67
\]
On en déduit la quantité totale de déchets en 2030
\[
300 + 678.67 = 978.67
\]
\item
\begin{enumerate}
\item ~
\begin{center}
\begin{tabularx}{0.4\linewidth}{|X|}\hline
$N \gets 2017$\\
$U \gets 15$ \\
$S \gets 300 + U$ \\
Tant que $S < 450$ \\
\hspace{1cm} $N \gets N + 1$\\
\hspace{1cm} $U \gets U * 1.19$\\
\hspace{1cm} $S \gets S + u$\\
Fin Tant que\\\hline
\end{tabularx}
\end{center}
\item \textit{Pas de correction automatisé}
\end{enumerate}
\end{enumerate}
\end{solution}
\end{document}
%%% Local Variables:
%%% mode: latex
%%% TeX-master: "master"
%%% End:

View File

@ -0,0 +1,262 @@
\documentclass[a4paper,10pt]{article}
\usepackage{myXsim}
% Title Page
\title{DS8 \hfill BENHATTAL Chakir}
\tribe{TST}
\date{\hfillÀ render pour le Mercredi 7 avril}
\xsimsetup{
solution/print = false
}
\begin{document}
\maketitle
\begin{exercise}[subtitle={Automatismes}]
\textit{Toutes les questions de cette exercice sont indépendantes et peuvent être répondus séparément}
\begin{enumerate}
\item De janvier à septembre, une quantité a augmenté de $28\,\%$. Faire un schéma pour représenter la situation puis calculer le taux d'évolution moyen mensuel.
\item Une quantité augmente de $28\,\%$ par ans. En 2020, elle est de 111\euro. Quelle était sa valeur en 2019? Faire un schéma pour représenter la situation.
\item Déterminer l'équation de la droite \\
\begin{tikzpicture}[xscale=0.8, yscale=0.5]
\tkzInit[xmin=-5,xmax=5,xstep=1,
ymin=-5,ymax=5,ystep=1]
\tkzGrid
\tkzAxeXY
\tkzFct[domain=-5:5,color=red,very thick]%
{1.0*\x -1};
\end{tikzpicture}
\item Résoudre l'équation $4 \times 0.77^x = 25$
\end{enumerate}
\end{exercise}
\begin{solution}
\begin{enumerate}
\item On veut partager cette évolution en 8 évolutions.
\[
\left(1 + \frac{28}{100}\right)^{\frac{1}{8}} = 1.0313
\]
Donc le taux d'évolution moyen est
\[
t_m = 1.0313 - 1 = 0.031300000000000106
\]
\item Coefficient multiplicateur pour revenir en arrière
\[
CM = (1 + \frac{28}{100})^{-1} = 0.7812
\]
On en déduit la quantité en 2019
\[
111 * 0.7812 = 86.7132
\]
\item L'équation de la droite est
\[
y = 1.0 x -1
\]
\item Il faut penser à faire la division à par $4$ avant d'utiliser le log car sinon, on ne peut pas utiliser la formule $\log(a^n) = n\times \log(a)$.
\[x = \frac{\log(6.25)}{\log(0.77)}\]
\end{enumerate}
\end{solution}
\begin{exercise}[subtitle={Restaurant}]
Un \emph{food truck}, ouvert le midi et le soir, propose deux types de formules :
\setlength\parindent{10mm}
\begin{itemize}
\item la formule \emph{Burger} ;
\item la formule \emph{Wok}.
\end{itemize}
\setlength\parindent{0mm}
\medskip
Le gérant a remarqué que 89\,\% de ses ventes ont lieu le midi. Le quart des ventes du midi correspondent à la formule \emph{Burger}, alors que 43\,\% des ventes du soir correspondent à la formule \emph{Wok}.
Le gérant se constitue un fichier en notant, pour chaque vente, la formule choisie et le moment de cette vente (midi ou soir).
On prélève une fiche de façon équiprobable. On définit les quatre évènements suivants:
\begin{enumerate}
\item $M$ : \og la fiche correspond à une vente du midi\fg{} ;
\item $S$ : \og la fiche correspond à une vente du soir\fg {};
\item $W$ : \og la fiche correspond à une formule \emph{Wok} \fg{} ;
\item $B$ : \og la fiche correspond à une formule \emph{Burger} \fg.
\end{enumerate}
\setlength\parindent{0mm}
\medskip
\begin{enumerate}
\item Recopier puis compléter l'arbre pondéré
\begin{center}
\begin{tikzpicture}[sloped]
\node {.}
child {node {$M$}
child {node {$W$}
edge from parent
node[above] {...}
}
child {node {$B$}
edge from parent
node[above] {...}
}
edge from parent
node[above] {...}
}
child[missing] {}
child { node {$S$}
child {node {$W$}
edge from parent
node[above] {...}
}
child {node {$B$}
edge from parent
node[above] {...}
}
edge from parent
node[above] {...}
} ;
\end{tikzpicture}
\end{center}
\item Calculer la probabilité de l'évènement $M \cap W$. Interpréter ce résultat dans le contexte de l'exercice.
\item Montrer que la probabilité que la fiche choisie corresponde à une formule \emph{Burger} est égale à $0.2852$.
\item On a prélevé une fiche correspondant à la formule \emph{Burger}. Quelle est la probabilité, arrondie au millième, que la vente ait eu lieu le soir?
\end{enumerate}
\end{exercise}
\begin{solution}
\begin{enumerate}
\item
\begin{center}
\begin{tikzpicture}[sloped]
\node {.}
child {node {$M$}
child {node {$W$}
edge from parent
node[above] {$0.75$}
}
child {node {$B$}
edge from parent
node[above] {$0.25$}
}
edge from parent
node[above] {$0.89$}
}
child[missing] {}
child { node {$S$}
child {node {$W$}
edge from parent
node[above] {$0.43$}
}
child {node {$B$}
edge from parent
node[above] {$0.57$}
}
edge from parent
node[above] {$0.11$}
} ;
\end{tikzpicture}
\end{center}
\item On calcule la probabilité que la vente soit un wok et ait eu lieu à midi
\[ P(M\cap W) = P(M) \times P_M(W) = 0.89 \times 0.75 = 0.6675 \]
\item Probabilité que la vente soit un burger.
\[
P(B) = P(M\cap B) + P(S\cap B) = 0.89 \times 0.75 + 0.11 \times 0.43 = 0.2852
\]
\item On cherche à calculer la quantité $P_B(S)$. Pour cela on utilise la formule de Bayes
\[
P_B(S) = \frac{P(B\cap S)}{P(B)} = \frac{P_S(B) \times P(S)}{P(B)} = \frac{0.57\times 0.11}{0.2852} = 0.2198457223001402 \approx 0.22
\]
\end{enumerate}
\end{solution}
\begin{exercise}[subtitle={Continent plastique}]
\textit{Les quantités évoqués dans cette exercice sont générés au hasard et sont donc complètement farfelus.}
\medskip
Le \og continent de plastique\fg{} est la plus grande des plaques de déchets plastiques évoluant sur les océans. Elle occupe actuellement dans l'océan Pacifique une surface dont l'aire est évaluée à plus de $1,6$ million de km$^2$, entre Hawaï et la Californie.
En 2017, des scientifiques ont estimé qu'il y avait $6$ millions de tonnes de déchets plastiques qui était déversé chaque année dans les océans et que cette quantité augmentait de $13\n\%$ par chaque année.
On modélise l'évolution de la masse de ces déchets plastiques déversée chaque année, si rien n'est fait pour la réduire, par une suite géométrique $\left(u_n\right)$. L'arrondi au centième du terme $u_n$ représente la masse de ces déchets déversée chaque année, exprimée en million de tonnes, pour l'année $(2017 + n)$.
\medskip
\begin{enumerate}
\item Expliquer pourquoi la suite $u_n$ est géométrique?
\item Calculer $u_1$ et $u_2$.
\item Exprimer $u_n$ en fonction de $n$.
\item Au début de l'année 2017, il y avait $300$ millions de tonnes de déchets plastique. Calculer la quantité totale de déchets plastiques en 2030.
\item On souhaite déterminer en quelle année la masse totale de ces déchets plastiques aura pour la première fois augmenté de $50$\,\% par rapport à sa valeur de 2017.
\begin{enumerate}
\item Recopier et compléter l'algorithme ci-dessous pour que la variable $N$ contienne la réponse au problème posé.
\begin{center}
\begin{tabularx}{0.4\linewidth}{|X|}\hline
$N = 2017$\\
$U = 6$ \\
$S = 300 + U$ \\
while $S < 450$: \\
\hspace{1cm} $N = \ldots$\\
\hspace{1cm} $U = \ldots$\\
\hspace{1cm} $S = \ldots$\\
\hline
\end{tabularx}
\end{center}
\item Que contiennent les variables $S$, $U$ et $N$ après exécution de cet algorithme ?
Interpréter les résultats dans le contexte de l'exercice.
\end{enumerate}
\end{enumerate}
\end{exercise}
\begin{solution}
\begin{enumerate}
\item Une augmentation de $13\,\%$ revient à multiplier la quantité par $1.13$. La suite est donc bien géométrique. Son premier terme est $u_0 = 6$ et sa raison est $q = 1.13$
\item
\[
u_1 = u_0 * 1.13 = 6.779999999999999
\]
\[
u_2 = u_0 * 1.13^2 = 7.6614
\]
\item
\[
u_n = u_0 \times q^n = 6 \times 1.13^n
\]
\item On calcule la quantité totale déversée entre 2017 et 2030.
\[
\sum_{n = 0}^{13} u_n = u_0 \times \frac{1-q^{13}}{1-q} = 6 \times \frac{1 - 1.13^{13}}{1 - 1.13} = 179.91
\]
On en déduit la quantité totale de déchets en 2030
\[
300 + 179.91 = 479.90999999999997
\]
\item
\begin{enumerate}
\item ~
\begin{center}
\begin{tabularx}{0.4\linewidth}{|X|}\hline
$N \gets 2017$\\
$U \gets 6$ \\
$S \gets 300 + U$ \\
Tant que $S < 450$ \\
\hspace{1cm} $N \gets N + 1$\\
\hspace{1cm} $U \gets U * 1.13$\\
\hspace{1cm} $S \gets S + u$\\
Fin Tant que\\\hline
\end{tabularx}
\end{center}
\item \textit{Pas de correction automatisé}
\end{enumerate}
\end{enumerate}
\end{solution}
\end{document}
%%% Local Variables:
%%% mode: latex
%%% TeX-master: "master"
%%% End:

View File

@ -0,0 +1,262 @@
\documentclass[a4paper,10pt]{article}
\usepackage{myXsim}
% Title Page
\title{DS8 \hfill CLAIN Avinash}
\tribe{TST}
\date{\hfillÀ render pour le Mercredi 7 avril}
\xsimsetup{
solution/print = false
}
\begin{document}
\maketitle
\begin{exercise}[subtitle={Automatismes}]
\textit{Toutes les questions de cette exercice sont indépendantes et peuvent être répondus séparément}
\begin{enumerate}
\item De janvier à septembre, une quantité a augmenté de $15\,\%$. Faire un schéma pour représenter la situation puis calculer le taux d'évolution moyen mensuel.
\item Une quantité augmente de $15\,\%$ par ans. En 2020, elle est de 112\euro. Quelle était sa valeur en 2019? Faire un schéma pour représenter la situation.
\item Déterminer l'équation de la droite \\
\begin{tikzpicture}[xscale=0.8, yscale=0.5]
\tkzInit[xmin=-5,xmax=5,xstep=1,
ymin=-5,ymax=5,ystep=1]
\tkzGrid
\tkzAxeXY
\tkzFct[domain=-5:5,color=red,very thick]%
{4.0*\x -4};
\end{tikzpicture}
\item Résoudre l'équation $6 \times 0.77^x = 23$
\end{enumerate}
\end{exercise}
\begin{solution}
\begin{enumerate}
\item On veut partager cette évolution en 8 évolutions.
\[
\left(1 + \frac{15}{100}\right)^{\frac{1}{8}} = 1.0176
\]
Donc le taux d'évolution moyen est
\[
t_m = 1.0176 - 1 = 0.01760000000000006
\]
\item Coefficient multiplicateur pour revenir en arrière
\[
CM = (1 + \frac{15}{100})^{-1} = 0.8696
\]
On en déduit la quantité en 2019
\[
112 * 0.8696 = 97.3952
\]
\item L'équation de la droite est
\[
y = 4.0 x -4
\]
\item Il faut penser à faire la division à par $6$ avant d'utiliser le log car sinon, on ne peut pas utiliser la formule $\log(a^n) = n\times \log(a)$.
\[x = \frac{\log(3.83)}{\log(0.77)}\]
\end{enumerate}
\end{solution}
\begin{exercise}[subtitle={Restaurant}]
Un \emph{food truck}, ouvert le midi et le soir, propose deux types de formules :
\setlength\parindent{10mm}
\begin{itemize}
\item la formule \emph{Burger} ;
\item la formule \emph{Wok}.
\end{itemize}
\setlength\parindent{0mm}
\medskip
Le gérant a remarqué que 92\,\% de ses ventes ont lieu le midi. Le quart des ventes du midi correspondent à la formule \emph{Burger}, alors que 54\,\% des ventes du soir correspondent à la formule \emph{Wok}.
Le gérant se constitue un fichier en notant, pour chaque vente, la formule choisie et le moment de cette vente (midi ou soir).
On prélève une fiche de façon équiprobable. On définit les quatre évènements suivants:
\begin{enumerate}
\item $M$ : \og la fiche correspond à une vente du midi\fg{} ;
\item $S$ : \og la fiche correspond à une vente du soir\fg {};
\item $W$ : \og la fiche correspond à une formule \emph{Wok} \fg{} ;
\item $B$ : \og la fiche correspond à une formule \emph{Burger} \fg.
\end{enumerate}
\setlength\parindent{0mm}
\medskip
\begin{enumerate}
\item Recopier puis compléter l'arbre pondéré
\begin{center}
\begin{tikzpicture}[sloped]
\node {.}
child {node {$M$}
child {node {$W$}
edge from parent
node[above] {...}
}
child {node {$B$}
edge from parent
node[above] {...}
}
edge from parent
node[above] {...}
}
child[missing] {}
child { node {$S$}
child {node {$W$}
edge from parent
node[above] {...}
}
child {node {$B$}
edge from parent
node[above] {...}
}
edge from parent
node[above] {...}
} ;
\end{tikzpicture}
\end{center}
\item Calculer la probabilité de l'évènement $M \cap W$. Interpréter ce résultat dans le contexte de l'exercice.
\item Montrer que la probabilité que la fiche choisie corresponde à une formule \emph{Burger} est égale à $0.2668$.
\item On a prélevé une fiche correspondant à la formule \emph{Burger}. Quelle est la probabilité, arrondie au millième, que la vente ait eu lieu le soir?
\end{enumerate}
\end{exercise}
\begin{solution}
\begin{enumerate}
\item
\begin{center}
\begin{tikzpicture}[sloped]
\node {.}
child {node {$M$}
child {node {$W$}
edge from parent
node[above] {$0.75$}
}
child {node {$B$}
edge from parent
node[above] {$0.25$}
}
edge from parent
node[above] {$0.92$}
}
child[missing] {}
child { node {$S$}
child {node {$W$}
edge from parent
node[above] {$0.54$}
}
child {node {$B$}
edge from parent
node[above] {$0.46$}
}
edge from parent
node[above] {$0.08$}
} ;
\end{tikzpicture}
\end{center}
\item On calcule la probabilité que la vente soit un wok et ait eu lieu à midi
\[ P(M\cap W) = P(M) \times P_M(W) = 0.92 \times 0.75 = 0.69 \]
\item Probabilité que la vente soit un burger.
\[
P(B) = P(M\cap B) + P(S\cap B) = 0.92 \times 0.75 + 0.08 \times 0.54 = 0.2668
\]
\item On cherche à calculer la quantité $P_B(S)$. Pour cela on utilise la formule de Bayes
\[
P_B(S) = \frac{P(B\cap S)}{P(B)} = \frac{P_S(B) \times P(S)}{P(B)} = \frac{0.46\times 0.08}{0.2668} = 0.13793103448275862 \approx 0.138
\]
\end{enumerate}
\end{solution}
\begin{exercise}[subtitle={Continent plastique}]
\textit{Les quantités évoqués dans cette exercice sont générés au hasard et sont donc complètement farfelus.}
\medskip
Le \og continent de plastique\fg{} est la plus grande des plaques de déchets plastiques évoluant sur les océans. Elle occupe actuellement dans l'océan Pacifique une surface dont l'aire est évaluée à plus de $1,6$ million de km$^2$, entre Hawaï et la Californie.
En 2017, des scientifiques ont estimé qu'il y avait $18$ millions de tonnes de déchets plastiques qui était déversé chaque année dans les océans et que cette quantité augmentait de $21\n\%$ par chaque année.
On modélise l'évolution de la masse de ces déchets plastiques déversée chaque année, si rien n'est fait pour la réduire, par une suite géométrique $\left(u_n\right)$. L'arrondi au centième du terme $u_n$ représente la masse de ces déchets déversée chaque année, exprimée en million de tonnes, pour l'année $(2017 + n)$.
\medskip
\begin{enumerate}
\item Expliquer pourquoi la suite $u_n$ est géométrique?
\item Calculer $u_1$ et $u_2$.
\item Exprimer $u_n$ en fonction de $n$.
\item Au début de l'année 2017, il y avait $300$ millions de tonnes de déchets plastique. Calculer la quantité totale de déchets plastiques en 2030.
\item On souhaite déterminer en quelle année la masse totale de ces déchets plastiques aura pour la première fois augmenté de $50$\,\% par rapport à sa valeur de 2017.
\begin{enumerate}
\item Recopier et compléter l'algorithme ci-dessous pour que la variable $N$ contienne la réponse au problème posé.
\begin{center}
\begin{tabularx}{0.4\linewidth}{|X|}\hline
$N = 2017$\\
$U = 18$ \\
$S = 300 + U$ \\
while $S < 450$: \\
\hspace{1cm} $N = \ldots$\\
\hspace{1cm} $U = \ldots$\\
\hspace{1cm} $S = \ldots$\\
\hline
\end{tabularx}
\end{center}
\item Que contiennent les variables $S$, $U$ et $N$ après exécution de cet algorithme ?
Interpréter les résultats dans le contexte de l'exercice.
\end{enumerate}
\end{enumerate}
\end{exercise}
\begin{solution}
\begin{enumerate}
\item Une augmentation de $21\,\%$ revient à multiplier la quantité par $1.21$. La suite est donc bien géométrique. Son premier terme est $u_0 = 18$ et sa raison est $q = 1.21$
\item
\[
u_1 = u_0 * 1.21 = 21.78
\]
\[
u_2 = u_0 * 1.21^2 = 26.3538
\]
\item
\[
u_n = u_0 \times q^n = 18 \times 1.21^n
\]
\item On calcule la quantité totale déversée entre 2017 et 2030.
\[
\sum_{n = 0}^{13} u_n = u_0 \times \frac{1-q^{13}}{1-q} = 18 \times \frac{1 - 1.21^{13}}{1 - 1.21} = 935.84
\]
On en déduit la quantité totale de déchets en 2030
\[
300 + 935.84 = 1235.8400000000001
\]
\item
\begin{enumerate}
\item ~
\begin{center}
\begin{tabularx}{0.4\linewidth}{|X|}\hline
$N \gets 2017$\\
$U \gets 18$ \\
$S \gets 300 + U$ \\
Tant que $S < 450$ \\
\hspace{1cm} $N \gets N + 1$\\
\hspace{1cm} $U \gets U * 1.21$\\
\hspace{1cm} $S \gets S + u$\\
Fin Tant que\\\hline
\end{tabularx}
\end{center}
\item \textit{Pas de correction automatisé}
\end{enumerate}
\end{enumerate}
\end{solution}
\end{document}
%%% Local Variables:
%%% mode: latex
%%% TeX-master: "master"
%%% End:

View File

@ -0,0 +1,262 @@
\documentclass[a4paper,10pt]{article}
\usepackage{myXsim}
% Title Page
\title{DS8 \hfill COLASSI Alexis}
\tribe{TST}
\date{\hfillÀ render pour le Mercredi 7 avril}
\xsimsetup{
solution/print = false
}
\begin{document}
\maketitle
\begin{exercise}[subtitle={Automatismes}]
\textit{Toutes les questions de cette exercice sont indépendantes et peuvent être répondus séparément}
\begin{enumerate}
\item De janvier à septembre, une quantité a augmenté de $27\,\%$. Faire un schéma pour représenter la situation puis calculer le taux d'évolution moyen mensuel.
\item Une quantité augmente de $27\,\%$ par ans. En 2020, elle est de 116\euro. Quelle était sa valeur en 2019? Faire un schéma pour représenter la situation.
\item Déterminer l'équation de la droite \\
\begin{tikzpicture}[xscale=0.8, yscale=0.5]
\tkzInit[xmin=-5,xmax=5,xstep=1,
ymin=-5,ymax=5,ystep=1]
\tkzGrid
\tkzAxeXY
\tkzFct[domain=-5:5,color=red,very thick]%
{2.0*\x -2};
\end{tikzpicture}
\item Résoudre l'équation $7 \times 0.23^x = 16$
\end{enumerate}
\end{exercise}
\begin{solution}
\begin{enumerate}
\item On veut partager cette évolution en 8 évolutions.
\[
\left(1 + \frac{27}{100}\right)^{\frac{1}{8}} = 1.0303
\]
Donc le taux d'évolution moyen est
\[
t_m = 1.0303 - 1 = 0.030299999999999994
\]
\item Coefficient multiplicateur pour revenir en arrière
\[
CM = (1 + \frac{27}{100})^{-1} = 0.7874
\]
On en déduit la quantité en 2019
\[
116 * 0.7874 = 91.3384
\]
\item L'équation de la droite est
\[
y = 2.0 x -2
\]
\item Il faut penser à faire la division à par $7$ avant d'utiliser le log car sinon, on ne peut pas utiliser la formule $\log(a^n) = n\times \log(a)$.
\[x = \frac{\log(2.29)}{\log(0.23)}\]
\end{enumerate}
\end{solution}
\begin{exercise}[subtitle={Restaurant}]
Un \emph{food truck}, ouvert le midi et le soir, propose deux types de formules :
\setlength\parindent{10mm}
\begin{itemize}
\item la formule \emph{Burger} ;
\item la formule \emph{Wok}.
\end{itemize}
\setlength\parindent{0mm}
\medskip
Le gérant a remarqué que 53\,\% de ses ventes ont lieu le midi. Le quart des ventes du midi correspondent à la formule \emph{Burger}, alors que 26\,\% des ventes du soir correspondent à la formule \emph{Wok}.
Le gérant se constitue un fichier en notant, pour chaque vente, la formule choisie et le moment de cette vente (midi ou soir).
On prélève une fiche de façon équiprobable. On définit les quatre évènements suivants:
\begin{enumerate}
\item $M$ : \og la fiche correspond à une vente du midi\fg{} ;
\item $S$ : \og la fiche correspond à une vente du soir\fg {};
\item $W$ : \og la fiche correspond à une formule \emph{Wok} \fg{} ;
\item $B$ : \og la fiche correspond à une formule \emph{Burger} \fg.
\end{enumerate}
\setlength\parindent{0mm}
\medskip
\begin{enumerate}
\item Recopier puis compléter l'arbre pondéré
\begin{center}
\begin{tikzpicture}[sloped]
\node {.}
child {node {$M$}
child {node {$W$}
edge from parent
node[above] {...}
}
child {node {$B$}
edge from parent
node[above] {...}
}
edge from parent
node[above] {...}
}
child[missing] {}
child { node {$S$}
child {node {$W$}
edge from parent
node[above] {...}
}
child {node {$B$}
edge from parent
node[above] {...}
}
edge from parent
node[above] {...}
} ;
\end{tikzpicture}
\end{center}
\item Calculer la probabilité de l'évènement $M \cap W$. Interpréter ce résultat dans le contexte de l'exercice.
\item Montrer que la probabilité que la fiche choisie corresponde à une formule \emph{Burger} est égale à $0.4803$.
\item On a prélevé une fiche correspondant à la formule \emph{Burger}. Quelle est la probabilité, arrondie au millième, que la vente ait eu lieu le soir?
\end{enumerate}
\end{exercise}
\begin{solution}
\begin{enumerate}
\item
\begin{center}
\begin{tikzpicture}[sloped]
\node {.}
child {node {$M$}
child {node {$W$}
edge from parent
node[above] {$0.75$}
}
child {node {$B$}
edge from parent
node[above] {$0.25$}
}
edge from parent
node[above] {$0.53$}
}
child[missing] {}
child { node {$S$}
child {node {$W$}
edge from parent
node[above] {$0.26$}
}
child {node {$B$}
edge from parent
node[above] {$0.74$}
}
edge from parent
node[above] {$0.47$}
} ;
\end{tikzpicture}
\end{center}
\item On calcule la probabilité que la vente soit un wok et ait eu lieu à midi
\[ P(M\cap W) = P(M) \times P_M(W) = 0.53 \times 0.75 = 0.3975 \]
\item Probabilité que la vente soit un burger.
\[
P(B) = P(M\cap B) + P(S\cap B) = 0.53 \times 0.75 + 0.47 \times 0.26 = 0.4803
\]
\item On cherche à calculer la quantité $P_B(S)$. Pour cela on utilise la formule de Bayes
\[
P_B(S) = \frac{P(B\cap S)}{P(B)} = \frac{P_S(B) \times P(S)}{P(B)} = \frac{0.74\times 0.47}{0.4803} = 0.7241307516135749 \approx 0.724
\]
\end{enumerate}
\end{solution}
\begin{exercise}[subtitle={Continent plastique}]
\textit{Les quantités évoqués dans cette exercice sont générés au hasard et sont donc complètement farfelus.}
\medskip
Le \og continent de plastique\fg{} est la plus grande des plaques de déchets plastiques évoluant sur les océans. Elle occupe actuellement dans l'océan Pacifique une surface dont l'aire est évaluée à plus de $1,6$ million de km$^2$, entre Hawaï et la Californie.
En 2017, des scientifiques ont estimé qu'il y avait $3$ millions de tonnes de déchets plastiques qui était déversé chaque année dans les océans et que cette quantité augmentait de $17\n\%$ par chaque année.
On modélise l'évolution de la masse de ces déchets plastiques déversée chaque année, si rien n'est fait pour la réduire, par une suite géométrique $\left(u_n\right)$. L'arrondi au centième du terme $u_n$ représente la masse de ces déchets déversée chaque année, exprimée en million de tonnes, pour l'année $(2017 + n)$.
\medskip
\begin{enumerate}
\item Expliquer pourquoi la suite $u_n$ est géométrique?
\item Calculer $u_1$ et $u_2$.
\item Exprimer $u_n$ en fonction de $n$.
\item Au début de l'année 2017, il y avait $300$ millions de tonnes de déchets plastique. Calculer la quantité totale de déchets plastiques en 2030.
\item On souhaite déterminer en quelle année la masse totale de ces déchets plastiques aura pour la première fois augmenté de $50$\,\% par rapport à sa valeur de 2017.
\begin{enumerate}
\item Recopier et compléter l'algorithme ci-dessous pour que la variable $N$ contienne la réponse au problème posé.
\begin{center}
\begin{tabularx}{0.4\linewidth}{|X|}\hline
$N = 2017$\\
$U = 3$ \\
$S = 300 + U$ \\
while $S < 450$: \\
\hspace{1cm} $N = \ldots$\\
\hspace{1cm} $U = \ldots$\\
\hspace{1cm} $S = \ldots$\\
\hline
\end{tabularx}
\end{center}
\item Que contiennent les variables $S$, $U$ et $N$ après exécution de cet algorithme ?
Interpréter les résultats dans le contexte de l'exercice.
\end{enumerate}
\end{enumerate}
\end{exercise}
\begin{solution}
\begin{enumerate}
\item Une augmentation de $17\,\%$ revient à multiplier la quantité par $1.17$. La suite est donc bien géométrique. Son premier terme est $u_0 = 3$ et sa raison est $q = 1.17$
\item
\[
u_1 = u_0 * 1.17 = 3.51
\]
\[
u_2 = u_0 * 1.17^2 = 4.1067
\]
\item
\[
u_n = u_0 \times q^n = 3 \times 1.17^n
\]
\item On calcule la quantité totale déversée entre 2017 et 2030.
\[
\sum_{n = 0}^{13} u_n = u_0 \times \frac{1-q^{13}}{1-q} = 3 \times \frac{1 - 1.17^{13}}{1 - 1.17} = 118.21
\]
On en déduit la quantité totale de déchets en 2030
\[
300 + 118.21 = 418.21
\]
\item
\begin{enumerate}
\item ~
\begin{center}
\begin{tabularx}{0.4\linewidth}{|X|}\hline
$N \gets 2017$\\
$U \gets 3$ \\
$S \gets 300 + U$ \\
Tant que $S < 450$ \\
\hspace{1cm} $N \gets N + 1$\\
\hspace{1cm} $U \gets U * 1.17$\\
\hspace{1cm} $S \gets S + u$\\
Fin Tant que\\\hline
\end{tabularx}
\end{center}
\item \textit{Pas de correction automatisé}
\end{enumerate}
\end{enumerate}
\end{solution}
\end{document}
%%% Local Variables:
%%% mode: latex
%%% TeX-master: "master"
%%% End:

View File

@ -0,0 +1,262 @@
\documentclass[a4paper,10pt]{article}
\usepackage{myXsim}
% Title Page
\title{DS8 \hfill COUBAT Alexis}
\tribe{TST}
\date{\hfillÀ render pour le Mercredi 7 avril}
\xsimsetup{
solution/print = false
}
\begin{document}
\maketitle
\begin{exercise}[subtitle={Automatismes}]
\textit{Toutes les questions de cette exercice sont indépendantes et peuvent être répondus séparément}
\begin{enumerate}
\item De janvier à septembre, une quantité a augmenté de $16\,\%$. Faire un schéma pour représenter la situation puis calculer le taux d'évolution moyen mensuel.
\item Une quantité augmente de $16\,\%$ par ans. En 2020, elle est de 120\euro. Quelle était sa valeur en 2019? Faire un schéma pour représenter la situation.
\item Déterminer l'équation de la droite \\
\begin{tikzpicture}[xscale=0.8, yscale=0.5]
\tkzInit[xmin=-5,xmax=5,xstep=1,
ymin=-5,ymax=5,ystep=1]
\tkzGrid
\tkzAxeXY
\tkzFct[domain=-5:5,color=red,very thick]%
{4.0*\x -4};
\end{tikzpicture}
\item Résoudre l'équation $10 \times 0.06^x = 34$
\end{enumerate}
\end{exercise}
\begin{solution}
\begin{enumerate}
\item On veut partager cette évolution en 8 évolutions.
\[
\left(1 + \frac{16}{100}\right)^{\frac{1}{8}} = 1.0187
\]
Donc le taux d'évolution moyen est
\[
t_m = 1.0187 - 1 = 0.01869999999999994
\]
\item Coefficient multiplicateur pour revenir en arrière
\[
CM = (1 + \frac{16}{100})^{-1} = 0.8621
\]
On en déduit la quantité en 2019
\[
120 * 0.8621 = 103.452
\]
\item L'équation de la droite est
\[
y = 4.0 x -4
\]
\item Il faut penser à faire la division à par $10$ avant d'utiliser le log car sinon, on ne peut pas utiliser la formule $\log(a^n) = n\times \log(a)$.
\[x = \frac{\log(3.4)}{\log(0.06)}\]
\end{enumerate}
\end{solution}
\begin{exercise}[subtitle={Restaurant}]
Un \emph{food truck}, ouvert le midi et le soir, propose deux types de formules :
\setlength\parindent{10mm}
\begin{itemize}
\item la formule \emph{Burger} ;
\item la formule \emph{Wok}.
\end{itemize}
\setlength\parindent{0mm}
\medskip
Le gérant a remarqué que 1\,\% de ses ventes ont lieu le midi. Le quart des ventes du midi correspondent à la formule \emph{Burger}, alors que 92\,\% des ventes du soir correspondent à la formule \emph{Wok}.
Le gérant se constitue un fichier en notant, pour chaque vente, la formule choisie et le moment de cette vente (midi ou soir).
On prélève une fiche de façon équiprobable. On définit les quatre évènements suivants:
\begin{enumerate}
\item $M$ : \og la fiche correspond à une vente du midi\fg{} ;
\item $S$ : \og la fiche correspond à une vente du soir\fg {};
\item $W$ : \og la fiche correspond à une formule \emph{Wok} \fg{} ;
\item $B$ : \og la fiche correspond à une formule \emph{Burger} \fg.
\end{enumerate}
\setlength\parindent{0mm}
\medskip
\begin{enumerate}
\item Recopier puis compléter l'arbre pondéré
\begin{center}
\begin{tikzpicture}[sloped]
\node {.}
child {node {$M$}
child {node {$W$}
edge from parent
node[above] {...}
}
child {node {$B$}
edge from parent
node[above] {...}
}
edge from parent
node[above] {...}
}
child[missing] {}
child { node {$S$}
child {node {$W$}
edge from parent
node[above] {...}
}
child {node {$B$}
edge from parent
node[above] {...}
}
edge from parent
node[above] {...}
} ;
\end{tikzpicture}
\end{center}
\item Calculer la probabilité de l'évènement $M \cap W$. Interpréter ce résultat dans le contexte de l'exercice.
\item Montrer que la probabilité que la fiche choisie corresponde à une formule \emph{Burger} est égale à $0.0817$.
\item On a prélevé une fiche correspondant à la formule \emph{Burger}. Quelle est la probabilité, arrondie au millième, que la vente ait eu lieu le soir?
\end{enumerate}
\end{exercise}
\begin{solution}
\begin{enumerate}
\item
\begin{center}
\begin{tikzpicture}[sloped]
\node {.}
child {node {$M$}
child {node {$W$}
edge from parent
node[above] {$0.75$}
}
child {node {$B$}
edge from parent
node[above] {$0.25$}
}
edge from parent
node[above] {$0.01$}
}
child[missing] {}
child { node {$S$}
child {node {$W$}
edge from parent
node[above] {$0.92$}
}
child {node {$B$}
edge from parent
node[above] {$0.08$}
}
edge from parent
node[above] {$0.99$}
} ;
\end{tikzpicture}
\end{center}
\item On calcule la probabilité que la vente soit un wok et ait eu lieu à midi
\[ P(M\cap W) = P(M) \times P_M(W) = 0.01 \times 0.75 = 0.0075 \]
\item Probabilité que la vente soit un burger.
\[
P(B) = P(M\cap B) + P(S\cap B) = 0.01 \times 0.75 + 0.99 \times 0.92 = 0.0817
\]
\item On cherche à calculer la quantité $P_B(S)$. Pour cela on utilise la formule de Bayes
\[
P_B(S) = \frac{P(B\cap S)}{P(B)} = \frac{P_S(B) \times P(S)}{P(B)} = \frac{0.08\times 0.99}{0.0817} = 0.9694002447980418 \approx 0.969
\]
\end{enumerate}
\end{solution}
\begin{exercise}[subtitle={Continent plastique}]
\textit{Les quantités évoqués dans cette exercice sont générés au hasard et sont donc complètement farfelus.}
\medskip
Le \og continent de plastique\fg{} est la plus grande des plaques de déchets plastiques évoluant sur les océans. Elle occupe actuellement dans l'océan Pacifique une surface dont l'aire est évaluée à plus de $1,6$ million de km$^2$, entre Hawaï et la Californie.
En 2017, des scientifiques ont estimé qu'il y avait $12$ millions de tonnes de déchets plastiques qui était déversé chaque année dans les océans et que cette quantité augmentait de $14\n\%$ par chaque année.
On modélise l'évolution de la masse de ces déchets plastiques déversée chaque année, si rien n'est fait pour la réduire, par une suite géométrique $\left(u_n\right)$. L'arrondi au centième du terme $u_n$ représente la masse de ces déchets déversée chaque année, exprimée en million de tonnes, pour l'année $(2017 + n)$.
\medskip
\begin{enumerate}
\item Expliquer pourquoi la suite $u_n$ est géométrique?
\item Calculer $u_1$ et $u_2$.
\item Exprimer $u_n$ en fonction de $n$.
\item Au début de l'année 2017, il y avait $300$ millions de tonnes de déchets plastique. Calculer la quantité totale de déchets plastiques en 2030.
\item On souhaite déterminer en quelle année la masse totale de ces déchets plastiques aura pour la première fois augmenté de $50$\,\% par rapport à sa valeur de 2017.
\begin{enumerate}
\item Recopier et compléter l'algorithme ci-dessous pour que la variable $N$ contienne la réponse au problème posé.
\begin{center}
\begin{tabularx}{0.4\linewidth}{|X|}\hline
$N = 2017$\\
$U = 12$ \\
$S = 300 + U$ \\
while $S < 450$: \\
\hspace{1cm} $N = \ldots$\\
\hspace{1cm} $U = \ldots$\\
\hspace{1cm} $S = \ldots$\\
\hline
\end{tabularx}
\end{center}
\item Que contiennent les variables $S$, $U$ et $N$ après exécution de cet algorithme ?
Interpréter les résultats dans le contexte de l'exercice.
\end{enumerate}
\end{enumerate}
\end{exercise}
\begin{solution}
\begin{enumerate}
\item Une augmentation de $14\,\%$ revient à multiplier la quantité par $1.1400000000000001$. La suite est donc bien géométrique. Son premier terme est $u_0 = 12$ et sa raison est $q = 1.1400000000000001$
\item
\[
u_1 = u_0 * 1.1400000000000001 = 13.680000000000001
\]
\[
u_2 = u_0 * 1.1400000000000001^2 = 15.5952
\]
\item
\[
u_n = u_0 \times q^n = 12 \times 1.1400000000000001^n
\]
\item On calcule la quantité totale déversée entre 2017 et 2030.
\[
\sum_{n = 0}^{13} u_n = u_0 \times \frac{1-q^{13}}{1-q} = 12 \times \frac{1 - 1.1400000000000001^{13}}{1 - 1.1400000000000001} = 385.06
\]
On en déduit la quantité totale de déchets en 2030
\[
300 + 385.06 = 685.06
\]
\item
\begin{enumerate}
\item ~
\begin{center}
\begin{tabularx}{0.4\linewidth}{|X|}\hline
$N \gets 2017$\\
$U \gets 12$ \\
$S \gets 300 + U$ \\
Tant que $S < 450$ \\
\hspace{1cm} $N \gets N + 1$\\
\hspace{1cm} $U \gets U * 1.1400000000000001$\\
\hspace{1cm} $S \gets S + u$\\
Fin Tant que\\\hline
\end{tabularx}
\end{center}
\item \textit{Pas de correction automatisé}
\end{enumerate}
\end{enumerate}
\end{solution}
\end{document}
%%% Local Variables:
%%% mode: latex
%%% TeX-master: "master"
%%% End:

View File

@ -0,0 +1,262 @@
\documentclass[a4paper,10pt]{article}
\usepackage{myXsim}
% Title Page
\title{DS8 \hfill COULLON Anis}
\tribe{TST}
\date{\hfillÀ render pour le Mercredi 7 avril}
\xsimsetup{
solution/print = false
}
\begin{document}
\maketitle
\begin{exercise}[subtitle={Automatismes}]
\textit{Toutes les questions de cette exercice sont indépendantes et peuvent être répondus séparément}
\begin{enumerate}
\item De janvier à septembre, une quantité a augmenté de $14\,\%$. Faire un schéma pour représenter la situation puis calculer le taux d'évolution moyen mensuel.
\item Une quantité augmente de $14\,\%$ par ans. En 2020, elle est de 125\euro. Quelle était sa valeur en 2019? Faire un schéma pour représenter la situation.
\item Déterminer l'équation de la droite \\
\begin{tikzpicture}[xscale=0.8, yscale=0.5]
\tkzInit[xmin=-5,xmax=5,xstep=1,
ymin=-5,ymax=5,ystep=1]
\tkzGrid
\tkzAxeXY
\tkzFct[domain=-5:5,color=red,very thick]%
{0.5*\x -1};
\end{tikzpicture}
\item Résoudre l'équation $6 \times 0.82^x = 19$
\end{enumerate}
\end{exercise}
\begin{solution}
\begin{enumerate}
\item On veut partager cette évolution en 8 évolutions.
\[
\left(1 + \frac{14}{100}\right)^{\frac{1}{8}} = 1.0165
\]
Donc le taux d'évolution moyen est
\[
t_m = 1.0165 - 1 = 0.01649999999999996
\]
\item Coefficient multiplicateur pour revenir en arrière
\[
CM = (1 + \frac{14}{100})^{-1} = 0.8772
\]
On en déduit la quantité en 2019
\[
125 * 0.8772 = 109.64999999999999
\]
\item L'équation de la droite est
\[
y = 0.5 x -1
\]
\item Il faut penser à faire la division à par $6$ avant d'utiliser le log car sinon, on ne peut pas utiliser la formule $\log(a^n) = n\times \log(a)$.
\[x = \frac{\log(3.17)}{\log(0.82)}\]
\end{enumerate}
\end{solution}
\begin{exercise}[subtitle={Restaurant}]
Un \emph{food truck}, ouvert le midi et le soir, propose deux types de formules :
\setlength\parindent{10mm}
\begin{itemize}
\item la formule \emph{Burger} ;
\item la formule \emph{Wok}.
\end{itemize}
\setlength\parindent{0mm}
\medskip
Le gérant a remarqué que 30\,\% de ses ventes ont lieu le midi. Le quart des ventes du midi correspondent à la formule \emph{Burger}, alors que 27\,\% des ventes du soir correspondent à la formule \emph{Wok}.
Le gérant se constitue un fichier en notant, pour chaque vente, la formule choisie et le moment de cette vente (midi ou soir).
On prélève une fiche de façon équiprobable. On définit les quatre évènements suivants:
\begin{enumerate}
\item $M$ : \og la fiche correspond à une vente du midi\fg{} ;
\item $S$ : \og la fiche correspond à une vente du soir\fg {};
\item $W$ : \og la fiche correspond à une formule \emph{Wok} \fg{} ;
\item $B$ : \og la fiche correspond à une formule \emph{Burger} \fg.
\end{enumerate}
\setlength\parindent{0mm}
\medskip
\begin{enumerate}
\item Recopier puis compléter l'arbre pondéré
\begin{center}
\begin{tikzpicture}[sloped]
\node {.}
child {node {$M$}
child {node {$W$}
edge from parent
node[above] {...}
}
child {node {$B$}
edge from parent
node[above] {...}
}
edge from parent
node[above] {...}
}
child[missing] {}
child { node {$S$}
child {node {$W$}
edge from parent
node[above] {...}
}
child {node {$B$}
edge from parent
node[above] {...}
}
edge from parent
node[above] {...}
} ;
\end{tikzpicture}
\end{center}
\item Calculer la probabilité de l'évènement $M \cap W$. Interpréter ce résultat dans le contexte de l'exercice.
\item Montrer que la probabilité que la fiche choisie corresponde à une formule \emph{Burger} est égale à $0.586$.
\item On a prélevé une fiche correspondant à la formule \emph{Burger}. Quelle est la probabilité, arrondie au millième, que la vente ait eu lieu le soir?
\end{enumerate}
\end{exercise}
\begin{solution}
\begin{enumerate}
\item
\begin{center}
\begin{tikzpicture}[sloped]
\node {.}
child {node {$M$}
child {node {$W$}
edge from parent
node[above] {$0.75$}
}
child {node {$B$}
edge from parent
node[above] {$0.25$}
}
edge from parent
node[above] {$0.3$}
}
child[missing] {}
child { node {$S$}
child {node {$W$}
edge from parent
node[above] {$0.27$}
}
child {node {$B$}
edge from parent
node[above] {$0.73$}
}
edge from parent
node[above] {$0.7$}
} ;
\end{tikzpicture}
\end{center}
\item On calcule la probabilité que la vente soit un wok et ait eu lieu à midi
\[ P(M\cap W) = P(M) \times P_M(W) = 0.3 \times 0.75 = 0.225 \]
\item Probabilité que la vente soit un burger.
\[
P(B) = P(M\cap B) + P(S\cap B) = 0.3 \times 0.75 + 0.7 \times 0.27 = 0.586
\]
\item On cherche à calculer la quantité $P_B(S)$. Pour cela on utilise la formule de Bayes
\[
P_B(S) = \frac{P(B\cap S)}{P(B)} = \frac{P_S(B) \times P(S)}{P(B)} = \frac{0.73\times 0.7}{0.586} = 0.8720136518771332 \approx 0.872
\]
\end{enumerate}
\end{solution}
\begin{exercise}[subtitle={Continent plastique}]
\textit{Les quantités évoqués dans cette exercice sont générés au hasard et sont donc complètement farfelus.}
\medskip
Le \og continent de plastique\fg{} est la plus grande des plaques de déchets plastiques évoluant sur les océans. Elle occupe actuellement dans l'océan Pacifique une surface dont l'aire est évaluée à plus de $1,6$ million de km$^2$, entre Hawaï et la Californie.
En 2017, des scientifiques ont estimé qu'il y avait $14$ millions de tonnes de déchets plastiques qui était déversé chaque année dans les océans et que cette quantité augmentait de $25\n\%$ par chaque année.
On modélise l'évolution de la masse de ces déchets plastiques déversée chaque année, si rien n'est fait pour la réduire, par une suite géométrique $\left(u_n\right)$. L'arrondi au centième du terme $u_n$ représente la masse de ces déchets déversée chaque année, exprimée en million de tonnes, pour l'année $(2017 + n)$.
\medskip
\begin{enumerate}
\item Expliquer pourquoi la suite $u_n$ est géométrique?
\item Calculer $u_1$ et $u_2$.
\item Exprimer $u_n$ en fonction de $n$.
\item Au début de l'année 2017, il y avait $300$ millions de tonnes de déchets plastique. Calculer la quantité totale de déchets plastiques en 2030.
\item On souhaite déterminer en quelle année la masse totale de ces déchets plastiques aura pour la première fois augmenté de $50$\,\% par rapport à sa valeur de 2017.
\begin{enumerate}
\item Recopier et compléter l'algorithme ci-dessous pour que la variable $N$ contienne la réponse au problème posé.
\begin{center}
\begin{tabularx}{0.4\linewidth}{|X|}\hline
$N = 2017$\\
$U = 14$ \\
$S = 300 + U$ \\
while $S < 450$: \\
\hspace{1cm} $N = \ldots$\\
\hspace{1cm} $U = \ldots$\\
\hspace{1cm} $S = \ldots$\\
\hline
\end{tabularx}
\end{center}
\item Que contiennent les variables $S$, $U$ et $N$ après exécution de cet algorithme ?
Interpréter les résultats dans le contexte de l'exercice.
\end{enumerate}
\end{enumerate}
\end{exercise}
\begin{solution}
\begin{enumerate}
\item Une augmentation de $25\,\%$ revient à multiplier la quantité par $1.25$. La suite est donc bien géométrique. Son premier terme est $u_0 = 14$ et sa raison est $q = 1.25$
\item
\[
u_1 = u_0 * 1.25 = 17.5
\]
\[
u_2 = u_0 * 1.25^2 = 21.875
\]
\item
\[
u_n = u_0 \times q^n = 14 \times 1.25^n
\]
\item On calcule la quantité totale déversée entre 2017 et 2030.
\[
\sum_{n = 0}^{13} u_n = u_0 \times \frac{1-q^{13}}{1-q} = 14 \times \frac{1 - 1.25^{13}}{1 - 1.25} = 962.63
\]
On en déduit la quantité totale de déchets en 2030
\[
300 + 962.63 = 1262.63
\]
\item
\begin{enumerate}
\item ~
\begin{center}
\begin{tabularx}{0.4\linewidth}{|X|}\hline
$N \gets 2017$\\
$U \gets 14$ \\
$S \gets 300 + U$ \\
Tant que $S < 450$ \\
\hspace{1cm} $N \gets N + 1$\\
\hspace{1cm} $U \gets U * 1.25$\\
\hspace{1cm} $S \gets S + u$\\
Fin Tant que\\\hline
\end{tabularx}
\end{center}
\item \textit{Pas de correction automatisé}
\end{enumerate}
\end{enumerate}
\end{solution}
\end{document}
%%% Local Variables:
%%% mode: latex
%%% TeX-master: "master"
%%% End:

View File

@ -0,0 +1,262 @@
\documentclass[a4paper,10pt]{article}
\usepackage{myXsim}
% Title Page
\title{DS8 \hfill DINGER Sölen}
\tribe{TST}
\date{\hfillÀ render pour le Mercredi 7 avril}
\xsimsetup{
solution/print = false
}
\begin{document}
\maketitle
\begin{exercise}[subtitle={Automatismes}]
\textit{Toutes les questions de cette exercice sont indépendantes et peuvent être répondus séparément}
\begin{enumerate}
\item De janvier à septembre, une quantité a augmenté de $30\,\%$. Faire un schéma pour représenter la situation puis calculer le taux d'évolution moyen mensuel.
\item Une quantité augmente de $30\,\%$ par ans. En 2020, elle est de 141\euro. Quelle était sa valeur en 2019? Faire un schéma pour représenter la situation.
\item Déterminer l'équation de la droite \\
\begin{tikzpicture}[xscale=0.8, yscale=0.5]
\tkzInit[xmin=-5,xmax=5,xstep=1,
ymin=-5,ymax=5,ystep=1]
\tkzGrid
\tkzAxeXY
\tkzFct[domain=-5:5,color=red,very thick]%
{2.0*\x -3};
\end{tikzpicture}
\item Résoudre l'équation $5 \times 0.8^x = 11$
\end{enumerate}
\end{exercise}
\begin{solution}
\begin{enumerate}
\item On veut partager cette évolution en 8 évolutions.
\[
\left(1 + \frac{30}{100}\right)^{\frac{1}{8}} = 1.0333
\]
Donc le taux d'évolution moyen est
\[
t_m = 1.0333 - 1 = 0.03330000000000011
\]
\item Coefficient multiplicateur pour revenir en arrière
\[
CM = (1 + \frac{30}{100})^{-1} = 0.7692
\]
On en déduit la quantité en 2019
\[
141 * 0.7692 = 108.4572
\]
\item L'équation de la droite est
\[
y = 2.0 x -3
\]
\item Il faut penser à faire la division à par $5$ avant d'utiliser le log car sinon, on ne peut pas utiliser la formule $\log(a^n) = n\times \log(a)$.
\[x = \frac{\log(2.2)}{\log(0.8)}\]
\end{enumerate}
\end{solution}
\begin{exercise}[subtitle={Restaurant}]
Un \emph{food truck}, ouvert le midi et le soir, propose deux types de formules :
\setlength\parindent{10mm}
\begin{itemize}
\item la formule \emph{Burger} ;
\item la formule \emph{Wok}.
\end{itemize}
\setlength\parindent{0mm}
\medskip
Le gérant a remarqué que 57\,\% de ses ventes ont lieu le midi. Le quart des ventes du midi correspondent à la formule \emph{Burger}, alors que 90\,\% des ventes du soir correspondent à la formule \emph{Wok}.
Le gérant se constitue un fichier en notant, pour chaque vente, la formule choisie et le moment de cette vente (midi ou soir).
On prélève une fiche de façon équiprobable. On définit les quatre évènements suivants:
\begin{enumerate}
\item $M$ : \og la fiche correspond à une vente du midi\fg{} ;
\item $S$ : \og la fiche correspond à une vente du soir\fg {};
\item $W$ : \og la fiche correspond à une formule \emph{Wok} \fg{} ;
\item $B$ : \og la fiche correspond à une formule \emph{Burger} \fg.
\end{enumerate}
\setlength\parindent{0mm}
\medskip
\begin{enumerate}
\item Recopier puis compléter l'arbre pondéré
\begin{center}
\begin{tikzpicture}[sloped]
\node {.}
child {node {$M$}
child {node {$W$}
edge from parent
node[above] {...}
}
child {node {$B$}
edge from parent
node[above] {...}
}
edge from parent
node[above] {...}
}
child[missing] {}
child { node {$S$}
child {node {$W$}
edge from parent
node[above] {...}
}
child {node {$B$}
edge from parent
node[above] {...}
}
edge from parent
node[above] {...}
} ;
\end{tikzpicture}
\end{center}
\item Calculer la probabilité de l'évènement $M \cap W$. Interpréter ce résultat dans le contexte de l'exercice.
\item Montrer que la probabilité que la fiche choisie corresponde à une formule \emph{Burger} est égale à $0.187$.
\item On a prélevé une fiche correspondant à la formule \emph{Burger}. Quelle est la probabilité, arrondie au millième, que la vente ait eu lieu le soir?
\end{enumerate}
\end{exercise}
\begin{solution}
\begin{enumerate}
\item
\begin{center}
\begin{tikzpicture}[sloped]
\node {.}
child {node {$M$}
child {node {$W$}
edge from parent
node[above] {$0.75$}
}
child {node {$B$}
edge from parent
node[above] {$0.25$}
}
edge from parent
node[above] {$0.58$}
}
child[missing] {}
child { node {$S$}
child {node {$W$}
edge from parent
node[above] {$0.9$}
}
child {node {$B$}
edge from parent
node[above] {$0.1$}
}
edge from parent
node[above] {$0.42$}
} ;
\end{tikzpicture}
\end{center}
\item On calcule la probabilité que la vente soit un wok et ait eu lieu à midi
\[ P(M\cap W) = P(M) \times P_M(W) = 0.58 \times 0.75 = 0.435 \]
\item Probabilité que la vente soit un burger.
\[
P(B) = P(M\cap B) + P(S\cap B) = 0.58 \times 0.75 + 0.42 \times 0.9 = 0.187
\]
\item On cherche à calculer la quantité $P_B(S)$. Pour cela on utilise la formule de Bayes
\[
P_B(S) = \frac{P(B\cap S)}{P(B)} = \frac{P_S(B) \times P(S)}{P(B)} = \frac{0.1\times 0.42}{0.187} = 0.22459893048128343 \approx 0.225
\]
\end{enumerate}
\end{solution}
\begin{exercise}[subtitle={Continent plastique}]
\textit{Les quantités évoqués dans cette exercice sont générés au hasard et sont donc complètement farfelus.}
\medskip
Le \og continent de plastique\fg{} est la plus grande des plaques de déchets plastiques évoluant sur les océans. Elle occupe actuellement dans l'océan Pacifique une surface dont l'aire est évaluée à plus de $1,6$ million de km$^2$, entre Hawaï et la Californie.
En 2017, des scientifiques ont estimé qu'il y avait $14$ millions de tonnes de déchets plastiques qui était déversé chaque année dans les océans et que cette quantité augmentait de $19\n\%$ par chaque année.
On modélise l'évolution de la masse de ces déchets plastiques déversée chaque année, si rien n'est fait pour la réduire, par une suite géométrique $\left(u_n\right)$. L'arrondi au centième du terme $u_n$ représente la masse de ces déchets déversée chaque année, exprimée en million de tonnes, pour l'année $(2017 + n)$.
\medskip
\begin{enumerate}
\item Expliquer pourquoi la suite $u_n$ est géométrique?
\item Calculer $u_1$ et $u_2$.
\item Exprimer $u_n$ en fonction de $n$.
\item Au début de l'année 2017, il y avait $300$ millions de tonnes de déchets plastique. Calculer la quantité totale de déchets plastiques en 2030.
\item On souhaite déterminer en quelle année la masse totale de ces déchets plastiques aura pour la première fois augmenté de $50$\,\% par rapport à sa valeur de 2017.
\begin{enumerate}
\item Recopier et compléter l'algorithme ci-dessous pour que la variable $N$ contienne la réponse au problème posé.
\begin{center}
\begin{tabularx}{0.4\linewidth}{|X|}\hline
$N = 2017$\\
$U = 14$ \\
$S = 300 + U$ \\
while $S < 450$: \\
\hspace{1cm} $N = \ldots$\\
\hspace{1cm} $U = \ldots$\\
\hspace{1cm} $S = \ldots$\\
\hline
\end{tabularx}
\end{center}
\item Que contiennent les variables $S$, $U$ et $N$ après exécution de cet algorithme ?
Interpréter les résultats dans le contexte de l'exercice.
\end{enumerate}
\end{enumerate}
\end{exercise}
\begin{solution}
\begin{enumerate}
\item Une augmentation de $19\,\%$ revient à multiplier la quantité par $1.19$. La suite est donc bien géométrique. Son premier terme est $u_0 = 14$ et sa raison est $q = 1.19$
\item
\[
u_1 = u_0 * 1.19 = 16.66
\]
\[
u_2 = u_0 * 1.19^2 = 19.8254
\]
\item
\[
u_n = u_0 \times q^n = 14 \times 1.19^n
\]
\item On calcule la quantité totale déversée entre 2017 et 2030.
\[
\sum_{n = 0}^{13} u_n = u_0 \times \frac{1-q^{13}}{1-q} = 14 \times \frac{1 - 1.19^{13}}{1 - 1.19} = 633.42
\]
On en déduit la quantité totale de déchets en 2030
\[
300 + 633.42 = 933.42
\]
\item
\begin{enumerate}
\item ~
\begin{center}
\begin{tabularx}{0.4\linewidth}{|X|}\hline
$N \gets 2017$\\
$U \gets 14$ \\
$S \gets 300 + U$ \\
Tant que $S < 450$ \\
\hspace{1cm} $N \gets N + 1$\\
\hspace{1cm} $U \gets U * 1.19$\\
\hspace{1cm} $S \gets S + u$\\
Fin Tant que\\\hline
\end{tabularx}
\end{center}
\item \textit{Pas de correction automatisé}
\end{enumerate}
\end{enumerate}
\end{solution}
\end{document}
%%% Local Variables:
%%% mode: latex
%%% TeX-master: "master"
%%% End:

View File

@ -0,0 +1,262 @@
\documentclass[a4paper,10pt]{article}
\usepackage{myXsim}
% Title Page
\title{DS8 \hfill EYRAUD Cynthia}
\tribe{TST}
\date{\hfillÀ render pour le Mercredi 7 avril}
\xsimsetup{
solution/print = false
}
\begin{document}
\maketitle
\begin{exercise}[subtitle={Automatismes}]
\textit{Toutes les questions de cette exercice sont indépendantes et peuvent être répondus séparément}
\begin{enumerate}
\item De janvier à septembre, une quantité a augmenté de $29\,\%$. Faire un schéma pour représenter la situation puis calculer le taux d'évolution moyen mensuel.
\item Une quantité augmente de $29\,\%$ par ans. En 2020, elle est de 130\euro. Quelle était sa valeur en 2019? Faire un schéma pour représenter la situation.
\item Déterminer l'équation de la droite \\
\begin{tikzpicture}[xscale=0.8, yscale=0.5]
\tkzInit[xmin=-5,xmax=5,xstep=1,
ymin=-5,ymax=5,ystep=1]
\tkzGrid
\tkzAxeXY
\tkzFct[domain=-5:5,color=red,very thick]%
{4.0*\x -4};
\end{tikzpicture}
\item Résoudre l'équation $6 \times 0.48^x = 28$
\end{enumerate}
\end{exercise}
\begin{solution}
\begin{enumerate}
\item On veut partager cette évolution en 8 évolutions.
\[
\left(1 + \frac{29}{100}\right)^{\frac{1}{8}} = 1.0323
\]
Donc le taux d'évolution moyen est
\[
t_m = 1.0323 - 1 = 0.032299999999999995
\]
\item Coefficient multiplicateur pour revenir en arrière
\[
CM = (1 + \frac{29}{100})^{-1} = 0.7752
\]
On en déduit la quantité en 2019
\[
130 * 0.7752 = 100.776
\]
\item L'équation de la droite est
\[
y = 4.0 x -4
\]
\item Il faut penser à faire la division à par $6$ avant d'utiliser le log car sinon, on ne peut pas utiliser la formule $\log(a^n) = n\times \log(a)$.
\[x = \frac{\log(4.67)}{\log(0.48)}\]
\end{enumerate}
\end{solution}
\begin{exercise}[subtitle={Restaurant}]
Un \emph{food truck}, ouvert le midi et le soir, propose deux types de formules :
\setlength\parindent{10mm}
\begin{itemize}
\item la formule \emph{Burger} ;
\item la formule \emph{Wok}.
\end{itemize}
\setlength\parindent{0mm}
\medskip
Le gérant a remarqué que 3\,\% de ses ventes ont lieu le midi. Le quart des ventes du midi correspondent à la formule \emph{Burger}, alors que 33\,\% des ventes du soir correspondent à la formule \emph{Wok}.
Le gérant se constitue un fichier en notant, pour chaque vente, la formule choisie et le moment de cette vente (midi ou soir).
On prélève une fiche de façon équiprobable. On définit les quatre évènements suivants:
\begin{enumerate}
\item $M$ : \og la fiche correspond à une vente du midi\fg{} ;
\item $S$ : \og la fiche correspond à une vente du soir\fg {};
\item $W$ : \og la fiche correspond à une formule \emph{Wok} \fg{} ;
\item $B$ : \og la fiche correspond à une formule \emph{Burger} \fg.
\end{enumerate}
\setlength\parindent{0mm}
\medskip
\begin{enumerate}
\item Recopier puis compléter l'arbre pondéré
\begin{center}
\begin{tikzpicture}[sloped]
\node {.}
child {node {$M$}
child {node {$W$}
edge from parent
node[above] {...}
}
child {node {$B$}
edge from parent
node[above] {...}
}
edge from parent
node[above] {...}
}
child[missing] {}
child { node {$S$}
child {node {$W$}
edge from parent
node[above] {...}
}
child {node {$B$}
edge from parent
node[above] {...}
}
edge from parent
node[above] {...}
} ;
\end{tikzpicture}
\end{center}
\item Calculer la probabilité de l'évènement $M \cap W$. Interpréter ce résultat dans le contexte de l'exercice.
\item Montrer que la probabilité que la fiche choisie corresponde à une formule \emph{Burger} est égale à $0.6574$.
\item On a prélevé une fiche correspondant à la formule \emph{Burger}. Quelle est la probabilité, arrondie au millième, que la vente ait eu lieu le soir?
\end{enumerate}
\end{exercise}
\begin{solution}
\begin{enumerate}
\item
\begin{center}
\begin{tikzpicture}[sloped]
\node {.}
child {node {$M$}
child {node {$W$}
edge from parent
node[above] {$0.75$}
}
child {node {$B$}
edge from parent
node[above] {$0.25$}
}
edge from parent
node[above] {$0.03$}
}
child[missing] {}
child { node {$S$}
child {node {$W$}
edge from parent
node[above] {$0.33$}
}
child {node {$B$}
edge from parent
node[above] {$0.67$}
}
edge from parent
node[above] {$0.97$}
} ;
\end{tikzpicture}
\end{center}
\item On calcule la probabilité que la vente soit un wok et ait eu lieu à midi
\[ P(M\cap W) = P(M) \times P_M(W) = 0.03 \times 0.75 = 0.0225 \]
\item Probabilité que la vente soit un burger.
\[
P(B) = P(M\cap B) + P(S\cap B) = 0.03 \times 0.75 + 0.97 \times 0.33 = 0.6574
\]
\item On cherche à calculer la quantité $P_B(S)$. Pour cela on utilise la formule de Bayes
\[
P_B(S) = \frac{P(B\cap S)}{P(B)} = \frac{P_S(B) \times P(S)}{P(B)} = \frac{0.67\times 0.97}{0.6574} = 0.9885914207484029 \approx 0.989
\]
\end{enumerate}
\end{solution}
\begin{exercise}[subtitle={Continent plastique}]
\textit{Les quantités évoqués dans cette exercice sont générés au hasard et sont donc complètement farfelus.}
\medskip
Le \og continent de plastique\fg{} est la plus grande des plaques de déchets plastiques évoluant sur les océans. Elle occupe actuellement dans l'océan Pacifique une surface dont l'aire est évaluée à plus de $1,6$ million de km$^2$, entre Hawaï et la Californie.
En 2017, des scientifiques ont estimé qu'il y avait $18$ millions de tonnes de déchets plastiques qui était déversé chaque année dans les océans et que cette quantité augmentait de $25\n\%$ par chaque année.
On modélise l'évolution de la masse de ces déchets plastiques déversée chaque année, si rien n'est fait pour la réduire, par une suite géométrique $\left(u_n\right)$. L'arrondi au centième du terme $u_n$ représente la masse de ces déchets déversée chaque année, exprimée en million de tonnes, pour l'année $(2017 + n)$.
\medskip
\begin{enumerate}
\item Expliquer pourquoi la suite $u_n$ est géométrique?
\item Calculer $u_1$ et $u_2$.
\item Exprimer $u_n$ en fonction de $n$.
\item Au début de l'année 2017, il y avait $300$ millions de tonnes de déchets plastique. Calculer la quantité totale de déchets plastiques en 2030.
\item On souhaite déterminer en quelle année la masse totale de ces déchets plastiques aura pour la première fois augmenté de $50$\,\% par rapport à sa valeur de 2017.
\begin{enumerate}
\item Recopier et compléter l'algorithme ci-dessous pour que la variable $N$ contienne la réponse au problème posé.
\begin{center}
\begin{tabularx}{0.4\linewidth}{|X|}\hline
$N = 2017$\\
$U = 18$ \\
$S = 300 + U$ \\
while $S < 450$: \\
\hspace{1cm} $N = \ldots$\\
\hspace{1cm} $U = \ldots$\\
\hspace{1cm} $S = \ldots$\\
\hline
\end{tabularx}
\end{center}
\item Que contiennent les variables $S$, $U$ et $N$ après exécution de cet algorithme ?
Interpréter les résultats dans le contexte de l'exercice.
\end{enumerate}
\end{enumerate}
\end{exercise}
\begin{solution}
\begin{enumerate}
\item Une augmentation de $25\,\%$ revient à multiplier la quantité par $1.25$. La suite est donc bien géométrique. Son premier terme est $u_0 = 18$ et sa raison est $q = 1.25$
\item
\[
u_1 = u_0 * 1.25 = 22.5
\]
\[
u_2 = u_0 * 1.25^2 = 28.125
\]
\item
\[
u_n = u_0 \times q^n = 18 \times 1.25^n
\]
\item On calcule la quantité totale déversée entre 2017 et 2030.
\[
\sum_{n = 0}^{13} u_n = u_0 \times \frac{1-q^{13}}{1-q} = 18 \times \frac{1 - 1.25^{13}}{1 - 1.25} = 1237.67
\]
On en déduit la quantité totale de déchets en 2030
\[
300 + 1237.67 = 1537.67
\]
\item
\begin{enumerate}
\item ~
\begin{center}
\begin{tabularx}{0.4\linewidth}{|X|}\hline
$N \gets 2017$\\
$U \gets 18$ \\
$S \gets 300 + U$ \\
Tant que $S < 450$ \\
\hspace{1cm} $N \gets N + 1$\\
\hspace{1cm} $U \gets U * 1.25$\\
\hspace{1cm} $S \gets S + u$\\
Fin Tant que\\\hline
\end{tabularx}
\end{center}
\item \textit{Pas de correction automatisé}
\end{enumerate}
\end{enumerate}
\end{solution}
\end{document}
%%% Local Variables:
%%% mode: latex
%%% TeX-master: "master"
%%% End:

View File

@ -0,0 +1,262 @@
\documentclass[a4paper,10pt]{article}
\usepackage{myXsim}
% Title Page
\title{DS8 \hfill FERREIRA Léo}
\tribe{TST}
\date{\hfillÀ render pour le Mercredi 7 avril}
\xsimsetup{
solution/print = false
}
\begin{document}
\maketitle
\begin{exercise}[subtitle={Automatismes}]
\textit{Toutes les questions de cette exercice sont indépendantes et peuvent être répondus séparément}
\begin{enumerate}
\item De janvier à septembre, une quantité a augmenté de $10\,\%$. Faire un schéma pour représenter la situation puis calculer le taux d'évolution moyen mensuel.
\item Une quantité augmente de $10\,\%$ par ans. En 2020, elle est de 123\euro. Quelle était sa valeur en 2019? Faire un schéma pour représenter la situation.
\item Déterminer l'équation de la droite \\
\begin{tikzpicture}[xscale=0.8, yscale=0.5]
\tkzInit[xmin=-5,xmax=5,xstep=1,
ymin=-5,ymax=5,ystep=1]
\tkzGrid
\tkzAxeXY
\tkzFct[domain=-5:5,color=red,very thick]%
{1.5*\x -3};
\end{tikzpicture}
\item Résoudre l'équation $5 \times 0.79^x = 30$
\end{enumerate}
\end{exercise}
\begin{solution}
\begin{enumerate}
\item On veut partager cette évolution en 8 évolutions.
\[
\left(1 + \frac{10}{100}\right)^{\frac{1}{8}} = 1.012
\]
Donc le taux d'évolution moyen est
\[
t_m = 1.012 - 1 = 0.01200000000000001
\]
\item Coefficient multiplicateur pour revenir en arrière
\[
CM = (1 + \frac{10}{100})^{-1} = 0.9091
\]
On en déduit la quantité en 2019
\[
123 * 0.9091 = 111.8193
\]
\item L'équation de la droite est
\[
y = 1.5 x -3
\]
\item Il faut penser à faire la division à par $5$ avant d'utiliser le log car sinon, on ne peut pas utiliser la formule $\log(a^n) = n\times \log(a)$.
\[x = \frac{\log(6.0)}{\log(0.79)}\]
\end{enumerate}
\end{solution}
\begin{exercise}[subtitle={Restaurant}]
Un \emph{food truck}, ouvert le midi et le soir, propose deux types de formules :
\setlength\parindent{10mm}
\begin{itemize}
\item la formule \emph{Burger} ;
\item la formule \emph{Wok}.
\end{itemize}
\setlength\parindent{0mm}
\medskip
Le gérant a remarqué que 18\,\% de ses ventes ont lieu le midi. Le quart des ventes du midi correspondent à la formule \emph{Burger}, alors que 37\,\% des ventes du soir correspondent à la formule \emph{Wok}.
Le gérant se constitue un fichier en notant, pour chaque vente, la formule choisie et le moment de cette vente (midi ou soir).
On prélève une fiche de façon équiprobable. On définit les quatre évènements suivants:
\begin{enumerate}
\item $M$ : \og la fiche correspond à une vente du midi\fg{} ;
\item $S$ : \og la fiche correspond à une vente du soir\fg {};
\item $W$ : \og la fiche correspond à une formule \emph{Wok} \fg{} ;
\item $B$ : \og la fiche correspond à une formule \emph{Burger} \fg.
\end{enumerate}
\setlength\parindent{0mm}
\medskip
\begin{enumerate}
\item Recopier puis compléter l'arbre pondéré
\begin{center}
\begin{tikzpicture}[sloped]
\node {.}
child {node {$M$}
child {node {$W$}
edge from parent
node[above] {...}
}
child {node {$B$}
edge from parent
node[above] {...}
}
edge from parent
node[above] {...}
}
child[missing] {}
child { node {$S$}
child {node {$W$}
edge from parent
node[above] {...}
}
child {node {$B$}
edge from parent
node[above] {...}
}
edge from parent
node[above] {...}
} ;
\end{tikzpicture}
\end{center}
\item Calculer la probabilité de l'évènement $M \cap W$. Interpréter ce résultat dans le contexte de l'exercice.
\item Montrer que la probabilité que la fiche choisie corresponde à une formule \emph{Burger} est égale à $0.5616$.
\item On a prélevé une fiche correspondant à la formule \emph{Burger}. Quelle est la probabilité, arrondie au millième, que la vente ait eu lieu le soir?
\end{enumerate}
\end{exercise}
\begin{solution}
\begin{enumerate}
\item
\begin{center}
\begin{tikzpicture}[sloped]
\node {.}
child {node {$M$}
child {node {$W$}
edge from parent
node[above] {$0.75$}
}
child {node {$B$}
edge from parent
node[above] {$0.25$}
}
edge from parent
node[above] {$0.18$}
}
child[missing] {}
child { node {$S$}
child {node {$W$}
edge from parent
node[above] {$0.37$}
}
child {node {$B$}
edge from parent
node[above] {$0.63$}
}
edge from parent
node[above] {$0.82$}
} ;
\end{tikzpicture}
\end{center}
\item On calcule la probabilité que la vente soit un wok et ait eu lieu à midi
\[ P(M\cap W) = P(M) \times P_M(W) = 0.18 \times 0.75 = 0.135 \]
\item Probabilité que la vente soit un burger.
\[
P(B) = P(M\cap B) + P(S\cap B) = 0.18 \times 0.75 + 0.82 \times 0.37 = 0.5616
\]
\item On cherche à calculer la quantité $P_B(S)$. Pour cela on utilise la formule de Bayes
\[
P_B(S) = \frac{P(B\cap S)}{P(B)} = \frac{P_S(B) \times P(S)}{P(B)} = \frac{0.63\times 0.82}{0.5616} = 0.9198717948717948 \approx 0.92
\]
\end{enumerate}
\end{solution}
\begin{exercise}[subtitle={Continent plastique}]
\textit{Les quantités évoqués dans cette exercice sont générés au hasard et sont donc complètement farfelus.}
\medskip
Le \og continent de plastique\fg{} est la plus grande des plaques de déchets plastiques évoluant sur les océans. Elle occupe actuellement dans l'océan Pacifique une surface dont l'aire est évaluée à plus de $1,6$ million de km$^2$, entre Hawaï et la Californie.
En 2017, des scientifiques ont estimé qu'il y avait $3$ millions de tonnes de déchets plastiques qui était déversé chaque année dans les océans et que cette quantité augmentait de $19\n\%$ par chaque année.
On modélise l'évolution de la masse de ces déchets plastiques déversée chaque année, si rien n'est fait pour la réduire, par une suite géométrique $\left(u_n\right)$. L'arrondi au centième du terme $u_n$ représente la masse de ces déchets déversée chaque année, exprimée en million de tonnes, pour l'année $(2017 + n)$.
\medskip
\begin{enumerate}
\item Expliquer pourquoi la suite $u_n$ est géométrique?
\item Calculer $u_1$ et $u_2$.
\item Exprimer $u_n$ en fonction de $n$.
\item Au début de l'année 2017, il y avait $300$ millions de tonnes de déchets plastique. Calculer la quantité totale de déchets plastiques en 2030.
\item On souhaite déterminer en quelle année la masse totale de ces déchets plastiques aura pour la première fois augmenté de $50$\,\% par rapport à sa valeur de 2017.
\begin{enumerate}
\item Recopier et compléter l'algorithme ci-dessous pour que la variable $N$ contienne la réponse au problème posé.
\begin{center}
\begin{tabularx}{0.4\linewidth}{|X|}\hline
$N = 2017$\\
$U = 3$ \\
$S = 300 + U$ \\
while $S < 450$: \\
\hspace{1cm} $N = \ldots$\\
\hspace{1cm} $U = \ldots$\\
\hspace{1cm} $S = \ldots$\\
\hline
\end{tabularx}
\end{center}
\item Que contiennent les variables $S$, $U$ et $N$ après exécution de cet algorithme ?
Interpréter les résultats dans le contexte de l'exercice.
\end{enumerate}
\end{enumerate}
\end{exercise}
\begin{solution}
\begin{enumerate}
\item Une augmentation de $19\,\%$ revient à multiplier la quantité par $1.19$. La suite est donc bien géométrique. Son premier terme est $u_0 = 3$ et sa raison est $q = 1.19$
\item
\[
u_1 = u_0 * 1.19 = 3.57
\]
\[
u_2 = u_0 * 1.19^2 = 4.2483
\]
\item
\[
u_n = u_0 \times q^n = 3 \times 1.19^n
\]
\item On calcule la quantité totale déversée entre 2017 et 2030.
\[
\sum_{n = 0}^{13} u_n = u_0 \times \frac{1-q^{13}}{1-q} = 3 \times \frac{1 - 1.19^{13}}{1 - 1.19} = 135.73
\]
On en déduit la quantité totale de déchets en 2030
\[
300 + 135.73 = 435.73
\]
\item
\begin{enumerate}
\item ~
\begin{center}
\begin{tabularx}{0.4\linewidth}{|X|}\hline
$N \gets 2017$\\
$U \gets 3$ \\
$S \gets 300 + U$ \\
Tant que $S < 450$ \\
\hspace{1cm} $N \gets N + 1$\\
\hspace{1cm} $U \gets U * 1.19$\\
\hspace{1cm} $S \gets S + u$\\
Fin Tant que\\\hline
\end{tabularx}
\end{center}
\item \textit{Pas de correction automatisé}
\end{enumerate}
\end{enumerate}
\end{solution}
\end{document}
%%% Local Variables:
%%% mode: latex
%%% TeX-master: "master"
%%% End:

View File

@ -0,0 +1,262 @@
\documentclass[a4paper,10pt]{article}
\usepackage{myXsim}
% Title Page
\title{DS8 \hfill FILALI Zakaria}
\tribe{TST}
\date{\hfillÀ render pour le Mercredi 7 avril}
\xsimsetup{
solution/print = false
}
\begin{document}
\maketitle
\begin{exercise}[subtitle={Automatismes}]
\textit{Toutes les questions de cette exercice sont indépendantes et peuvent être répondus séparément}
\begin{enumerate}
\item De janvier à septembre, une quantité a augmenté de $19\,\%$. Faire un schéma pour représenter la situation puis calculer le taux d'évolution moyen mensuel.
\item Une quantité augmente de $19\,\%$ par ans. En 2020, elle est de 122\euro. Quelle était sa valeur en 2019? Faire un schéma pour représenter la situation.
\item Déterminer l'équation de la droite \\
\begin{tikzpicture}[xscale=0.8, yscale=0.5]
\tkzInit[xmin=-5,xmax=5,xstep=1,
ymin=-5,ymax=5,ystep=1]
\tkzGrid
\tkzAxeXY
\tkzFct[domain=-5:5,color=red,very thick]%
{1.0*\x -2};
\end{tikzpicture}
\item Résoudre l'équation $8 \times 0.25^x = 25$
\end{enumerate}
\end{exercise}
\begin{solution}
\begin{enumerate}
\item On veut partager cette évolution en 8 évolutions.
\[
\left(1 + \frac{19}{100}\right)^{\frac{1}{8}} = 1.022
\]
Donc le taux d'évolution moyen est
\[
t_m = 1.022 - 1 = 0.02200000000000002
\]
\item Coefficient multiplicateur pour revenir en arrière
\[
CM = (1 + \frac{19}{100})^{-1} = 0.8403
\]
On en déduit la quantité en 2019
\[
122 * 0.8403 = 102.51660000000001
\]
\item L'équation de la droite est
\[
y = 1.0 x -2
\]
\item Il faut penser à faire la division à par $8$ avant d'utiliser le log car sinon, on ne peut pas utiliser la formule $\log(a^n) = n\times \log(a)$.
\[x = \frac{\log(3.12)}{\log(0.25)}\]
\end{enumerate}
\end{solution}
\begin{exercise}[subtitle={Restaurant}]
Un \emph{food truck}, ouvert le midi et le soir, propose deux types de formules :
\setlength\parindent{10mm}
\begin{itemize}
\item la formule \emph{Burger} ;
\item la formule \emph{Wok}.
\end{itemize}
\setlength\parindent{0mm}
\medskip
Le gérant a remarqué que 47\,\% de ses ventes ont lieu le midi. Le quart des ventes du midi correspondent à la formule \emph{Burger}, alors que 69\,\% des ventes du soir correspondent à la formule \emph{Wok}.
Le gérant se constitue un fichier en notant, pour chaque vente, la formule choisie et le moment de cette vente (midi ou soir).
On prélève une fiche de façon équiprobable. On définit les quatre évènements suivants:
\begin{enumerate}
\item $M$ : \og la fiche correspond à une vente du midi\fg{} ;
\item $S$ : \og la fiche correspond à une vente du soir\fg {};
\item $W$ : \og la fiche correspond à une formule \emph{Wok} \fg{} ;
\item $B$ : \og la fiche correspond à une formule \emph{Burger} \fg.
\end{enumerate}
\setlength\parindent{0mm}
\medskip
\begin{enumerate}
\item Recopier puis compléter l'arbre pondéré
\begin{center}
\begin{tikzpicture}[sloped]
\node {.}
child {node {$M$}
child {node {$W$}
edge from parent
node[above] {...}
}
child {node {$B$}
edge from parent
node[above] {...}
}
edge from parent
node[above] {...}
}
child[missing] {}
child { node {$S$}
child {node {$W$}
edge from parent
node[above] {...}
}
child {node {$B$}
edge from parent
node[above] {...}
}
edge from parent
node[above] {...}
} ;
\end{tikzpicture}
\end{center}
\item Calculer la probabilité de l'évènement $M \cap W$. Interpréter ce résultat dans le contexte de l'exercice.
\item Montrer que la probabilité que la fiche choisie corresponde à une formule \emph{Burger} est égale à $0.2818$.
\item On a prélevé une fiche correspondant à la formule \emph{Burger}. Quelle est la probabilité, arrondie au millième, que la vente ait eu lieu le soir?
\end{enumerate}
\end{exercise}
\begin{solution}
\begin{enumerate}
\item
\begin{center}
\begin{tikzpicture}[sloped]
\node {.}
child {node {$M$}
child {node {$W$}
edge from parent
node[above] {$0.75$}
}
child {node {$B$}
edge from parent
node[above] {$0.25$}
}
edge from parent
node[above] {$0.47$}
}
child[missing] {}
child { node {$S$}
child {node {$W$}
edge from parent
node[above] {$0.69$}
}
child {node {$B$}
edge from parent
node[above] {$0.31$}
}
edge from parent
node[above] {$0.53$}
} ;
\end{tikzpicture}
\end{center}
\item On calcule la probabilité que la vente soit un wok et ait eu lieu à midi
\[ P(M\cap W) = P(M) \times P_M(W) = 0.47 \times 0.75 = 0.3525 \]
\item Probabilité que la vente soit un burger.
\[
P(B) = P(M\cap B) + P(S\cap B) = 0.47 \times 0.75 + 0.53 \times 0.69 = 0.2818
\]
\item On cherche à calculer la quantité $P_B(S)$. Pour cela on utilise la formule de Bayes
\[
P_B(S) = \frac{P(B\cap S)}{P(B)} = \frac{P_S(B) \times P(S)}{P(B)} = \frac{0.31\times 0.53}{0.2818} = 0.5830376153300213 \approx 0.583
\]
\end{enumerate}
\end{solution}
\begin{exercise}[subtitle={Continent plastique}]
\textit{Les quantités évoqués dans cette exercice sont générés au hasard et sont donc complètement farfelus.}
\medskip
Le \og continent de plastique\fg{} est la plus grande des plaques de déchets plastiques évoluant sur les océans. Elle occupe actuellement dans l'océan Pacifique une surface dont l'aire est évaluée à plus de $1,6$ million de km$^2$, entre Hawaï et la Californie.
En 2017, des scientifiques ont estimé qu'il y avait $11$ millions de tonnes de déchets plastiques qui était déversé chaque année dans les océans et que cette quantité augmentait de $16\n\%$ par chaque année.
On modélise l'évolution de la masse de ces déchets plastiques déversée chaque année, si rien n'est fait pour la réduire, par une suite géométrique $\left(u_n\right)$. L'arrondi au centième du terme $u_n$ représente la masse de ces déchets déversée chaque année, exprimée en million de tonnes, pour l'année $(2017 + n)$.
\medskip
\begin{enumerate}
\item Expliquer pourquoi la suite $u_n$ est géométrique?
\item Calculer $u_1$ et $u_2$.
\item Exprimer $u_n$ en fonction de $n$.
\item Au début de l'année 2017, il y avait $300$ millions de tonnes de déchets plastique. Calculer la quantité totale de déchets plastiques en 2030.
\item On souhaite déterminer en quelle année la masse totale de ces déchets plastiques aura pour la première fois augmenté de $50$\,\% par rapport à sa valeur de 2017.
\begin{enumerate}
\item Recopier et compléter l'algorithme ci-dessous pour que la variable $N$ contienne la réponse au problème posé.
\begin{center}
\begin{tabularx}{0.4\linewidth}{|X|}\hline
$N = 2017$\\
$U = 11$ \\
$S = 300 + U$ \\
while $S < 450$: \\
\hspace{1cm} $N = \ldots$\\
\hspace{1cm} $U = \ldots$\\
\hspace{1cm} $S = \ldots$\\
\hline
\end{tabularx}
\end{center}
\item Que contiennent les variables $S$, $U$ et $N$ après exécution de cet algorithme ?
Interpréter les résultats dans le contexte de l'exercice.
\end{enumerate}
\end{enumerate}
\end{exercise}
\begin{solution}
\begin{enumerate}
\item Une augmentation de $16\,\%$ revient à multiplier la quantité par $1.16$. La suite est donc bien géométrique. Son premier terme est $u_0 = 11$ et sa raison est $q = 1.16$
\item
\[
u_1 = u_0 * 1.16 = 12.76
\]
\[
u_2 = u_0 * 1.16^2 = 14.8016
\]
\item
\[
u_n = u_0 \times q^n = 11 \times 1.16^n
\]
\item On calcule la quantité totale déversée entre 2017 et 2030.
\[
\sum_{n = 0}^{13} u_n = u_0 \times \frac{1-q^{13}}{1-q} = 11 \times \frac{1 - 1.16^{13}}{1 - 1.16} = 404.65
\]
On en déduit la quantité totale de déchets en 2030
\[
300 + 404.65 = 704.65
\]
\item
\begin{enumerate}
\item ~
\begin{center}
\begin{tabularx}{0.4\linewidth}{|X|}\hline
$N \gets 2017$\\
$U \gets 11$ \\
$S \gets 300 + U$ \\
Tant que $S < 450$ \\
\hspace{1cm} $N \gets N + 1$\\
\hspace{1cm} $U \gets U * 1.16$\\
\hspace{1cm} $S \gets S + u$\\
Fin Tant que\\\hline
\end{tabularx}
\end{center}
\item \textit{Pas de correction automatisé}
\end{enumerate}
\end{enumerate}
\end{solution}
\end{document}
%%% Local Variables:
%%% mode: latex
%%% TeX-master: "master"
%%% End:

View File

@ -0,0 +1,262 @@
\documentclass[a4paper,10pt]{article}
\usepackage{myXsim}
% Title Page
\title{DS8 \hfill FOIGNY Romain}
\tribe{TST}
\date{\hfillÀ render pour le Mercredi 7 avril}
\xsimsetup{
solution/print = false
}
\begin{document}
\maketitle
\begin{exercise}[subtitle={Automatismes}]
\textit{Toutes les questions de cette exercice sont indépendantes et peuvent être répondus séparément}
\begin{enumerate}
\item De janvier à septembre, une quantité a augmenté de $22\,\%$. Faire un schéma pour représenter la situation puis calculer le taux d'évolution moyen mensuel.
\item Une quantité augmente de $22\,\%$ par ans. En 2020, elle est de 125\euro. Quelle était sa valeur en 2019? Faire un schéma pour représenter la situation.
\item Déterminer l'équation de la droite \\
\begin{tikzpicture}[xscale=0.8, yscale=0.5]
\tkzInit[xmin=-5,xmax=5,xstep=1,
ymin=-5,ymax=5,ystep=1]
\tkzGrid
\tkzAxeXY
\tkzFct[domain=-5:5,color=red,very thick]%
{2.6666666666666665*\x -4};
\end{tikzpicture}
\item Résoudre l'équation $9 \times 0.35^x = 44$
\end{enumerate}
\end{exercise}
\begin{solution}
\begin{enumerate}
\item On veut partager cette évolution en 8 évolutions.
\[
\left(1 + \frac{22}{100}\right)^{\frac{1}{8}} = 1.0252
\]
Donc le taux d'évolution moyen est
\[
t_m = 1.0252 - 1 = 0.02519999999999989
\]
\item Coefficient multiplicateur pour revenir en arrière
\[
CM = (1 + \frac{22}{100})^{-1} = 0.8197
\]
On en déduit la quantité en 2019
\[
125 * 0.8197 = 102.46249999999999
\]
\item L'équation de la droite est
\[
y = 2.6666666666666665 x -4
\]
\item Il faut penser à faire la division à par $9$ avant d'utiliser le log car sinon, on ne peut pas utiliser la formule $\log(a^n) = n\times \log(a)$.
\[x = \frac{\log(4.89)}{\log(0.35)}\]
\end{enumerate}
\end{solution}
\begin{exercise}[subtitle={Restaurant}]
Un \emph{food truck}, ouvert le midi et le soir, propose deux types de formules :
\setlength\parindent{10mm}
\begin{itemize}
\item la formule \emph{Burger} ;
\item la formule \emph{Wok}.
\end{itemize}
\setlength\parindent{0mm}
\medskip
Le gérant a remarqué que 14\,\% de ses ventes ont lieu le midi. Le quart des ventes du midi correspondent à la formule \emph{Burger}, alors que 17\,\% des ventes du soir correspondent à la formule \emph{Wok}.
Le gérant se constitue un fichier en notant, pour chaque vente, la formule choisie et le moment de cette vente (midi ou soir).
On prélève une fiche de façon équiprobable. On définit les quatre évènements suivants:
\begin{enumerate}
\item $M$ : \og la fiche correspond à une vente du midi\fg{} ;
\item $S$ : \og la fiche correspond à une vente du soir\fg {};
\item $W$ : \og la fiche correspond à une formule \emph{Wok} \fg{} ;
\item $B$ : \og la fiche correspond à une formule \emph{Burger} \fg.
\end{enumerate}
\setlength\parindent{0mm}
\medskip
\begin{enumerate}
\item Recopier puis compléter l'arbre pondéré
\begin{center}
\begin{tikzpicture}[sloped]
\node {.}
child {node {$M$}
child {node {$W$}
edge from parent
node[above] {...}
}
child {node {$B$}
edge from parent
node[above] {...}
}
edge from parent
node[above] {...}
}
child[missing] {}
child { node {$S$}
child {node {$W$}
edge from parent
node[above] {...}
}
child {node {$B$}
edge from parent
node[above] {...}
}
edge from parent
node[above] {...}
} ;
\end{tikzpicture}
\end{center}
\item Calculer la probabilité de l'évènement $M \cap W$. Interpréter ce résultat dans le contexte de l'exercice.
\item Montrer que la probabilité que la fiche choisie corresponde à une formule \emph{Burger} est égale à $0.7488$.
\item On a prélevé une fiche correspondant à la formule \emph{Burger}. Quelle est la probabilité, arrondie au millième, que la vente ait eu lieu le soir?
\end{enumerate}
\end{exercise}
\begin{solution}
\begin{enumerate}
\item
\begin{center}
\begin{tikzpicture}[sloped]
\node {.}
child {node {$M$}
child {node {$W$}
edge from parent
node[above] {$0.75$}
}
child {node {$B$}
edge from parent
node[above] {$0.25$}
}
edge from parent
node[above] {$0.14$}
}
child[missing] {}
child { node {$S$}
child {node {$W$}
edge from parent
node[above] {$0.17$}
}
child {node {$B$}
edge from parent
node[above] {$0.83$}
}
edge from parent
node[above] {$0.86$}
} ;
\end{tikzpicture}
\end{center}
\item On calcule la probabilité que la vente soit un wok et ait eu lieu à midi
\[ P(M\cap W) = P(M) \times P_M(W) = 0.14 \times 0.75 = 0.105 \]
\item Probabilité que la vente soit un burger.
\[
P(B) = P(M\cap B) + P(S\cap B) = 0.14 \times 0.75 + 0.86 \times 0.17 = 0.7488
\]
\item On cherche à calculer la quantité $P_B(S)$. Pour cela on utilise la formule de Bayes
\[
P_B(S) = \frac{P(B\cap S)}{P(B)} = \frac{P_S(B) \times P(S)}{P(B)} = \frac{0.83\times 0.86}{0.7488} = 0.953258547008547 \approx 0.953
\]
\end{enumerate}
\end{solution}
\begin{exercise}[subtitle={Continent plastique}]
\textit{Les quantités évoqués dans cette exercice sont générés au hasard et sont donc complètement farfelus.}
\medskip
Le \og continent de plastique\fg{} est la plus grande des plaques de déchets plastiques évoluant sur les océans. Elle occupe actuellement dans l'océan Pacifique une surface dont l'aire est évaluée à plus de $1,6$ million de km$^2$, entre Hawaï et la Californie.
En 2017, des scientifiques ont estimé qu'il y avait $11$ millions de tonnes de déchets plastiques qui était déversé chaque année dans les océans et que cette quantité augmentait de $12\n\%$ par chaque année.
On modélise l'évolution de la masse de ces déchets plastiques déversée chaque année, si rien n'est fait pour la réduire, par une suite géométrique $\left(u_n\right)$. L'arrondi au centième du terme $u_n$ représente la masse de ces déchets déversée chaque année, exprimée en million de tonnes, pour l'année $(2017 + n)$.
\medskip
\begin{enumerate}
\item Expliquer pourquoi la suite $u_n$ est géométrique?
\item Calculer $u_1$ et $u_2$.
\item Exprimer $u_n$ en fonction de $n$.
\item Au début de l'année 2017, il y avait $300$ millions de tonnes de déchets plastique. Calculer la quantité totale de déchets plastiques en 2030.
\item On souhaite déterminer en quelle année la masse totale de ces déchets plastiques aura pour la première fois augmenté de $50$\,\% par rapport à sa valeur de 2017.
\begin{enumerate}
\item Recopier et compléter l'algorithme ci-dessous pour que la variable $N$ contienne la réponse au problème posé.
\begin{center}
\begin{tabularx}{0.4\linewidth}{|X|}\hline
$N = 2017$\\
$U = 11$ \\
$S = 300 + U$ \\
while $S < 450$: \\
\hspace{1cm} $N = \ldots$\\
\hspace{1cm} $U = \ldots$\\
\hspace{1cm} $S = \ldots$\\
\hline
\end{tabularx}
\end{center}
\item Que contiennent les variables $S$, $U$ et $N$ après exécution de cet algorithme ?
Interpréter les résultats dans le contexte de l'exercice.
\end{enumerate}
\end{enumerate}
\end{exercise}
\begin{solution}
\begin{enumerate}
\item Une augmentation de $12\,\%$ revient à multiplier la quantité par $1.12$. La suite est donc bien géométrique. Son premier terme est $u_0 = 11$ et sa raison est $q = 1.12$
\item
\[
u_1 = u_0 * 1.12 = 12.32
\]
\[
u_2 = u_0 * 1.12^2 = 13.7984
\]
\item
\[
u_n = u_0 \times q^n = 11 \times 1.12^n
\]
\item On calcule la quantité totale déversée entre 2017 et 2030.
\[
\sum_{n = 0}^{13} u_n = u_0 \times \frac{1-q^{13}}{1-q} = 11 \times \frac{1 - 1.12^{13}}{1 - 1.12} = 308.32
\]
On en déduit la quantité totale de déchets en 2030
\[
300 + 308.32 = 608.3199999999999
\]
\item
\begin{enumerate}
\item ~
\begin{center}
\begin{tabularx}{0.4\linewidth}{|X|}\hline
$N \gets 2017$\\
$U \gets 11$ \\
$S \gets 300 + U$ \\
Tant que $S < 450$ \\
\hspace{1cm} $N \gets N + 1$\\
\hspace{1cm} $U \gets U * 1.12$\\
\hspace{1cm} $S \gets S + u$\\
Fin Tant que\\\hline
\end{tabularx}
\end{center}
\item \textit{Pas de correction automatisé}
\end{enumerate}
\end{enumerate}
\end{solution}
\end{document}
%%% Local Variables:
%%% mode: latex
%%% TeX-master: "master"
%%% End:

View File

@ -0,0 +1,262 @@
\documentclass[a4paper,10pt]{article}
\usepackage{myXsim}
% Title Page
\title{DS8 \hfill HIPOLITO DA SILVA Andréa}
\tribe{TST}
\date{\hfillÀ render pour le Mercredi 7 avril}
\xsimsetup{
solution/print = false
}
\begin{document}
\maketitle
\begin{exercise}[subtitle={Automatismes}]
\textit{Toutes les questions de cette exercice sont indépendantes et peuvent être répondus séparément}
\begin{enumerate}
\item De janvier à septembre, une quantité a augmenté de $23\,\%$. Faire un schéma pour représenter la situation puis calculer le taux d'évolution moyen mensuel.
\item Une quantité augmente de $23\,\%$ par ans. En 2020, elle est de 149\euro. Quelle était sa valeur en 2019? Faire un schéma pour représenter la situation.
\item Déterminer l'équation de la droite \\
\begin{tikzpicture}[xscale=0.8, yscale=0.5]
\tkzInit[xmin=-5,xmax=5,xstep=1,
ymin=-5,ymax=5,ystep=1]
\tkzGrid
\tkzAxeXY
\tkzFct[domain=-5:5,color=red,very thick]%
{0.6666666666666666*\x -1};
\end{tikzpicture}
\item Résoudre l'équation $7 \times 0.38^x = 21$
\end{enumerate}
\end{exercise}
\begin{solution}
\begin{enumerate}
\item On veut partager cette évolution en 8 évolutions.
\[
\left(1 + \frac{23}{100}\right)^{\frac{1}{8}} = 1.0262
\]
Donc le taux d'évolution moyen est
\[
t_m = 1.0262 - 1 = 0.0262
\]
\item Coefficient multiplicateur pour revenir en arrière
\[
CM = (1 + \frac{23}{100})^{-1} = 0.813
\]
On en déduit la quantité en 2019
\[
149 * 0.813 = 121.13699999999999
\]
\item L'équation de la droite est
\[
y = 0.6666666666666666 x -1
\]
\item Il faut penser à faire la division à par $7$ avant d'utiliser le log car sinon, on ne peut pas utiliser la formule $\log(a^n) = n\times \log(a)$.
\[x = \frac{\log(3.0)}{\log(0.38)}\]
\end{enumerate}
\end{solution}
\begin{exercise}[subtitle={Restaurant}]
Un \emph{food truck}, ouvert le midi et le soir, propose deux types de formules :
\setlength\parindent{10mm}
\begin{itemize}
\item la formule \emph{Burger} ;
\item la formule \emph{Wok}.
\end{itemize}
\setlength\parindent{0mm}
\medskip
Le gérant a remarqué que 35\,\% de ses ventes ont lieu le midi. Le quart des ventes du midi correspondent à la formule \emph{Burger}, alors que 13\,\% des ventes du soir correspondent à la formule \emph{Wok}.
Le gérant se constitue un fichier en notant, pour chaque vente, la formule choisie et le moment de cette vente (midi ou soir).
On prélève une fiche de façon équiprobable. On définit les quatre évènements suivants:
\begin{enumerate}
\item $M$ : \og la fiche correspond à une vente du midi\fg{} ;
\item $S$ : \og la fiche correspond à une vente du soir\fg {};
\item $W$ : \og la fiche correspond à une formule \emph{Wok} \fg{} ;
\item $B$ : \og la fiche correspond à une formule \emph{Burger} \fg.
\end{enumerate}
\setlength\parindent{0mm}
\medskip
\begin{enumerate}
\item Recopier puis compléter l'arbre pondéré
\begin{center}
\begin{tikzpicture}[sloped]
\node {.}
child {node {$M$}
child {node {$W$}
edge from parent
node[above] {...}
}
child {node {$B$}
edge from parent
node[above] {...}
}
edge from parent
node[above] {...}
}
child[missing] {}
child { node {$S$}
child {node {$W$}
edge from parent
node[above] {...}
}
child {node {$B$}
edge from parent
node[above] {...}
}
edge from parent
node[above] {...}
} ;
\end{tikzpicture}
\end{center}
\item Calculer la probabilité de l'évènement $M \cap W$. Interpréter ce résultat dans le contexte de l'exercice.
\item Montrer que la probabilité que la fiche choisie corresponde à une formule \emph{Burger} est égale à $0.653$.
\item On a prélevé une fiche correspondant à la formule \emph{Burger}. Quelle est la probabilité, arrondie au millième, que la vente ait eu lieu le soir?
\end{enumerate}
\end{exercise}
\begin{solution}
\begin{enumerate}
\item
\begin{center}
\begin{tikzpicture}[sloped]
\node {.}
child {node {$M$}
child {node {$W$}
edge from parent
node[above] {$0.75$}
}
child {node {$B$}
edge from parent
node[above] {$0.25$}
}
edge from parent
node[above] {$0.35$}
}
child[missing] {}
child { node {$S$}
child {node {$W$}
edge from parent
node[above] {$0.13$}
}
child {node {$B$}
edge from parent
node[above] {$0.87$}
}
edge from parent
node[above] {$0.65$}
} ;
\end{tikzpicture}
\end{center}
\item On calcule la probabilité que la vente soit un wok et ait eu lieu à midi
\[ P(M\cap W) = P(M) \times P_M(W) = 0.35 \times 0.75 = 0.2625 \]
\item Probabilité que la vente soit un burger.
\[
P(B) = P(M\cap B) + P(S\cap B) = 0.35 \times 0.75 + 0.65 \times 0.13 = 0.653
\]
\item On cherche à calculer la quantité $P_B(S)$. Pour cela on utilise la formule de Bayes
\[
P_B(S) = \frac{P(B\cap S)}{P(B)} = \frac{P_S(B) \times P(S)}{P(B)} = \frac{0.87\times 0.65}{0.653} = 0.8660030627871362 \approx 0.866
\]
\end{enumerate}
\end{solution}
\begin{exercise}[subtitle={Continent plastique}]
\textit{Les quantités évoqués dans cette exercice sont générés au hasard et sont donc complètement farfelus.}
\medskip
Le \og continent de plastique\fg{} est la plus grande des plaques de déchets plastiques évoluant sur les océans. Elle occupe actuellement dans l'océan Pacifique une surface dont l'aire est évaluée à plus de $1,6$ million de km$^2$, entre Hawaï et la Californie.
En 2017, des scientifiques ont estimé qu'il y avait $4$ millions de tonnes de déchets plastiques qui était déversé chaque année dans les océans et que cette quantité augmentait de $23\n\%$ par chaque année.
On modélise l'évolution de la masse de ces déchets plastiques déversée chaque année, si rien n'est fait pour la réduire, par une suite géométrique $\left(u_n\right)$. L'arrondi au centième du terme $u_n$ représente la masse de ces déchets déversée chaque année, exprimée en million de tonnes, pour l'année $(2017 + n)$.
\medskip
\begin{enumerate}
\item Expliquer pourquoi la suite $u_n$ est géométrique?
\item Calculer $u_1$ et $u_2$.
\item Exprimer $u_n$ en fonction de $n$.
\item Au début de l'année 2017, il y avait $300$ millions de tonnes de déchets plastique. Calculer la quantité totale de déchets plastiques en 2030.
\item On souhaite déterminer en quelle année la masse totale de ces déchets plastiques aura pour la première fois augmenté de $50$\,\% par rapport à sa valeur de 2017.
\begin{enumerate}
\item Recopier et compléter l'algorithme ci-dessous pour que la variable $N$ contienne la réponse au problème posé.
\begin{center}
\begin{tabularx}{0.4\linewidth}{|X|}\hline
$N = 2017$\\
$U = 4$ \\
$S = 300 + U$ \\
while $S < 450$: \\
\hspace{1cm} $N = \ldots$\\
\hspace{1cm} $U = \ldots$\\
\hspace{1cm} $S = \ldots$\\
\hline
\end{tabularx}
\end{center}
\item Que contiennent les variables $S$, $U$ et $N$ après exécution de cet algorithme ?
Interpréter les résultats dans le contexte de l'exercice.
\end{enumerate}
\end{enumerate}
\end{exercise}
\begin{solution}
\begin{enumerate}
\item Une augmentation de $23\,\%$ revient à multiplier la quantité par $1.23$. La suite est donc bien géométrique. Son premier terme est $u_0 = 4$ et sa raison est $q = 1.23$
\item
\[
u_1 = u_0 * 1.23 = 4.92
\]
\[
u_2 = u_0 * 1.23^2 = 6.0516
\]
\item
\[
u_n = u_0 \times q^n = 4 \times 1.23^n
\]
\item On calcule la quantité totale déversée entre 2017 et 2030.
\[
\sum_{n = 0}^{13} u_n = u_0 \times \frac{1-q^{13}}{1-q} = 4 \times \frac{1 - 1.23^{13}}{1 - 1.23} = 239.12
\]
On en déduit la quantité totale de déchets en 2030
\[
300 + 239.12 = 539.12
\]
\item
\begin{enumerate}
\item ~
\begin{center}
\begin{tabularx}{0.4\linewidth}{|X|}\hline
$N \gets 2017$\\
$U \gets 4$ \\
$S \gets 300 + U$ \\
Tant que $S < 450$ \\
\hspace{1cm} $N \gets N + 1$\\
\hspace{1cm} $U \gets U * 1.23$\\
\hspace{1cm} $S \gets S + u$\\
Fin Tant que\\\hline
\end{tabularx}
\end{center}
\item \textit{Pas de correction automatisé}
\end{enumerate}
\end{enumerate}
\end{solution}
\end{document}
%%% Local Variables:
%%% mode: latex
%%% TeX-master: "master"
%%% End:

View File

@ -0,0 +1,262 @@
\documentclass[a4paper,10pt]{article}
\usepackage{myXsim}
% Title Page
\title{DS8 \hfill HUMBERT Rayan}
\tribe{TST}
\date{\hfillÀ render pour le Mercredi 7 avril}
\xsimsetup{
solution/print = false
}
\begin{document}
\maketitle
\begin{exercise}[subtitle={Automatismes}]
\textit{Toutes les questions de cette exercice sont indépendantes et peuvent être répondus séparément}
\begin{enumerate}
\item De janvier à septembre, une quantité a augmenté de $17\,\%$. Faire un schéma pour représenter la situation puis calculer le taux d'évolution moyen mensuel.
\item Une quantité augmente de $17\,\%$ par ans. En 2020, elle est de 149\euro. Quelle était sa valeur en 2019? Faire un schéma pour représenter la situation.
\item Déterminer l'équation de la droite \\
\begin{tikzpicture}[xscale=0.8, yscale=0.5]
\tkzInit[xmin=-5,xmax=5,xstep=1,
ymin=-5,ymax=5,ystep=1]
\tkzGrid
\tkzAxeXY
\tkzFct[domain=-5:5,color=red,very thick]%
{0.5*\x -1};
\end{tikzpicture}
\item Résoudre l'équation $5 \times 0.02^x = 46$
\end{enumerate}
\end{exercise}
\begin{solution}
\begin{enumerate}
\item On veut partager cette évolution en 8 évolutions.
\[
\left(1 + \frac{17}{100}\right)^{\frac{1}{8}} = 1.0198
\]
Donc le taux d'évolution moyen est
\[
t_m = 1.0198 - 1 = 0.01980000000000004
\]
\item Coefficient multiplicateur pour revenir en arrière
\[
CM = (1 + \frac{17}{100})^{-1} = 0.8547
\]
On en déduit la quantité en 2019
\[
149 * 0.8547 = 127.3503
\]
\item L'équation de la droite est
\[
y = 0.5 x -1
\]
\item Il faut penser à faire la division à par $5$ avant d'utiliser le log car sinon, on ne peut pas utiliser la formule $\log(a^n) = n\times \log(a)$.
\[x = \frac{\log(9.2)}{\log(0.02)}\]
\end{enumerate}
\end{solution}
\begin{exercise}[subtitle={Restaurant}]
Un \emph{food truck}, ouvert le midi et le soir, propose deux types de formules :
\setlength\parindent{10mm}
\begin{itemize}
\item la formule \emph{Burger} ;
\item la formule \emph{Wok}.
\end{itemize}
\setlength\parindent{0mm}
\medskip
Le gérant a remarqué que 65\,\% de ses ventes ont lieu le midi. Le quart des ventes du midi correspondent à la formule \emph{Burger}, alors que 93\,\% des ventes du soir correspondent à la formule \emph{Wok}.
Le gérant se constitue un fichier en notant, pour chaque vente, la formule choisie et le moment de cette vente (midi ou soir).
On prélève une fiche de façon équiprobable. On définit les quatre évènements suivants:
\begin{enumerate}
\item $M$ : \og la fiche correspond à une vente du midi\fg{} ;
\item $S$ : \og la fiche correspond à une vente du soir\fg {};
\item $W$ : \og la fiche correspond à une formule \emph{Wok} \fg{} ;
\item $B$ : \og la fiche correspond à une formule \emph{Burger} \fg.
\end{enumerate}
\setlength\parindent{0mm}
\medskip
\begin{enumerate}
\item Recopier puis compléter l'arbre pondéré
\begin{center}
\begin{tikzpicture}[sloped]
\node {.}
child {node {$M$}
child {node {$W$}
edge from parent
node[above] {...}
}
child {node {$B$}
edge from parent
node[above] {...}
}
edge from parent
node[above] {...}
}
child[missing] {}
child { node {$S$}
child {node {$W$}
edge from parent
node[above] {...}
}
child {node {$B$}
edge from parent
node[above] {...}
}
edge from parent
node[above] {...}
} ;
\end{tikzpicture}
\end{center}
\item Calculer la probabilité de l'évènement $M \cap W$. Interpréter ce résultat dans le contexte de l'exercice.
\item Montrer que la probabilité que la fiche choisie corresponde à une formule \emph{Burger} est égale à $0.187$.
\item On a prélevé une fiche correspondant à la formule \emph{Burger}. Quelle est la probabilité, arrondie au millième, que la vente ait eu lieu le soir?
\end{enumerate}
\end{exercise}
\begin{solution}
\begin{enumerate}
\item
\begin{center}
\begin{tikzpicture}[sloped]
\node {.}
child {node {$M$}
child {node {$W$}
edge from parent
node[above] {$0.75$}
}
child {node {$B$}
edge from parent
node[above] {$0.25$}
}
edge from parent
node[above] {$0.65$}
}
child[missing] {}
child { node {$S$}
child {node {$W$}
edge from parent
node[above] {$0.93$}
}
child {node {$B$}
edge from parent
node[above] {$0.07$}
}
edge from parent
node[above] {$0.35$}
} ;
\end{tikzpicture}
\end{center}
\item On calcule la probabilité que la vente soit un wok et ait eu lieu à midi
\[ P(M\cap W) = P(M) \times P_M(W) = 0.65 \times 0.75 = 0.4875 \]
\item Probabilité que la vente soit un burger.
\[
P(B) = P(M\cap B) + P(S\cap B) = 0.65 \times 0.75 + 0.35 \times 0.93 = 0.187
\]
\item On cherche à calculer la quantité $P_B(S)$. Pour cela on utilise la formule de Bayes
\[
P_B(S) = \frac{P(B\cap S)}{P(B)} = \frac{P_S(B) \times P(S)}{P(B)} = \frac{0.07\times 0.35}{0.187} = 0.13101604278074866 \approx 0.131
\]
\end{enumerate}
\end{solution}
\begin{exercise}[subtitle={Continent plastique}]
\textit{Les quantités évoqués dans cette exercice sont générés au hasard et sont donc complètement farfelus.}
\medskip
Le \og continent de plastique\fg{} est la plus grande des plaques de déchets plastiques évoluant sur les océans. Elle occupe actuellement dans l'océan Pacifique une surface dont l'aire est évaluée à plus de $1,6$ million de km$^2$, entre Hawaï et la Californie.
En 2017, des scientifiques ont estimé qu'il y avait $15$ millions de tonnes de déchets plastiques qui était déversé chaque année dans les océans et que cette quantité augmentait de $12\n\%$ par chaque année.
On modélise l'évolution de la masse de ces déchets plastiques déversée chaque année, si rien n'est fait pour la réduire, par une suite géométrique $\left(u_n\right)$. L'arrondi au centième du terme $u_n$ représente la masse de ces déchets déversée chaque année, exprimée en million de tonnes, pour l'année $(2017 + n)$.
\medskip
\begin{enumerate}
\item Expliquer pourquoi la suite $u_n$ est géométrique?
\item Calculer $u_1$ et $u_2$.
\item Exprimer $u_n$ en fonction de $n$.
\item Au début de l'année 2017, il y avait $300$ millions de tonnes de déchets plastique. Calculer la quantité totale de déchets plastiques en 2030.
\item On souhaite déterminer en quelle année la masse totale de ces déchets plastiques aura pour la première fois augmenté de $50$\,\% par rapport à sa valeur de 2017.
\begin{enumerate}
\item Recopier et compléter l'algorithme ci-dessous pour que la variable $N$ contienne la réponse au problème posé.
\begin{center}
\begin{tabularx}{0.4\linewidth}{|X|}\hline
$N = 2017$\\
$U = 15$ \\
$S = 300 + U$ \\
while $S < 450$: \\
\hspace{1cm} $N = \ldots$\\
\hspace{1cm} $U = \ldots$\\
\hspace{1cm} $S = \ldots$\\
\hline
\end{tabularx}
\end{center}
\item Que contiennent les variables $S$, $U$ et $N$ après exécution de cet algorithme ?
Interpréter les résultats dans le contexte de l'exercice.
\end{enumerate}
\end{enumerate}
\end{exercise}
\begin{solution}
\begin{enumerate}
\item Une augmentation de $12\,\%$ revient à multiplier la quantité par $1.12$. La suite est donc bien géométrique. Son premier terme est $u_0 = 15$ et sa raison est $q = 1.12$
\item
\[
u_1 = u_0 * 1.12 = 16.8
\]
\[
u_2 = u_0 * 1.12^2 = 18.816
\]
\item
\[
u_n = u_0 \times q^n = 15 \times 1.12^n
\]
\item On calcule la quantité totale déversée entre 2017 et 2030.
\[
\sum_{n = 0}^{13} u_n = u_0 \times \frac{1-q^{13}}{1-q} = 15 \times \frac{1 - 1.12^{13}}{1 - 1.12} = 420.44
\]
On en déduit la quantité totale de déchets en 2030
\[
300 + 420.44 = 720.44
\]
\item
\begin{enumerate}
\item ~
\begin{center}
\begin{tabularx}{0.4\linewidth}{|X|}\hline
$N \gets 2017$\\
$U \gets 15$ \\
$S \gets 300 + U$ \\
Tant que $S < 450$ \\
\hspace{1cm} $N \gets N + 1$\\
\hspace{1cm} $U \gets U * 1.12$\\
\hspace{1cm} $S \gets S + u$\\
Fin Tant que\\\hline
\end{tabularx}
\end{center}
\item \textit{Pas de correction automatisé}
\end{enumerate}
\end{enumerate}
\end{solution}
\end{document}
%%% Local Variables:
%%% mode: latex
%%% TeX-master: "master"
%%% End:

View File

@ -0,0 +1,262 @@
\documentclass[a4paper,10pt]{article}
\usepackage{myXsim}
% Title Page
\title{DS8 \hfill MASSON Grace}
\tribe{TST}
\date{\hfillÀ render pour le Mercredi 7 avril}
\xsimsetup{
solution/print = false
}
\begin{document}
\maketitle
\begin{exercise}[subtitle={Automatismes}]
\textit{Toutes les questions de cette exercice sont indépendantes et peuvent être répondus séparément}
\begin{enumerate}
\item De janvier à septembre, une quantité a augmenté de $24\,\%$. Faire un schéma pour représenter la situation puis calculer le taux d'évolution moyen mensuel.
\item Une quantité augmente de $24\,\%$ par ans. En 2020, elle est de 145\euro. Quelle était sa valeur en 2019? Faire un schéma pour représenter la situation.
\item Déterminer l'équation de la droite \\
\begin{tikzpicture}[xscale=0.8, yscale=0.5]
\tkzInit[xmin=-5,xmax=5,xstep=1,
ymin=-5,ymax=5,ystep=1]
\tkzGrid
\tkzAxeXY
\tkzFct[domain=-5:5,color=red,very thick]%
{3.0*\x -3};
\end{tikzpicture}
\item Résoudre l'équation $5 \times 0.53^x = 18$
\end{enumerate}
\end{exercise}
\begin{solution}
\begin{enumerate}
\item On veut partager cette évolution en 8 évolutions.
\[
\left(1 + \frac{24}{100}\right)^{\frac{1}{8}} = 1.0273
\]
Donc le taux d'évolution moyen est
\[
t_m = 1.0273 - 1 = 0.027300000000000102
\]
\item Coefficient multiplicateur pour revenir en arrière
\[
CM = (1 + \frac{24}{100})^{-1} = 0.8065
\]
On en déduit la quantité en 2019
\[
145 * 0.8065 = 116.9425
\]
\item L'équation de la droite est
\[
y = 3.0 x -3
\]
\item Il faut penser à faire la division à par $5$ avant d'utiliser le log car sinon, on ne peut pas utiliser la formule $\log(a^n) = n\times \log(a)$.
\[x = \frac{\log(3.6)}{\log(0.53)}\]
\end{enumerate}
\end{solution}
\begin{exercise}[subtitle={Restaurant}]
Un \emph{food truck}, ouvert le midi et le soir, propose deux types de formules :
\setlength\parindent{10mm}
\begin{itemize}
\item la formule \emph{Burger} ;
\item la formule \emph{Wok}.
\end{itemize}
\setlength\parindent{0mm}
\medskip
Le gérant a remarqué que 73\,\% de ses ventes ont lieu le midi. Le quart des ventes du midi correspondent à la formule \emph{Burger}, alors que 56\,\% des ventes du soir correspondent à la formule \emph{Wok}.
Le gérant se constitue un fichier en notant, pour chaque vente, la formule choisie et le moment de cette vente (midi ou soir).
On prélève une fiche de façon équiprobable. On définit les quatre évènements suivants:
\begin{enumerate}
\item $M$ : \og la fiche correspond à une vente du midi\fg{} ;
\item $S$ : \og la fiche correspond à une vente du soir\fg {};
\item $W$ : \og la fiche correspond à une formule \emph{Wok} \fg{} ;
\item $B$ : \og la fiche correspond à une formule \emph{Burger} \fg.
\end{enumerate}
\setlength\parindent{0mm}
\medskip
\begin{enumerate}
\item Recopier puis compléter l'arbre pondéré
\begin{center}
\begin{tikzpicture}[sloped]
\node {.}
child {node {$M$}
child {node {$W$}
edge from parent
node[above] {...}
}
child {node {$B$}
edge from parent
node[above] {...}
}
edge from parent
node[above] {...}
}
child[missing] {}
child { node {$S$}
child {node {$W$}
edge from parent
node[above] {...}
}
child {node {$B$}
edge from parent
node[above] {...}
}
edge from parent
node[above] {...}
} ;
\end{tikzpicture}
\end{center}
\item Calculer la probabilité de l'évènement $M \cap W$. Interpréter ce résultat dans le contexte de l'exercice.
\item Montrer que la probabilité que la fiche choisie corresponde à une formule \emph{Burger} est égale à $0.2986$.
\item On a prélevé une fiche correspondant à la formule \emph{Burger}. Quelle est la probabilité, arrondie au millième, que la vente ait eu lieu le soir?
\end{enumerate}
\end{exercise}
\begin{solution}
\begin{enumerate}
\item
\begin{center}
\begin{tikzpicture}[sloped]
\node {.}
child {node {$M$}
child {node {$W$}
edge from parent
node[above] {$0.75$}
}
child {node {$B$}
edge from parent
node[above] {$0.25$}
}
edge from parent
node[above] {$0.73$}
}
child[missing] {}
child { node {$S$}
child {node {$W$}
edge from parent
node[above] {$0.57$}
}
child {node {$B$}
edge from parent
node[above] {$0.43$}
}
edge from parent
node[above] {$0.27$}
} ;
\end{tikzpicture}
\end{center}
\item On calcule la probabilité que la vente soit un wok et ait eu lieu à midi
\[ P(M\cap W) = P(M) \times P_M(W) = 0.73 \times 0.75 = 0.5475 \]
\item Probabilité que la vente soit un burger.
\[
P(B) = P(M\cap B) + P(S\cap B) = 0.73 \times 0.75 + 0.27 \times 0.57 = 0.2986
\]
\item On cherche à calculer la quantité $P_B(S)$. Pour cela on utilise la formule de Bayes
\[
P_B(S) = \frac{P(B\cap S)}{P(B)} = \frac{P_S(B) \times P(S)}{P(B)} = \frac{0.43\times 0.27}{0.2986} = 0.38881446751507037 \approx 0.389
\]
\end{enumerate}
\end{solution}
\begin{exercise}[subtitle={Continent plastique}]
\textit{Les quantités évoqués dans cette exercice sont générés au hasard et sont donc complètement farfelus.}
\medskip
Le \og continent de plastique\fg{} est la plus grande des plaques de déchets plastiques évoluant sur les océans. Elle occupe actuellement dans l'océan Pacifique une surface dont l'aire est évaluée à plus de $1,6$ million de km$^2$, entre Hawaï et la Californie.
En 2017, des scientifiques ont estimé qu'il y avait $4$ millions de tonnes de déchets plastiques qui était déversé chaque année dans les océans et que cette quantité augmentait de $19\n\%$ par chaque année.
On modélise l'évolution de la masse de ces déchets plastiques déversée chaque année, si rien n'est fait pour la réduire, par une suite géométrique $\left(u_n\right)$. L'arrondi au centième du terme $u_n$ représente la masse de ces déchets déversée chaque année, exprimée en million de tonnes, pour l'année $(2017 + n)$.
\medskip
\begin{enumerate}
\item Expliquer pourquoi la suite $u_n$ est géométrique?
\item Calculer $u_1$ et $u_2$.
\item Exprimer $u_n$ en fonction de $n$.
\item Au début de l'année 2017, il y avait $300$ millions de tonnes de déchets plastique. Calculer la quantité totale de déchets plastiques en 2030.
\item On souhaite déterminer en quelle année la masse totale de ces déchets plastiques aura pour la première fois augmenté de $50$\,\% par rapport à sa valeur de 2017.
\begin{enumerate}
\item Recopier et compléter l'algorithme ci-dessous pour que la variable $N$ contienne la réponse au problème posé.
\begin{center}
\begin{tabularx}{0.4\linewidth}{|X|}\hline
$N = 2017$\\
$U = 4$ \\
$S = 300 + U$ \\
while $S < 450$: \\
\hspace{1cm} $N = \ldots$\\
\hspace{1cm} $U = \ldots$\\
\hspace{1cm} $S = \ldots$\\
\hline
\end{tabularx}
\end{center}
\item Que contiennent les variables $S$, $U$ et $N$ après exécution de cet algorithme ?
Interpréter les résultats dans le contexte de l'exercice.
\end{enumerate}
\end{enumerate}
\end{exercise}
\begin{solution}
\begin{enumerate}
\item Une augmentation de $19\,\%$ revient à multiplier la quantité par $1.19$. La suite est donc bien géométrique. Son premier terme est $u_0 = 4$ et sa raison est $q = 1.19$
\item
\[
u_1 = u_0 * 1.19 = 4.76
\]
\[
u_2 = u_0 * 1.19^2 = 5.6644
\]
\item
\[
u_n = u_0 \times q^n = 4 \times 1.19^n
\]
\item On calcule la quantité totale déversée entre 2017 et 2030.
\[
\sum_{n = 0}^{13} u_n = u_0 \times \frac{1-q^{13}}{1-q} = 4 \times \frac{1 - 1.19^{13}}{1 - 1.19} = 180.98
\]
On en déduit la quantité totale de déchets en 2030
\[
300 + 180.98 = 480.98
\]
\item
\begin{enumerate}
\item ~
\begin{center}
\begin{tabularx}{0.4\linewidth}{|X|}\hline
$N \gets 2017$\\
$U \gets 4$ \\
$S \gets 300 + U$ \\
Tant que $S < 450$ \\
\hspace{1cm} $N \gets N + 1$\\
\hspace{1cm} $U \gets U * 1.19$\\
\hspace{1cm} $S \gets S + u$\\
Fin Tant que\\\hline
\end{tabularx}
\end{center}
\item \textit{Pas de correction automatisé}
\end{enumerate}
\end{enumerate}
\end{solution}
\end{document}
%%% Local Variables:
%%% mode: latex
%%% TeX-master: "master"
%%% End:

View File

@ -0,0 +1,262 @@
\documentclass[a4paper,10pt]{article}
\usepackage{myXsim}
% Title Page
\title{DS8 \hfill MOKHTARI Nissrine}
\tribe{TST}
\date{\hfillÀ render pour le Mercredi 7 avril}
\xsimsetup{
solution/print = false
}
\begin{document}
\maketitle
\begin{exercise}[subtitle={Automatismes}]
\textit{Toutes les questions de cette exercice sont indépendantes et peuvent être répondus séparément}
\begin{enumerate}
\item De janvier à septembre, une quantité a augmenté de $11\,\%$. Faire un schéma pour représenter la situation puis calculer le taux d'évolution moyen mensuel.
\item Une quantité augmente de $11\,\%$ par ans. En 2020, elle est de 138\euro. Quelle était sa valeur en 2019? Faire un schéma pour représenter la situation.
\item Déterminer l'équation de la droite \\
\begin{tikzpicture}[xscale=0.8, yscale=0.5]
\tkzInit[xmin=-5,xmax=5,xstep=1,
ymin=-5,ymax=5,ystep=1]
\tkzGrid
\tkzAxeXY
\tkzFct[domain=-5:5,color=red,very thick]%
{2.0*\x -3};
\end{tikzpicture}
\item Résoudre l'équation $8 \times 0.17^x = 35$
\end{enumerate}
\end{exercise}
\begin{solution}
\begin{enumerate}
\item On veut partager cette évolution en 8 évolutions.
\[
\left(1 + \frac{11}{100}\right)^{\frac{1}{8}} = 1.0131
\]
Donc le taux d'évolution moyen est
\[
t_m = 1.0131 - 1 = 0.01309999999999989
\]
\item Coefficient multiplicateur pour revenir en arrière
\[
CM = (1 + \frac{11}{100})^{-1} = 0.9009
\]
On en déduit la quantité en 2019
\[
138 * 0.9009 = 124.3242
\]
\item L'équation de la droite est
\[
y = 2.0 x -3
\]
\item Il faut penser à faire la division à par $8$ avant d'utiliser le log car sinon, on ne peut pas utiliser la formule $\log(a^n) = n\times \log(a)$.
\[x = \frac{\log(4.38)}{\log(0.17)}\]
\end{enumerate}
\end{solution}
\begin{exercise}[subtitle={Restaurant}]
Un \emph{food truck}, ouvert le midi et le soir, propose deux types de formules :
\setlength\parindent{10mm}
\begin{itemize}
\item la formule \emph{Burger} ;
\item la formule \emph{Wok}.
\end{itemize}
\setlength\parindent{0mm}
\medskip
Le gérant a remarqué que 62\,\% de ses ventes ont lieu le midi. Le quart des ventes du midi correspondent à la formule \emph{Burger}, alors que 28\,\% des ventes du soir correspondent à la formule \emph{Wok}.
Le gérant se constitue un fichier en notant, pour chaque vente, la formule choisie et le moment de cette vente (midi ou soir).
On prélève une fiche de façon équiprobable. On définit les quatre évènements suivants:
\begin{enumerate}
\item $M$ : \og la fiche correspond à une vente du midi\fg{} ;
\item $S$ : \og la fiche correspond à une vente du soir\fg {};
\item $W$ : \og la fiche correspond à une formule \emph{Wok} \fg{} ;
\item $B$ : \og la fiche correspond à une formule \emph{Burger} \fg.
\end{enumerate}
\setlength\parindent{0mm}
\medskip
\begin{enumerate}
\item Recopier puis compléter l'arbre pondéré
\begin{center}
\begin{tikzpicture}[sloped]
\node {.}
child {node {$M$}
child {node {$W$}
edge from parent
node[above] {...}
}
child {node {$B$}
edge from parent
node[above] {...}
}
edge from parent
node[above] {...}
}
child[missing] {}
child { node {$S$}
child {node {$W$}
edge from parent
node[above] {...}
}
child {node {$B$}
edge from parent
node[above] {...}
}
edge from parent
node[above] {...}
} ;
\end{tikzpicture}
\end{center}
\item Calculer la probabilité de l'évènement $M \cap W$. Interpréter ce résultat dans le contexte de l'exercice.
\item Montrer que la probabilité que la fiche choisie corresponde à une formule \emph{Burger} est égale à $0.4248$.
\item On a prélevé une fiche correspondant à la formule \emph{Burger}. Quelle est la probabilité, arrondie au millième, que la vente ait eu lieu le soir?
\end{enumerate}
\end{exercise}
\begin{solution}
\begin{enumerate}
\item
\begin{center}
\begin{tikzpicture}[sloped]
\node {.}
child {node {$M$}
child {node {$W$}
edge from parent
node[above] {$0.75$}
}
child {node {$B$}
edge from parent
node[above] {$0.25$}
}
edge from parent
node[above] {$0.62$}
}
child[missing] {}
child { node {$S$}
child {node {$W$}
edge from parent
node[above] {$0.29$}
}
child {node {$B$}
edge from parent
node[above] {$0.71$}
}
edge from parent
node[above] {$0.38$}
} ;
\end{tikzpicture}
\end{center}
\item On calcule la probabilité que la vente soit un wok et ait eu lieu à midi
\[ P(M\cap W) = P(M) \times P_M(W) = 0.62 \times 0.75 = 0.465 \]
\item Probabilité que la vente soit un burger.
\[
P(B) = P(M\cap B) + P(S\cap B) = 0.62 \times 0.75 + 0.38 \times 0.29 = 0.4248
\]
\item On cherche à calculer la quantité $P_B(S)$. Pour cela on utilise la formule de Bayes
\[
P_B(S) = \frac{P(B\cap S)}{P(B)} = \frac{P_S(B) \times P(S)}{P(B)} = \frac{0.71\times 0.38}{0.4248} = 0.6351224105461393 \approx 0.635
\]
\end{enumerate}
\end{solution}
\begin{exercise}[subtitle={Continent plastique}]
\textit{Les quantités évoqués dans cette exercice sont générés au hasard et sont donc complètement farfelus.}
\medskip
Le \og continent de plastique\fg{} est la plus grande des plaques de déchets plastiques évoluant sur les océans. Elle occupe actuellement dans l'océan Pacifique une surface dont l'aire est évaluée à plus de $1,6$ million de km$^2$, entre Hawaï et la Californie.
En 2017, des scientifiques ont estimé qu'il y avait $4$ millions de tonnes de déchets plastiques qui était déversé chaque année dans les océans et que cette quantité augmentait de $30\n\%$ par chaque année.
On modélise l'évolution de la masse de ces déchets plastiques déversée chaque année, si rien n'est fait pour la réduire, par une suite géométrique $\left(u_n\right)$. L'arrondi au centième du terme $u_n$ représente la masse de ces déchets déversée chaque année, exprimée en million de tonnes, pour l'année $(2017 + n)$.
\medskip
\begin{enumerate}
\item Expliquer pourquoi la suite $u_n$ est géométrique?
\item Calculer $u_1$ et $u_2$.
\item Exprimer $u_n$ en fonction de $n$.
\item Au début de l'année 2017, il y avait $300$ millions de tonnes de déchets plastique. Calculer la quantité totale de déchets plastiques en 2030.
\item On souhaite déterminer en quelle année la masse totale de ces déchets plastiques aura pour la première fois augmenté de $50$\,\% par rapport à sa valeur de 2017.
\begin{enumerate}
\item Recopier et compléter l'algorithme ci-dessous pour que la variable $N$ contienne la réponse au problème posé.
\begin{center}
\begin{tabularx}{0.4\linewidth}{|X|}\hline
$N = 2017$\\
$U = 4$ \\
$S = 300 + U$ \\
while $S < 450$: \\
\hspace{1cm} $N = \ldots$\\
\hspace{1cm} $U = \ldots$\\
\hspace{1cm} $S = \ldots$\\
\hline
\end{tabularx}
\end{center}
\item Que contiennent les variables $S$, $U$ et $N$ après exécution de cet algorithme ?
Interpréter les résultats dans le contexte de l'exercice.
\end{enumerate}
\end{enumerate}
\end{exercise}
\begin{solution}
\begin{enumerate}
\item Une augmentation de $30\,\%$ revient à multiplier la quantité par $1.3$. La suite est donc bien géométrique. Son premier terme est $u_0 = 4$ et sa raison est $q = 1.3$
\item
\[
u_1 = u_0 * 1.3 = 5.2
\]
\[
u_2 = u_0 * 1.3^2 = 6.76
\]
\item
\[
u_n = u_0 \times q^n = 4 \times 1.3^n
\]
\item On calcule la quantité totale déversée entre 2017 et 2030.
\[
\sum_{n = 0}^{13} u_n = u_0 \times \frac{1-q^{13}}{1-q} = 4 \times \frac{1 - 1.3^{13}}{1 - 1.3} = 390.5
\]
On en déduit la quantité totale de déchets en 2030
\[
300 + 390.5 = 690.5
\]
\item
\begin{enumerate}
\item ~
\begin{center}
\begin{tabularx}{0.4\linewidth}{|X|}\hline
$N \gets 2017$\\
$U \gets 4$ \\
$S \gets 300 + U$ \\
Tant que $S < 450$ \\
\hspace{1cm} $N \gets N + 1$\\
\hspace{1cm} $U \gets U * 1.3$\\
\hspace{1cm} $S \gets S + u$\\
Fin Tant que\\\hline
\end{tabularx}
\end{center}
\item \textit{Pas de correction automatisé}
\end{enumerate}
\end{enumerate}
\end{solution}
\end{document}
%%% Local Variables:
%%% mode: latex
%%% TeX-master: "master"
%%% End:

View File

@ -0,0 +1,262 @@
\documentclass[a4paper,10pt]{article}
\usepackage{myXsim}
% Title Page
\title{DS8 \hfill MOUFAQ Amine}
\tribe{TST}
\date{\hfillÀ render pour le Mercredi 7 avril}
\xsimsetup{
solution/print = false
}
\begin{document}
\maketitle
\begin{exercise}[subtitle={Automatismes}]
\textit{Toutes les questions de cette exercice sont indépendantes et peuvent être répondus séparément}
\begin{enumerate}
\item De janvier à septembre, une quantité a augmenté de $16\,\%$. Faire un schéma pour représenter la situation puis calculer le taux d'évolution moyen mensuel.
\item Une quantité augmente de $16\,\%$ par ans. En 2020, elle est de 115\euro. Quelle était sa valeur en 2019? Faire un schéma pour représenter la situation.
\item Déterminer l'équation de la droite \\
\begin{tikzpicture}[xscale=0.8, yscale=0.5]
\tkzInit[xmin=-5,xmax=5,xstep=1,
ymin=-5,ymax=5,ystep=1]
\tkzGrid
\tkzAxeXY
\tkzFct[domain=-5:5,color=red,very thick]%
{0.5*\x -1};
\end{tikzpicture}
\item Résoudre l'équation $6 \times 0.24^x = 41$
\end{enumerate}
\end{exercise}
\begin{solution}
\begin{enumerate}
\item On veut partager cette évolution en 8 évolutions.
\[
\left(1 + \frac{16}{100}\right)^{\frac{1}{8}} = 1.0187
\]
Donc le taux d'évolution moyen est
\[
t_m = 1.0187 - 1 = 0.01869999999999994
\]
\item Coefficient multiplicateur pour revenir en arrière
\[
CM = (1 + \frac{16}{100})^{-1} = 0.8621
\]
On en déduit la quantité en 2019
\[
115 * 0.8621 = 99.1415
\]
\item L'équation de la droite est
\[
y = 0.5 x -1
\]
\item Il faut penser à faire la division à par $6$ avant d'utiliser le log car sinon, on ne peut pas utiliser la formule $\log(a^n) = n\times \log(a)$.
\[x = \frac{\log(6.83)}{\log(0.24)}\]
\end{enumerate}
\end{solution}
\begin{exercise}[subtitle={Restaurant}]
Un \emph{food truck}, ouvert le midi et le soir, propose deux types de formules :
\setlength\parindent{10mm}
\begin{itemize}
\item la formule \emph{Burger} ;
\item la formule \emph{Wok}.
\end{itemize}
\setlength\parindent{0mm}
\medskip
Le gérant a remarqué que 78\,\% de ses ventes ont lieu le midi. Le quart des ventes du midi correspondent à la formule \emph{Burger}, alors que 37\,\% des ventes du soir correspondent à la formule \emph{Wok}.
Le gérant se constitue un fichier en notant, pour chaque vente, la formule choisie et le moment de cette vente (midi ou soir).
On prélève une fiche de façon équiprobable. On définit les quatre évènements suivants:
\begin{enumerate}
\item $M$ : \og la fiche correspond à une vente du midi\fg{} ;
\item $S$ : \og la fiche correspond à une vente du soir\fg {};
\item $W$ : \og la fiche correspond à une formule \emph{Wok} \fg{} ;
\item $B$ : \og la fiche correspond à une formule \emph{Burger} \fg.
\end{enumerate}
\setlength\parindent{0mm}
\medskip
\begin{enumerate}
\item Recopier puis compléter l'arbre pondéré
\begin{center}
\begin{tikzpicture}[sloped]
\node {.}
child {node {$M$}
child {node {$W$}
edge from parent
node[above] {...}
}
child {node {$B$}
edge from parent
node[above] {...}
}
edge from parent
node[above] {...}
}
child[missing] {}
child { node {$S$}
child {node {$W$}
edge from parent
node[above] {...}
}
child {node {$B$}
edge from parent
node[above] {...}
}
edge from parent
node[above] {...}
} ;
\end{tikzpicture}
\end{center}
\item Calculer la probabilité de l'évènement $M \cap W$. Interpréter ce résultat dans le contexte de l'exercice.
\item Montrer que la probabilité que la fiche choisie corresponde à une formule \emph{Burger} est égale à $0.3336$.
\item On a prélevé une fiche correspondant à la formule \emph{Burger}. Quelle est la probabilité, arrondie au millième, que la vente ait eu lieu le soir?
\end{enumerate}
\end{exercise}
\begin{solution}
\begin{enumerate}
\item
\begin{center}
\begin{tikzpicture}[sloped]
\node {.}
child {node {$M$}
child {node {$W$}
edge from parent
node[above] {$0.75$}
}
child {node {$B$}
edge from parent
node[above] {$0.25$}
}
edge from parent
node[above] {$0.78$}
}
child[missing] {}
child { node {$S$}
child {node {$W$}
edge from parent
node[above] {$0.37$}
}
child {node {$B$}
edge from parent
node[above] {$0.63$}
}
edge from parent
node[above] {$0.22$}
} ;
\end{tikzpicture}
\end{center}
\item On calcule la probabilité que la vente soit un wok et ait eu lieu à midi
\[ P(M\cap W) = P(M) \times P_M(W) = 0.78 \times 0.75 = 0.585 \]
\item Probabilité que la vente soit un burger.
\[
P(B) = P(M\cap B) + P(S\cap B) = 0.78 \times 0.75 + 0.22 \times 0.37 = 0.3336
\]
\item On cherche à calculer la quantité $P_B(S)$. Pour cela on utilise la formule de Bayes
\[
P_B(S) = \frac{P(B\cap S)}{P(B)} = \frac{P_S(B) \times P(S)}{P(B)} = \frac{0.63\times 0.22}{0.3336} = 0.41546762589928055 \approx 0.415
\]
\end{enumerate}
\end{solution}
\begin{exercise}[subtitle={Continent plastique}]
\textit{Les quantités évoqués dans cette exercice sont générés au hasard et sont donc complètement farfelus.}
\medskip
Le \og continent de plastique\fg{} est la plus grande des plaques de déchets plastiques évoluant sur les océans. Elle occupe actuellement dans l'océan Pacifique une surface dont l'aire est évaluée à plus de $1,6$ million de km$^2$, entre Hawaï et la Californie.
En 2017, des scientifiques ont estimé qu'il y avait $10$ millions de tonnes de déchets plastiques qui était déversé chaque année dans les océans et que cette quantité augmentait de $24\n\%$ par chaque année.
On modélise l'évolution de la masse de ces déchets plastiques déversée chaque année, si rien n'est fait pour la réduire, par une suite géométrique $\left(u_n\right)$. L'arrondi au centième du terme $u_n$ représente la masse de ces déchets déversée chaque année, exprimée en million de tonnes, pour l'année $(2017 + n)$.
\medskip
\begin{enumerate}
\item Expliquer pourquoi la suite $u_n$ est géométrique?
\item Calculer $u_1$ et $u_2$.
\item Exprimer $u_n$ en fonction de $n$.
\item Au début de l'année 2017, il y avait $300$ millions de tonnes de déchets plastique. Calculer la quantité totale de déchets plastiques en 2030.
\item On souhaite déterminer en quelle année la masse totale de ces déchets plastiques aura pour la première fois augmenté de $50$\,\% par rapport à sa valeur de 2017.
\begin{enumerate}
\item Recopier et compléter l'algorithme ci-dessous pour que la variable $N$ contienne la réponse au problème posé.
\begin{center}
\begin{tabularx}{0.4\linewidth}{|X|}\hline
$N = 2017$\\
$U = 10$ \\
$S = 300 + U$ \\
while $S < 450$: \\
\hspace{1cm} $N = \ldots$\\
\hspace{1cm} $U = \ldots$\\
\hspace{1cm} $S = \ldots$\\
\hline
\end{tabularx}
\end{center}
\item Que contiennent les variables $S$, $U$ et $N$ après exécution de cet algorithme ?
Interpréter les résultats dans le contexte de l'exercice.
\end{enumerate}
\end{enumerate}
\end{exercise}
\begin{solution}
\begin{enumerate}
\item Une augmentation de $24\,\%$ revient à multiplier la quantité par $1.24$. La suite est donc bien géométrique. Son premier terme est $u_0 = 10$ et sa raison est $q = 1.24$
\item
\[
u_1 = u_0 * 1.24 = 12.4
\]
\[
u_2 = u_0 * 1.24^2 = 15.376
\]
\item
\[
u_n = u_0 \times q^n = 10 \times 1.24^n
\]
\item On calcule la quantité totale déversée entre 2017 et 2030.
\[
\sum_{n = 0}^{13} u_n = u_0 \times \frac{1-q^{13}}{1-q} = 10 \times \frac{1 - 1.24^{13}}{1 - 1.24} = 641.1
\]
On en déduit la quantité totale de déchets en 2030
\[
300 + 641.1 = 941.1
\]
\item
\begin{enumerate}
\item ~
\begin{center}
\begin{tabularx}{0.4\linewidth}{|X|}\hline
$N \gets 2017$\\
$U \gets 10$ \\
$S \gets 300 + U$ \\
Tant que $S < 450$ \\
\hspace{1cm} $N \gets N + 1$\\
\hspace{1cm} $U \gets U * 1.24$\\
\hspace{1cm} $S \gets S + u$\\
Fin Tant que\\\hline
\end{tabularx}
\end{center}
\item \textit{Pas de correction automatisé}
\end{enumerate}
\end{enumerate}
\end{solution}
\end{document}
%%% Local Variables:
%%% mode: latex
%%% TeX-master: "master"
%%% End:

View File

@ -0,0 +1,262 @@
\documentclass[a4paper,10pt]{article}
\usepackage{myXsim}
% Title Page
\title{DS8 \hfill ONAL Yakub}
\tribe{TST}
\date{\hfillÀ render pour le Mercredi 7 avril}
\xsimsetup{
solution/print = false
}
\begin{document}
\maketitle
\begin{exercise}[subtitle={Automatismes}]
\textit{Toutes les questions de cette exercice sont indépendantes et peuvent être répondus séparément}
\begin{enumerate}
\item De janvier à septembre, une quantité a augmenté de $27\,\%$. Faire un schéma pour représenter la situation puis calculer le taux d'évolution moyen mensuel.
\item Une quantité augmente de $27\,\%$ par ans. En 2020, elle est de 132\euro. Quelle était sa valeur en 2019? Faire un schéma pour représenter la situation.
\item Déterminer l'équation de la droite \\
\begin{tikzpicture}[xscale=0.8, yscale=0.5]
\tkzInit[xmin=-5,xmax=5,xstep=1,
ymin=-5,ymax=5,ystep=1]
\tkzGrid
\tkzAxeXY
\tkzFct[domain=-5:5,color=red,very thick]%
{1.0*\x -1};
\end{tikzpicture}
\item Résoudre l'équation $10 \times 0.18^x = 7$
\end{enumerate}
\end{exercise}
\begin{solution}
\begin{enumerate}
\item On veut partager cette évolution en 8 évolutions.
\[
\left(1 + \frac{27}{100}\right)^{\frac{1}{8}} = 1.0303
\]
Donc le taux d'évolution moyen est
\[
t_m = 1.0303 - 1 = 0.030299999999999994
\]
\item Coefficient multiplicateur pour revenir en arrière
\[
CM = (1 + \frac{27}{100})^{-1} = 0.7874
\]
On en déduit la quantité en 2019
\[
132 * 0.7874 = 103.9368
\]
\item L'équation de la droite est
\[
y = 1.0 x -1
\]
\item Il faut penser à faire la division à par $10$ avant d'utiliser le log car sinon, on ne peut pas utiliser la formule $\log(a^n) = n\times \log(a)$.
\[x = \frac{\log(0.7)}{\log(0.18)}\]
\end{enumerate}
\end{solution}
\begin{exercise}[subtitle={Restaurant}]
Un \emph{food truck}, ouvert le midi et le soir, propose deux types de formules :
\setlength\parindent{10mm}
\begin{itemize}
\item la formule \emph{Burger} ;
\item la formule \emph{Wok}.
\end{itemize}
\setlength\parindent{0mm}
\medskip
Le gérant a remarqué que 78\,\% de ses ventes ont lieu le midi. Le quart des ventes du midi correspondent à la formule \emph{Burger}, alors que 28\,\% des ventes du soir correspondent à la formule \emph{Wok}.
Le gérant se constitue un fichier en notant, pour chaque vente, la formule choisie et le moment de cette vente (midi ou soir).
On prélève une fiche de façon équiprobable. On définit les quatre évènements suivants:
\begin{enumerate}
\item $M$ : \og la fiche correspond à une vente du midi\fg{} ;
\item $S$ : \og la fiche correspond à une vente du soir\fg {};
\item $W$ : \og la fiche correspond à une formule \emph{Wok} \fg{} ;
\item $B$ : \og la fiche correspond à une formule \emph{Burger} \fg.
\end{enumerate}
\setlength\parindent{0mm}
\medskip
\begin{enumerate}
\item Recopier puis compléter l'arbre pondéré
\begin{center}
\begin{tikzpicture}[sloped]
\node {.}
child {node {$M$}
child {node {$W$}
edge from parent
node[above] {...}
}
child {node {$B$}
edge from parent
node[above] {...}
}
edge from parent
node[above] {...}
}
child[missing] {}
child { node {$S$}
child {node {$W$}
edge from parent
node[above] {...}
}
child {node {$B$}
edge from parent
node[above] {...}
}
edge from parent
node[above] {...}
} ;
\end{tikzpicture}
\end{center}
\item Calculer la probabilité de l'évènement $M \cap W$. Interpréter ce résultat dans le contexte de l'exercice.
\item Montrer que la probabilité que la fiche choisie corresponde à une formule \emph{Burger} est égale à $0.3534$.
\item On a prélevé une fiche correspondant à la formule \emph{Burger}. Quelle est la probabilité, arrondie au millième, que la vente ait eu lieu le soir?
\end{enumerate}
\end{exercise}
\begin{solution}
\begin{enumerate}
\item
\begin{center}
\begin{tikzpicture}[sloped]
\node {.}
child {node {$M$}
child {node {$W$}
edge from parent
node[above] {$0.75$}
}
child {node {$B$}
edge from parent
node[above] {$0.25$}
}
edge from parent
node[above] {$0.78$}
}
child[missing] {}
child { node {$S$}
child {node {$W$}
edge from parent
node[above] {$0.28$}
}
child {node {$B$}
edge from parent
node[above] {$0.72$}
}
edge from parent
node[above] {$0.22$}
} ;
\end{tikzpicture}
\end{center}
\item On calcule la probabilité que la vente soit un wok et ait eu lieu à midi
\[ P(M\cap W) = P(M) \times P_M(W) = 0.78 \times 0.75 = 0.585 \]
\item Probabilité que la vente soit un burger.
\[
P(B) = P(M\cap B) + P(S\cap B) = 0.78 \times 0.75 + 0.22 \times 0.28 = 0.3534
\]
\item On cherche à calculer la quantité $P_B(S)$. Pour cela on utilise la formule de Bayes
\[
P_B(S) = \frac{P(B\cap S)}{P(B)} = \frac{P_S(B) \times P(S)}{P(B)} = \frac{0.72\times 0.22}{0.3534} = 0.4482173174872665 \approx 0.448
\]
\end{enumerate}
\end{solution}
\begin{exercise}[subtitle={Continent plastique}]
\textit{Les quantités évoqués dans cette exercice sont générés au hasard et sont donc complètement farfelus.}
\medskip
Le \og continent de plastique\fg{} est la plus grande des plaques de déchets plastiques évoluant sur les océans. Elle occupe actuellement dans l'océan Pacifique une surface dont l'aire est évaluée à plus de $1,6$ million de km$^2$, entre Hawaï et la Californie.
En 2017, des scientifiques ont estimé qu'il y avait $3$ millions de tonnes de déchets plastiques qui était déversé chaque année dans les océans et que cette quantité augmentait de $21\n\%$ par chaque année.
On modélise l'évolution de la masse de ces déchets plastiques déversée chaque année, si rien n'est fait pour la réduire, par une suite géométrique $\left(u_n\right)$. L'arrondi au centième du terme $u_n$ représente la masse de ces déchets déversée chaque année, exprimée en million de tonnes, pour l'année $(2017 + n)$.
\medskip
\begin{enumerate}
\item Expliquer pourquoi la suite $u_n$ est géométrique?
\item Calculer $u_1$ et $u_2$.
\item Exprimer $u_n$ en fonction de $n$.
\item Au début de l'année 2017, il y avait $300$ millions de tonnes de déchets plastique. Calculer la quantité totale de déchets plastiques en 2030.
\item On souhaite déterminer en quelle année la masse totale de ces déchets plastiques aura pour la première fois augmenté de $50$\,\% par rapport à sa valeur de 2017.
\begin{enumerate}
\item Recopier et compléter l'algorithme ci-dessous pour que la variable $N$ contienne la réponse au problème posé.
\begin{center}
\begin{tabularx}{0.4\linewidth}{|X|}\hline
$N = 2017$\\
$U = 3$ \\
$S = 300 + U$ \\
while $S < 450$: \\
\hspace{1cm} $N = \ldots$\\
\hspace{1cm} $U = \ldots$\\
\hspace{1cm} $S = \ldots$\\
\hline
\end{tabularx}
\end{center}
\item Que contiennent les variables $S$, $U$ et $N$ après exécution de cet algorithme ?
Interpréter les résultats dans le contexte de l'exercice.
\end{enumerate}
\end{enumerate}
\end{exercise}
\begin{solution}
\begin{enumerate}
\item Une augmentation de $21\,\%$ revient à multiplier la quantité par $1.21$. La suite est donc bien géométrique. Son premier terme est $u_0 = 3$ et sa raison est $q = 1.21$
\item
\[
u_1 = u_0 * 1.21 = 3.63
\]
\[
u_2 = u_0 * 1.21^2 = 4.3923
\]
\item
\[
u_n = u_0 \times q^n = 3 \times 1.21^n
\]
\item On calcule la quantité totale déversée entre 2017 et 2030.
\[
\sum_{n = 0}^{13} u_n = u_0 \times \frac{1-q^{13}}{1-q} = 3 \times \frac{1 - 1.21^{13}}{1 - 1.21} = 155.97
\]
On en déduit la quantité totale de déchets en 2030
\[
300 + 155.97 = 455.97
\]
\item
\begin{enumerate}
\item ~
\begin{center}
\begin{tabularx}{0.4\linewidth}{|X|}\hline
$N \gets 2017$\\
$U \gets 3$ \\
$S \gets 300 + U$ \\
Tant que $S < 450$ \\
\hspace{1cm} $N \gets N + 1$\\
\hspace{1cm} $U \gets U * 1.21$\\
\hspace{1cm} $S \gets S + u$\\
Fin Tant que\\\hline
\end{tabularx}
\end{center}
\item \textit{Pas de correction automatisé}
\end{enumerate}
\end{enumerate}
\end{solution}
\end{document}
%%% Local Variables:
%%% mode: latex
%%% TeX-master: "master"
%%% End:

View File

@ -0,0 +1,262 @@
\documentclass[a4paper,10pt]{article}
\usepackage{myXsim}
% Title Page
\title{DS8 \hfill SORIANO Laura}
\tribe{TST}
\date{\hfillÀ render pour le Mercredi 7 avril}
\xsimsetup{
solution/print = false
}
\begin{document}
\maketitle
\begin{exercise}[subtitle={Automatismes}]
\textit{Toutes les questions de cette exercice sont indépendantes et peuvent être répondus séparément}
\begin{enumerate}
\item De janvier à septembre, une quantité a augmenté de $26\,\%$. Faire un schéma pour représenter la situation puis calculer le taux d'évolution moyen mensuel.
\item Une quantité augmente de $26\,\%$ par ans. En 2020, elle est de 122\euro. Quelle était sa valeur en 2019? Faire un schéma pour représenter la situation.
\item Déterminer l'équation de la droite \\
\begin{tikzpicture}[xscale=0.8, yscale=0.5]
\tkzInit[xmin=-5,xmax=5,xstep=1,
ymin=-5,ymax=5,ystep=1]
\tkzGrid
\tkzAxeXY
\tkzFct[domain=-5:5,color=red,very thick]%
{3.0*\x -3};
\end{tikzpicture}
\item Résoudre l'équation $2 \times 0.02^x = 10$
\end{enumerate}
\end{exercise}
\begin{solution}
\begin{enumerate}
\item On veut partager cette évolution en 8 évolutions.
\[
\left(1 + \frac{26}{100}\right)^{\frac{1}{8}} = 1.0293
\]
Donc le taux d'évolution moyen est
\[
t_m = 1.0293 - 1 = 0.029300000000000104
\]
\item Coefficient multiplicateur pour revenir en arrière
\[
CM = (1 + \frac{26}{100})^{-1} = 0.7937
\]
On en déduit la quantité en 2019
\[
122 * 0.7937 = 96.8314
\]
\item L'équation de la droite est
\[
y = 3.0 x -3
\]
\item Il faut penser à faire la division à par $2$ avant d'utiliser le log car sinon, on ne peut pas utiliser la formule $\log(a^n) = n\times \log(a)$.
\[x = \frac{\log(5.0)}{\log(0.02)}\]
\end{enumerate}
\end{solution}
\begin{exercise}[subtitle={Restaurant}]
Un \emph{food truck}, ouvert le midi et le soir, propose deux types de formules :
\setlength\parindent{10mm}
\begin{itemize}
\item la formule \emph{Burger} ;
\item la formule \emph{Wok}.
\end{itemize}
\setlength\parindent{0mm}
\medskip
Le gérant a remarqué que 0\,\% de ses ventes ont lieu le midi. Le quart des ventes du midi correspondent à la formule \emph{Burger}, alors que 21\,\% des ventes du soir correspondent à la formule \emph{Wok}.
Le gérant se constitue un fichier en notant, pour chaque vente, la formule choisie et le moment de cette vente (midi ou soir).
On prélève une fiche de façon équiprobable. On définit les quatre évènements suivants:
\begin{enumerate}
\item $M$ : \og la fiche correspond à une vente du midi\fg{} ;
\item $S$ : \og la fiche correspond à une vente du soir\fg {};
\item $W$ : \og la fiche correspond à une formule \emph{Wok} \fg{} ;
\item $B$ : \og la fiche correspond à une formule \emph{Burger} \fg.
\end{enumerate}
\setlength\parindent{0mm}
\medskip
\begin{enumerate}
\item Recopier puis compléter l'arbre pondéré
\begin{center}
\begin{tikzpicture}[sloped]
\node {.}
child {node {$M$}
child {node {$W$}
edge from parent
node[above] {...}
}
child {node {$B$}
edge from parent
node[above] {...}
}
edge from parent
node[above] {...}
}
child[missing] {}
child { node {$S$}
child {node {$W$}
edge from parent
node[above] {...}
}
child {node {$B$}
edge from parent
node[above] {...}
}
edge from parent
node[above] {...}
} ;
\end{tikzpicture}
\end{center}
\item Calculer la probabilité de l'évènement $M \cap W$. Interpréter ce résultat dans le contexte de l'exercice.
\item Montrer que la probabilité que la fiche choisie corresponde à une formule \emph{Burger} est égale à $0.79$.
\item On a prélevé une fiche correspondant à la formule \emph{Burger}. Quelle est la probabilité, arrondie au millième, que la vente ait eu lieu le soir?
\end{enumerate}
\end{exercise}
\begin{solution}
\begin{enumerate}
\item
\begin{center}
\begin{tikzpicture}[sloped]
\node {.}
child {node {$M$}
child {node {$W$}
edge from parent
node[above] {$0.75$}
}
child {node {$B$}
edge from parent
node[above] {$0.25$}
}
edge from parent
node[above] {$0.0$}
}
child[missing] {}
child { node {$S$}
child {node {$W$}
edge from parent
node[above] {$0.21$}
}
child {node {$B$}
edge from parent
node[above] {$0.79$}
}
edge from parent
node[above] {$1.0$}
} ;
\end{tikzpicture}
\end{center}
\item On calcule la probabilité que la vente soit un wok et ait eu lieu à midi
\[ P(M\cap W) = P(M) \times P_M(W) = 0.0 \times 0.75 = 0.0 \]
\item Probabilité que la vente soit un burger.
\[
P(B) = P(M\cap B) + P(S\cap B) = 0.0 \times 0.75 + 1.0 \times 0.21 = 0.79
\]
\item On cherche à calculer la quantité $P_B(S)$. Pour cela on utilise la formule de Bayes
\[
P_B(S) = \frac{P(B\cap S)}{P(B)} = \frac{P_S(B) \times P(S)}{P(B)} = \frac{0.79\times 1.0}{0.79} = 1.0 \approx 1.0
\]
\end{enumerate}
\end{solution}
\begin{exercise}[subtitle={Continent plastique}]
\textit{Les quantités évoqués dans cette exercice sont générés au hasard et sont donc complètement farfelus.}
\medskip
Le \og continent de plastique\fg{} est la plus grande des plaques de déchets plastiques évoluant sur les océans. Elle occupe actuellement dans l'océan Pacifique une surface dont l'aire est évaluée à plus de $1,6$ million de km$^2$, entre Hawaï et la Californie.
En 2017, des scientifiques ont estimé qu'il y avait $11$ millions de tonnes de déchets plastiques qui était déversé chaque année dans les océans et que cette quantité augmentait de $16\n\%$ par chaque année.
On modélise l'évolution de la masse de ces déchets plastiques déversée chaque année, si rien n'est fait pour la réduire, par une suite géométrique $\left(u_n\right)$. L'arrondi au centième du terme $u_n$ représente la masse de ces déchets déversée chaque année, exprimée en million de tonnes, pour l'année $(2017 + n)$.
\medskip
\begin{enumerate}
\item Expliquer pourquoi la suite $u_n$ est géométrique?
\item Calculer $u_1$ et $u_2$.
\item Exprimer $u_n$ en fonction de $n$.
\item Au début de l'année 2017, il y avait $300$ millions de tonnes de déchets plastique. Calculer la quantité totale de déchets plastiques en 2030.
\item On souhaite déterminer en quelle année la masse totale de ces déchets plastiques aura pour la première fois augmenté de $50$\,\% par rapport à sa valeur de 2017.
\begin{enumerate}
\item Recopier et compléter l'algorithme ci-dessous pour que la variable $N$ contienne la réponse au problème posé.
\begin{center}
\begin{tabularx}{0.4\linewidth}{|X|}\hline
$N = 2017$\\
$U = 11$ \\
$S = 300 + U$ \\
while $S < 450$: \\
\hspace{1cm} $N = \ldots$\\
\hspace{1cm} $U = \ldots$\\
\hspace{1cm} $S = \ldots$\\
\hline
\end{tabularx}
\end{center}
\item Que contiennent les variables $S$, $U$ et $N$ après exécution de cet algorithme ?
Interpréter les résultats dans le contexte de l'exercice.
\end{enumerate}
\end{enumerate}
\end{exercise}
\begin{solution}
\begin{enumerate}
\item Une augmentation de $16\,\%$ revient à multiplier la quantité par $1.16$. La suite est donc bien géométrique. Son premier terme est $u_0 = 11$ et sa raison est $q = 1.16$
\item
\[
u_1 = u_0 * 1.16 = 12.76
\]
\[
u_2 = u_0 * 1.16^2 = 14.8016
\]
\item
\[
u_n = u_0 \times q^n = 11 \times 1.16^n
\]
\item On calcule la quantité totale déversée entre 2017 et 2030.
\[
\sum_{n = 0}^{13} u_n = u_0 \times \frac{1-q^{13}}{1-q} = 11 \times \frac{1 - 1.16^{13}}{1 - 1.16} = 404.65
\]
On en déduit la quantité totale de déchets en 2030
\[
300 + 404.65 = 704.65
\]
\item
\begin{enumerate}
\item ~
\begin{center}
\begin{tabularx}{0.4\linewidth}{|X|}\hline
$N \gets 2017$\\
$U \gets 11$ \\
$S \gets 300 + U$ \\
Tant que $S < 450$ \\
\hspace{1cm} $N \gets N + 1$\\
\hspace{1cm} $U \gets U * 1.16$\\
\hspace{1cm} $S \gets S + u$\\
Fin Tant que\\\hline
\end{tabularx}
\end{center}
\item \textit{Pas de correction automatisé}
\end{enumerate}
\end{enumerate}
\end{solution}
\end{document}
%%% Local Variables:
%%% mode: latex
%%% TeX-master: "master"
%%% End:

View File

@ -0,0 +1,262 @@
\documentclass[a4paper,10pt]{article}
\usepackage{myXsim}
% Title Page
\title{DS8 \hfill VECCHIO Léa}
\tribe{TST}
\date{\hfillÀ render pour le Mercredi 7 avril}
\xsimsetup{
solution/print = false
}
\begin{document}
\maketitle
\begin{exercise}[subtitle={Automatismes}]
\textit{Toutes les questions de cette exercice sont indépendantes et peuvent être répondus séparément}
\begin{enumerate}
\item De janvier à septembre, une quantité a augmenté de $28\,\%$. Faire un schéma pour représenter la situation puis calculer le taux d'évolution moyen mensuel.
\item Une quantité augmente de $28\,\%$ par ans. En 2020, elle est de 113\euro. Quelle était sa valeur en 2019? Faire un schéma pour représenter la situation.
\item Déterminer l'équation de la droite \\
\begin{tikzpicture}[xscale=0.8, yscale=0.5]
\tkzInit[xmin=-5,xmax=5,xstep=1,
ymin=-5,ymax=5,ystep=1]
\tkzGrid
\tkzAxeXY
\tkzFct[domain=-5:5,color=red,very thick]%
{2.0*\x -2};
\end{tikzpicture}
\item Résoudre l'équation $8 \times 0.84^x = 26$
\end{enumerate}
\end{exercise}
\begin{solution}
\begin{enumerate}
\item On veut partager cette évolution en 8 évolutions.
\[
\left(1 + \frac{28}{100}\right)^{\frac{1}{8}} = 1.0313
\]
Donc le taux d'évolution moyen est
\[
t_m = 1.0313 - 1 = 0.031300000000000106
\]
\item Coefficient multiplicateur pour revenir en arrière
\[
CM = (1 + \frac{28}{100})^{-1} = 0.7812
\]
On en déduit la quantité en 2019
\[
113 * 0.7812 = 88.2756
\]
\item L'équation de la droite est
\[
y = 2.0 x -2
\]
\item Il faut penser à faire la division à par $8$ avant d'utiliser le log car sinon, on ne peut pas utiliser la formule $\log(a^n) = n\times \log(a)$.
\[x = \frac{\log(3.25)}{\log(0.84)}\]
\end{enumerate}
\end{solution}
\begin{exercise}[subtitle={Restaurant}]
Un \emph{food truck}, ouvert le midi et le soir, propose deux types de formules :
\setlength\parindent{10mm}
\begin{itemize}
\item la formule \emph{Burger} ;
\item la formule \emph{Wok}.
\end{itemize}
\setlength\parindent{0mm}
\medskip
Le gérant a remarqué que 18\,\% de ses ventes ont lieu le midi. Le quart des ventes du midi correspondent à la formule \emph{Burger}, alors que 16\,\% des ventes du soir correspondent à la formule \emph{Wok}.
Le gérant se constitue un fichier en notant, pour chaque vente, la formule choisie et le moment de cette vente (midi ou soir).
On prélève une fiche de façon équiprobable. On définit les quatre évènements suivants:
\begin{enumerate}
\item $M$ : \og la fiche correspond à une vente du midi\fg{} ;
\item $S$ : \og la fiche correspond à une vente du soir\fg {};
\item $W$ : \og la fiche correspond à une formule \emph{Wok} \fg{} ;
\item $B$ : \og la fiche correspond à une formule \emph{Burger} \fg.
\end{enumerate}
\setlength\parindent{0mm}
\medskip
\begin{enumerate}
\item Recopier puis compléter l'arbre pondéré
\begin{center}
\begin{tikzpicture}[sloped]
\node {.}
child {node {$M$}
child {node {$W$}
edge from parent
node[above] {...}
}
child {node {$B$}
edge from parent
node[above] {...}
}
edge from parent
node[above] {...}
}
child[missing] {}
child { node {$S$}
child {node {$W$}
edge from parent
node[above] {...}
}
child {node {$B$}
edge from parent
node[above] {...}
}
edge from parent
node[above] {...}
} ;
\end{tikzpicture}
\end{center}
\item Calculer la probabilité de l'évènement $M \cap W$. Interpréter ce résultat dans le contexte de l'exercice.
\item Montrer que la probabilité que la fiche choisie corresponde à une formule \emph{Burger} est égale à $0.7338$.
\item On a prélevé une fiche correspondant à la formule \emph{Burger}. Quelle est la probabilité, arrondie au millième, que la vente ait eu lieu le soir?
\end{enumerate}
\end{exercise}
\begin{solution}
\begin{enumerate}
\item
\begin{center}
\begin{tikzpicture}[sloped]
\node {.}
child {node {$M$}
child {node {$W$}
edge from parent
node[above] {$0.75$}
}
child {node {$B$}
edge from parent
node[above] {$0.25$}
}
edge from parent
node[above] {$0.18$}
}
child[missing] {}
child { node {$S$}
child {node {$W$}
edge from parent
node[above] {$0.16$}
}
child {node {$B$}
edge from parent
node[above] {$0.84$}
}
edge from parent
node[above] {$0.82$}
} ;
\end{tikzpicture}
\end{center}
\item On calcule la probabilité que la vente soit un wok et ait eu lieu à midi
\[ P(M\cap W) = P(M) \times P_M(W) = 0.18 \times 0.75 = 0.135 \]
\item Probabilité que la vente soit un burger.
\[
P(B) = P(M\cap B) + P(S\cap B) = 0.18 \times 0.75 + 0.82 \times 0.16 = 0.7338
\]
\item On cherche à calculer la quantité $P_B(S)$. Pour cela on utilise la formule de Bayes
\[
P_B(S) = \frac{P(B\cap S)}{P(B)} = \frac{P_S(B) \times P(S)}{P(B)} = \frac{0.84\times 0.82}{0.7338} = 0.9386753883892068 \approx 0.939
\]
\end{enumerate}
\end{solution}
\begin{exercise}[subtitle={Continent plastique}]
\textit{Les quantités évoqués dans cette exercice sont générés au hasard et sont donc complètement farfelus.}
\medskip
Le \og continent de plastique\fg{} est la plus grande des plaques de déchets plastiques évoluant sur les océans. Elle occupe actuellement dans l'océan Pacifique une surface dont l'aire est évaluée à plus de $1,6$ million de km$^2$, entre Hawaï et la Californie.
En 2017, des scientifiques ont estimé qu'il y avait $2$ millions de tonnes de déchets plastiques qui était déversé chaque année dans les océans et que cette quantité augmentait de $24\n\%$ par chaque année.
On modélise l'évolution de la masse de ces déchets plastiques déversée chaque année, si rien n'est fait pour la réduire, par une suite géométrique $\left(u_n\right)$. L'arrondi au centième du terme $u_n$ représente la masse de ces déchets déversée chaque année, exprimée en million de tonnes, pour l'année $(2017 + n)$.
\medskip
\begin{enumerate}
\item Expliquer pourquoi la suite $u_n$ est géométrique?
\item Calculer $u_1$ et $u_2$.
\item Exprimer $u_n$ en fonction de $n$.
\item Au début de l'année 2017, il y avait $300$ millions de tonnes de déchets plastique. Calculer la quantité totale de déchets plastiques en 2030.
\item On souhaite déterminer en quelle année la masse totale de ces déchets plastiques aura pour la première fois augmenté de $50$\,\% par rapport à sa valeur de 2017.
\begin{enumerate}
\item Recopier et compléter l'algorithme ci-dessous pour que la variable $N$ contienne la réponse au problème posé.
\begin{center}
\begin{tabularx}{0.4\linewidth}{|X|}\hline
$N = 2017$\\
$U = 2$ \\
$S = 300 + U$ \\
while $S < 450$: \\
\hspace{1cm} $N = \ldots$\\
\hspace{1cm} $U = \ldots$\\
\hspace{1cm} $S = \ldots$\\
\hline
\end{tabularx}
\end{center}
\item Que contiennent les variables $S$, $U$ et $N$ après exécution de cet algorithme ?
Interpréter les résultats dans le contexte de l'exercice.
\end{enumerate}
\end{enumerate}
\end{exercise}
\begin{solution}
\begin{enumerate}
\item Une augmentation de $24\,\%$ revient à multiplier la quantité par $1.24$. La suite est donc bien géométrique. Son premier terme est $u_0 = 2$ et sa raison est $q = 1.24$
\item
\[
u_1 = u_0 * 1.24 = 2.48
\]
\[
u_2 = u_0 * 1.24^2 = 3.0752
\]
\item
\[
u_n = u_0 \times q^n = 2 \times 1.24^n
\]
\item On calcule la quantité totale déversée entre 2017 et 2030.
\[
\sum_{n = 0}^{13} u_n = u_0 \times \frac{1-q^{13}}{1-q} = 2 \times \frac{1 - 1.24^{13}}{1 - 1.24} = 128.22
\]
On en déduit la quantité totale de déchets en 2030
\[
300 + 128.22 = 428.22
\]
\item
\begin{enumerate}
\item ~
\begin{center}
\begin{tabularx}{0.4\linewidth}{|X|}\hline
$N \gets 2017$\\
$U \gets 2$ \\
$S \gets 300 + U$ \\
Tant que $S < 450$ \\
\hspace{1cm} $N \gets N + 1$\\
\hspace{1cm} $U \gets U * 1.24$\\
\hspace{1cm} $S \gets S + u$\\
Fin Tant que\\\hline
\end{tabularx}
\end{center}
\item \textit{Pas de correction automatisé}
\end{enumerate}
\end{enumerate}
\end{solution}
\end{document}
%%% Local Variables:
%%% mode: latex
%%% TeX-master: "master"
%%% End:

Binary file not shown.

View File

@ -0,0 +1,262 @@
\documentclass[a4paper,10pt]{article}
\usepackage{myXsim}
% Title Page
\title{DS8 \hfill AIOUAZ Ahmed}
\tribe{TST}
\date{\hfillÀ render pour le Mercredi 7 avril}
\xsimsetup{
solution/print = true
}
\begin{document}
\maketitle
\begin{exercise}[subtitle={Automatismes}]
\textit{Toutes les questions de cette exercice sont indépendantes et peuvent être répondus séparément}
\begin{enumerate}
\item De janvier à septembre, une quantité a augmenté de $20\,\%$. Faire un schéma pour représenter la situation puis calculer le taux d'évolution moyen mensuel.
\item Une quantité augmente de $20\,\%$ par ans. En 2020, elle est de 110\euro. Quelle était sa valeur en 2019? Faire un schéma pour représenter la situation.
\item Déterminer l'équation de la droite \\
\begin{tikzpicture}[xscale=0.8, yscale=0.5]
\tkzInit[xmin=-5,xmax=5,xstep=1,
ymin=-5,ymax=5,ystep=1]
\tkzGrid
\tkzAxeXY
\tkzFct[domain=-5:5,color=red,very thick]%
{3.0*\x -3};
\end{tikzpicture}
\item Résoudre l'équation $2 \times 0.07^x = 20$
\end{enumerate}
\end{exercise}
\begin{solution}
\begin{enumerate}
\item On veut partager cette évolution en 8 évolutions.
\[
\left(1 + \frac{20}{100}\right)^{\frac{1}{8}} = 1.0231
\]
Donc le taux d'évolution moyen est
\[
t_m = 1.0231 - 1 = 0.0230999999999999
\]
\item Coefficient multiplicateur pour revenir en arrière
\[
CM = (1 + \frac{20}{100})^{-1} = 0.8333
\]
On en déduit la quantité en 2019
\[
110 * 0.8333 = 91.66300000000001
\]
\item L'équation de la droite est
\[
y = 3.0 x -3
\]
\item Il faut penser à faire la division à par $2$ avant d'utiliser le log car sinon, on ne peut pas utiliser la formule $\log(a^n) = n\times \log(a)$.
\[x = \frac{\log(10.0)}{\log(0.07)}\]
\end{enumerate}
\end{solution}
\begin{exercise}[subtitle={Restaurant}]
Un \emph{food truck}, ouvert le midi et le soir, propose deux types de formules :
\setlength\parindent{10mm}
\begin{itemize}
\item la formule \emph{Burger} ;
\item la formule \emph{Wok}.
\end{itemize}
\setlength\parindent{0mm}
\medskip
Le gérant a remarqué que 9\,\% de ses ventes ont lieu le midi. Le quart des ventes du midi correspondent à la formule \emph{Burger}, alors que 3\,\% des ventes du soir correspondent à la formule \emph{Wok}.
Le gérant se constitue un fichier en notant, pour chaque vente, la formule choisie et le moment de cette vente (midi ou soir).
On prélève une fiche de façon équiprobable. On définit les quatre évènements suivants:
\begin{enumerate}
\item $M$ : \og la fiche correspond à une vente du midi\fg{} ;
\item $S$ : \og la fiche correspond à une vente du soir\fg {};
\item $W$ : \og la fiche correspond à une formule \emph{Wok} \fg{} ;
\item $B$ : \og la fiche correspond à une formule \emph{Burger} \fg.
\end{enumerate}
\setlength\parindent{0mm}
\medskip
\begin{enumerate}
\item Recopier puis compléter l'arbre pondéré
\begin{center}
\begin{tikzpicture}[sloped]
\node {.}
child {node {$M$}
child {node {$W$}
edge from parent
node[above] {...}
}
child {node {$B$}
edge from parent
node[above] {...}
}
edge from parent
node[above] {...}
}
child[missing] {}
child { node {$S$}
child {node {$W$}
edge from parent
node[above] {...}
}
child {node {$B$}
edge from parent
node[above] {...}
}
edge from parent
node[above] {...}
} ;
\end{tikzpicture}
\end{center}
\item Calculer la probabilité de l'évènement $M \cap W$. Interpréter ce résultat dans le contexte de l'exercice.
\item Montrer que la probabilité que la fiche choisie corresponde à une formule \emph{Burger} est égale à $0.9052$.
\item On a prélevé une fiche correspondant à la formule \emph{Burger}. Quelle est la probabilité, arrondie au millième, que la vente ait eu lieu le soir?
\end{enumerate}
\end{exercise}
\begin{solution}
\begin{enumerate}
\item
\begin{center}
\begin{tikzpicture}[sloped]
\node {.}
child {node {$M$}
child {node {$W$}
edge from parent
node[above] {$0.75$}
}
child {node {$B$}
edge from parent
node[above] {$0.25$}
}
edge from parent
node[above] {$0.09$}
}
child[missing] {}
child { node {$S$}
child {node {$W$}
edge from parent
node[above] {$0.03$}
}
child {node {$B$}
edge from parent
node[above] {$0.97$}
}
edge from parent
node[above] {$0.91$}
} ;
\end{tikzpicture}
\end{center}
\item On calcule la probabilité que la vente soit un wok et ait eu lieu à midi
\[ P(M\cap W) = P(M) \times P_M(W) = 0.09 \times 0.75 = 0.0675 \]
\item Probabilité que la vente soit un burger.
\[
P(B) = P(M\cap B) + P(S\cap B) = 0.09 \times 0.75 + 0.91 \times 0.03 = 0.9052
\]
\item On cherche à calculer la quantité $P_B(S)$. Pour cela on utilise la formule de Bayes
\[
P_B(S) = \frac{P(B\cap S)}{P(B)} = \frac{P_S(B) \times P(S)}{P(B)} = \frac{0.97\times 0.91}{0.9052} = 0.975143614670791 \approx 0.975
\]
\end{enumerate}
\end{solution}
\begin{exercise}[subtitle={Continent plastique}]
\textit{Les quantités évoqués dans cette exercice sont générés au hasard et sont donc complètement farfelus.}
\medskip
Le \og continent de plastique\fg{} est la plus grande des plaques de déchets plastiques évoluant sur les océans. Elle occupe actuellement dans l'océan Pacifique une surface dont l'aire est évaluée à plus de $1,6$ million de km$^2$, entre Hawaï et la Californie.
En 2017, des scientifiques ont estimé qu'il y avait $14$ millions de tonnes de déchets plastiques qui était déversé chaque année dans les océans et que cette quantité augmentait de $28\n\%$ par chaque année.
On modélise l'évolution de la masse de ces déchets plastiques déversée chaque année, si rien n'est fait pour la réduire, par une suite géométrique $\left(u_n\right)$. L'arrondi au centième du terme $u_n$ représente la masse de ces déchets déversée chaque année, exprimée en million de tonnes, pour l'année $(2017 + n)$.
\medskip
\begin{enumerate}
\item Expliquer pourquoi la suite $u_n$ est géométrique?
\item Calculer $u_1$ et $u_2$.
\item Exprimer $u_n$ en fonction de $n$.
\item Au début de l'année 2017, il y avait $300$ millions de tonnes de déchets plastique. Calculer la quantité totale de déchets plastiques en 2030.
\item On souhaite déterminer en quelle année la masse totale de ces déchets plastiques aura pour la première fois augmenté de $50$\,\% par rapport à sa valeur de 2017.
\begin{enumerate}
\item Recopier et compléter l'algorithme ci-dessous pour que la variable $N$ contienne la réponse au problème posé.
\begin{center}
\begin{tabularx}{0.4\linewidth}{|X|}\hline
$N = 2017$\\
$U = 14$ \\
$S = 300 + U$ \\
while $S < 450$: \\
\hspace{1cm} $N = \ldots$\\
\hspace{1cm} $U = \ldots$\\
\hspace{1cm} $S = \ldots$\\
\hline
\end{tabularx}
\end{center}
\item Que contiennent les variables $S$, $U$ et $N$ après exécution de cet algorithme ?
Interpréter les résultats dans le contexte de l'exercice.
\end{enumerate}
\end{enumerate}
\end{exercise}
\begin{solution}
\begin{enumerate}
\item Une augmentation de $28\,\%$ revient à multiplier la quantité par $1.28$. La suite est donc bien géométrique. Son premier terme est $u_0 = 14$ et sa raison est $q = 1.28$
\item
\[
u_1 = u_0 * 1.28 = 17.92
\]
\[
u_2 = u_0 * 1.28^2 = 22.9376
\]
\item
\[
u_n = u_0 \times q^n = 14 \times 1.28^n
\]
\item On calcule la quantité totale déversée entre 2017 et 2030.
\[
\sum_{n = 0}^{13} u_n = u_0 \times \frac{1-q^{13}}{1-q} = 14 \times \frac{1 - 1.28^{13}}{1 - 1.28} = 1187.94
\]
On en déduit la quantité totale de déchets en 2030
\[
300 + 1187.94 = 1487.94
\]
\item
\begin{enumerate}
\item ~
\begin{center}
\begin{tabularx}{0.4\linewidth}{|X|}\hline
$N \gets 2017$\\
$U \gets 14$ \\
$S \gets 300 + U$ \\
Tant que $S < 450$ \\
\hspace{1cm} $N \gets N + 1$\\
\hspace{1cm} $U \gets U * 1.28$\\
\hspace{1cm} $S \gets S + u$\\
Fin Tant que\\\hline
\end{tabularx}
\end{center}
\item \textit{Pas de correction automatisé}
\end{enumerate}
\end{enumerate}
\end{solution}
\end{document}
%%% Local Variables:
%%% mode: latex
%%% TeX-master: "master"
%%% End:

View File

@ -0,0 +1,262 @@
\documentclass[a4paper,10pt]{article}
\usepackage{myXsim}
% Title Page
\title{DS8 \hfill BAHBAH Zakaria}
\tribe{TST}
\date{\hfillÀ render pour le Mercredi 7 avril}
\xsimsetup{
solution/print = true
}
\begin{document}
\maketitle
\begin{exercise}[subtitle={Automatismes}]
\textit{Toutes les questions de cette exercice sont indépendantes et peuvent être répondus séparément}
\begin{enumerate}
\item De janvier à septembre, une quantité a augmenté de $28\,\%$. Faire un schéma pour représenter la situation puis calculer le taux d'évolution moyen mensuel.
\item Une quantité augmente de $28\,\%$ par ans. En 2020, elle est de 138\euro. Quelle était sa valeur en 2019? Faire un schéma pour représenter la situation.
\item Déterminer l'équation de la droite \\
\begin{tikzpicture}[xscale=0.8, yscale=0.5]
\tkzInit[xmin=-5,xmax=5,xstep=1,
ymin=-5,ymax=5,ystep=1]
\tkzGrid
\tkzAxeXY
\tkzFct[domain=-5:5,color=red,very thick]%
{1.0*\x -2};
\end{tikzpicture}
\item Résoudre l'équation $7 \times 0.54^x = 18$
\end{enumerate}
\end{exercise}
\begin{solution}
\begin{enumerate}
\item On veut partager cette évolution en 8 évolutions.
\[
\left(1 + \frac{28}{100}\right)^{\frac{1}{8}} = 1.0313
\]
Donc le taux d'évolution moyen est
\[
t_m = 1.0313 - 1 = 0.031300000000000106
\]
\item Coefficient multiplicateur pour revenir en arrière
\[
CM = (1 + \frac{28}{100})^{-1} = 0.7812
\]
On en déduit la quantité en 2019
\[
138 * 0.7812 = 107.8056
\]
\item L'équation de la droite est
\[
y = 1.0 x -2
\]
\item Il faut penser à faire la division à par $7$ avant d'utiliser le log car sinon, on ne peut pas utiliser la formule $\log(a^n) = n\times \log(a)$.
\[x = \frac{\log(2.57)}{\log(0.54)}\]
\end{enumerate}
\end{solution}
\begin{exercise}[subtitle={Restaurant}]
Un \emph{food truck}, ouvert le midi et le soir, propose deux types de formules :
\setlength\parindent{10mm}
\begin{itemize}
\item la formule \emph{Burger} ;
\item la formule \emph{Wok}.
\end{itemize}
\setlength\parindent{0mm}
\medskip
Le gérant a remarqué que 57\,\% de ses ventes ont lieu le midi. Le quart des ventes du midi correspondent à la formule \emph{Burger}, alors que 49\,\% des ventes du soir correspondent à la formule \emph{Wok}.
Le gérant se constitue un fichier en notant, pour chaque vente, la formule choisie et le moment de cette vente (midi ou soir).
On prélève une fiche de façon équiprobable. On définit les quatre évènements suivants:
\begin{enumerate}
\item $M$ : \og la fiche correspond à une vente du midi\fg{} ;
\item $S$ : \og la fiche correspond à une vente du soir\fg {};
\item $W$ : \og la fiche correspond à une formule \emph{Wok} \fg{} ;
\item $B$ : \og la fiche correspond à une formule \emph{Burger} \fg.
\end{enumerate}
\setlength\parindent{0mm}
\medskip
\begin{enumerate}
\item Recopier puis compléter l'arbre pondéré
\begin{center}
\begin{tikzpicture}[sloped]
\node {.}
child {node {$M$}
child {node {$W$}
edge from parent
node[above] {...}
}
child {node {$B$}
edge from parent
node[above] {...}
}
edge from parent
node[above] {...}
}
child[missing] {}
child { node {$S$}
child {node {$W$}
edge from parent
node[above] {...}
}
child {node {$B$}
edge from parent
node[above] {...}
}
edge from parent
node[above] {...}
} ;
\end{tikzpicture}
\end{center}
\item Calculer la probabilité de l'évènement $M \cap W$. Interpréter ce résultat dans le contexte de l'exercice.
\item Montrer que la probabilité que la fiche choisie corresponde à une formule \emph{Burger} est égale à $0.3592$.
\item On a prélevé une fiche correspondant à la formule \emph{Burger}. Quelle est la probabilité, arrondie au millième, que la vente ait eu lieu le soir?
\end{enumerate}
\end{exercise}
\begin{solution}
\begin{enumerate}
\item
\begin{center}
\begin{tikzpicture}[sloped]
\node {.}
child {node {$M$}
child {node {$W$}
edge from parent
node[above] {$0.75$}
}
child {node {$B$}
edge from parent
node[above] {$0.25$}
}
edge from parent
node[above] {$0.58$}
}
child[missing] {}
child { node {$S$}
child {node {$W$}
edge from parent
node[above] {$0.49$}
}
child {node {$B$}
edge from parent
node[above] {$0.51$}
}
edge from parent
node[above] {$0.42$}
} ;
\end{tikzpicture}
\end{center}
\item On calcule la probabilité que la vente soit un wok et ait eu lieu à midi
\[ P(M\cap W) = P(M) \times P_M(W) = 0.58 \times 0.75 = 0.435 \]
\item Probabilité que la vente soit un burger.
\[
P(B) = P(M\cap B) + P(S\cap B) = 0.58 \times 0.75 + 0.42 \times 0.49 = 0.3592
\]
\item On cherche à calculer la quantité $P_B(S)$. Pour cela on utilise la formule de Bayes
\[
P_B(S) = \frac{P(B\cap S)}{P(B)} = \frac{P_S(B) \times P(S)}{P(B)} = \frac{0.51\times 0.42}{0.3592} = 0.5963251670378619 \approx 0.596
\]
\end{enumerate}
\end{solution}
\begin{exercise}[subtitle={Continent plastique}]
\textit{Les quantités évoqués dans cette exercice sont générés au hasard et sont donc complètement farfelus.}
\medskip
Le \og continent de plastique\fg{} est la plus grande des plaques de déchets plastiques évoluant sur les océans. Elle occupe actuellement dans l'océan Pacifique une surface dont l'aire est évaluée à plus de $1,6$ million de km$^2$, entre Hawaï et la Californie.
En 2017, des scientifiques ont estimé qu'il y avait $9$ millions de tonnes de déchets plastiques qui était déversé chaque année dans les océans et que cette quantité augmentait de $26\n\%$ par chaque année.
On modélise l'évolution de la masse de ces déchets plastiques déversée chaque année, si rien n'est fait pour la réduire, par une suite géométrique $\left(u_n\right)$. L'arrondi au centième du terme $u_n$ représente la masse de ces déchets déversée chaque année, exprimée en million de tonnes, pour l'année $(2017 + n)$.
\medskip
\begin{enumerate}
\item Expliquer pourquoi la suite $u_n$ est géométrique?
\item Calculer $u_1$ et $u_2$.
\item Exprimer $u_n$ en fonction de $n$.
\item Au début de l'année 2017, il y avait $300$ millions de tonnes de déchets plastique. Calculer la quantité totale de déchets plastiques en 2030.
\item On souhaite déterminer en quelle année la masse totale de ces déchets plastiques aura pour la première fois augmenté de $50$\,\% par rapport à sa valeur de 2017.
\begin{enumerate}
\item Recopier et compléter l'algorithme ci-dessous pour que la variable $N$ contienne la réponse au problème posé.
\begin{center}
\begin{tabularx}{0.4\linewidth}{|X|}\hline
$N = 2017$\\
$U = 9$ \\
$S = 300 + U$ \\
while $S < 450$: \\
\hspace{1cm} $N = \ldots$\\
\hspace{1cm} $U = \ldots$\\
\hspace{1cm} $S = \ldots$\\
\hline
\end{tabularx}
\end{center}
\item Que contiennent les variables $S$, $U$ et $N$ après exécution de cet algorithme ?
Interpréter les résultats dans le contexte de l'exercice.
\end{enumerate}
\end{enumerate}
\end{exercise}
\begin{solution}
\begin{enumerate}
\item Une augmentation de $26\,\%$ revient à multiplier la quantité par $1.26$. La suite est donc bien géométrique. Son premier terme est $u_0 = 9$ et sa raison est $q = 1.26$
\item
\[
u_1 = u_0 * 1.26 = 11.34
\]
\[
u_2 = u_0 * 1.26^2 = 14.2884
\]
\item
\[
u_n = u_0 \times q^n = 9 \times 1.26^n
\]
\item On calcule la quantité totale déversée entre 2017 et 2030.
\[
\sum_{n = 0}^{13} u_n = u_0 \times \frac{1-q^{13}}{1-q} = 9 \times \frac{1 - 1.26^{13}}{1 - 1.26} = 663.76
\]
On en déduit la quantité totale de déchets en 2030
\[
300 + 663.76 = 963.76
\]
\item
\begin{enumerate}
\item ~
\begin{center}
\begin{tabularx}{0.4\linewidth}{|X|}\hline
$N \gets 2017$\\
$U \gets 9$ \\
$S \gets 300 + U$ \\
Tant que $S < 450$ \\
\hspace{1cm} $N \gets N + 1$\\
\hspace{1cm} $U \gets U * 1.26$\\
\hspace{1cm} $S \gets S + u$\\
Fin Tant que\\\hline
\end{tabularx}
\end{center}
\item \textit{Pas de correction automatisé}
\end{enumerate}
\end{enumerate}
\end{solution}
\end{document}
%%% Local Variables:
%%% mode: latex
%%% TeX-master: "master"
%%% End:

View File

@ -0,0 +1,262 @@
\documentclass[a4paper,10pt]{article}
\usepackage{myXsim}
% Title Page
\title{DS8 \hfill BALLOFFET Kenza}
\tribe{TST}
\date{\hfillÀ render pour le Mercredi 7 avril}
\xsimsetup{
solution/print = true
}
\begin{document}
\maketitle
\begin{exercise}[subtitle={Automatismes}]
\textit{Toutes les questions de cette exercice sont indépendantes et peuvent être répondus séparément}
\begin{enumerate}
\item De janvier à septembre, une quantité a augmenté de $12\,\%$. Faire un schéma pour représenter la situation puis calculer le taux d'évolution moyen mensuel.
\item Une quantité augmente de $12\,\%$ par ans. En 2020, elle est de 126\euro. Quelle était sa valeur en 2019? Faire un schéma pour représenter la situation.
\item Déterminer l'équation de la droite \\
\begin{tikzpicture}[xscale=0.8, yscale=0.5]
\tkzInit[xmin=-5,xmax=5,xstep=1,
ymin=-5,ymax=5,ystep=1]
\tkzGrid
\tkzAxeXY
\tkzFct[domain=-5:5,color=red,very thick]%
{2.6666666666666665*\x -4};
\end{tikzpicture}
\item Résoudre l'équation $8 \times 0.54^x = 18$
\end{enumerate}
\end{exercise}
\begin{solution}
\begin{enumerate}
\item On veut partager cette évolution en 8 évolutions.
\[
\left(1 + \frac{12}{100}\right)^{\frac{1}{8}} = 1.0143
\]
Donc le taux d'évolution moyen est
\[
t_m = 1.0143 - 1 = 0.01429999999999998
\]
\item Coefficient multiplicateur pour revenir en arrière
\[
CM = (1 + \frac{12}{100})^{-1} = 0.8929
\]
On en déduit la quantité en 2019
\[
126 * 0.8929 = 112.50540000000001
\]
\item L'équation de la droite est
\[
y = 2.6666666666666665 x -4
\]
\item Il faut penser à faire la division à par $8$ avant d'utiliser le log car sinon, on ne peut pas utiliser la formule $\log(a^n) = n\times \log(a)$.
\[x = \frac{\log(2.25)}{\log(0.54)}\]
\end{enumerate}
\end{solution}
\begin{exercise}[subtitle={Restaurant}]
Un \emph{food truck}, ouvert le midi et le soir, propose deux types de formules :
\setlength\parindent{10mm}
\begin{itemize}
\item la formule \emph{Burger} ;
\item la formule \emph{Wok}.
\end{itemize}
\setlength\parindent{0mm}
\medskip
Le gérant a remarqué que 94\,\% de ses ventes ont lieu le midi. Le quart des ventes du midi correspondent à la formule \emph{Burger}, alors que 37\,\% des ventes du soir correspondent à la formule \emph{Wok}.
Le gérant se constitue un fichier en notant, pour chaque vente, la formule choisie et le moment de cette vente (midi ou soir).
On prélève une fiche de façon équiprobable. On définit les quatre évènements suivants:
\begin{enumerate}
\item $M$ : \og la fiche correspond à une vente du midi\fg{} ;
\item $S$ : \og la fiche correspond à une vente du soir\fg {};
\item $W$ : \og la fiche correspond à une formule \emph{Wok} \fg{} ;
\item $B$ : \og la fiche correspond à une formule \emph{Burger} \fg.
\end{enumerate}
\setlength\parindent{0mm}
\medskip
\begin{enumerate}
\item Recopier puis compléter l'arbre pondéré
\begin{center}
\begin{tikzpicture}[sloped]
\node {.}
child {node {$M$}
child {node {$W$}
edge from parent
node[above] {...}
}
child {node {$B$}
edge from parent
node[above] {...}
}
edge from parent
node[above] {...}
}
child[missing] {}
child { node {$S$}
child {node {$W$}
edge from parent
node[above] {...}
}
child {node {$B$}
edge from parent
node[above] {...}
}
edge from parent
node[above] {...}
} ;
\end{tikzpicture}
\end{center}
\item Calculer la probabilité de l'évènement $M \cap W$. Interpréter ce résultat dans le contexte de l'exercice.
\item Montrer que la probabilité que la fiche choisie corresponde à une formule \emph{Burger} est égale à $0.2728$.
\item On a prélevé une fiche correspondant à la formule \emph{Burger}. Quelle est la probabilité, arrondie au millième, que la vente ait eu lieu le soir?
\end{enumerate}
\end{exercise}
\begin{solution}
\begin{enumerate}
\item
\begin{center}
\begin{tikzpicture}[sloped]
\node {.}
child {node {$M$}
child {node {$W$}
edge from parent
node[above] {$0.75$}
}
child {node {$B$}
edge from parent
node[above] {$0.25$}
}
edge from parent
node[above] {$0.94$}
}
child[missing] {}
child { node {$S$}
child {node {$W$}
edge from parent
node[above] {$0.37$}
}
child {node {$B$}
edge from parent
node[above] {$0.63$}
}
edge from parent
node[above] {$0.06$}
} ;
\end{tikzpicture}
\end{center}
\item On calcule la probabilité que la vente soit un wok et ait eu lieu à midi
\[ P(M\cap W) = P(M) \times P_M(W) = 0.94 \times 0.75 = 0.705 \]
\item Probabilité que la vente soit un burger.
\[
P(B) = P(M\cap B) + P(S\cap B) = 0.94 \times 0.75 + 0.06 \times 0.37 = 0.2728
\]
\item On cherche à calculer la quantité $P_B(S)$. Pour cela on utilise la formule de Bayes
\[
P_B(S) = \frac{P(B\cap S)}{P(B)} = \frac{P_S(B) \times P(S)}{P(B)} = \frac{0.63\times 0.06}{0.2728} = 0.13856304985337245 \approx 0.139
\]
\end{enumerate}
\end{solution}
\begin{exercise}[subtitle={Continent plastique}]
\textit{Les quantités évoqués dans cette exercice sont générés au hasard et sont donc complètement farfelus.}
\medskip
Le \og continent de plastique\fg{} est la plus grande des plaques de déchets plastiques évoluant sur les océans. Elle occupe actuellement dans l'océan Pacifique une surface dont l'aire est évaluée à plus de $1,6$ million de km$^2$, entre Hawaï et la Californie.
En 2017, des scientifiques ont estimé qu'il y avait $15$ millions de tonnes de déchets plastiques qui était déversé chaque année dans les océans et que cette quantité augmentait de $19\n\%$ par chaque année.
On modélise l'évolution de la masse de ces déchets plastiques déversée chaque année, si rien n'est fait pour la réduire, par une suite géométrique $\left(u_n\right)$. L'arrondi au centième du terme $u_n$ représente la masse de ces déchets déversée chaque année, exprimée en million de tonnes, pour l'année $(2017 + n)$.
\medskip
\begin{enumerate}
\item Expliquer pourquoi la suite $u_n$ est géométrique?
\item Calculer $u_1$ et $u_2$.
\item Exprimer $u_n$ en fonction de $n$.
\item Au début de l'année 2017, il y avait $300$ millions de tonnes de déchets plastique. Calculer la quantité totale de déchets plastiques en 2030.
\item On souhaite déterminer en quelle année la masse totale de ces déchets plastiques aura pour la première fois augmenté de $50$\,\% par rapport à sa valeur de 2017.
\begin{enumerate}
\item Recopier et compléter l'algorithme ci-dessous pour que la variable $N$ contienne la réponse au problème posé.
\begin{center}
\begin{tabularx}{0.4\linewidth}{|X|}\hline
$N = 2017$\\
$U = 15$ \\
$S = 300 + U$ \\
while $S < 450$: \\
\hspace{1cm} $N = \ldots$\\
\hspace{1cm} $U = \ldots$\\
\hspace{1cm} $S = \ldots$\\
\hline
\end{tabularx}
\end{center}
\item Que contiennent les variables $S$, $U$ et $N$ après exécution de cet algorithme ?
Interpréter les résultats dans le contexte de l'exercice.
\end{enumerate}
\end{enumerate}
\end{exercise}
\begin{solution}
\begin{enumerate}
\item Une augmentation de $19\,\%$ revient à multiplier la quantité par $1.19$. La suite est donc bien géométrique. Son premier terme est $u_0 = 15$ et sa raison est $q = 1.19$
\item
\[
u_1 = u_0 * 1.19 = 17.849999999999998
\]
\[
u_2 = u_0 * 1.19^2 = 21.2415
\]
\item
\[
u_n = u_0 \times q^n = 15 \times 1.19^n
\]
\item On calcule la quantité totale déversée entre 2017 et 2030.
\[
\sum_{n = 0}^{13} u_n = u_0 \times \frac{1-q^{13}}{1-q} = 15 \times \frac{1 - 1.19^{13}}{1 - 1.19} = 678.67
\]
On en déduit la quantité totale de déchets en 2030
\[
300 + 678.67 = 978.67
\]
\item
\begin{enumerate}
\item ~
\begin{center}
\begin{tabularx}{0.4\linewidth}{|X|}\hline
$N \gets 2017$\\
$U \gets 15$ \\
$S \gets 300 + U$ \\
Tant que $S < 450$ \\
\hspace{1cm} $N \gets N + 1$\\
\hspace{1cm} $U \gets U * 1.19$\\
\hspace{1cm} $S \gets S + u$\\
Fin Tant que\\\hline
\end{tabularx}
\end{center}
\item \textit{Pas de correction automatisé}
\end{enumerate}
\end{enumerate}
\end{solution}
\end{document}
%%% Local Variables:
%%% mode: latex
%%% TeX-master: "master"
%%% End:

View File

@ -0,0 +1,262 @@
\documentclass[a4paper,10pt]{article}
\usepackage{myXsim}
% Title Page
\title{DS8 \hfill BENHATTAL Chakir}
\tribe{TST}
\date{\hfillÀ render pour le Mercredi 7 avril}
\xsimsetup{
solution/print = true
}
\begin{document}
\maketitle
\begin{exercise}[subtitle={Automatismes}]
\textit{Toutes les questions de cette exercice sont indépendantes et peuvent être répondus séparément}
\begin{enumerate}
\item De janvier à septembre, une quantité a augmenté de $28\,\%$. Faire un schéma pour représenter la situation puis calculer le taux d'évolution moyen mensuel.
\item Une quantité augmente de $28\,\%$ par ans. En 2020, elle est de 111\euro. Quelle était sa valeur en 2019? Faire un schéma pour représenter la situation.
\item Déterminer l'équation de la droite \\
\begin{tikzpicture}[xscale=0.8, yscale=0.5]
\tkzInit[xmin=-5,xmax=5,xstep=1,
ymin=-5,ymax=5,ystep=1]
\tkzGrid
\tkzAxeXY
\tkzFct[domain=-5:5,color=red,very thick]%
{1.0*\x -1};
\end{tikzpicture}
\item Résoudre l'équation $4 \times 0.77^x = 25$
\end{enumerate}
\end{exercise}
\begin{solution}
\begin{enumerate}
\item On veut partager cette évolution en 8 évolutions.
\[
\left(1 + \frac{28}{100}\right)^{\frac{1}{8}} = 1.0313
\]
Donc le taux d'évolution moyen est
\[
t_m = 1.0313 - 1 = 0.031300000000000106
\]
\item Coefficient multiplicateur pour revenir en arrière
\[
CM = (1 + \frac{28}{100})^{-1} = 0.7812
\]
On en déduit la quantité en 2019
\[
111 * 0.7812 = 86.7132
\]
\item L'équation de la droite est
\[
y = 1.0 x -1
\]
\item Il faut penser à faire la division à par $4$ avant d'utiliser le log car sinon, on ne peut pas utiliser la formule $\log(a^n) = n\times \log(a)$.
\[x = \frac{\log(6.25)}{\log(0.77)}\]
\end{enumerate}
\end{solution}
\begin{exercise}[subtitle={Restaurant}]
Un \emph{food truck}, ouvert le midi et le soir, propose deux types de formules :
\setlength\parindent{10mm}
\begin{itemize}
\item la formule \emph{Burger} ;
\item la formule \emph{Wok}.
\end{itemize}
\setlength\parindent{0mm}
\medskip
Le gérant a remarqué que 89\,\% de ses ventes ont lieu le midi. Le quart des ventes du midi correspondent à la formule \emph{Burger}, alors que 43\,\% des ventes du soir correspondent à la formule \emph{Wok}.
Le gérant se constitue un fichier en notant, pour chaque vente, la formule choisie et le moment de cette vente (midi ou soir).
On prélève une fiche de façon équiprobable. On définit les quatre évènements suivants:
\begin{enumerate}
\item $M$ : \og la fiche correspond à une vente du midi\fg{} ;
\item $S$ : \og la fiche correspond à une vente du soir\fg {};
\item $W$ : \og la fiche correspond à une formule \emph{Wok} \fg{} ;
\item $B$ : \og la fiche correspond à une formule \emph{Burger} \fg.
\end{enumerate}
\setlength\parindent{0mm}
\medskip
\begin{enumerate}
\item Recopier puis compléter l'arbre pondéré
\begin{center}
\begin{tikzpicture}[sloped]
\node {.}
child {node {$M$}
child {node {$W$}
edge from parent
node[above] {...}
}
child {node {$B$}
edge from parent
node[above] {...}
}
edge from parent
node[above] {...}
}
child[missing] {}
child { node {$S$}
child {node {$W$}
edge from parent
node[above] {...}
}
child {node {$B$}
edge from parent
node[above] {...}
}
edge from parent
node[above] {...}
} ;
\end{tikzpicture}
\end{center}
\item Calculer la probabilité de l'évènement $M \cap W$. Interpréter ce résultat dans le contexte de l'exercice.
\item Montrer que la probabilité que la fiche choisie corresponde à une formule \emph{Burger} est égale à $0.2852$.
\item On a prélevé une fiche correspondant à la formule \emph{Burger}. Quelle est la probabilité, arrondie au millième, que la vente ait eu lieu le soir?
\end{enumerate}
\end{exercise}
\begin{solution}
\begin{enumerate}
\item
\begin{center}
\begin{tikzpicture}[sloped]
\node {.}
child {node {$M$}
child {node {$W$}
edge from parent
node[above] {$0.75$}
}
child {node {$B$}
edge from parent
node[above] {$0.25$}
}
edge from parent
node[above] {$0.89$}
}
child[missing] {}
child { node {$S$}
child {node {$W$}
edge from parent
node[above] {$0.43$}
}
child {node {$B$}
edge from parent
node[above] {$0.57$}
}
edge from parent
node[above] {$0.11$}
} ;
\end{tikzpicture}
\end{center}
\item On calcule la probabilité que la vente soit un wok et ait eu lieu à midi
\[ P(M\cap W) = P(M) \times P_M(W) = 0.89 \times 0.75 = 0.6675 \]
\item Probabilité que la vente soit un burger.
\[
P(B) = P(M\cap B) + P(S\cap B) = 0.89 \times 0.75 + 0.11 \times 0.43 = 0.2852
\]
\item On cherche à calculer la quantité $P_B(S)$. Pour cela on utilise la formule de Bayes
\[
P_B(S) = \frac{P(B\cap S)}{P(B)} = \frac{P_S(B) \times P(S)}{P(B)} = \frac{0.57\times 0.11}{0.2852} = 0.2198457223001402 \approx 0.22
\]
\end{enumerate}
\end{solution}
\begin{exercise}[subtitle={Continent plastique}]
\textit{Les quantités évoqués dans cette exercice sont générés au hasard et sont donc complètement farfelus.}
\medskip
Le \og continent de plastique\fg{} est la plus grande des plaques de déchets plastiques évoluant sur les océans. Elle occupe actuellement dans l'océan Pacifique une surface dont l'aire est évaluée à plus de $1,6$ million de km$^2$, entre Hawaï et la Californie.
En 2017, des scientifiques ont estimé qu'il y avait $6$ millions de tonnes de déchets plastiques qui était déversé chaque année dans les océans et que cette quantité augmentait de $13\n\%$ par chaque année.
On modélise l'évolution de la masse de ces déchets plastiques déversée chaque année, si rien n'est fait pour la réduire, par une suite géométrique $\left(u_n\right)$. L'arrondi au centième du terme $u_n$ représente la masse de ces déchets déversée chaque année, exprimée en million de tonnes, pour l'année $(2017 + n)$.
\medskip
\begin{enumerate}
\item Expliquer pourquoi la suite $u_n$ est géométrique?
\item Calculer $u_1$ et $u_2$.
\item Exprimer $u_n$ en fonction de $n$.
\item Au début de l'année 2017, il y avait $300$ millions de tonnes de déchets plastique. Calculer la quantité totale de déchets plastiques en 2030.
\item On souhaite déterminer en quelle année la masse totale de ces déchets plastiques aura pour la première fois augmenté de $50$\,\% par rapport à sa valeur de 2017.
\begin{enumerate}
\item Recopier et compléter l'algorithme ci-dessous pour que la variable $N$ contienne la réponse au problème posé.
\begin{center}
\begin{tabularx}{0.4\linewidth}{|X|}\hline
$N = 2017$\\
$U = 6$ \\
$S = 300 + U$ \\
while $S < 450$: \\
\hspace{1cm} $N = \ldots$\\
\hspace{1cm} $U = \ldots$\\
\hspace{1cm} $S = \ldots$\\
\hline
\end{tabularx}
\end{center}
\item Que contiennent les variables $S$, $U$ et $N$ après exécution de cet algorithme ?
Interpréter les résultats dans le contexte de l'exercice.
\end{enumerate}
\end{enumerate}
\end{exercise}
\begin{solution}
\begin{enumerate}
\item Une augmentation de $13\,\%$ revient à multiplier la quantité par $1.13$. La suite est donc bien géométrique. Son premier terme est $u_0 = 6$ et sa raison est $q = 1.13$
\item
\[
u_1 = u_0 * 1.13 = 6.779999999999999
\]
\[
u_2 = u_0 * 1.13^2 = 7.6614
\]
\item
\[
u_n = u_0 \times q^n = 6 \times 1.13^n
\]
\item On calcule la quantité totale déversée entre 2017 et 2030.
\[
\sum_{n = 0}^{13} u_n = u_0 \times \frac{1-q^{13}}{1-q} = 6 \times \frac{1 - 1.13^{13}}{1 - 1.13} = 179.91
\]
On en déduit la quantité totale de déchets en 2030
\[
300 + 179.91 = 479.90999999999997
\]
\item
\begin{enumerate}
\item ~
\begin{center}
\begin{tabularx}{0.4\linewidth}{|X|}\hline
$N \gets 2017$\\
$U \gets 6$ \\
$S \gets 300 + U$ \\
Tant que $S < 450$ \\
\hspace{1cm} $N \gets N + 1$\\
\hspace{1cm} $U \gets U * 1.13$\\
\hspace{1cm} $S \gets S + u$\\
Fin Tant que\\\hline
\end{tabularx}
\end{center}
\item \textit{Pas de correction automatisé}
\end{enumerate}
\end{enumerate}
\end{solution}
\end{document}
%%% Local Variables:
%%% mode: latex
%%% TeX-master: "master"
%%% End:

View File

@ -0,0 +1,262 @@
\documentclass[a4paper,10pt]{article}
\usepackage{myXsim}
% Title Page
\title{DS8 \hfill CLAIN Avinash}
\tribe{TST}
\date{\hfillÀ render pour le Mercredi 7 avril}
\xsimsetup{
solution/print = true
}
\begin{document}
\maketitle
\begin{exercise}[subtitle={Automatismes}]
\textit{Toutes les questions de cette exercice sont indépendantes et peuvent être répondus séparément}
\begin{enumerate}
\item De janvier à septembre, une quantité a augmenté de $15\,\%$. Faire un schéma pour représenter la situation puis calculer le taux d'évolution moyen mensuel.
\item Une quantité augmente de $15\,\%$ par ans. En 2020, elle est de 112\euro. Quelle était sa valeur en 2019? Faire un schéma pour représenter la situation.
\item Déterminer l'équation de la droite \\
\begin{tikzpicture}[xscale=0.8, yscale=0.5]
\tkzInit[xmin=-5,xmax=5,xstep=1,
ymin=-5,ymax=5,ystep=1]
\tkzGrid
\tkzAxeXY
\tkzFct[domain=-5:5,color=red,very thick]%
{4.0*\x -4};
\end{tikzpicture}
\item Résoudre l'équation $6 \times 0.77^x = 23$
\end{enumerate}
\end{exercise}
\begin{solution}
\begin{enumerate}
\item On veut partager cette évolution en 8 évolutions.
\[
\left(1 + \frac{15}{100}\right)^{\frac{1}{8}} = 1.0176
\]
Donc le taux d'évolution moyen est
\[
t_m = 1.0176 - 1 = 0.01760000000000006
\]
\item Coefficient multiplicateur pour revenir en arrière
\[
CM = (1 + \frac{15}{100})^{-1} = 0.8696
\]
On en déduit la quantité en 2019
\[
112 * 0.8696 = 97.3952
\]
\item L'équation de la droite est
\[
y = 4.0 x -4
\]
\item Il faut penser à faire la division à par $6$ avant d'utiliser le log car sinon, on ne peut pas utiliser la formule $\log(a^n) = n\times \log(a)$.
\[x = \frac{\log(3.83)}{\log(0.77)}\]
\end{enumerate}
\end{solution}
\begin{exercise}[subtitle={Restaurant}]
Un \emph{food truck}, ouvert le midi et le soir, propose deux types de formules :
\setlength\parindent{10mm}
\begin{itemize}
\item la formule \emph{Burger} ;
\item la formule \emph{Wok}.
\end{itemize}
\setlength\parindent{0mm}
\medskip
Le gérant a remarqué que 92\,\% de ses ventes ont lieu le midi. Le quart des ventes du midi correspondent à la formule \emph{Burger}, alors que 54\,\% des ventes du soir correspondent à la formule \emph{Wok}.
Le gérant se constitue un fichier en notant, pour chaque vente, la formule choisie et le moment de cette vente (midi ou soir).
On prélève une fiche de façon équiprobable. On définit les quatre évènements suivants:
\begin{enumerate}
\item $M$ : \og la fiche correspond à une vente du midi\fg{} ;
\item $S$ : \og la fiche correspond à une vente du soir\fg {};
\item $W$ : \og la fiche correspond à une formule \emph{Wok} \fg{} ;
\item $B$ : \og la fiche correspond à une formule \emph{Burger} \fg.
\end{enumerate}
\setlength\parindent{0mm}
\medskip
\begin{enumerate}
\item Recopier puis compléter l'arbre pondéré
\begin{center}
\begin{tikzpicture}[sloped]
\node {.}
child {node {$M$}
child {node {$W$}
edge from parent
node[above] {...}
}
child {node {$B$}
edge from parent
node[above] {...}
}
edge from parent
node[above] {...}
}
child[missing] {}
child { node {$S$}
child {node {$W$}
edge from parent
node[above] {...}
}
child {node {$B$}
edge from parent
node[above] {...}
}
edge from parent
node[above] {...}
} ;
\end{tikzpicture}
\end{center}
\item Calculer la probabilité de l'évènement $M \cap W$. Interpréter ce résultat dans le contexte de l'exercice.
\item Montrer que la probabilité que la fiche choisie corresponde à une formule \emph{Burger} est égale à $0.2668$.
\item On a prélevé une fiche correspondant à la formule \emph{Burger}. Quelle est la probabilité, arrondie au millième, que la vente ait eu lieu le soir?
\end{enumerate}
\end{exercise}
\begin{solution}
\begin{enumerate}
\item
\begin{center}
\begin{tikzpicture}[sloped]
\node {.}
child {node {$M$}
child {node {$W$}
edge from parent
node[above] {$0.75$}
}
child {node {$B$}
edge from parent
node[above] {$0.25$}
}
edge from parent
node[above] {$0.92$}
}
child[missing] {}
child { node {$S$}
child {node {$W$}
edge from parent
node[above] {$0.54$}
}
child {node {$B$}
edge from parent
node[above] {$0.46$}
}
edge from parent
node[above] {$0.08$}
} ;
\end{tikzpicture}
\end{center}
\item On calcule la probabilité que la vente soit un wok et ait eu lieu à midi
\[ P(M\cap W) = P(M) \times P_M(W) = 0.92 \times 0.75 = 0.69 \]
\item Probabilité que la vente soit un burger.
\[
P(B) = P(M\cap B) + P(S\cap B) = 0.92 \times 0.75 + 0.08 \times 0.54 = 0.2668
\]
\item On cherche à calculer la quantité $P_B(S)$. Pour cela on utilise la formule de Bayes
\[
P_B(S) = \frac{P(B\cap S)}{P(B)} = \frac{P_S(B) \times P(S)}{P(B)} = \frac{0.46\times 0.08}{0.2668} = 0.13793103448275862 \approx 0.138
\]
\end{enumerate}
\end{solution}
\begin{exercise}[subtitle={Continent plastique}]
\textit{Les quantités évoqués dans cette exercice sont générés au hasard et sont donc complètement farfelus.}
\medskip
Le \og continent de plastique\fg{} est la plus grande des plaques de déchets plastiques évoluant sur les océans. Elle occupe actuellement dans l'océan Pacifique une surface dont l'aire est évaluée à plus de $1,6$ million de km$^2$, entre Hawaï et la Californie.
En 2017, des scientifiques ont estimé qu'il y avait $18$ millions de tonnes de déchets plastiques qui était déversé chaque année dans les océans et que cette quantité augmentait de $21\n\%$ par chaque année.
On modélise l'évolution de la masse de ces déchets plastiques déversée chaque année, si rien n'est fait pour la réduire, par une suite géométrique $\left(u_n\right)$. L'arrondi au centième du terme $u_n$ représente la masse de ces déchets déversée chaque année, exprimée en million de tonnes, pour l'année $(2017 + n)$.
\medskip
\begin{enumerate}
\item Expliquer pourquoi la suite $u_n$ est géométrique?
\item Calculer $u_1$ et $u_2$.
\item Exprimer $u_n$ en fonction de $n$.
\item Au début de l'année 2017, il y avait $300$ millions de tonnes de déchets plastique. Calculer la quantité totale de déchets plastiques en 2030.
\item On souhaite déterminer en quelle année la masse totale de ces déchets plastiques aura pour la première fois augmenté de $50$\,\% par rapport à sa valeur de 2017.
\begin{enumerate}
\item Recopier et compléter l'algorithme ci-dessous pour que la variable $N$ contienne la réponse au problème posé.
\begin{center}
\begin{tabularx}{0.4\linewidth}{|X|}\hline
$N = 2017$\\
$U = 18$ \\
$S = 300 + U$ \\
while $S < 450$: \\
\hspace{1cm} $N = \ldots$\\
\hspace{1cm} $U = \ldots$\\
\hspace{1cm} $S = \ldots$\\
\hline
\end{tabularx}
\end{center}
\item Que contiennent les variables $S$, $U$ et $N$ après exécution de cet algorithme ?
Interpréter les résultats dans le contexte de l'exercice.
\end{enumerate}
\end{enumerate}
\end{exercise}
\begin{solution}
\begin{enumerate}
\item Une augmentation de $21\,\%$ revient à multiplier la quantité par $1.21$. La suite est donc bien géométrique. Son premier terme est $u_0 = 18$ et sa raison est $q = 1.21$
\item
\[
u_1 = u_0 * 1.21 = 21.78
\]
\[
u_2 = u_0 * 1.21^2 = 26.3538
\]
\item
\[
u_n = u_0 \times q^n = 18 \times 1.21^n
\]
\item On calcule la quantité totale déversée entre 2017 et 2030.
\[
\sum_{n = 0}^{13} u_n = u_0 \times \frac{1-q^{13}}{1-q} = 18 \times \frac{1 - 1.21^{13}}{1 - 1.21} = 935.84
\]
On en déduit la quantité totale de déchets en 2030
\[
300 + 935.84 = 1235.8400000000001
\]
\item
\begin{enumerate}
\item ~
\begin{center}
\begin{tabularx}{0.4\linewidth}{|X|}\hline
$N \gets 2017$\\
$U \gets 18$ \\
$S \gets 300 + U$ \\
Tant que $S < 450$ \\
\hspace{1cm} $N \gets N + 1$\\
\hspace{1cm} $U \gets U * 1.21$\\
\hspace{1cm} $S \gets S + u$\\
Fin Tant que\\\hline
\end{tabularx}
\end{center}
\item \textit{Pas de correction automatisé}
\end{enumerate}
\end{enumerate}
\end{solution}
\end{document}
%%% Local Variables:
%%% mode: latex
%%% TeX-master: "master"
%%% End:

View File

@ -0,0 +1,262 @@
\documentclass[a4paper,10pt]{article}
\usepackage{myXsim}
% Title Page
\title{DS8 \hfill COLASSI Alexis}
\tribe{TST}
\date{\hfillÀ render pour le Mercredi 7 avril}
\xsimsetup{
solution/print = true
}
\begin{document}
\maketitle
\begin{exercise}[subtitle={Automatismes}]
\textit{Toutes les questions de cette exercice sont indépendantes et peuvent être répondus séparément}
\begin{enumerate}
\item De janvier à septembre, une quantité a augmenté de $27\,\%$. Faire un schéma pour représenter la situation puis calculer le taux d'évolution moyen mensuel.
\item Une quantité augmente de $27\,\%$ par ans. En 2020, elle est de 116\euro. Quelle était sa valeur en 2019? Faire un schéma pour représenter la situation.
\item Déterminer l'équation de la droite \\
\begin{tikzpicture}[xscale=0.8, yscale=0.5]
\tkzInit[xmin=-5,xmax=5,xstep=1,
ymin=-5,ymax=5,ystep=1]
\tkzGrid
\tkzAxeXY
\tkzFct[domain=-5:5,color=red,very thick]%
{2.0*\x -2};
\end{tikzpicture}
\item Résoudre l'équation $7 \times 0.23^x = 16$
\end{enumerate}
\end{exercise}
\begin{solution}
\begin{enumerate}
\item On veut partager cette évolution en 8 évolutions.
\[
\left(1 + \frac{27}{100}\right)^{\frac{1}{8}} = 1.0303
\]
Donc le taux d'évolution moyen est
\[
t_m = 1.0303 - 1 = 0.030299999999999994
\]
\item Coefficient multiplicateur pour revenir en arrière
\[
CM = (1 + \frac{27}{100})^{-1} = 0.7874
\]
On en déduit la quantité en 2019
\[
116 * 0.7874 = 91.3384
\]
\item L'équation de la droite est
\[
y = 2.0 x -2
\]
\item Il faut penser à faire la division à par $7$ avant d'utiliser le log car sinon, on ne peut pas utiliser la formule $\log(a^n) = n\times \log(a)$.
\[x = \frac{\log(2.29)}{\log(0.23)}\]
\end{enumerate}
\end{solution}
\begin{exercise}[subtitle={Restaurant}]
Un \emph{food truck}, ouvert le midi et le soir, propose deux types de formules :
\setlength\parindent{10mm}
\begin{itemize}
\item la formule \emph{Burger} ;
\item la formule \emph{Wok}.
\end{itemize}
\setlength\parindent{0mm}
\medskip
Le gérant a remarqué que 53\,\% de ses ventes ont lieu le midi. Le quart des ventes du midi correspondent à la formule \emph{Burger}, alors que 26\,\% des ventes du soir correspondent à la formule \emph{Wok}.
Le gérant se constitue un fichier en notant, pour chaque vente, la formule choisie et le moment de cette vente (midi ou soir).
On prélève une fiche de façon équiprobable. On définit les quatre évènements suivants:
\begin{enumerate}
\item $M$ : \og la fiche correspond à une vente du midi\fg{} ;
\item $S$ : \og la fiche correspond à une vente du soir\fg {};
\item $W$ : \og la fiche correspond à une formule \emph{Wok} \fg{} ;
\item $B$ : \og la fiche correspond à une formule \emph{Burger} \fg.
\end{enumerate}
\setlength\parindent{0mm}
\medskip
\begin{enumerate}
\item Recopier puis compléter l'arbre pondéré
\begin{center}
\begin{tikzpicture}[sloped]
\node {.}
child {node {$M$}
child {node {$W$}
edge from parent
node[above] {...}
}
child {node {$B$}
edge from parent
node[above] {...}
}
edge from parent
node[above] {...}
}
child[missing] {}
child { node {$S$}
child {node {$W$}
edge from parent
node[above] {...}
}
child {node {$B$}
edge from parent
node[above] {...}
}
edge from parent
node[above] {...}
} ;
\end{tikzpicture}
\end{center}
\item Calculer la probabilité de l'évènement $M \cap W$. Interpréter ce résultat dans le contexte de l'exercice.
\item Montrer que la probabilité que la fiche choisie corresponde à une formule \emph{Burger} est égale à $0.4803$.
\item On a prélevé une fiche correspondant à la formule \emph{Burger}. Quelle est la probabilité, arrondie au millième, que la vente ait eu lieu le soir?
\end{enumerate}
\end{exercise}
\begin{solution}
\begin{enumerate}
\item
\begin{center}
\begin{tikzpicture}[sloped]
\node {.}
child {node {$M$}
child {node {$W$}
edge from parent
node[above] {$0.75$}
}
child {node {$B$}
edge from parent
node[above] {$0.25$}
}
edge from parent
node[above] {$0.53$}
}
child[missing] {}
child { node {$S$}
child {node {$W$}
edge from parent
node[above] {$0.26$}
}
child {node {$B$}
edge from parent
node[above] {$0.74$}
}
edge from parent
node[above] {$0.47$}
} ;
\end{tikzpicture}
\end{center}
\item On calcule la probabilité que la vente soit un wok et ait eu lieu à midi
\[ P(M\cap W) = P(M) \times P_M(W) = 0.53 \times 0.75 = 0.3975 \]
\item Probabilité que la vente soit un burger.
\[
P(B) = P(M\cap B) + P(S\cap B) = 0.53 \times 0.75 + 0.47 \times 0.26 = 0.4803
\]
\item On cherche à calculer la quantité $P_B(S)$. Pour cela on utilise la formule de Bayes
\[
P_B(S) = \frac{P(B\cap S)}{P(B)} = \frac{P_S(B) \times P(S)}{P(B)} = \frac{0.74\times 0.47}{0.4803} = 0.7241307516135749 \approx 0.724
\]
\end{enumerate}
\end{solution}
\begin{exercise}[subtitle={Continent plastique}]
\textit{Les quantités évoqués dans cette exercice sont générés au hasard et sont donc complètement farfelus.}
\medskip
Le \og continent de plastique\fg{} est la plus grande des plaques de déchets plastiques évoluant sur les océans. Elle occupe actuellement dans l'océan Pacifique une surface dont l'aire est évaluée à plus de $1,6$ million de km$^2$, entre Hawaï et la Californie.
En 2017, des scientifiques ont estimé qu'il y avait $3$ millions de tonnes de déchets plastiques qui était déversé chaque année dans les océans et que cette quantité augmentait de $17\n\%$ par chaque année.
On modélise l'évolution de la masse de ces déchets plastiques déversée chaque année, si rien n'est fait pour la réduire, par une suite géométrique $\left(u_n\right)$. L'arrondi au centième du terme $u_n$ représente la masse de ces déchets déversée chaque année, exprimée en million de tonnes, pour l'année $(2017 + n)$.
\medskip
\begin{enumerate}
\item Expliquer pourquoi la suite $u_n$ est géométrique?
\item Calculer $u_1$ et $u_2$.
\item Exprimer $u_n$ en fonction de $n$.
\item Au début de l'année 2017, il y avait $300$ millions de tonnes de déchets plastique. Calculer la quantité totale de déchets plastiques en 2030.
\item On souhaite déterminer en quelle année la masse totale de ces déchets plastiques aura pour la première fois augmenté de $50$\,\% par rapport à sa valeur de 2017.
\begin{enumerate}
\item Recopier et compléter l'algorithme ci-dessous pour que la variable $N$ contienne la réponse au problème posé.
\begin{center}
\begin{tabularx}{0.4\linewidth}{|X|}\hline
$N = 2017$\\
$U = 3$ \\
$S = 300 + U$ \\
while $S < 450$: \\
\hspace{1cm} $N = \ldots$\\
\hspace{1cm} $U = \ldots$\\
\hspace{1cm} $S = \ldots$\\
\hline
\end{tabularx}
\end{center}
\item Que contiennent les variables $S$, $U$ et $N$ après exécution de cet algorithme ?
Interpréter les résultats dans le contexte de l'exercice.
\end{enumerate}
\end{enumerate}
\end{exercise}
\begin{solution}
\begin{enumerate}
\item Une augmentation de $17\,\%$ revient à multiplier la quantité par $1.17$. La suite est donc bien géométrique. Son premier terme est $u_0 = 3$ et sa raison est $q = 1.17$
\item
\[
u_1 = u_0 * 1.17 = 3.51
\]
\[
u_2 = u_0 * 1.17^2 = 4.1067
\]
\item
\[
u_n = u_0 \times q^n = 3 \times 1.17^n
\]
\item On calcule la quantité totale déversée entre 2017 et 2030.
\[
\sum_{n = 0}^{13} u_n = u_0 \times \frac{1-q^{13}}{1-q} = 3 \times \frac{1 - 1.17^{13}}{1 - 1.17} = 118.21
\]
On en déduit la quantité totale de déchets en 2030
\[
300 + 118.21 = 418.21
\]
\item
\begin{enumerate}
\item ~
\begin{center}
\begin{tabularx}{0.4\linewidth}{|X|}\hline
$N \gets 2017$\\
$U \gets 3$ \\
$S \gets 300 + U$ \\
Tant que $S < 450$ \\
\hspace{1cm} $N \gets N + 1$\\
\hspace{1cm} $U \gets U * 1.17$\\
\hspace{1cm} $S \gets S + u$\\
Fin Tant que\\\hline
\end{tabularx}
\end{center}
\item \textit{Pas de correction automatisé}
\end{enumerate}
\end{enumerate}
\end{solution}
\end{document}
%%% Local Variables:
%%% mode: latex
%%% TeX-master: "master"
%%% End:

View File

@ -0,0 +1,262 @@
\documentclass[a4paper,10pt]{article}
\usepackage{myXsim}
% Title Page
\title{DS8 \hfill COUBAT Alexis}
\tribe{TST}
\date{\hfillÀ render pour le Mercredi 7 avril}
\xsimsetup{
solution/print = true
}
\begin{document}
\maketitle
\begin{exercise}[subtitle={Automatismes}]
\textit{Toutes les questions de cette exercice sont indépendantes et peuvent être répondus séparément}
\begin{enumerate}
\item De janvier à septembre, une quantité a augmenté de $16\,\%$. Faire un schéma pour représenter la situation puis calculer le taux d'évolution moyen mensuel.
\item Une quantité augmente de $16\,\%$ par ans. En 2020, elle est de 120\euro. Quelle était sa valeur en 2019? Faire un schéma pour représenter la situation.
\item Déterminer l'équation de la droite \\
\begin{tikzpicture}[xscale=0.8, yscale=0.5]
\tkzInit[xmin=-5,xmax=5,xstep=1,
ymin=-5,ymax=5,ystep=1]
\tkzGrid
\tkzAxeXY
\tkzFct[domain=-5:5,color=red,very thick]%
{4.0*\x -4};
\end{tikzpicture}
\item Résoudre l'équation $10 \times 0.06^x = 34$
\end{enumerate}
\end{exercise}
\begin{solution}
\begin{enumerate}
\item On veut partager cette évolution en 8 évolutions.
\[
\left(1 + \frac{16}{100}\right)^{\frac{1}{8}} = 1.0187
\]
Donc le taux d'évolution moyen est
\[
t_m = 1.0187 - 1 = 0.01869999999999994
\]
\item Coefficient multiplicateur pour revenir en arrière
\[
CM = (1 + \frac{16}{100})^{-1} = 0.8621
\]
On en déduit la quantité en 2019
\[
120 * 0.8621 = 103.452
\]
\item L'équation de la droite est
\[
y = 4.0 x -4
\]
\item Il faut penser à faire la division à par $10$ avant d'utiliser le log car sinon, on ne peut pas utiliser la formule $\log(a^n) = n\times \log(a)$.
\[x = \frac{\log(3.4)}{\log(0.06)}\]
\end{enumerate}
\end{solution}
\begin{exercise}[subtitle={Restaurant}]
Un \emph{food truck}, ouvert le midi et le soir, propose deux types de formules :
\setlength\parindent{10mm}
\begin{itemize}
\item la formule \emph{Burger} ;
\item la formule \emph{Wok}.
\end{itemize}
\setlength\parindent{0mm}
\medskip
Le gérant a remarqué que 1\,\% de ses ventes ont lieu le midi. Le quart des ventes du midi correspondent à la formule \emph{Burger}, alors que 92\,\% des ventes du soir correspondent à la formule \emph{Wok}.
Le gérant se constitue un fichier en notant, pour chaque vente, la formule choisie et le moment de cette vente (midi ou soir).
On prélève une fiche de façon équiprobable. On définit les quatre évènements suivants:
\begin{enumerate}
\item $M$ : \og la fiche correspond à une vente du midi\fg{} ;
\item $S$ : \og la fiche correspond à une vente du soir\fg {};
\item $W$ : \og la fiche correspond à une formule \emph{Wok} \fg{} ;
\item $B$ : \og la fiche correspond à une formule \emph{Burger} \fg.
\end{enumerate}
\setlength\parindent{0mm}
\medskip
\begin{enumerate}
\item Recopier puis compléter l'arbre pondéré
\begin{center}
\begin{tikzpicture}[sloped]
\node {.}
child {node {$M$}
child {node {$W$}
edge from parent
node[above] {...}
}
child {node {$B$}
edge from parent
node[above] {...}
}
edge from parent
node[above] {...}
}
child[missing] {}
child { node {$S$}
child {node {$W$}
edge from parent
node[above] {...}
}
child {node {$B$}
edge from parent
node[above] {...}
}
edge from parent
node[above] {...}
} ;
\end{tikzpicture}
\end{center}
\item Calculer la probabilité de l'évènement $M \cap W$. Interpréter ce résultat dans le contexte de l'exercice.
\item Montrer que la probabilité que la fiche choisie corresponde à une formule \emph{Burger} est égale à $0.0817$.
\item On a prélevé une fiche correspondant à la formule \emph{Burger}. Quelle est la probabilité, arrondie au millième, que la vente ait eu lieu le soir?
\end{enumerate}
\end{exercise}
\begin{solution}
\begin{enumerate}
\item
\begin{center}
\begin{tikzpicture}[sloped]
\node {.}
child {node {$M$}
child {node {$W$}
edge from parent
node[above] {$0.75$}
}
child {node {$B$}
edge from parent
node[above] {$0.25$}
}
edge from parent
node[above] {$0.01$}
}
child[missing] {}
child { node {$S$}
child {node {$W$}
edge from parent
node[above] {$0.92$}
}
child {node {$B$}
edge from parent
node[above] {$0.08$}
}
edge from parent
node[above] {$0.99$}
} ;
\end{tikzpicture}
\end{center}
\item On calcule la probabilité que la vente soit un wok et ait eu lieu à midi
\[ P(M\cap W) = P(M) \times P_M(W) = 0.01 \times 0.75 = 0.0075 \]
\item Probabilité que la vente soit un burger.
\[
P(B) = P(M\cap B) + P(S\cap B) = 0.01 \times 0.75 + 0.99 \times 0.92 = 0.0817
\]
\item On cherche à calculer la quantité $P_B(S)$. Pour cela on utilise la formule de Bayes
\[
P_B(S) = \frac{P(B\cap S)}{P(B)} = \frac{P_S(B) \times P(S)}{P(B)} = \frac{0.08\times 0.99}{0.0817} = 0.9694002447980418 \approx 0.969
\]
\end{enumerate}
\end{solution}
\begin{exercise}[subtitle={Continent plastique}]
\textit{Les quantités évoqués dans cette exercice sont générés au hasard et sont donc complètement farfelus.}
\medskip
Le \og continent de plastique\fg{} est la plus grande des plaques de déchets plastiques évoluant sur les océans. Elle occupe actuellement dans l'océan Pacifique une surface dont l'aire est évaluée à plus de $1,6$ million de km$^2$, entre Hawaï et la Californie.
En 2017, des scientifiques ont estimé qu'il y avait $12$ millions de tonnes de déchets plastiques qui était déversé chaque année dans les océans et que cette quantité augmentait de $14\n\%$ par chaque année.
On modélise l'évolution de la masse de ces déchets plastiques déversée chaque année, si rien n'est fait pour la réduire, par une suite géométrique $\left(u_n\right)$. L'arrondi au centième du terme $u_n$ représente la masse de ces déchets déversée chaque année, exprimée en million de tonnes, pour l'année $(2017 + n)$.
\medskip
\begin{enumerate}
\item Expliquer pourquoi la suite $u_n$ est géométrique?
\item Calculer $u_1$ et $u_2$.
\item Exprimer $u_n$ en fonction de $n$.
\item Au début de l'année 2017, il y avait $300$ millions de tonnes de déchets plastique. Calculer la quantité totale de déchets plastiques en 2030.
\item On souhaite déterminer en quelle année la masse totale de ces déchets plastiques aura pour la première fois augmenté de $50$\,\% par rapport à sa valeur de 2017.
\begin{enumerate}
\item Recopier et compléter l'algorithme ci-dessous pour que la variable $N$ contienne la réponse au problème posé.
\begin{center}
\begin{tabularx}{0.4\linewidth}{|X|}\hline
$N = 2017$\\
$U = 12$ \\
$S = 300 + U$ \\
while $S < 450$: \\
\hspace{1cm} $N = \ldots$\\
\hspace{1cm} $U = \ldots$\\
\hspace{1cm} $S = \ldots$\\
\hline
\end{tabularx}
\end{center}
\item Que contiennent les variables $S$, $U$ et $N$ après exécution de cet algorithme ?
Interpréter les résultats dans le contexte de l'exercice.
\end{enumerate}
\end{enumerate}
\end{exercise}
\begin{solution}
\begin{enumerate}
\item Une augmentation de $14\,\%$ revient à multiplier la quantité par $1.1400000000000001$. La suite est donc bien géométrique. Son premier terme est $u_0 = 12$ et sa raison est $q = 1.1400000000000001$
\item
\[
u_1 = u_0 * 1.1400000000000001 = 13.680000000000001
\]
\[
u_2 = u_0 * 1.1400000000000001^2 = 15.5952
\]
\item
\[
u_n = u_0 \times q^n = 12 \times 1.1400000000000001^n
\]
\item On calcule la quantité totale déversée entre 2017 et 2030.
\[
\sum_{n = 0}^{13} u_n = u_0 \times \frac{1-q^{13}}{1-q} = 12 \times \frac{1 - 1.1400000000000001^{13}}{1 - 1.1400000000000001} = 385.06
\]
On en déduit la quantité totale de déchets en 2030
\[
300 + 385.06 = 685.06
\]
\item
\begin{enumerate}
\item ~
\begin{center}
\begin{tabularx}{0.4\linewidth}{|X|}\hline
$N \gets 2017$\\
$U \gets 12$ \\
$S \gets 300 + U$ \\
Tant que $S < 450$ \\
\hspace{1cm} $N \gets N + 1$\\
\hspace{1cm} $U \gets U * 1.1400000000000001$\\
\hspace{1cm} $S \gets S + u$\\
Fin Tant que\\\hline
\end{tabularx}
\end{center}
\item \textit{Pas de correction automatisé}
\end{enumerate}
\end{enumerate}
\end{solution}
\end{document}
%%% Local Variables:
%%% mode: latex
%%% TeX-master: "master"
%%% End:

View File

@ -0,0 +1,262 @@
\documentclass[a4paper,10pt]{article}
\usepackage{myXsim}
% Title Page
\title{DS8 \hfill COULLON Anis}
\tribe{TST}
\date{\hfillÀ render pour le Mercredi 7 avril}
\xsimsetup{
solution/print = true
}
\begin{document}
\maketitle
\begin{exercise}[subtitle={Automatismes}]
\textit{Toutes les questions de cette exercice sont indépendantes et peuvent être répondus séparément}
\begin{enumerate}
\item De janvier à septembre, une quantité a augmenté de $14\,\%$. Faire un schéma pour représenter la situation puis calculer le taux d'évolution moyen mensuel.
\item Une quantité augmente de $14\,\%$ par ans. En 2020, elle est de 125\euro. Quelle était sa valeur en 2019? Faire un schéma pour représenter la situation.
\item Déterminer l'équation de la droite \\
\begin{tikzpicture}[xscale=0.8, yscale=0.5]
\tkzInit[xmin=-5,xmax=5,xstep=1,
ymin=-5,ymax=5,ystep=1]
\tkzGrid
\tkzAxeXY
\tkzFct[domain=-5:5,color=red,very thick]%
{0.5*\x -1};
\end{tikzpicture}
\item Résoudre l'équation $6 \times 0.82^x = 19$
\end{enumerate}
\end{exercise}
\begin{solution}
\begin{enumerate}
\item On veut partager cette évolution en 8 évolutions.
\[
\left(1 + \frac{14}{100}\right)^{\frac{1}{8}} = 1.0165
\]
Donc le taux d'évolution moyen est
\[
t_m = 1.0165 - 1 = 0.01649999999999996
\]
\item Coefficient multiplicateur pour revenir en arrière
\[
CM = (1 + \frac{14}{100})^{-1} = 0.8772
\]
On en déduit la quantité en 2019
\[
125 * 0.8772 = 109.64999999999999
\]
\item L'équation de la droite est
\[
y = 0.5 x -1
\]
\item Il faut penser à faire la division à par $6$ avant d'utiliser le log car sinon, on ne peut pas utiliser la formule $\log(a^n) = n\times \log(a)$.
\[x = \frac{\log(3.17)}{\log(0.82)}\]
\end{enumerate}
\end{solution}
\begin{exercise}[subtitle={Restaurant}]
Un \emph{food truck}, ouvert le midi et le soir, propose deux types de formules :
\setlength\parindent{10mm}
\begin{itemize}
\item la formule \emph{Burger} ;
\item la formule \emph{Wok}.
\end{itemize}
\setlength\parindent{0mm}
\medskip
Le gérant a remarqué que 30\,\% de ses ventes ont lieu le midi. Le quart des ventes du midi correspondent à la formule \emph{Burger}, alors que 27\,\% des ventes du soir correspondent à la formule \emph{Wok}.
Le gérant se constitue un fichier en notant, pour chaque vente, la formule choisie et le moment de cette vente (midi ou soir).
On prélève une fiche de façon équiprobable. On définit les quatre évènements suivants:
\begin{enumerate}
\item $M$ : \og la fiche correspond à une vente du midi\fg{} ;
\item $S$ : \og la fiche correspond à une vente du soir\fg {};
\item $W$ : \og la fiche correspond à une formule \emph{Wok} \fg{} ;
\item $B$ : \og la fiche correspond à une formule \emph{Burger} \fg.
\end{enumerate}
\setlength\parindent{0mm}
\medskip
\begin{enumerate}
\item Recopier puis compléter l'arbre pondéré
\begin{center}
\begin{tikzpicture}[sloped]
\node {.}
child {node {$M$}
child {node {$W$}
edge from parent
node[above] {...}
}
child {node {$B$}
edge from parent
node[above] {...}
}
edge from parent
node[above] {...}
}
child[missing] {}
child { node {$S$}
child {node {$W$}
edge from parent
node[above] {...}
}
child {node {$B$}
edge from parent
node[above] {...}
}
edge from parent
node[above] {...}
} ;
\end{tikzpicture}
\end{center}
\item Calculer la probabilité de l'évènement $M \cap W$. Interpréter ce résultat dans le contexte de l'exercice.
\item Montrer que la probabilité que la fiche choisie corresponde à une formule \emph{Burger} est égale à $0.586$.
\item On a prélevé une fiche correspondant à la formule \emph{Burger}. Quelle est la probabilité, arrondie au millième, que la vente ait eu lieu le soir?
\end{enumerate}
\end{exercise}
\begin{solution}
\begin{enumerate}
\item
\begin{center}
\begin{tikzpicture}[sloped]
\node {.}
child {node {$M$}
child {node {$W$}
edge from parent
node[above] {$0.75$}
}
child {node {$B$}
edge from parent
node[above] {$0.25$}
}
edge from parent
node[above] {$0.3$}
}
child[missing] {}
child { node {$S$}
child {node {$W$}
edge from parent
node[above] {$0.27$}
}
child {node {$B$}
edge from parent
node[above] {$0.73$}
}
edge from parent
node[above] {$0.7$}
} ;
\end{tikzpicture}
\end{center}
\item On calcule la probabilité que la vente soit un wok et ait eu lieu à midi
\[ P(M\cap W) = P(M) \times P_M(W) = 0.3 \times 0.75 = 0.225 \]
\item Probabilité que la vente soit un burger.
\[
P(B) = P(M\cap B) + P(S\cap B) = 0.3 \times 0.75 + 0.7 \times 0.27 = 0.586
\]
\item On cherche à calculer la quantité $P_B(S)$. Pour cela on utilise la formule de Bayes
\[
P_B(S) = \frac{P(B\cap S)}{P(B)} = \frac{P_S(B) \times P(S)}{P(B)} = \frac{0.73\times 0.7}{0.586} = 0.8720136518771332 \approx 0.872
\]
\end{enumerate}
\end{solution}
\begin{exercise}[subtitle={Continent plastique}]
\textit{Les quantités évoqués dans cette exercice sont générés au hasard et sont donc complètement farfelus.}
\medskip
Le \og continent de plastique\fg{} est la plus grande des plaques de déchets plastiques évoluant sur les océans. Elle occupe actuellement dans l'océan Pacifique une surface dont l'aire est évaluée à plus de $1,6$ million de km$^2$, entre Hawaï et la Californie.
En 2017, des scientifiques ont estimé qu'il y avait $14$ millions de tonnes de déchets plastiques qui était déversé chaque année dans les océans et que cette quantité augmentait de $25\n\%$ par chaque année.
On modélise l'évolution de la masse de ces déchets plastiques déversée chaque année, si rien n'est fait pour la réduire, par une suite géométrique $\left(u_n\right)$. L'arrondi au centième du terme $u_n$ représente la masse de ces déchets déversée chaque année, exprimée en million de tonnes, pour l'année $(2017 + n)$.
\medskip
\begin{enumerate}
\item Expliquer pourquoi la suite $u_n$ est géométrique?
\item Calculer $u_1$ et $u_2$.
\item Exprimer $u_n$ en fonction de $n$.
\item Au début de l'année 2017, il y avait $300$ millions de tonnes de déchets plastique. Calculer la quantité totale de déchets plastiques en 2030.
\item On souhaite déterminer en quelle année la masse totale de ces déchets plastiques aura pour la première fois augmenté de $50$\,\% par rapport à sa valeur de 2017.
\begin{enumerate}
\item Recopier et compléter l'algorithme ci-dessous pour que la variable $N$ contienne la réponse au problème posé.
\begin{center}
\begin{tabularx}{0.4\linewidth}{|X|}\hline
$N = 2017$\\
$U = 14$ \\
$S = 300 + U$ \\
while $S < 450$: \\
\hspace{1cm} $N = \ldots$\\
\hspace{1cm} $U = \ldots$\\
\hspace{1cm} $S = \ldots$\\
\hline
\end{tabularx}
\end{center}
\item Que contiennent les variables $S$, $U$ et $N$ après exécution de cet algorithme ?
Interpréter les résultats dans le contexte de l'exercice.
\end{enumerate}
\end{enumerate}
\end{exercise}
\begin{solution}
\begin{enumerate}
\item Une augmentation de $25\,\%$ revient à multiplier la quantité par $1.25$. La suite est donc bien géométrique. Son premier terme est $u_0 = 14$ et sa raison est $q = 1.25$
\item
\[
u_1 = u_0 * 1.25 = 17.5
\]
\[
u_2 = u_0 * 1.25^2 = 21.875
\]
\item
\[
u_n = u_0 \times q^n = 14 \times 1.25^n
\]
\item On calcule la quantité totale déversée entre 2017 et 2030.
\[
\sum_{n = 0}^{13} u_n = u_0 \times \frac{1-q^{13}}{1-q} = 14 \times \frac{1 - 1.25^{13}}{1 - 1.25} = 962.63
\]
On en déduit la quantité totale de déchets en 2030
\[
300 + 962.63 = 1262.63
\]
\item
\begin{enumerate}
\item ~
\begin{center}
\begin{tabularx}{0.4\linewidth}{|X|}\hline
$N \gets 2017$\\
$U \gets 14$ \\
$S \gets 300 + U$ \\
Tant que $S < 450$ \\
\hspace{1cm} $N \gets N + 1$\\
\hspace{1cm} $U \gets U * 1.25$\\
\hspace{1cm} $S \gets S + u$\\
Fin Tant que\\\hline
\end{tabularx}
\end{center}
\item \textit{Pas de correction automatisé}
\end{enumerate}
\end{enumerate}
\end{solution}
\end{document}
%%% Local Variables:
%%% mode: latex
%%% TeX-master: "master"
%%% End:

View File

@ -0,0 +1,262 @@
\documentclass[a4paper,10pt]{article}
\usepackage{myXsim}
% Title Page
\title{DS8 \hfill DINGER Sölen}
\tribe{TST}
\date{\hfillÀ render pour le Mercredi 7 avril}
\xsimsetup{
solution/print = true
}
\begin{document}
\maketitle
\begin{exercise}[subtitle={Automatismes}]
\textit{Toutes les questions de cette exercice sont indépendantes et peuvent être répondus séparément}
\begin{enumerate}
\item De janvier à septembre, une quantité a augmenté de $30\,\%$. Faire un schéma pour représenter la situation puis calculer le taux d'évolution moyen mensuel.
\item Une quantité augmente de $30\,\%$ par ans. En 2020, elle est de 141\euro. Quelle était sa valeur en 2019? Faire un schéma pour représenter la situation.
\item Déterminer l'équation de la droite \\
\begin{tikzpicture}[xscale=0.8, yscale=0.5]
\tkzInit[xmin=-5,xmax=5,xstep=1,
ymin=-5,ymax=5,ystep=1]
\tkzGrid
\tkzAxeXY
\tkzFct[domain=-5:5,color=red,very thick]%
{2.0*\x -3};
\end{tikzpicture}
\item Résoudre l'équation $5 \times 0.8^x = 11$
\end{enumerate}
\end{exercise}
\begin{solution}
\begin{enumerate}
\item On veut partager cette évolution en 8 évolutions.
\[
\left(1 + \frac{30}{100}\right)^{\frac{1}{8}} = 1.0333
\]
Donc le taux d'évolution moyen est
\[
t_m = 1.0333 - 1 = 0.03330000000000011
\]
\item Coefficient multiplicateur pour revenir en arrière
\[
CM = (1 + \frac{30}{100})^{-1} = 0.7692
\]
On en déduit la quantité en 2019
\[
141 * 0.7692 = 108.4572
\]
\item L'équation de la droite est
\[
y = 2.0 x -3
\]
\item Il faut penser à faire la division à par $5$ avant d'utiliser le log car sinon, on ne peut pas utiliser la formule $\log(a^n) = n\times \log(a)$.
\[x = \frac{\log(2.2)}{\log(0.8)}\]
\end{enumerate}
\end{solution}
\begin{exercise}[subtitle={Restaurant}]
Un \emph{food truck}, ouvert le midi et le soir, propose deux types de formules :
\setlength\parindent{10mm}
\begin{itemize}
\item la formule \emph{Burger} ;
\item la formule \emph{Wok}.
\end{itemize}
\setlength\parindent{0mm}
\medskip
Le gérant a remarqué que 57\,\% de ses ventes ont lieu le midi. Le quart des ventes du midi correspondent à la formule \emph{Burger}, alors que 90\,\% des ventes du soir correspondent à la formule \emph{Wok}.
Le gérant se constitue un fichier en notant, pour chaque vente, la formule choisie et le moment de cette vente (midi ou soir).
On prélève une fiche de façon équiprobable. On définit les quatre évènements suivants:
\begin{enumerate}
\item $M$ : \og la fiche correspond à une vente du midi\fg{} ;
\item $S$ : \og la fiche correspond à une vente du soir\fg {};
\item $W$ : \og la fiche correspond à une formule \emph{Wok} \fg{} ;
\item $B$ : \og la fiche correspond à une formule \emph{Burger} \fg.
\end{enumerate}
\setlength\parindent{0mm}
\medskip
\begin{enumerate}
\item Recopier puis compléter l'arbre pondéré
\begin{center}
\begin{tikzpicture}[sloped]
\node {.}
child {node {$M$}
child {node {$W$}
edge from parent
node[above] {...}
}
child {node {$B$}
edge from parent
node[above] {...}
}
edge from parent
node[above] {...}
}
child[missing] {}
child { node {$S$}
child {node {$W$}
edge from parent
node[above] {...}
}
child {node {$B$}
edge from parent
node[above] {...}
}
edge from parent
node[above] {...}
} ;
\end{tikzpicture}
\end{center}
\item Calculer la probabilité de l'évènement $M \cap W$. Interpréter ce résultat dans le contexte de l'exercice.
\item Montrer que la probabilité que la fiche choisie corresponde à une formule \emph{Burger} est égale à $0.187$.
\item On a prélevé une fiche correspondant à la formule \emph{Burger}. Quelle est la probabilité, arrondie au millième, que la vente ait eu lieu le soir?
\end{enumerate}
\end{exercise}
\begin{solution}
\begin{enumerate}
\item
\begin{center}
\begin{tikzpicture}[sloped]
\node {.}
child {node {$M$}
child {node {$W$}
edge from parent
node[above] {$0.75$}
}
child {node {$B$}
edge from parent
node[above] {$0.25$}
}
edge from parent
node[above] {$0.58$}
}
child[missing] {}
child { node {$S$}
child {node {$W$}
edge from parent
node[above] {$0.9$}
}
child {node {$B$}
edge from parent
node[above] {$0.1$}
}
edge from parent
node[above] {$0.42$}
} ;
\end{tikzpicture}
\end{center}
\item On calcule la probabilité que la vente soit un wok et ait eu lieu à midi
\[ P(M\cap W) = P(M) \times P_M(W) = 0.58 \times 0.75 = 0.435 \]
\item Probabilité que la vente soit un burger.
\[
P(B) = P(M\cap B) + P(S\cap B) = 0.58 \times 0.75 + 0.42 \times 0.9 = 0.187
\]
\item On cherche à calculer la quantité $P_B(S)$. Pour cela on utilise la formule de Bayes
\[
P_B(S) = \frac{P(B\cap S)}{P(B)} = \frac{P_S(B) \times P(S)}{P(B)} = \frac{0.1\times 0.42}{0.187} = 0.22459893048128343 \approx 0.225
\]
\end{enumerate}
\end{solution}
\begin{exercise}[subtitle={Continent plastique}]
\textit{Les quantités évoqués dans cette exercice sont générés au hasard et sont donc complètement farfelus.}
\medskip
Le \og continent de plastique\fg{} est la plus grande des plaques de déchets plastiques évoluant sur les océans. Elle occupe actuellement dans l'océan Pacifique une surface dont l'aire est évaluée à plus de $1,6$ million de km$^2$, entre Hawaï et la Californie.
En 2017, des scientifiques ont estimé qu'il y avait $14$ millions de tonnes de déchets plastiques qui était déversé chaque année dans les océans et que cette quantité augmentait de $19\n\%$ par chaque année.
On modélise l'évolution de la masse de ces déchets plastiques déversée chaque année, si rien n'est fait pour la réduire, par une suite géométrique $\left(u_n\right)$. L'arrondi au centième du terme $u_n$ représente la masse de ces déchets déversée chaque année, exprimée en million de tonnes, pour l'année $(2017 + n)$.
\medskip
\begin{enumerate}
\item Expliquer pourquoi la suite $u_n$ est géométrique?
\item Calculer $u_1$ et $u_2$.
\item Exprimer $u_n$ en fonction de $n$.
\item Au début de l'année 2017, il y avait $300$ millions de tonnes de déchets plastique. Calculer la quantité totale de déchets plastiques en 2030.
\item On souhaite déterminer en quelle année la masse totale de ces déchets plastiques aura pour la première fois augmenté de $50$\,\% par rapport à sa valeur de 2017.
\begin{enumerate}
\item Recopier et compléter l'algorithme ci-dessous pour que la variable $N$ contienne la réponse au problème posé.
\begin{center}
\begin{tabularx}{0.4\linewidth}{|X|}\hline
$N = 2017$\\
$U = 14$ \\
$S = 300 + U$ \\
while $S < 450$: \\
\hspace{1cm} $N = \ldots$\\
\hspace{1cm} $U = \ldots$\\
\hspace{1cm} $S = \ldots$\\
\hline
\end{tabularx}
\end{center}
\item Que contiennent les variables $S$, $U$ et $N$ après exécution de cet algorithme ?
Interpréter les résultats dans le contexte de l'exercice.
\end{enumerate}
\end{enumerate}
\end{exercise}
\begin{solution}
\begin{enumerate}
\item Une augmentation de $19\,\%$ revient à multiplier la quantité par $1.19$. La suite est donc bien géométrique. Son premier terme est $u_0 = 14$ et sa raison est $q = 1.19$
\item
\[
u_1 = u_0 * 1.19 = 16.66
\]
\[
u_2 = u_0 * 1.19^2 = 19.8254
\]
\item
\[
u_n = u_0 \times q^n = 14 \times 1.19^n
\]
\item On calcule la quantité totale déversée entre 2017 et 2030.
\[
\sum_{n = 0}^{13} u_n = u_0 \times \frac{1-q^{13}}{1-q} = 14 \times \frac{1 - 1.19^{13}}{1 - 1.19} = 633.42
\]
On en déduit la quantité totale de déchets en 2030
\[
300 + 633.42 = 933.42
\]
\item
\begin{enumerate}
\item ~
\begin{center}
\begin{tabularx}{0.4\linewidth}{|X|}\hline
$N \gets 2017$\\
$U \gets 14$ \\
$S \gets 300 + U$ \\
Tant que $S < 450$ \\
\hspace{1cm} $N \gets N + 1$\\
\hspace{1cm} $U \gets U * 1.19$\\
\hspace{1cm} $S \gets S + u$\\
Fin Tant que\\\hline
\end{tabularx}
\end{center}
\item \textit{Pas de correction automatisé}
\end{enumerate}
\end{enumerate}
\end{solution}
\end{document}
%%% Local Variables:
%%% mode: latex
%%% TeX-master: "master"
%%% End:

View File

@ -0,0 +1,262 @@
\documentclass[a4paper,10pt]{article}
\usepackage{myXsim}
% Title Page
\title{DS8 \hfill EYRAUD Cynthia}
\tribe{TST}
\date{\hfillÀ render pour le Mercredi 7 avril}
\xsimsetup{
solution/print = true
}
\begin{document}
\maketitle
\begin{exercise}[subtitle={Automatismes}]
\textit{Toutes les questions de cette exercice sont indépendantes et peuvent être répondus séparément}
\begin{enumerate}
\item De janvier à septembre, une quantité a augmenté de $29\,\%$. Faire un schéma pour représenter la situation puis calculer le taux d'évolution moyen mensuel.
\item Une quantité augmente de $29\,\%$ par ans. En 2020, elle est de 130\euro. Quelle était sa valeur en 2019? Faire un schéma pour représenter la situation.
\item Déterminer l'équation de la droite \\
\begin{tikzpicture}[xscale=0.8, yscale=0.5]
\tkzInit[xmin=-5,xmax=5,xstep=1,
ymin=-5,ymax=5,ystep=1]
\tkzGrid
\tkzAxeXY
\tkzFct[domain=-5:5,color=red,very thick]%
{4.0*\x -4};
\end{tikzpicture}
\item Résoudre l'équation $6 \times 0.48^x = 28$
\end{enumerate}
\end{exercise}
\begin{solution}
\begin{enumerate}
\item On veut partager cette évolution en 8 évolutions.
\[
\left(1 + \frac{29}{100}\right)^{\frac{1}{8}} = 1.0323
\]
Donc le taux d'évolution moyen est
\[
t_m = 1.0323 - 1 = 0.032299999999999995
\]
\item Coefficient multiplicateur pour revenir en arrière
\[
CM = (1 + \frac{29}{100})^{-1} = 0.7752
\]
On en déduit la quantité en 2019
\[
130 * 0.7752 = 100.776
\]
\item L'équation de la droite est
\[
y = 4.0 x -4
\]
\item Il faut penser à faire la division à par $6$ avant d'utiliser le log car sinon, on ne peut pas utiliser la formule $\log(a^n) = n\times \log(a)$.
\[x = \frac{\log(4.67)}{\log(0.48)}\]
\end{enumerate}
\end{solution}
\begin{exercise}[subtitle={Restaurant}]
Un \emph{food truck}, ouvert le midi et le soir, propose deux types de formules :
\setlength\parindent{10mm}
\begin{itemize}
\item la formule \emph{Burger} ;
\item la formule \emph{Wok}.
\end{itemize}
\setlength\parindent{0mm}
\medskip
Le gérant a remarqué que 3\,\% de ses ventes ont lieu le midi. Le quart des ventes du midi correspondent à la formule \emph{Burger}, alors que 33\,\% des ventes du soir correspondent à la formule \emph{Wok}.
Le gérant se constitue un fichier en notant, pour chaque vente, la formule choisie et le moment de cette vente (midi ou soir).
On prélève une fiche de façon équiprobable. On définit les quatre évènements suivants:
\begin{enumerate}
\item $M$ : \og la fiche correspond à une vente du midi\fg{} ;
\item $S$ : \og la fiche correspond à une vente du soir\fg {};
\item $W$ : \og la fiche correspond à une formule \emph{Wok} \fg{} ;
\item $B$ : \og la fiche correspond à une formule \emph{Burger} \fg.
\end{enumerate}
\setlength\parindent{0mm}
\medskip
\begin{enumerate}
\item Recopier puis compléter l'arbre pondéré
\begin{center}
\begin{tikzpicture}[sloped]
\node {.}
child {node {$M$}
child {node {$W$}
edge from parent
node[above] {...}
}
child {node {$B$}
edge from parent
node[above] {...}
}
edge from parent
node[above] {...}
}
child[missing] {}
child { node {$S$}
child {node {$W$}
edge from parent
node[above] {...}
}
child {node {$B$}
edge from parent
node[above] {...}
}
edge from parent
node[above] {...}
} ;
\end{tikzpicture}
\end{center}
\item Calculer la probabilité de l'évènement $M \cap W$. Interpréter ce résultat dans le contexte de l'exercice.
\item Montrer que la probabilité que la fiche choisie corresponde à une formule \emph{Burger} est égale à $0.6574$.
\item On a prélevé une fiche correspondant à la formule \emph{Burger}. Quelle est la probabilité, arrondie au millième, que la vente ait eu lieu le soir?
\end{enumerate}
\end{exercise}
\begin{solution}
\begin{enumerate}
\item
\begin{center}
\begin{tikzpicture}[sloped]
\node {.}
child {node {$M$}
child {node {$W$}
edge from parent
node[above] {$0.75$}
}
child {node {$B$}
edge from parent
node[above] {$0.25$}
}
edge from parent
node[above] {$0.03$}
}
child[missing] {}
child { node {$S$}
child {node {$W$}
edge from parent
node[above] {$0.33$}
}
child {node {$B$}
edge from parent
node[above] {$0.67$}
}
edge from parent
node[above] {$0.97$}
} ;
\end{tikzpicture}
\end{center}
\item On calcule la probabilité que la vente soit un wok et ait eu lieu à midi
\[ P(M\cap W) = P(M) \times P_M(W) = 0.03 \times 0.75 = 0.0225 \]
\item Probabilité que la vente soit un burger.
\[
P(B) = P(M\cap B) + P(S\cap B) = 0.03 \times 0.75 + 0.97 \times 0.33 = 0.6574
\]
\item On cherche à calculer la quantité $P_B(S)$. Pour cela on utilise la formule de Bayes
\[
P_B(S) = \frac{P(B\cap S)}{P(B)} = \frac{P_S(B) \times P(S)}{P(B)} = \frac{0.67\times 0.97}{0.6574} = 0.9885914207484029 \approx 0.989
\]
\end{enumerate}
\end{solution}
\begin{exercise}[subtitle={Continent plastique}]
\textit{Les quantités évoqués dans cette exercice sont générés au hasard et sont donc complètement farfelus.}
\medskip
Le \og continent de plastique\fg{} est la plus grande des plaques de déchets plastiques évoluant sur les océans. Elle occupe actuellement dans l'océan Pacifique une surface dont l'aire est évaluée à plus de $1,6$ million de km$^2$, entre Hawaï et la Californie.
En 2017, des scientifiques ont estimé qu'il y avait $18$ millions de tonnes de déchets plastiques qui était déversé chaque année dans les océans et que cette quantité augmentait de $25\n\%$ par chaque année.
On modélise l'évolution de la masse de ces déchets plastiques déversée chaque année, si rien n'est fait pour la réduire, par une suite géométrique $\left(u_n\right)$. L'arrondi au centième du terme $u_n$ représente la masse de ces déchets déversée chaque année, exprimée en million de tonnes, pour l'année $(2017 + n)$.
\medskip
\begin{enumerate}
\item Expliquer pourquoi la suite $u_n$ est géométrique?
\item Calculer $u_1$ et $u_2$.
\item Exprimer $u_n$ en fonction de $n$.
\item Au début de l'année 2017, il y avait $300$ millions de tonnes de déchets plastique. Calculer la quantité totale de déchets plastiques en 2030.
\item On souhaite déterminer en quelle année la masse totale de ces déchets plastiques aura pour la première fois augmenté de $50$\,\% par rapport à sa valeur de 2017.
\begin{enumerate}
\item Recopier et compléter l'algorithme ci-dessous pour que la variable $N$ contienne la réponse au problème posé.
\begin{center}
\begin{tabularx}{0.4\linewidth}{|X|}\hline
$N = 2017$\\
$U = 18$ \\
$S = 300 + U$ \\
while $S < 450$: \\
\hspace{1cm} $N = \ldots$\\
\hspace{1cm} $U = \ldots$\\
\hspace{1cm} $S = \ldots$\\
\hline
\end{tabularx}
\end{center}
\item Que contiennent les variables $S$, $U$ et $N$ après exécution de cet algorithme ?
Interpréter les résultats dans le contexte de l'exercice.
\end{enumerate}
\end{enumerate}
\end{exercise}
\begin{solution}
\begin{enumerate}
\item Une augmentation de $25\,\%$ revient à multiplier la quantité par $1.25$. La suite est donc bien géométrique. Son premier terme est $u_0 = 18$ et sa raison est $q = 1.25$
\item
\[
u_1 = u_0 * 1.25 = 22.5
\]
\[
u_2 = u_0 * 1.25^2 = 28.125
\]
\item
\[
u_n = u_0 \times q^n = 18 \times 1.25^n
\]
\item On calcule la quantité totale déversée entre 2017 et 2030.
\[
\sum_{n = 0}^{13} u_n = u_0 \times \frac{1-q^{13}}{1-q} = 18 \times \frac{1 - 1.25^{13}}{1 - 1.25} = 1237.67
\]
On en déduit la quantité totale de déchets en 2030
\[
300 + 1237.67 = 1537.67
\]
\item
\begin{enumerate}
\item ~
\begin{center}
\begin{tabularx}{0.4\linewidth}{|X|}\hline
$N \gets 2017$\\
$U \gets 18$ \\
$S \gets 300 + U$ \\
Tant que $S < 450$ \\
\hspace{1cm} $N \gets N + 1$\\
\hspace{1cm} $U \gets U * 1.25$\\
\hspace{1cm} $S \gets S + u$\\
Fin Tant que\\\hline
\end{tabularx}
\end{center}
\item \textit{Pas de correction automatisé}
\end{enumerate}
\end{enumerate}
\end{solution}
\end{document}
%%% Local Variables:
%%% mode: latex
%%% TeX-master: "master"
%%% End:

View File

@ -0,0 +1,262 @@
\documentclass[a4paper,10pt]{article}
\usepackage{myXsim}
% Title Page
\title{DS8 \hfill FERREIRA Léo}
\tribe{TST}
\date{\hfillÀ render pour le Mercredi 7 avril}
\xsimsetup{
solution/print = true
}
\begin{document}
\maketitle
\begin{exercise}[subtitle={Automatismes}]
\textit{Toutes les questions de cette exercice sont indépendantes et peuvent être répondus séparément}
\begin{enumerate}
\item De janvier à septembre, une quantité a augmenté de $10\,\%$. Faire un schéma pour représenter la situation puis calculer le taux d'évolution moyen mensuel.
\item Une quantité augmente de $10\,\%$ par ans. En 2020, elle est de 123\euro. Quelle était sa valeur en 2019? Faire un schéma pour représenter la situation.
\item Déterminer l'équation de la droite \\
\begin{tikzpicture}[xscale=0.8, yscale=0.5]
\tkzInit[xmin=-5,xmax=5,xstep=1,
ymin=-5,ymax=5,ystep=1]
\tkzGrid
\tkzAxeXY
\tkzFct[domain=-5:5,color=red,very thick]%
{1.5*\x -3};
\end{tikzpicture}
\item Résoudre l'équation $5 \times 0.79^x = 30$
\end{enumerate}
\end{exercise}
\begin{solution}
\begin{enumerate}
\item On veut partager cette évolution en 8 évolutions.
\[
\left(1 + \frac{10}{100}\right)^{\frac{1}{8}} = 1.012
\]
Donc le taux d'évolution moyen est
\[
t_m = 1.012 - 1 = 0.01200000000000001
\]
\item Coefficient multiplicateur pour revenir en arrière
\[
CM = (1 + \frac{10}{100})^{-1} = 0.9091
\]
On en déduit la quantité en 2019
\[
123 * 0.9091 = 111.8193
\]
\item L'équation de la droite est
\[
y = 1.5 x -3
\]
\item Il faut penser à faire la division à par $5$ avant d'utiliser le log car sinon, on ne peut pas utiliser la formule $\log(a^n) = n\times \log(a)$.
\[x = \frac{\log(6.0)}{\log(0.79)}\]
\end{enumerate}
\end{solution}
\begin{exercise}[subtitle={Restaurant}]
Un \emph{food truck}, ouvert le midi et le soir, propose deux types de formules :
\setlength\parindent{10mm}
\begin{itemize}
\item la formule \emph{Burger} ;
\item la formule \emph{Wok}.
\end{itemize}
\setlength\parindent{0mm}
\medskip
Le gérant a remarqué que 18\,\% de ses ventes ont lieu le midi. Le quart des ventes du midi correspondent à la formule \emph{Burger}, alors que 37\,\% des ventes du soir correspondent à la formule \emph{Wok}.
Le gérant se constitue un fichier en notant, pour chaque vente, la formule choisie et le moment de cette vente (midi ou soir).
On prélève une fiche de façon équiprobable. On définit les quatre évènements suivants:
\begin{enumerate}
\item $M$ : \og la fiche correspond à une vente du midi\fg{} ;
\item $S$ : \og la fiche correspond à une vente du soir\fg {};
\item $W$ : \og la fiche correspond à une formule \emph{Wok} \fg{} ;
\item $B$ : \og la fiche correspond à une formule \emph{Burger} \fg.
\end{enumerate}
\setlength\parindent{0mm}
\medskip
\begin{enumerate}
\item Recopier puis compléter l'arbre pondéré
\begin{center}
\begin{tikzpicture}[sloped]
\node {.}
child {node {$M$}
child {node {$W$}
edge from parent
node[above] {...}
}
child {node {$B$}
edge from parent
node[above] {...}
}
edge from parent
node[above] {...}
}
child[missing] {}
child { node {$S$}
child {node {$W$}
edge from parent
node[above] {...}
}
child {node {$B$}
edge from parent
node[above] {...}
}
edge from parent
node[above] {...}
} ;
\end{tikzpicture}
\end{center}
\item Calculer la probabilité de l'évènement $M \cap W$. Interpréter ce résultat dans le contexte de l'exercice.
\item Montrer que la probabilité que la fiche choisie corresponde à une formule \emph{Burger} est égale à $0.5616$.
\item On a prélevé une fiche correspondant à la formule \emph{Burger}. Quelle est la probabilité, arrondie au millième, que la vente ait eu lieu le soir?
\end{enumerate}
\end{exercise}
\begin{solution}
\begin{enumerate}
\item
\begin{center}
\begin{tikzpicture}[sloped]
\node {.}
child {node {$M$}
child {node {$W$}
edge from parent
node[above] {$0.75$}
}
child {node {$B$}
edge from parent
node[above] {$0.25$}
}
edge from parent
node[above] {$0.18$}
}
child[missing] {}
child { node {$S$}
child {node {$W$}
edge from parent
node[above] {$0.37$}
}
child {node {$B$}
edge from parent
node[above] {$0.63$}
}
edge from parent
node[above] {$0.82$}
} ;
\end{tikzpicture}
\end{center}
\item On calcule la probabilité que la vente soit un wok et ait eu lieu à midi
\[ P(M\cap W) = P(M) \times P_M(W) = 0.18 \times 0.75 = 0.135 \]
\item Probabilité que la vente soit un burger.
\[
P(B) = P(M\cap B) + P(S\cap B) = 0.18 \times 0.75 + 0.82 \times 0.37 = 0.5616
\]
\item On cherche à calculer la quantité $P_B(S)$. Pour cela on utilise la formule de Bayes
\[
P_B(S) = \frac{P(B\cap S)}{P(B)} = \frac{P_S(B) \times P(S)}{P(B)} = \frac{0.63\times 0.82}{0.5616} = 0.9198717948717948 \approx 0.92
\]
\end{enumerate}
\end{solution}
\begin{exercise}[subtitle={Continent plastique}]
\textit{Les quantités évoqués dans cette exercice sont générés au hasard et sont donc complètement farfelus.}
\medskip
Le \og continent de plastique\fg{} est la plus grande des plaques de déchets plastiques évoluant sur les océans. Elle occupe actuellement dans l'océan Pacifique une surface dont l'aire est évaluée à plus de $1,6$ million de km$^2$, entre Hawaï et la Californie.
En 2017, des scientifiques ont estimé qu'il y avait $3$ millions de tonnes de déchets plastiques qui était déversé chaque année dans les océans et que cette quantité augmentait de $19\n\%$ par chaque année.
On modélise l'évolution de la masse de ces déchets plastiques déversée chaque année, si rien n'est fait pour la réduire, par une suite géométrique $\left(u_n\right)$. L'arrondi au centième du terme $u_n$ représente la masse de ces déchets déversée chaque année, exprimée en million de tonnes, pour l'année $(2017 + n)$.
\medskip
\begin{enumerate}
\item Expliquer pourquoi la suite $u_n$ est géométrique?
\item Calculer $u_1$ et $u_2$.
\item Exprimer $u_n$ en fonction de $n$.
\item Au début de l'année 2017, il y avait $300$ millions de tonnes de déchets plastique. Calculer la quantité totale de déchets plastiques en 2030.
\item On souhaite déterminer en quelle année la masse totale de ces déchets plastiques aura pour la première fois augmenté de $50$\,\% par rapport à sa valeur de 2017.
\begin{enumerate}
\item Recopier et compléter l'algorithme ci-dessous pour que la variable $N$ contienne la réponse au problème posé.
\begin{center}
\begin{tabularx}{0.4\linewidth}{|X|}\hline
$N = 2017$\\
$U = 3$ \\
$S = 300 + U$ \\
while $S < 450$: \\
\hspace{1cm} $N = \ldots$\\
\hspace{1cm} $U = \ldots$\\
\hspace{1cm} $S = \ldots$\\
\hline
\end{tabularx}
\end{center}
\item Que contiennent les variables $S$, $U$ et $N$ après exécution de cet algorithme ?
Interpréter les résultats dans le contexte de l'exercice.
\end{enumerate}
\end{enumerate}
\end{exercise}
\begin{solution}
\begin{enumerate}
\item Une augmentation de $19\,\%$ revient à multiplier la quantité par $1.19$. La suite est donc bien géométrique. Son premier terme est $u_0 = 3$ et sa raison est $q = 1.19$
\item
\[
u_1 = u_0 * 1.19 = 3.57
\]
\[
u_2 = u_0 * 1.19^2 = 4.2483
\]
\item
\[
u_n = u_0 \times q^n = 3 \times 1.19^n
\]
\item On calcule la quantité totale déversée entre 2017 et 2030.
\[
\sum_{n = 0}^{13} u_n = u_0 \times \frac{1-q^{13}}{1-q} = 3 \times \frac{1 - 1.19^{13}}{1 - 1.19} = 135.73
\]
On en déduit la quantité totale de déchets en 2030
\[
300 + 135.73 = 435.73
\]
\item
\begin{enumerate}
\item ~
\begin{center}
\begin{tabularx}{0.4\linewidth}{|X|}\hline
$N \gets 2017$\\
$U \gets 3$ \\
$S \gets 300 + U$ \\
Tant que $S < 450$ \\
\hspace{1cm} $N \gets N + 1$\\
\hspace{1cm} $U \gets U * 1.19$\\
\hspace{1cm} $S \gets S + u$\\
Fin Tant que\\\hline
\end{tabularx}
\end{center}
\item \textit{Pas de correction automatisé}
\end{enumerate}
\end{enumerate}
\end{solution}
\end{document}
%%% Local Variables:
%%% mode: latex
%%% TeX-master: "master"
%%% End:

View File

@ -0,0 +1,262 @@
\documentclass[a4paper,10pt]{article}
\usepackage{myXsim}
% Title Page
\title{DS8 \hfill FILALI Zakaria}
\tribe{TST}
\date{\hfillÀ render pour le Mercredi 7 avril}
\xsimsetup{
solution/print = true
}
\begin{document}
\maketitle
\begin{exercise}[subtitle={Automatismes}]
\textit{Toutes les questions de cette exercice sont indépendantes et peuvent être répondus séparément}
\begin{enumerate}
\item De janvier à septembre, une quantité a augmenté de $19\,\%$. Faire un schéma pour représenter la situation puis calculer le taux d'évolution moyen mensuel.
\item Une quantité augmente de $19\,\%$ par ans. En 2020, elle est de 122\euro. Quelle était sa valeur en 2019? Faire un schéma pour représenter la situation.
\item Déterminer l'équation de la droite \\
\begin{tikzpicture}[xscale=0.8, yscale=0.5]
\tkzInit[xmin=-5,xmax=5,xstep=1,
ymin=-5,ymax=5,ystep=1]
\tkzGrid
\tkzAxeXY
\tkzFct[domain=-5:5,color=red,very thick]%
{1.0*\x -2};
\end{tikzpicture}
\item Résoudre l'équation $8 \times 0.25^x = 25$
\end{enumerate}
\end{exercise}
\begin{solution}
\begin{enumerate}
\item On veut partager cette évolution en 8 évolutions.
\[
\left(1 + \frac{19}{100}\right)^{\frac{1}{8}} = 1.022
\]
Donc le taux d'évolution moyen est
\[
t_m = 1.022 - 1 = 0.02200000000000002
\]
\item Coefficient multiplicateur pour revenir en arrière
\[
CM = (1 + \frac{19}{100})^{-1} = 0.8403
\]
On en déduit la quantité en 2019
\[
122 * 0.8403 = 102.51660000000001
\]
\item L'équation de la droite est
\[
y = 1.0 x -2
\]
\item Il faut penser à faire la division à par $8$ avant d'utiliser le log car sinon, on ne peut pas utiliser la formule $\log(a^n) = n\times \log(a)$.
\[x = \frac{\log(3.12)}{\log(0.25)}\]
\end{enumerate}
\end{solution}
\begin{exercise}[subtitle={Restaurant}]
Un \emph{food truck}, ouvert le midi et le soir, propose deux types de formules :
\setlength\parindent{10mm}
\begin{itemize}
\item la formule \emph{Burger} ;
\item la formule \emph{Wok}.
\end{itemize}
\setlength\parindent{0mm}
\medskip
Le gérant a remarqué que 47\,\% de ses ventes ont lieu le midi. Le quart des ventes du midi correspondent à la formule \emph{Burger}, alors que 69\,\% des ventes du soir correspondent à la formule \emph{Wok}.
Le gérant se constitue un fichier en notant, pour chaque vente, la formule choisie et le moment de cette vente (midi ou soir).
On prélève une fiche de façon équiprobable. On définit les quatre évènements suivants:
\begin{enumerate}
\item $M$ : \og la fiche correspond à une vente du midi\fg{} ;
\item $S$ : \og la fiche correspond à une vente du soir\fg {};
\item $W$ : \og la fiche correspond à une formule \emph{Wok} \fg{} ;
\item $B$ : \og la fiche correspond à une formule \emph{Burger} \fg.
\end{enumerate}
\setlength\parindent{0mm}
\medskip
\begin{enumerate}
\item Recopier puis compléter l'arbre pondéré
\begin{center}
\begin{tikzpicture}[sloped]
\node {.}
child {node {$M$}
child {node {$W$}
edge from parent
node[above] {...}
}
child {node {$B$}
edge from parent
node[above] {...}
}
edge from parent
node[above] {...}
}
child[missing] {}
child { node {$S$}
child {node {$W$}
edge from parent
node[above] {...}
}
child {node {$B$}
edge from parent
node[above] {...}
}
edge from parent
node[above] {...}
} ;
\end{tikzpicture}
\end{center}
\item Calculer la probabilité de l'évènement $M \cap W$. Interpréter ce résultat dans le contexte de l'exercice.
\item Montrer que la probabilité que la fiche choisie corresponde à une formule \emph{Burger} est égale à $0.2818$.
\item On a prélevé une fiche correspondant à la formule \emph{Burger}. Quelle est la probabilité, arrondie au millième, que la vente ait eu lieu le soir?
\end{enumerate}
\end{exercise}
\begin{solution}
\begin{enumerate}
\item
\begin{center}
\begin{tikzpicture}[sloped]
\node {.}
child {node {$M$}
child {node {$W$}
edge from parent
node[above] {$0.75$}
}
child {node {$B$}
edge from parent
node[above] {$0.25$}
}
edge from parent
node[above] {$0.47$}
}
child[missing] {}
child { node {$S$}
child {node {$W$}
edge from parent
node[above] {$0.69$}
}
child {node {$B$}
edge from parent
node[above] {$0.31$}
}
edge from parent
node[above] {$0.53$}
} ;
\end{tikzpicture}
\end{center}
\item On calcule la probabilité que la vente soit un wok et ait eu lieu à midi
\[ P(M\cap W) = P(M) \times P_M(W) = 0.47 \times 0.75 = 0.3525 \]
\item Probabilité que la vente soit un burger.
\[
P(B) = P(M\cap B) + P(S\cap B) = 0.47 \times 0.75 + 0.53 \times 0.69 = 0.2818
\]
\item On cherche à calculer la quantité $P_B(S)$. Pour cela on utilise la formule de Bayes
\[
P_B(S) = \frac{P(B\cap S)}{P(B)} = \frac{P_S(B) \times P(S)}{P(B)} = \frac{0.31\times 0.53}{0.2818} = 0.5830376153300213 \approx 0.583
\]
\end{enumerate}
\end{solution}
\begin{exercise}[subtitle={Continent plastique}]
\textit{Les quantités évoqués dans cette exercice sont générés au hasard et sont donc complètement farfelus.}
\medskip
Le \og continent de plastique\fg{} est la plus grande des plaques de déchets plastiques évoluant sur les océans. Elle occupe actuellement dans l'océan Pacifique une surface dont l'aire est évaluée à plus de $1,6$ million de km$^2$, entre Hawaï et la Californie.
En 2017, des scientifiques ont estimé qu'il y avait $11$ millions de tonnes de déchets plastiques qui était déversé chaque année dans les océans et que cette quantité augmentait de $16\n\%$ par chaque année.
On modélise l'évolution de la masse de ces déchets plastiques déversée chaque année, si rien n'est fait pour la réduire, par une suite géométrique $\left(u_n\right)$. L'arrondi au centième du terme $u_n$ représente la masse de ces déchets déversée chaque année, exprimée en million de tonnes, pour l'année $(2017 + n)$.
\medskip
\begin{enumerate}
\item Expliquer pourquoi la suite $u_n$ est géométrique?
\item Calculer $u_1$ et $u_2$.
\item Exprimer $u_n$ en fonction de $n$.
\item Au début de l'année 2017, il y avait $300$ millions de tonnes de déchets plastique. Calculer la quantité totale de déchets plastiques en 2030.
\item On souhaite déterminer en quelle année la masse totale de ces déchets plastiques aura pour la première fois augmenté de $50$\,\% par rapport à sa valeur de 2017.
\begin{enumerate}
\item Recopier et compléter l'algorithme ci-dessous pour que la variable $N$ contienne la réponse au problème posé.
\begin{center}
\begin{tabularx}{0.4\linewidth}{|X|}\hline
$N = 2017$\\
$U = 11$ \\
$S = 300 + U$ \\
while $S < 450$: \\
\hspace{1cm} $N = \ldots$\\
\hspace{1cm} $U = \ldots$\\
\hspace{1cm} $S = \ldots$\\
\hline
\end{tabularx}
\end{center}
\item Que contiennent les variables $S$, $U$ et $N$ après exécution de cet algorithme ?
Interpréter les résultats dans le contexte de l'exercice.
\end{enumerate}
\end{enumerate}
\end{exercise}
\begin{solution}
\begin{enumerate}
\item Une augmentation de $16\,\%$ revient à multiplier la quantité par $1.16$. La suite est donc bien géométrique. Son premier terme est $u_0 = 11$ et sa raison est $q = 1.16$
\item
\[
u_1 = u_0 * 1.16 = 12.76
\]
\[
u_2 = u_0 * 1.16^2 = 14.8016
\]
\item
\[
u_n = u_0 \times q^n = 11 \times 1.16^n
\]
\item On calcule la quantité totale déversée entre 2017 et 2030.
\[
\sum_{n = 0}^{13} u_n = u_0 \times \frac{1-q^{13}}{1-q} = 11 \times \frac{1 - 1.16^{13}}{1 - 1.16} = 404.65
\]
On en déduit la quantité totale de déchets en 2030
\[
300 + 404.65 = 704.65
\]
\item
\begin{enumerate}
\item ~
\begin{center}
\begin{tabularx}{0.4\linewidth}{|X|}\hline
$N \gets 2017$\\
$U \gets 11$ \\
$S \gets 300 + U$ \\
Tant que $S < 450$ \\
\hspace{1cm} $N \gets N + 1$\\
\hspace{1cm} $U \gets U * 1.16$\\
\hspace{1cm} $S \gets S + u$\\
Fin Tant que\\\hline
\end{tabularx}
\end{center}
\item \textit{Pas de correction automatisé}
\end{enumerate}
\end{enumerate}
\end{solution}
\end{document}
%%% Local Variables:
%%% mode: latex
%%% TeX-master: "master"
%%% End:

View File

@ -0,0 +1,262 @@
\documentclass[a4paper,10pt]{article}
\usepackage{myXsim}
% Title Page
\title{DS8 \hfill FOIGNY Romain}
\tribe{TST}
\date{\hfillÀ render pour le Mercredi 7 avril}
\xsimsetup{
solution/print = true
}
\begin{document}
\maketitle
\begin{exercise}[subtitle={Automatismes}]
\textit{Toutes les questions de cette exercice sont indépendantes et peuvent être répondus séparément}
\begin{enumerate}
\item De janvier à septembre, une quantité a augmenté de $22\,\%$. Faire un schéma pour représenter la situation puis calculer le taux d'évolution moyen mensuel.
\item Une quantité augmente de $22\,\%$ par ans. En 2020, elle est de 125\euro. Quelle était sa valeur en 2019? Faire un schéma pour représenter la situation.
\item Déterminer l'équation de la droite \\
\begin{tikzpicture}[xscale=0.8, yscale=0.5]
\tkzInit[xmin=-5,xmax=5,xstep=1,
ymin=-5,ymax=5,ystep=1]
\tkzGrid
\tkzAxeXY
\tkzFct[domain=-5:5,color=red,very thick]%
{2.6666666666666665*\x -4};
\end{tikzpicture}
\item Résoudre l'équation $9 \times 0.35^x = 44$
\end{enumerate}
\end{exercise}
\begin{solution}
\begin{enumerate}
\item On veut partager cette évolution en 8 évolutions.
\[
\left(1 + \frac{22}{100}\right)^{\frac{1}{8}} = 1.0252
\]
Donc le taux d'évolution moyen est
\[
t_m = 1.0252 - 1 = 0.02519999999999989
\]
\item Coefficient multiplicateur pour revenir en arrière
\[
CM = (1 + \frac{22}{100})^{-1} = 0.8197
\]
On en déduit la quantité en 2019
\[
125 * 0.8197 = 102.46249999999999
\]
\item L'équation de la droite est
\[
y = 2.6666666666666665 x -4
\]
\item Il faut penser à faire la division à par $9$ avant d'utiliser le log car sinon, on ne peut pas utiliser la formule $\log(a^n) = n\times \log(a)$.
\[x = \frac{\log(4.89)}{\log(0.35)}\]
\end{enumerate}
\end{solution}
\begin{exercise}[subtitle={Restaurant}]
Un \emph{food truck}, ouvert le midi et le soir, propose deux types de formules :
\setlength\parindent{10mm}
\begin{itemize}
\item la formule \emph{Burger} ;
\item la formule \emph{Wok}.
\end{itemize}
\setlength\parindent{0mm}
\medskip
Le gérant a remarqué que 14\,\% de ses ventes ont lieu le midi. Le quart des ventes du midi correspondent à la formule \emph{Burger}, alors que 17\,\% des ventes du soir correspondent à la formule \emph{Wok}.
Le gérant se constitue un fichier en notant, pour chaque vente, la formule choisie et le moment de cette vente (midi ou soir).
On prélève une fiche de façon équiprobable. On définit les quatre évènements suivants:
\begin{enumerate}
\item $M$ : \og la fiche correspond à une vente du midi\fg{} ;
\item $S$ : \og la fiche correspond à une vente du soir\fg {};
\item $W$ : \og la fiche correspond à une formule \emph{Wok} \fg{} ;
\item $B$ : \og la fiche correspond à une formule \emph{Burger} \fg.
\end{enumerate}
\setlength\parindent{0mm}
\medskip
\begin{enumerate}
\item Recopier puis compléter l'arbre pondéré
\begin{center}
\begin{tikzpicture}[sloped]
\node {.}
child {node {$M$}
child {node {$W$}
edge from parent
node[above] {...}
}
child {node {$B$}
edge from parent
node[above] {...}
}
edge from parent
node[above] {...}
}
child[missing] {}
child { node {$S$}
child {node {$W$}
edge from parent
node[above] {...}
}
child {node {$B$}
edge from parent
node[above] {...}
}
edge from parent
node[above] {...}
} ;
\end{tikzpicture}
\end{center}
\item Calculer la probabilité de l'évènement $M \cap W$. Interpréter ce résultat dans le contexte de l'exercice.
\item Montrer que la probabilité que la fiche choisie corresponde à une formule \emph{Burger} est égale à $0.7488$.
\item On a prélevé une fiche correspondant à la formule \emph{Burger}. Quelle est la probabilité, arrondie au millième, que la vente ait eu lieu le soir?
\end{enumerate}
\end{exercise}
\begin{solution}
\begin{enumerate}
\item
\begin{center}
\begin{tikzpicture}[sloped]
\node {.}
child {node {$M$}
child {node {$W$}
edge from parent
node[above] {$0.75$}
}
child {node {$B$}
edge from parent
node[above] {$0.25$}
}
edge from parent
node[above] {$0.14$}
}
child[missing] {}
child { node {$S$}
child {node {$W$}
edge from parent
node[above] {$0.17$}
}
child {node {$B$}
edge from parent
node[above] {$0.83$}
}
edge from parent
node[above] {$0.86$}
} ;
\end{tikzpicture}
\end{center}
\item On calcule la probabilité que la vente soit un wok et ait eu lieu à midi
\[ P(M\cap W) = P(M) \times P_M(W) = 0.14 \times 0.75 = 0.105 \]
\item Probabilité que la vente soit un burger.
\[
P(B) = P(M\cap B) + P(S\cap B) = 0.14 \times 0.75 + 0.86 \times 0.17 = 0.7488
\]
\item On cherche à calculer la quantité $P_B(S)$. Pour cela on utilise la formule de Bayes
\[
P_B(S) = \frac{P(B\cap S)}{P(B)} = \frac{P_S(B) \times P(S)}{P(B)} = \frac{0.83\times 0.86}{0.7488} = 0.953258547008547 \approx 0.953
\]
\end{enumerate}
\end{solution}
\begin{exercise}[subtitle={Continent plastique}]
\textit{Les quantités évoqués dans cette exercice sont générés au hasard et sont donc complètement farfelus.}
\medskip
Le \og continent de plastique\fg{} est la plus grande des plaques de déchets plastiques évoluant sur les océans. Elle occupe actuellement dans l'océan Pacifique une surface dont l'aire est évaluée à plus de $1,6$ million de km$^2$, entre Hawaï et la Californie.
En 2017, des scientifiques ont estimé qu'il y avait $11$ millions de tonnes de déchets plastiques qui était déversé chaque année dans les océans et que cette quantité augmentait de $12\n\%$ par chaque année.
On modélise l'évolution de la masse de ces déchets plastiques déversée chaque année, si rien n'est fait pour la réduire, par une suite géométrique $\left(u_n\right)$. L'arrondi au centième du terme $u_n$ représente la masse de ces déchets déversée chaque année, exprimée en million de tonnes, pour l'année $(2017 + n)$.
\medskip
\begin{enumerate}
\item Expliquer pourquoi la suite $u_n$ est géométrique?
\item Calculer $u_1$ et $u_2$.
\item Exprimer $u_n$ en fonction de $n$.
\item Au début de l'année 2017, il y avait $300$ millions de tonnes de déchets plastique. Calculer la quantité totale de déchets plastiques en 2030.
\item On souhaite déterminer en quelle année la masse totale de ces déchets plastiques aura pour la première fois augmenté de $50$\,\% par rapport à sa valeur de 2017.
\begin{enumerate}
\item Recopier et compléter l'algorithme ci-dessous pour que la variable $N$ contienne la réponse au problème posé.
\begin{center}
\begin{tabularx}{0.4\linewidth}{|X|}\hline
$N = 2017$\\
$U = 11$ \\
$S = 300 + U$ \\
while $S < 450$: \\
\hspace{1cm} $N = \ldots$\\
\hspace{1cm} $U = \ldots$\\
\hspace{1cm} $S = \ldots$\\
\hline
\end{tabularx}
\end{center}
\item Que contiennent les variables $S$, $U$ et $N$ après exécution de cet algorithme ?
Interpréter les résultats dans le contexte de l'exercice.
\end{enumerate}
\end{enumerate}
\end{exercise}
\begin{solution}
\begin{enumerate}
\item Une augmentation de $12\,\%$ revient à multiplier la quantité par $1.12$. La suite est donc bien géométrique. Son premier terme est $u_0 = 11$ et sa raison est $q = 1.12$
\item
\[
u_1 = u_0 * 1.12 = 12.32
\]
\[
u_2 = u_0 * 1.12^2 = 13.7984
\]
\item
\[
u_n = u_0 \times q^n = 11 \times 1.12^n
\]
\item On calcule la quantité totale déversée entre 2017 et 2030.
\[
\sum_{n = 0}^{13} u_n = u_0 \times \frac{1-q^{13}}{1-q} = 11 \times \frac{1 - 1.12^{13}}{1 - 1.12} = 308.32
\]
On en déduit la quantité totale de déchets en 2030
\[
300 + 308.32 = 608.3199999999999
\]
\item
\begin{enumerate}
\item ~
\begin{center}
\begin{tabularx}{0.4\linewidth}{|X|}\hline
$N \gets 2017$\\
$U \gets 11$ \\
$S \gets 300 + U$ \\
Tant que $S < 450$ \\
\hspace{1cm} $N \gets N + 1$\\
\hspace{1cm} $U \gets U * 1.12$\\
\hspace{1cm} $S \gets S + u$\\
Fin Tant que\\\hline
\end{tabularx}
\end{center}
\item \textit{Pas de correction automatisé}
\end{enumerate}
\end{enumerate}
\end{solution}
\end{document}
%%% Local Variables:
%%% mode: latex
%%% TeX-master: "master"
%%% End:

View File

@ -0,0 +1,262 @@
\documentclass[a4paper,10pt]{article}
\usepackage{myXsim}
% Title Page
\title{DS8 \hfill HIPOLITO DA SILVA Andréa}
\tribe{TST}
\date{\hfillÀ render pour le Mercredi 7 avril}
\xsimsetup{
solution/print = true
}
\begin{document}
\maketitle
\begin{exercise}[subtitle={Automatismes}]
\textit{Toutes les questions de cette exercice sont indépendantes et peuvent être répondus séparément}
\begin{enumerate}
\item De janvier à septembre, une quantité a augmenté de $23\,\%$. Faire un schéma pour représenter la situation puis calculer le taux d'évolution moyen mensuel.
\item Une quantité augmente de $23\,\%$ par ans. En 2020, elle est de 149\euro. Quelle était sa valeur en 2019? Faire un schéma pour représenter la situation.
\item Déterminer l'équation de la droite \\
\begin{tikzpicture}[xscale=0.8, yscale=0.5]
\tkzInit[xmin=-5,xmax=5,xstep=1,
ymin=-5,ymax=5,ystep=1]
\tkzGrid
\tkzAxeXY
\tkzFct[domain=-5:5,color=red,very thick]%
{0.6666666666666666*\x -1};
\end{tikzpicture}
\item Résoudre l'équation $7 \times 0.38^x = 21$
\end{enumerate}
\end{exercise}
\begin{solution}
\begin{enumerate}
\item On veut partager cette évolution en 8 évolutions.
\[
\left(1 + \frac{23}{100}\right)^{\frac{1}{8}} = 1.0262
\]
Donc le taux d'évolution moyen est
\[
t_m = 1.0262 - 1 = 0.0262
\]
\item Coefficient multiplicateur pour revenir en arrière
\[
CM = (1 + \frac{23}{100})^{-1} = 0.813
\]
On en déduit la quantité en 2019
\[
149 * 0.813 = 121.13699999999999
\]
\item L'équation de la droite est
\[
y = 0.6666666666666666 x -1
\]
\item Il faut penser à faire la division à par $7$ avant d'utiliser le log car sinon, on ne peut pas utiliser la formule $\log(a^n) = n\times \log(a)$.
\[x = \frac{\log(3.0)}{\log(0.38)}\]
\end{enumerate}
\end{solution}
\begin{exercise}[subtitle={Restaurant}]
Un \emph{food truck}, ouvert le midi et le soir, propose deux types de formules :
\setlength\parindent{10mm}
\begin{itemize}
\item la formule \emph{Burger} ;
\item la formule \emph{Wok}.
\end{itemize}
\setlength\parindent{0mm}
\medskip
Le gérant a remarqué que 35\,\% de ses ventes ont lieu le midi. Le quart des ventes du midi correspondent à la formule \emph{Burger}, alors que 13\,\% des ventes du soir correspondent à la formule \emph{Wok}.
Le gérant se constitue un fichier en notant, pour chaque vente, la formule choisie et le moment de cette vente (midi ou soir).
On prélève une fiche de façon équiprobable. On définit les quatre évènements suivants:
\begin{enumerate}
\item $M$ : \og la fiche correspond à une vente du midi\fg{} ;
\item $S$ : \og la fiche correspond à une vente du soir\fg {};
\item $W$ : \og la fiche correspond à une formule \emph{Wok} \fg{} ;
\item $B$ : \og la fiche correspond à une formule \emph{Burger} \fg.
\end{enumerate}
\setlength\parindent{0mm}
\medskip
\begin{enumerate}
\item Recopier puis compléter l'arbre pondéré
\begin{center}
\begin{tikzpicture}[sloped]
\node {.}
child {node {$M$}
child {node {$W$}
edge from parent
node[above] {...}
}
child {node {$B$}
edge from parent
node[above] {...}
}
edge from parent
node[above] {...}
}
child[missing] {}
child { node {$S$}
child {node {$W$}
edge from parent
node[above] {...}
}
child {node {$B$}
edge from parent
node[above] {...}
}
edge from parent
node[above] {...}
} ;
\end{tikzpicture}
\end{center}
\item Calculer la probabilité de l'évènement $M \cap W$. Interpréter ce résultat dans le contexte de l'exercice.
\item Montrer que la probabilité que la fiche choisie corresponde à une formule \emph{Burger} est égale à $0.653$.
\item On a prélevé une fiche correspondant à la formule \emph{Burger}. Quelle est la probabilité, arrondie au millième, que la vente ait eu lieu le soir?
\end{enumerate}
\end{exercise}
\begin{solution}
\begin{enumerate}
\item
\begin{center}
\begin{tikzpicture}[sloped]
\node {.}
child {node {$M$}
child {node {$W$}
edge from parent
node[above] {$0.75$}
}
child {node {$B$}
edge from parent
node[above] {$0.25$}
}
edge from parent
node[above] {$0.35$}
}
child[missing] {}
child { node {$S$}
child {node {$W$}
edge from parent
node[above] {$0.13$}
}
child {node {$B$}
edge from parent
node[above] {$0.87$}
}
edge from parent
node[above] {$0.65$}
} ;
\end{tikzpicture}
\end{center}
\item On calcule la probabilité que la vente soit un wok et ait eu lieu à midi
\[ P(M\cap W) = P(M) \times P_M(W) = 0.35 \times 0.75 = 0.2625 \]
\item Probabilité que la vente soit un burger.
\[
P(B) = P(M\cap B) + P(S\cap B) = 0.35 \times 0.75 + 0.65 \times 0.13 = 0.653
\]
\item On cherche à calculer la quantité $P_B(S)$. Pour cela on utilise la formule de Bayes
\[
P_B(S) = \frac{P(B\cap S)}{P(B)} = \frac{P_S(B) \times P(S)}{P(B)} = \frac{0.87\times 0.65}{0.653} = 0.8660030627871362 \approx 0.866
\]
\end{enumerate}
\end{solution}
\begin{exercise}[subtitle={Continent plastique}]
\textit{Les quantités évoqués dans cette exercice sont générés au hasard et sont donc complètement farfelus.}
\medskip
Le \og continent de plastique\fg{} est la plus grande des plaques de déchets plastiques évoluant sur les océans. Elle occupe actuellement dans l'océan Pacifique une surface dont l'aire est évaluée à plus de $1,6$ million de km$^2$, entre Hawaï et la Californie.
En 2017, des scientifiques ont estimé qu'il y avait $4$ millions de tonnes de déchets plastiques qui était déversé chaque année dans les océans et que cette quantité augmentait de $23\n\%$ par chaque année.
On modélise l'évolution de la masse de ces déchets plastiques déversée chaque année, si rien n'est fait pour la réduire, par une suite géométrique $\left(u_n\right)$. L'arrondi au centième du terme $u_n$ représente la masse de ces déchets déversée chaque année, exprimée en million de tonnes, pour l'année $(2017 + n)$.
\medskip
\begin{enumerate}
\item Expliquer pourquoi la suite $u_n$ est géométrique?
\item Calculer $u_1$ et $u_2$.
\item Exprimer $u_n$ en fonction de $n$.
\item Au début de l'année 2017, il y avait $300$ millions de tonnes de déchets plastique. Calculer la quantité totale de déchets plastiques en 2030.
\item On souhaite déterminer en quelle année la masse totale de ces déchets plastiques aura pour la première fois augmenté de $50$\,\% par rapport à sa valeur de 2017.
\begin{enumerate}
\item Recopier et compléter l'algorithme ci-dessous pour que la variable $N$ contienne la réponse au problème posé.
\begin{center}
\begin{tabularx}{0.4\linewidth}{|X|}\hline
$N = 2017$\\
$U = 4$ \\
$S = 300 + U$ \\
while $S < 450$: \\
\hspace{1cm} $N = \ldots$\\
\hspace{1cm} $U = \ldots$\\
\hspace{1cm} $S = \ldots$\\
\hline
\end{tabularx}
\end{center}
\item Que contiennent les variables $S$, $U$ et $N$ après exécution de cet algorithme ?
Interpréter les résultats dans le contexte de l'exercice.
\end{enumerate}
\end{enumerate}
\end{exercise}
\begin{solution}
\begin{enumerate}
\item Une augmentation de $23\,\%$ revient à multiplier la quantité par $1.23$. La suite est donc bien géométrique. Son premier terme est $u_0 = 4$ et sa raison est $q = 1.23$
\item
\[
u_1 = u_0 * 1.23 = 4.92
\]
\[
u_2 = u_0 * 1.23^2 = 6.0516
\]
\item
\[
u_n = u_0 \times q^n = 4 \times 1.23^n
\]
\item On calcule la quantité totale déversée entre 2017 et 2030.
\[
\sum_{n = 0}^{13} u_n = u_0 \times \frac{1-q^{13}}{1-q} = 4 \times \frac{1 - 1.23^{13}}{1 - 1.23} = 239.12
\]
On en déduit la quantité totale de déchets en 2030
\[
300 + 239.12 = 539.12
\]
\item
\begin{enumerate}
\item ~
\begin{center}
\begin{tabularx}{0.4\linewidth}{|X|}\hline
$N \gets 2017$\\
$U \gets 4$ \\
$S \gets 300 + U$ \\
Tant que $S < 450$ \\
\hspace{1cm} $N \gets N + 1$\\
\hspace{1cm} $U \gets U * 1.23$\\
\hspace{1cm} $S \gets S + u$\\
Fin Tant que\\\hline
\end{tabularx}
\end{center}
\item \textit{Pas de correction automatisé}
\end{enumerate}
\end{enumerate}
\end{solution}
\end{document}
%%% Local Variables:
%%% mode: latex
%%% TeX-master: "master"
%%% End:

View File

@ -0,0 +1,262 @@
\documentclass[a4paper,10pt]{article}
\usepackage{myXsim}
% Title Page
\title{DS8 \hfill HUMBERT Rayan}
\tribe{TST}
\date{\hfillÀ render pour le Mercredi 7 avril}
\xsimsetup{
solution/print = true
}
\begin{document}
\maketitle
\begin{exercise}[subtitle={Automatismes}]
\textit{Toutes les questions de cette exercice sont indépendantes et peuvent être répondus séparément}
\begin{enumerate}
\item De janvier à septembre, une quantité a augmenté de $17\,\%$. Faire un schéma pour représenter la situation puis calculer le taux d'évolution moyen mensuel.
\item Une quantité augmente de $17\,\%$ par ans. En 2020, elle est de 149\euro. Quelle était sa valeur en 2019? Faire un schéma pour représenter la situation.
\item Déterminer l'équation de la droite \\
\begin{tikzpicture}[xscale=0.8, yscale=0.5]
\tkzInit[xmin=-5,xmax=5,xstep=1,
ymin=-5,ymax=5,ystep=1]
\tkzGrid
\tkzAxeXY
\tkzFct[domain=-5:5,color=red,very thick]%
{0.5*\x -1};
\end{tikzpicture}
\item Résoudre l'équation $5 \times 0.02^x = 46$
\end{enumerate}
\end{exercise}
\begin{solution}
\begin{enumerate}
\item On veut partager cette évolution en 8 évolutions.
\[
\left(1 + \frac{17}{100}\right)^{\frac{1}{8}} = 1.0198
\]
Donc le taux d'évolution moyen est
\[
t_m = 1.0198 - 1 = 0.01980000000000004
\]
\item Coefficient multiplicateur pour revenir en arrière
\[
CM = (1 + \frac{17}{100})^{-1} = 0.8547
\]
On en déduit la quantité en 2019
\[
149 * 0.8547 = 127.3503
\]
\item L'équation de la droite est
\[
y = 0.5 x -1
\]
\item Il faut penser à faire la division à par $5$ avant d'utiliser le log car sinon, on ne peut pas utiliser la formule $\log(a^n) = n\times \log(a)$.
\[x = \frac{\log(9.2)}{\log(0.02)}\]
\end{enumerate}
\end{solution}
\begin{exercise}[subtitle={Restaurant}]
Un \emph{food truck}, ouvert le midi et le soir, propose deux types de formules :
\setlength\parindent{10mm}
\begin{itemize}
\item la formule \emph{Burger} ;
\item la formule \emph{Wok}.
\end{itemize}
\setlength\parindent{0mm}
\medskip
Le gérant a remarqué que 65\,\% de ses ventes ont lieu le midi. Le quart des ventes du midi correspondent à la formule \emph{Burger}, alors que 93\,\% des ventes du soir correspondent à la formule \emph{Wok}.
Le gérant se constitue un fichier en notant, pour chaque vente, la formule choisie et le moment de cette vente (midi ou soir).
On prélève une fiche de façon équiprobable. On définit les quatre évènements suivants:
\begin{enumerate}
\item $M$ : \og la fiche correspond à une vente du midi\fg{} ;
\item $S$ : \og la fiche correspond à une vente du soir\fg {};
\item $W$ : \og la fiche correspond à une formule \emph{Wok} \fg{} ;
\item $B$ : \og la fiche correspond à une formule \emph{Burger} \fg.
\end{enumerate}
\setlength\parindent{0mm}
\medskip
\begin{enumerate}
\item Recopier puis compléter l'arbre pondéré
\begin{center}
\begin{tikzpicture}[sloped]
\node {.}
child {node {$M$}
child {node {$W$}
edge from parent
node[above] {...}
}
child {node {$B$}
edge from parent
node[above] {...}
}
edge from parent
node[above] {...}
}
child[missing] {}
child { node {$S$}
child {node {$W$}
edge from parent
node[above] {...}
}
child {node {$B$}
edge from parent
node[above] {...}
}
edge from parent
node[above] {...}
} ;
\end{tikzpicture}
\end{center}
\item Calculer la probabilité de l'évènement $M \cap W$. Interpréter ce résultat dans le contexte de l'exercice.
\item Montrer que la probabilité que la fiche choisie corresponde à une formule \emph{Burger} est égale à $0.187$.
\item On a prélevé une fiche correspondant à la formule \emph{Burger}. Quelle est la probabilité, arrondie au millième, que la vente ait eu lieu le soir?
\end{enumerate}
\end{exercise}
\begin{solution}
\begin{enumerate}
\item
\begin{center}
\begin{tikzpicture}[sloped]
\node {.}
child {node {$M$}
child {node {$W$}
edge from parent
node[above] {$0.75$}
}
child {node {$B$}
edge from parent
node[above] {$0.25$}
}
edge from parent
node[above] {$0.65$}
}
child[missing] {}
child { node {$S$}
child {node {$W$}
edge from parent
node[above] {$0.93$}
}
child {node {$B$}
edge from parent
node[above] {$0.07$}
}
edge from parent
node[above] {$0.35$}
} ;
\end{tikzpicture}
\end{center}
\item On calcule la probabilité que la vente soit un wok et ait eu lieu à midi
\[ P(M\cap W) = P(M) \times P_M(W) = 0.65 \times 0.75 = 0.4875 \]
\item Probabilité que la vente soit un burger.
\[
P(B) = P(M\cap B) + P(S\cap B) = 0.65 \times 0.75 + 0.35 \times 0.93 = 0.187
\]
\item On cherche à calculer la quantité $P_B(S)$. Pour cela on utilise la formule de Bayes
\[
P_B(S) = \frac{P(B\cap S)}{P(B)} = \frac{P_S(B) \times P(S)}{P(B)} = \frac{0.07\times 0.35}{0.187} = 0.13101604278074866 \approx 0.131
\]
\end{enumerate}
\end{solution}
\begin{exercise}[subtitle={Continent plastique}]
\textit{Les quantités évoqués dans cette exercice sont générés au hasard et sont donc complètement farfelus.}
\medskip
Le \og continent de plastique\fg{} est la plus grande des plaques de déchets plastiques évoluant sur les océans. Elle occupe actuellement dans l'océan Pacifique une surface dont l'aire est évaluée à plus de $1,6$ million de km$^2$, entre Hawaï et la Californie.
En 2017, des scientifiques ont estimé qu'il y avait $15$ millions de tonnes de déchets plastiques qui était déversé chaque année dans les océans et que cette quantité augmentait de $12\n\%$ par chaque année.
On modélise l'évolution de la masse de ces déchets plastiques déversée chaque année, si rien n'est fait pour la réduire, par une suite géométrique $\left(u_n\right)$. L'arrondi au centième du terme $u_n$ représente la masse de ces déchets déversée chaque année, exprimée en million de tonnes, pour l'année $(2017 + n)$.
\medskip
\begin{enumerate}
\item Expliquer pourquoi la suite $u_n$ est géométrique?
\item Calculer $u_1$ et $u_2$.
\item Exprimer $u_n$ en fonction de $n$.
\item Au début de l'année 2017, il y avait $300$ millions de tonnes de déchets plastique. Calculer la quantité totale de déchets plastiques en 2030.
\item On souhaite déterminer en quelle année la masse totale de ces déchets plastiques aura pour la première fois augmenté de $50$\,\% par rapport à sa valeur de 2017.
\begin{enumerate}
\item Recopier et compléter l'algorithme ci-dessous pour que la variable $N$ contienne la réponse au problème posé.
\begin{center}
\begin{tabularx}{0.4\linewidth}{|X|}\hline
$N = 2017$\\
$U = 15$ \\
$S = 300 + U$ \\
while $S < 450$: \\
\hspace{1cm} $N = \ldots$\\
\hspace{1cm} $U = \ldots$\\
\hspace{1cm} $S = \ldots$\\
\hline
\end{tabularx}
\end{center}
\item Que contiennent les variables $S$, $U$ et $N$ après exécution de cet algorithme ?
Interpréter les résultats dans le contexte de l'exercice.
\end{enumerate}
\end{enumerate}
\end{exercise}
\begin{solution}
\begin{enumerate}
\item Une augmentation de $12\,\%$ revient à multiplier la quantité par $1.12$. La suite est donc bien géométrique. Son premier terme est $u_0 = 15$ et sa raison est $q = 1.12$
\item
\[
u_1 = u_0 * 1.12 = 16.8
\]
\[
u_2 = u_0 * 1.12^2 = 18.816
\]
\item
\[
u_n = u_0 \times q^n = 15 \times 1.12^n
\]
\item On calcule la quantité totale déversée entre 2017 et 2030.
\[
\sum_{n = 0}^{13} u_n = u_0 \times \frac{1-q^{13}}{1-q} = 15 \times \frac{1 - 1.12^{13}}{1 - 1.12} = 420.44
\]
On en déduit la quantité totale de déchets en 2030
\[
300 + 420.44 = 720.44
\]
\item
\begin{enumerate}
\item ~
\begin{center}
\begin{tabularx}{0.4\linewidth}{|X|}\hline
$N \gets 2017$\\
$U \gets 15$ \\
$S \gets 300 + U$ \\
Tant que $S < 450$ \\
\hspace{1cm} $N \gets N + 1$\\
\hspace{1cm} $U \gets U * 1.12$\\
\hspace{1cm} $S \gets S + u$\\
Fin Tant que\\\hline
\end{tabularx}
\end{center}
\item \textit{Pas de correction automatisé}
\end{enumerate}
\end{enumerate}
\end{solution}
\end{document}
%%% Local Variables:
%%% mode: latex
%%% TeX-master: "master"
%%% End:

View File

@ -0,0 +1,262 @@
\documentclass[a4paper,10pt]{article}
\usepackage{myXsim}
% Title Page
\title{DS8 \hfill MASSON Grace}
\tribe{TST}
\date{\hfillÀ render pour le Mercredi 7 avril}
\xsimsetup{
solution/print = true
}
\begin{document}
\maketitle
\begin{exercise}[subtitle={Automatismes}]
\textit{Toutes les questions de cette exercice sont indépendantes et peuvent être répondus séparément}
\begin{enumerate}
\item De janvier à septembre, une quantité a augmenté de $24\,\%$. Faire un schéma pour représenter la situation puis calculer le taux d'évolution moyen mensuel.
\item Une quantité augmente de $24\,\%$ par ans. En 2020, elle est de 145\euro. Quelle était sa valeur en 2019? Faire un schéma pour représenter la situation.
\item Déterminer l'équation de la droite \\
\begin{tikzpicture}[xscale=0.8, yscale=0.5]
\tkzInit[xmin=-5,xmax=5,xstep=1,
ymin=-5,ymax=5,ystep=1]
\tkzGrid
\tkzAxeXY
\tkzFct[domain=-5:5,color=red,very thick]%
{3.0*\x -3};
\end{tikzpicture}
\item Résoudre l'équation $5 \times 0.53^x = 18$
\end{enumerate}
\end{exercise}
\begin{solution}
\begin{enumerate}
\item On veut partager cette évolution en 8 évolutions.
\[
\left(1 + \frac{24}{100}\right)^{\frac{1}{8}} = 1.0273
\]
Donc le taux d'évolution moyen est
\[
t_m = 1.0273 - 1 = 0.027300000000000102
\]
\item Coefficient multiplicateur pour revenir en arrière
\[
CM = (1 + \frac{24}{100})^{-1} = 0.8065
\]
On en déduit la quantité en 2019
\[
145 * 0.8065 = 116.9425
\]
\item L'équation de la droite est
\[
y = 3.0 x -3
\]
\item Il faut penser à faire la division à par $5$ avant d'utiliser le log car sinon, on ne peut pas utiliser la formule $\log(a^n) = n\times \log(a)$.
\[x = \frac{\log(3.6)}{\log(0.53)}\]
\end{enumerate}
\end{solution}
\begin{exercise}[subtitle={Restaurant}]
Un \emph{food truck}, ouvert le midi et le soir, propose deux types de formules :
\setlength\parindent{10mm}
\begin{itemize}
\item la formule \emph{Burger} ;
\item la formule \emph{Wok}.
\end{itemize}
\setlength\parindent{0mm}
\medskip
Le gérant a remarqué que 73\,\% de ses ventes ont lieu le midi. Le quart des ventes du midi correspondent à la formule \emph{Burger}, alors que 56\,\% des ventes du soir correspondent à la formule \emph{Wok}.
Le gérant se constitue un fichier en notant, pour chaque vente, la formule choisie et le moment de cette vente (midi ou soir).
On prélève une fiche de façon équiprobable. On définit les quatre évènements suivants:
\begin{enumerate}
\item $M$ : \og la fiche correspond à une vente du midi\fg{} ;
\item $S$ : \og la fiche correspond à une vente du soir\fg {};
\item $W$ : \og la fiche correspond à une formule \emph{Wok} \fg{} ;
\item $B$ : \og la fiche correspond à une formule \emph{Burger} \fg.
\end{enumerate}
\setlength\parindent{0mm}
\medskip
\begin{enumerate}
\item Recopier puis compléter l'arbre pondéré
\begin{center}
\begin{tikzpicture}[sloped]
\node {.}
child {node {$M$}
child {node {$W$}
edge from parent
node[above] {...}
}
child {node {$B$}
edge from parent
node[above] {...}
}
edge from parent
node[above] {...}
}
child[missing] {}
child { node {$S$}
child {node {$W$}
edge from parent
node[above] {...}
}
child {node {$B$}
edge from parent
node[above] {...}
}
edge from parent
node[above] {...}
} ;
\end{tikzpicture}
\end{center}
\item Calculer la probabilité de l'évènement $M \cap W$. Interpréter ce résultat dans le contexte de l'exercice.
\item Montrer que la probabilité que la fiche choisie corresponde à une formule \emph{Burger} est égale à $0.2986$.
\item On a prélevé une fiche correspondant à la formule \emph{Burger}. Quelle est la probabilité, arrondie au millième, que la vente ait eu lieu le soir?
\end{enumerate}
\end{exercise}
\begin{solution}
\begin{enumerate}
\item
\begin{center}
\begin{tikzpicture}[sloped]
\node {.}
child {node {$M$}
child {node {$W$}
edge from parent
node[above] {$0.75$}
}
child {node {$B$}
edge from parent
node[above] {$0.25$}
}
edge from parent
node[above] {$0.73$}
}
child[missing] {}
child { node {$S$}
child {node {$W$}
edge from parent
node[above] {$0.57$}
}
child {node {$B$}
edge from parent
node[above] {$0.43$}
}
edge from parent
node[above] {$0.27$}
} ;
\end{tikzpicture}
\end{center}
\item On calcule la probabilité que la vente soit un wok et ait eu lieu à midi
\[ P(M\cap W) = P(M) \times P_M(W) = 0.73 \times 0.75 = 0.5475 \]
\item Probabilité que la vente soit un burger.
\[
P(B) = P(M\cap B) + P(S\cap B) = 0.73 \times 0.75 + 0.27 \times 0.57 = 0.2986
\]
\item On cherche à calculer la quantité $P_B(S)$. Pour cela on utilise la formule de Bayes
\[
P_B(S) = \frac{P(B\cap S)}{P(B)} = \frac{P_S(B) \times P(S)}{P(B)} = \frac{0.43\times 0.27}{0.2986} = 0.38881446751507037 \approx 0.389
\]
\end{enumerate}
\end{solution}
\begin{exercise}[subtitle={Continent plastique}]
\textit{Les quantités évoqués dans cette exercice sont générés au hasard et sont donc complètement farfelus.}
\medskip
Le \og continent de plastique\fg{} est la plus grande des plaques de déchets plastiques évoluant sur les océans. Elle occupe actuellement dans l'océan Pacifique une surface dont l'aire est évaluée à plus de $1,6$ million de km$^2$, entre Hawaï et la Californie.
En 2017, des scientifiques ont estimé qu'il y avait $4$ millions de tonnes de déchets plastiques qui était déversé chaque année dans les océans et que cette quantité augmentait de $19\n\%$ par chaque année.
On modélise l'évolution de la masse de ces déchets plastiques déversée chaque année, si rien n'est fait pour la réduire, par une suite géométrique $\left(u_n\right)$. L'arrondi au centième du terme $u_n$ représente la masse de ces déchets déversée chaque année, exprimée en million de tonnes, pour l'année $(2017 + n)$.
\medskip
\begin{enumerate}
\item Expliquer pourquoi la suite $u_n$ est géométrique?
\item Calculer $u_1$ et $u_2$.
\item Exprimer $u_n$ en fonction de $n$.
\item Au début de l'année 2017, il y avait $300$ millions de tonnes de déchets plastique. Calculer la quantité totale de déchets plastiques en 2030.
\item On souhaite déterminer en quelle année la masse totale de ces déchets plastiques aura pour la première fois augmenté de $50$\,\% par rapport à sa valeur de 2017.
\begin{enumerate}
\item Recopier et compléter l'algorithme ci-dessous pour que la variable $N$ contienne la réponse au problème posé.
\begin{center}
\begin{tabularx}{0.4\linewidth}{|X|}\hline
$N = 2017$\\
$U = 4$ \\
$S = 300 + U$ \\
while $S < 450$: \\
\hspace{1cm} $N = \ldots$\\
\hspace{1cm} $U = \ldots$\\
\hspace{1cm} $S = \ldots$\\
\hline
\end{tabularx}
\end{center}
\item Que contiennent les variables $S$, $U$ et $N$ après exécution de cet algorithme ?
Interpréter les résultats dans le contexte de l'exercice.
\end{enumerate}
\end{enumerate}
\end{exercise}
\begin{solution}
\begin{enumerate}
\item Une augmentation de $19\,\%$ revient à multiplier la quantité par $1.19$. La suite est donc bien géométrique. Son premier terme est $u_0 = 4$ et sa raison est $q = 1.19$
\item
\[
u_1 = u_0 * 1.19 = 4.76
\]
\[
u_2 = u_0 * 1.19^2 = 5.6644
\]
\item
\[
u_n = u_0 \times q^n = 4 \times 1.19^n
\]
\item On calcule la quantité totale déversée entre 2017 et 2030.
\[
\sum_{n = 0}^{13} u_n = u_0 \times \frac{1-q^{13}}{1-q} = 4 \times \frac{1 - 1.19^{13}}{1 - 1.19} = 180.98
\]
On en déduit la quantité totale de déchets en 2030
\[
300 + 180.98 = 480.98
\]
\item
\begin{enumerate}
\item ~
\begin{center}
\begin{tabularx}{0.4\linewidth}{|X|}\hline
$N \gets 2017$\\
$U \gets 4$ \\
$S \gets 300 + U$ \\
Tant que $S < 450$ \\
\hspace{1cm} $N \gets N + 1$\\
\hspace{1cm} $U \gets U * 1.19$\\
\hspace{1cm} $S \gets S + u$\\
Fin Tant que\\\hline
\end{tabularx}
\end{center}
\item \textit{Pas de correction automatisé}
\end{enumerate}
\end{enumerate}
\end{solution}
\end{document}
%%% Local Variables:
%%% mode: latex
%%% TeX-master: "master"
%%% End:

View File

@ -0,0 +1,262 @@
\documentclass[a4paper,10pt]{article}
\usepackage{myXsim}
% Title Page
\title{DS8 \hfill MOKHTARI Nissrine}
\tribe{TST}
\date{\hfillÀ render pour le Mercredi 7 avril}
\xsimsetup{
solution/print = true
}
\begin{document}
\maketitle
\begin{exercise}[subtitle={Automatismes}]
\textit{Toutes les questions de cette exercice sont indépendantes et peuvent être répondus séparément}
\begin{enumerate}
\item De janvier à septembre, une quantité a augmenté de $11\,\%$. Faire un schéma pour représenter la situation puis calculer le taux d'évolution moyen mensuel.
\item Une quantité augmente de $11\,\%$ par ans. En 2020, elle est de 138\euro. Quelle était sa valeur en 2019? Faire un schéma pour représenter la situation.
\item Déterminer l'équation de la droite \\
\begin{tikzpicture}[xscale=0.8, yscale=0.5]
\tkzInit[xmin=-5,xmax=5,xstep=1,
ymin=-5,ymax=5,ystep=1]
\tkzGrid
\tkzAxeXY
\tkzFct[domain=-5:5,color=red,very thick]%
{2.0*\x -3};
\end{tikzpicture}
\item Résoudre l'équation $8 \times 0.17^x = 35$
\end{enumerate}
\end{exercise}
\begin{solution}
\begin{enumerate}
\item On veut partager cette évolution en 8 évolutions.
\[
\left(1 + \frac{11}{100}\right)^{\frac{1}{8}} = 1.0131
\]
Donc le taux d'évolution moyen est
\[
t_m = 1.0131 - 1 = 0.01309999999999989
\]
\item Coefficient multiplicateur pour revenir en arrière
\[
CM = (1 + \frac{11}{100})^{-1} = 0.9009
\]
On en déduit la quantité en 2019
\[
138 * 0.9009 = 124.3242
\]
\item L'équation de la droite est
\[
y = 2.0 x -3
\]
\item Il faut penser à faire la division à par $8$ avant d'utiliser le log car sinon, on ne peut pas utiliser la formule $\log(a^n) = n\times \log(a)$.
\[x = \frac{\log(4.38)}{\log(0.17)}\]
\end{enumerate}
\end{solution}
\begin{exercise}[subtitle={Restaurant}]
Un \emph{food truck}, ouvert le midi et le soir, propose deux types de formules :
\setlength\parindent{10mm}
\begin{itemize}
\item la formule \emph{Burger} ;
\item la formule \emph{Wok}.
\end{itemize}
\setlength\parindent{0mm}
\medskip
Le gérant a remarqué que 62\,\% de ses ventes ont lieu le midi. Le quart des ventes du midi correspondent à la formule \emph{Burger}, alors que 28\,\% des ventes du soir correspondent à la formule \emph{Wok}.
Le gérant se constitue un fichier en notant, pour chaque vente, la formule choisie et le moment de cette vente (midi ou soir).
On prélève une fiche de façon équiprobable. On définit les quatre évènements suivants:
\begin{enumerate}
\item $M$ : \og la fiche correspond à une vente du midi\fg{} ;
\item $S$ : \og la fiche correspond à une vente du soir\fg {};
\item $W$ : \og la fiche correspond à une formule \emph{Wok} \fg{} ;
\item $B$ : \og la fiche correspond à une formule \emph{Burger} \fg.
\end{enumerate}
\setlength\parindent{0mm}
\medskip
\begin{enumerate}
\item Recopier puis compléter l'arbre pondéré
\begin{center}
\begin{tikzpicture}[sloped]
\node {.}
child {node {$M$}
child {node {$W$}
edge from parent
node[above] {...}
}
child {node {$B$}
edge from parent
node[above] {...}
}
edge from parent
node[above] {...}
}
child[missing] {}
child { node {$S$}
child {node {$W$}
edge from parent
node[above] {...}
}
child {node {$B$}
edge from parent
node[above] {...}
}
edge from parent
node[above] {...}
} ;
\end{tikzpicture}
\end{center}
\item Calculer la probabilité de l'évènement $M \cap W$. Interpréter ce résultat dans le contexte de l'exercice.
\item Montrer que la probabilité que la fiche choisie corresponde à une formule \emph{Burger} est égale à $0.4248$.
\item On a prélevé une fiche correspondant à la formule \emph{Burger}. Quelle est la probabilité, arrondie au millième, que la vente ait eu lieu le soir?
\end{enumerate}
\end{exercise}
\begin{solution}
\begin{enumerate}
\item
\begin{center}
\begin{tikzpicture}[sloped]
\node {.}
child {node {$M$}
child {node {$W$}
edge from parent
node[above] {$0.75$}
}
child {node {$B$}
edge from parent
node[above] {$0.25$}
}
edge from parent
node[above] {$0.62$}
}
child[missing] {}
child { node {$S$}
child {node {$W$}
edge from parent
node[above] {$0.29$}
}
child {node {$B$}
edge from parent
node[above] {$0.71$}
}
edge from parent
node[above] {$0.38$}
} ;
\end{tikzpicture}
\end{center}
\item On calcule la probabilité que la vente soit un wok et ait eu lieu à midi
\[ P(M\cap W) = P(M) \times P_M(W) = 0.62 \times 0.75 = 0.465 \]
\item Probabilité que la vente soit un burger.
\[
P(B) = P(M\cap B) + P(S\cap B) = 0.62 \times 0.75 + 0.38 \times 0.29 = 0.4248
\]
\item On cherche à calculer la quantité $P_B(S)$. Pour cela on utilise la formule de Bayes
\[
P_B(S) = \frac{P(B\cap S)}{P(B)} = \frac{P_S(B) \times P(S)}{P(B)} = \frac{0.71\times 0.38}{0.4248} = 0.6351224105461393 \approx 0.635
\]
\end{enumerate}
\end{solution}
\begin{exercise}[subtitle={Continent plastique}]
\textit{Les quantités évoqués dans cette exercice sont générés au hasard et sont donc complètement farfelus.}
\medskip
Le \og continent de plastique\fg{} est la plus grande des plaques de déchets plastiques évoluant sur les océans. Elle occupe actuellement dans l'océan Pacifique une surface dont l'aire est évaluée à plus de $1,6$ million de km$^2$, entre Hawaï et la Californie.
En 2017, des scientifiques ont estimé qu'il y avait $4$ millions de tonnes de déchets plastiques qui était déversé chaque année dans les océans et que cette quantité augmentait de $30\n\%$ par chaque année.
On modélise l'évolution de la masse de ces déchets plastiques déversée chaque année, si rien n'est fait pour la réduire, par une suite géométrique $\left(u_n\right)$. L'arrondi au centième du terme $u_n$ représente la masse de ces déchets déversée chaque année, exprimée en million de tonnes, pour l'année $(2017 + n)$.
\medskip
\begin{enumerate}
\item Expliquer pourquoi la suite $u_n$ est géométrique?
\item Calculer $u_1$ et $u_2$.
\item Exprimer $u_n$ en fonction de $n$.
\item Au début de l'année 2017, il y avait $300$ millions de tonnes de déchets plastique. Calculer la quantité totale de déchets plastiques en 2030.
\item On souhaite déterminer en quelle année la masse totale de ces déchets plastiques aura pour la première fois augmenté de $50$\,\% par rapport à sa valeur de 2017.
\begin{enumerate}
\item Recopier et compléter l'algorithme ci-dessous pour que la variable $N$ contienne la réponse au problème posé.
\begin{center}
\begin{tabularx}{0.4\linewidth}{|X|}\hline
$N = 2017$\\
$U = 4$ \\
$S = 300 + U$ \\
while $S < 450$: \\
\hspace{1cm} $N = \ldots$\\
\hspace{1cm} $U = \ldots$\\
\hspace{1cm} $S = \ldots$\\
\hline
\end{tabularx}
\end{center}
\item Que contiennent les variables $S$, $U$ et $N$ après exécution de cet algorithme ?
Interpréter les résultats dans le contexte de l'exercice.
\end{enumerate}
\end{enumerate}
\end{exercise}
\begin{solution}
\begin{enumerate}
\item Une augmentation de $30\,\%$ revient à multiplier la quantité par $1.3$. La suite est donc bien géométrique. Son premier terme est $u_0 = 4$ et sa raison est $q = 1.3$
\item
\[
u_1 = u_0 * 1.3 = 5.2
\]
\[
u_2 = u_0 * 1.3^2 = 6.76
\]
\item
\[
u_n = u_0 \times q^n = 4 \times 1.3^n
\]
\item On calcule la quantité totale déversée entre 2017 et 2030.
\[
\sum_{n = 0}^{13} u_n = u_0 \times \frac{1-q^{13}}{1-q} = 4 \times \frac{1 - 1.3^{13}}{1 - 1.3} = 390.5
\]
On en déduit la quantité totale de déchets en 2030
\[
300 + 390.5 = 690.5
\]
\item
\begin{enumerate}
\item ~
\begin{center}
\begin{tabularx}{0.4\linewidth}{|X|}\hline
$N \gets 2017$\\
$U \gets 4$ \\
$S \gets 300 + U$ \\
Tant que $S < 450$ \\
\hspace{1cm} $N \gets N + 1$\\
\hspace{1cm} $U \gets U * 1.3$\\
\hspace{1cm} $S \gets S + u$\\
Fin Tant que\\\hline
\end{tabularx}
\end{center}
\item \textit{Pas de correction automatisé}
\end{enumerate}
\end{enumerate}
\end{solution}
\end{document}
%%% Local Variables:
%%% mode: latex
%%% TeX-master: "master"
%%% End:

View File

@ -0,0 +1,262 @@
\documentclass[a4paper,10pt]{article}
\usepackage{myXsim}
% Title Page
\title{DS8 \hfill MOUFAQ Amine}
\tribe{TST}
\date{\hfillÀ render pour le Mercredi 7 avril}
\xsimsetup{
solution/print = true
}
\begin{document}
\maketitle
\begin{exercise}[subtitle={Automatismes}]
\textit{Toutes les questions de cette exercice sont indépendantes et peuvent être répondus séparément}
\begin{enumerate}
\item De janvier à septembre, une quantité a augmenté de $16\,\%$. Faire un schéma pour représenter la situation puis calculer le taux d'évolution moyen mensuel.
\item Une quantité augmente de $16\,\%$ par ans. En 2020, elle est de 115\euro. Quelle était sa valeur en 2019? Faire un schéma pour représenter la situation.
\item Déterminer l'équation de la droite \\
\begin{tikzpicture}[xscale=0.8, yscale=0.5]
\tkzInit[xmin=-5,xmax=5,xstep=1,
ymin=-5,ymax=5,ystep=1]
\tkzGrid
\tkzAxeXY
\tkzFct[domain=-5:5,color=red,very thick]%
{0.5*\x -1};
\end{tikzpicture}
\item Résoudre l'équation $6 \times 0.24^x = 41$
\end{enumerate}
\end{exercise}
\begin{solution}
\begin{enumerate}
\item On veut partager cette évolution en 8 évolutions.
\[
\left(1 + \frac{16}{100}\right)^{\frac{1}{8}} = 1.0187
\]
Donc le taux d'évolution moyen est
\[
t_m = 1.0187 - 1 = 0.01869999999999994
\]
\item Coefficient multiplicateur pour revenir en arrière
\[
CM = (1 + \frac{16}{100})^{-1} = 0.8621
\]
On en déduit la quantité en 2019
\[
115 * 0.8621 = 99.1415
\]
\item L'équation de la droite est
\[
y = 0.5 x -1
\]
\item Il faut penser à faire la division à par $6$ avant d'utiliser le log car sinon, on ne peut pas utiliser la formule $\log(a^n) = n\times \log(a)$.
\[x = \frac{\log(6.83)}{\log(0.24)}\]
\end{enumerate}
\end{solution}
\begin{exercise}[subtitle={Restaurant}]
Un \emph{food truck}, ouvert le midi et le soir, propose deux types de formules :
\setlength\parindent{10mm}
\begin{itemize}
\item la formule \emph{Burger} ;
\item la formule \emph{Wok}.
\end{itemize}
\setlength\parindent{0mm}
\medskip
Le gérant a remarqué que 78\,\% de ses ventes ont lieu le midi. Le quart des ventes du midi correspondent à la formule \emph{Burger}, alors que 37\,\% des ventes du soir correspondent à la formule \emph{Wok}.
Le gérant se constitue un fichier en notant, pour chaque vente, la formule choisie et le moment de cette vente (midi ou soir).
On prélève une fiche de façon équiprobable. On définit les quatre évènements suivants:
\begin{enumerate}
\item $M$ : \og la fiche correspond à une vente du midi\fg{} ;
\item $S$ : \og la fiche correspond à une vente du soir\fg {};
\item $W$ : \og la fiche correspond à une formule \emph{Wok} \fg{} ;
\item $B$ : \og la fiche correspond à une formule \emph{Burger} \fg.
\end{enumerate}
\setlength\parindent{0mm}
\medskip
\begin{enumerate}
\item Recopier puis compléter l'arbre pondéré
\begin{center}
\begin{tikzpicture}[sloped]
\node {.}
child {node {$M$}
child {node {$W$}
edge from parent
node[above] {...}
}
child {node {$B$}
edge from parent
node[above] {...}
}
edge from parent
node[above] {...}
}
child[missing] {}
child { node {$S$}
child {node {$W$}
edge from parent
node[above] {...}
}
child {node {$B$}
edge from parent
node[above] {...}
}
edge from parent
node[above] {...}
} ;
\end{tikzpicture}
\end{center}
\item Calculer la probabilité de l'évènement $M \cap W$. Interpréter ce résultat dans le contexte de l'exercice.
\item Montrer que la probabilité que la fiche choisie corresponde à une formule \emph{Burger} est égale à $0.3336$.
\item On a prélevé une fiche correspondant à la formule \emph{Burger}. Quelle est la probabilité, arrondie au millième, que la vente ait eu lieu le soir?
\end{enumerate}
\end{exercise}
\begin{solution}
\begin{enumerate}
\item
\begin{center}
\begin{tikzpicture}[sloped]
\node {.}
child {node {$M$}
child {node {$W$}
edge from parent
node[above] {$0.75$}
}
child {node {$B$}
edge from parent
node[above] {$0.25$}
}
edge from parent
node[above] {$0.78$}
}
child[missing] {}
child { node {$S$}
child {node {$W$}
edge from parent
node[above] {$0.37$}
}
child {node {$B$}
edge from parent
node[above] {$0.63$}
}
edge from parent
node[above] {$0.22$}
} ;
\end{tikzpicture}
\end{center}
\item On calcule la probabilité que la vente soit un wok et ait eu lieu à midi
\[ P(M\cap W) = P(M) \times P_M(W) = 0.78 \times 0.75 = 0.585 \]
\item Probabilité que la vente soit un burger.
\[
P(B) = P(M\cap B) + P(S\cap B) = 0.78 \times 0.75 + 0.22 \times 0.37 = 0.3336
\]
\item On cherche à calculer la quantité $P_B(S)$. Pour cela on utilise la formule de Bayes
\[
P_B(S) = \frac{P(B\cap S)}{P(B)} = \frac{P_S(B) \times P(S)}{P(B)} = \frac{0.63\times 0.22}{0.3336} = 0.41546762589928055 \approx 0.415
\]
\end{enumerate}
\end{solution}
\begin{exercise}[subtitle={Continent plastique}]
\textit{Les quantités évoqués dans cette exercice sont générés au hasard et sont donc complètement farfelus.}
\medskip
Le \og continent de plastique\fg{} est la plus grande des plaques de déchets plastiques évoluant sur les océans. Elle occupe actuellement dans l'océan Pacifique une surface dont l'aire est évaluée à plus de $1,6$ million de km$^2$, entre Hawaï et la Californie.
En 2017, des scientifiques ont estimé qu'il y avait $10$ millions de tonnes de déchets plastiques qui était déversé chaque année dans les océans et que cette quantité augmentait de $24\n\%$ par chaque année.
On modélise l'évolution de la masse de ces déchets plastiques déversée chaque année, si rien n'est fait pour la réduire, par une suite géométrique $\left(u_n\right)$. L'arrondi au centième du terme $u_n$ représente la masse de ces déchets déversée chaque année, exprimée en million de tonnes, pour l'année $(2017 + n)$.
\medskip
\begin{enumerate}
\item Expliquer pourquoi la suite $u_n$ est géométrique?
\item Calculer $u_1$ et $u_2$.
\item Exprimer $u_n$ en fonction de $n$.
\item Au début de l'année 2017, il y avait $300$ millions de tonnes de déchets plastique. Calculer la quantité totale de déchets plastiques en 2030.
\item On souhaite déterminer en quelle année la masse totale de ces déchets plastiques aura pour la première fois augmenté de $50$\,\% par rapport à sa valeur de 2017.
\begin{enumerate}
\item Recopier et compléter l'algorithme ci-dessous pour que la variable $N$ contienne la réponse au problème posé.
\begin{center}
\begin{tabularx}{0.4\linewidth}{|X|}\hline
$N = 2017$\\
$U = 10$ \\
$S = 300 + U$ \\
while $S < 450$: \\
\hspace{1cm} $N = \ldots$\\
\hspace{1cm} $U = \ldots$\\
\hspace{1cm} $S = \ldots$\\
\hline
\end{tabularx}
\end{center}
\item Que contiennent les variables $S$, $U$ et $N$ après exécution de cet algorithme ?
Interpréter les résultats dans le contexte de l'exercice.
\end{enumerate}
\end{enumerate}
\end{exercise}
\begin{solution}
\begin{enumerate}
\item Une augmentation de $24\,\%$ revient à multiplier la quantité par $1.24$. La suite est donc bien géométrique. Son premier terme est $u_0 = 10$ et sa raison est $q = 1.24$
\item
\[
u_1 = u_0 * 1.24 = 12.4
\]
\[
u_2 = u_0 * 1.24^2 = 15.376
\]
\item
\[
u_n = u_0 \times q^n = 10 \times 1.24^n
\]
\item On calcule la quantité totale déversée entre 2017 et 2030.
\[
\sum_{n = 0}^{13} u_n = u_0 \times \frac{1-q^{13}}{1-q} = 10 \times \frac{1 - 1.24^{13}}{1 - 1.24} = 641.1
\]
On en déduit la quantité totale de déchets en 2030
\[
300 + 641.1 = 941.1
\]
\item
\begin{enumerate}
\item ~
\begin{center}
\begin{tabularx}{0.4\linewidth}{|X|}\hline
$N \gets 2017$\\
$U \gets 10$ \\
$S \gets 300 + U$ \\
Tant que $S < 450$ \\
\hspace{1cm} $N \gets N + 1$\\
\hspace{1cm} $U \gets U * 1.24$\\
\hspace{1cm} $S \gets S + u$\\
Fin Tant que\\\hline
\end{tabularx}
\end{center}
\item \textit{Pas de correction automatisé}
\end{enumerate}
\end{enumerate}
\end{solution}
\end{document}
%%% Local Variables:
%%% mode: latex
%%% TeX-master: "master"
%%% End:

View File

@ -0,0 +1,262 @@
\documentclass[a4paper,10pt]{article}
\usepackage{myXsim}
% Title Page
\title{DS8 \hfill ONAL Yakub}
\tribe{TST}
\date{\hfillÀ render pour le Mercredi 7 avril}
\xsimsetup{
solution/print = true
}
\begin{document}
\maketitle
\begin{exercise}[subtitle={Automatismes}]
\textit{Toutes les questions de cette exercice sont indépendantes et peuvent être répondus séparément}
\begin{enumerate}
\item De janvier à septembre, une quantité a augmenté de $27\,\%$. Faire un schéma pour représenter la situation puis calculer le taux d'évolution moyen mensuel.
\item Une quantité augmente de $27\,\%$ par ans. En 2020, elle est de 132\euro. Quelle était sa valeur en 2019? Faire un schéma pour représenter la situation.
\item Déterminer l'équation de la droite \\
\begin{tikzpicture}[xscale=0.8, yscale=0.5]
\tkzInit[xmin=-5,xmax=5,xstep=1,
ymin=-5,ymax=5,ystep=1]
\tkzGrid
\tkzAxeXY
\tkzFct[domain=-5:5,color=red,very thick]%
{1.0*\x -1};
\end{tikzpicture}
\item Résoudre l'équation $10 \times 0.18^x = 7$
\end{enumerate}
\end{exercise}
\begin{solution}
\begin{enumerate}
\item On veut partager cette évolution en 8 évolutions.
\[
\left(1 + \frac{27}{100}\right)^{\frac{1}{8}} = 1.0303
\]
Donc le taux d'évolution moyen est
\[
t_m = 1.0303 - 1 = 0.030299999999999994
\]
\item Coefficient multiplicateur pour revenir en arrière
\[
CM = (1 + \frac{27}{100})^{-1} = 0.7874
\]
On en déduit la quantité en 2019
\[
132 * 0.7874 = 103.9368
\]
\item L'équation de la droite est
\[
y = 1.0 x -1
\]
\item Il faut penser à faire la division à par $10$ avant d'utiliser le log car sinon, on ne peut pas utiliser la formule $\log(a^n) = n\times \log(a)$.
\[x = \frac{\log(0.7)}{\log(0.18)}\]
\end{enumerate}
\end{solution}
\begin{exercise}[subtitle={Restaurant}]
Un \emph{food truck}, ouvert le midi et le soir, propose deux types de formules :
\setlength\parindent{10mm}
\begin{itemize}
\item la formule \emph{Burger} ;
\item la formule \emph{Wok}.
\end{itemize}
\setlength\parindent{0mm}
\medskip
Le gérant a remarqué que 78\,\% de ses ventes ont lieu le midi. Le quart des ventes du midi correspondent à la formule \emph{Burger}, alors que 28\,\% des ventes du soir correspondent à la formule \emph{Wok}.
Le gérant se constitue un fichier en notant, pour chaque vente, la formule choisie et le moment de cette vente (midi ou soir).
On prélève une fiche de façon équiprobable. On définit les quatre évènements suivants:
\begin{enumerate}
\item $M$ : \og la fiche correspond à une vente du midi\fg{} ;
\item $S$ : \og la fiche correspond à une vente du soir\fg {};
\item $W$ : \og la fiche correspond à une formule \emph{Wok} \fg{} ;
\item $B$ : \og la fiche correspond à une formule \emph{Burger} \fg.
\end{enumerate}
\setlength\parindent{0mm}
\medskip
\begin{enumerate}
\item Recopier puis compléter l'arbre pondéré
\begin{center}
\begin{tikzpicture}[sloped]
\node {.}
child {node {$M$}
child {node {$W$}
edge from parent
node[above] {...}
}
child {node {$B$}
edge from parent
node[above] {...}
}
edge from parent
node[above] {...}
}
child[missing] {}
child { node {$S$}
child {node {$W$}
edge from parent
node[above] {...}
}
child {node {$B$}
edge from parent
node[above] {...}
}
edge from parent
node[above] {...}
} ;
\end{tikzpicture}
\end{center}
\item Calculer la probabilité de l'évènement $M \cap W$. Interpréter ce résultat dans le contexte de l'exercice.
\item Montrer que la probabilité que la fiche choisie corresponde à une formule \emph{Burger} est égale à $0.3534$.
\item On a prélevé une fiche correspondant à la formule \emph{Burger}. Quelle est la probabilité, arrondie au millième, que la vente ait eu lieu le soir?
\end{enumerate}
\end{exercise}
\begin{solution}
\begin{enumerate}
\item
\begin{center}
\begin{tikzpicture}[sloped]
\node {.}
child {node {$M$}
child {node {$W$}
edge from parent
node[above] {$0.75$}
}
child {node {$B$}
edge from parent
node[above] {$0.25$}
}
edge from parent
node[above] {$0.78$}
}
child[missing] {}
child { node {$S$}
child {node {$W$}
edge from parent
node[above] {$0.28$}
}
child {node {$B$}
edge from parent
node[above] {$0.72$}
}
edge from parent
node[above] {$0.22$}
} ;
\end{tikzpicture}
\end{center}
\item On calcule la probabilité que la vente soit un wok et ait eu lieu à midi
\[ P(M\cap W) = P(M) \times P_M(W) = 0.78 \times 0.75 = 0.585 \]
\item Probabilité que la vente soit un burger.
\[
P(B) = P(M\cap B) + P(S\cap B) = 0.78 \times 0.75 + 0.22 \times 0.28 = 0.3534
\]
\item On cherche à calculer la quantité $P_B(S)$. Pour cela on utilise la formule de Bayes
\[
P_B(S) = \frac{P(B\cap S)}{P(B)} = \frac{P_S(B) \times P(S)}{P(B)} = \frac{0.72\times 0.22}{0.3534} = 0.4482173174872665 \approx 0.448
\]
\end{enumerate}
\end{solution}
\begin{exercise}[subtitle={Continent plastique}]
\textit{Les quantités évoqués dans cette exercice sont générés au hasard et sont donc complètement farfelus.}
\medskip
Le \og continent de plastique\fg{} est la plus grande des plaques de déchets plastiques évoluant sur les océans. Elle occupe actuellement dans l'océan Pacifique une surface dont l'aire est évaluée à plus de $1,6$ million de km$^2$, entre Hawaï et la Californie.
En 2017, des scientifiques ont estimé qu'il y avait $3$ millions de tonnes de déchets plastiques qui était déversé chaque année dans les océans et que cette quantité augmentait de $21\n\%$ par chaque année.
On modélise l'évolution de la masse de ces déchets plastiques déversée chaque année, si rien n'est fait pour la réduire, par une suite géométrique $\left(u_n\right)$. L'arrondi au centième du terme $u_n$ représente la masse de ces déchets déversée chaque année, exprimée en million de tonnes, pour l'année $(2017 + n)$.
\medskip
\begin{enumerate}
\item Expliquer pourquoi la suite $u_n$ est géométrique?
\item Calculer $u_1$ et $u_2$.
\item Exprimer $u_n$ en fonction de $n$.
\item Au début de l'année 2017, il y avait $300$ millions de tonnes de déchets plastique. Calculer la quantité totale de déchets plastiques en 2030.
\item On souhaite déterminer en quelle année la masse totale de ces déchets plastiques aura pour la première fois augmenté de $50$\,\% par rapport à sa valeur de 2017.
\begin{enumerate}
\item Recopier et compléter l'algorithme ci-dessous pour que la variable $N$ contienne la réponse au problème posé.
\begin{center}
\begin{tabularx}{0.4\linewidth}{|X|}\hline
$N = 2017$\\
$U = 3$ \\
$S = 300 + U$ \\
while $S < 450$: \\
\hspace{1cm} $N = \ldots$\\
\hspace{1cm} $U = \ldots$\\
\hspace{1cm} $S = \ldots$\\
\hline
\end{tabularx}
\end{center}
\item Que contiennent les variables $S$, $U$ et $N$ après exécution de cet algorithme ?
Interpréter les résultats dans le contexte de l'exercice.
\end{enumerate}
\end{enumerate}
\end{exercise}
\begin{solution}
\begin{enumerate}
\item Une augmentation de $21\,\%$ revient à multiplier la quantité par $1.21$. La suite est donc bien géométrique. Son premier terme est $u_0 = 3$ et sa raison est $q = 1.21$
\item
\[
u_1 = u_0 * 1.21 = 3.63
\]
\[
u_2 = u_0 * 1.21^2 = 4.3923
\]
\item
\[
u_n = u_0 \times q^n = 3 \times 1.21^n
\]
\item On calcule la quantité totale déversée entre 2017 et 2030.
\[
\sum_{n = 0}^{13} u_n = u_0 \times \frac{1-q^{13}}{1-q} = 3 \times \frac{1 - 1.21^{13}}{1 - 1.21} = 155.97
\]
On en déduit la quantité totale de déchets en 2030
\[
300 + 155.97 = 455.97
\]
\item
\begin{enumerate}
\item ~
\begin{center}
\begin{tabularx}{0.4\linewidth}{|X|}\hline
$N \gets 2017$\\
$U \gets 3$ \\
$S \gets 300 + U$ \\
Tant que $S < 450$ \\
\hspace{1cm} $N \gets N + 1$\\
\hspace{1cm} $U \gets U * 1.21$\\
\hspace{1cm} $S \gets S + u$\\
Fin Tant que\\\hline
\end{tabularx}
\end{center}
\item \textit{Pas de correction automatisé}
\end{enumerate}
\end{enumerate}
\end{solution}
\end{document}
%%% Local Variables:
%%% mode: latex
%%% TeX-master: "master"
%%% End:

View File

@ -0,0 +1,262 @@
\documentclass[a4paper,10pt]{article}
\usepackage{myXsim}
% Title Page
\title{DS8 \hfill SORIANO Laura}
\tribe{TST}
\date{\hfillÀ render pour le Mercredi 7 avril}
\xsimsetup{
solution/print = true
}
\begin{document}
\maketitle
\begin{exercise}[subtitle={Automatismes}]
\textit{Toutes les questions de cette exercice sont indépendantes et peuvent être répondus séparément}
\begin{enumerate}
\item De janvier à septembre, une quantité a augmenté de $26\,\%$. Faire un schéma pour représenter la situation puis calculer le taux d'évolution moyen mensuel.
\item Une quantité augmente de $26\,\%$ par ans. En 2020, elle est de 122\euro. Quelle était sa valeur en 2019? Faire un schéma pour représenter la situation.
\item Déterminer l'équation de la droite \\
\begin{tikzpicture}[xscale=0.8, yscale=0.5]
\tkzInit[xmin=-5,xmax=5,xstep=1,
ymin=-5,ymax=5,ystep=1]
\tkzGrid
\tkzAxeXY
\tkzFct[domain=-5:5,color=red,very thick]%
{3.0*\x -3};
\end{tikzpicture}
\item Résoudre l'équation $2 \times 0.02^x = 10$
\end{enumerate}
\end{exercise}
\begin{solution}
\begin{enumerate}
\item On veut partager cette évolution en 8 évolutions.
\[
\left(1 + \frac{26}{100}\right)^{\frac{1}{8}} = 1.0293
\]
Donc le taux d'évolution moyen est
\[
t_m = 1.0293 - 1 = 0.029300000000000104
\]
\item Coefficient multiplicateur pour revenir en arrière
\[
CM = (1 + \frac{26}{100})^{-1} = 0.7937
\]
On en déduit la quantité en 2019
\[
122 * 0.7937 = 96.8314
\]
\item L'équation de la droite est
\[
y = 3.0 x -3
\]
\item Il faut penser à faire la division à par $2$ avant d'utiliser le log car sinon, on ne peut pas utiliser la formule $\log(a^n) = n\times \log(a)$.
\[x = \frac{\log(5.0)}{\log(0.02)}\]
\end{enumerate}
\end{solution}
\begin{exercise}[subtitle={Restaurant}]
Un \emph{food truck}, ouvert le midi et le soir, propose deux types de formules :
\setlength\parindent{10mm}
\begin{itemize}
\item la formule \emph{Burger} ;
\item la formule \emph{Wok}.
\end{itemize}
\setlength\parindent{0mm}
\medskip
Le gérant a remarqué que 0\,\% de ses ventes ont lieu le midi. Le quart des ventes du midi correspondent à la formule \emph{Burger}, alors que 21\,\% des ventes du soir correspondent à la formule \emph{Wok}.
Le gérant se constitue un fichier en notant, pour chaque vente, la formule choisie et le moment de cette vente (midi ou soir).
On prélève une fiche de façon équiprobable. On définit les quatre évènements suivants:
\begin{enumerate}
\item $M$ : \og la fiche correspond à une vente du midi\fg{} ;
\item $S$ : \og la fiche correspond à une vente du soir\fg {};
\item $W$ : \og la fiche correspond à une formule \emph{Wok} \fg{} ;
\item $B$ : \og la fiche correspond à une formule \emph{Burger} \fg.
\end{enumerate}
\setlength\parindent{0mm}
\medskip
\begin{enumerate}
\item Recopier puis compléter l'arbre pondéré
\begin{center}
\begin{tikzpicture}[sloped]
\node {.}
child {node {$M$}
child {node {$W$}
edge from parent
node[above] {...}
}
child {node {$B$}
edge from parent
node[above] {...}
}
edge from parent
node[above] {...}
}
child[missing] {}
child { node {$S$}
child {node {$W$}
edge from parent
node[above] {...}
}
child {node {$B$}
edge from parent
node[above] {...}
}
edge from parent
node[above] {...}
} ;
\end{tikzpicture}
\end{center}
\item Calculer la probabilité de l'évènement $M \cap W$. Interpréter ce résultat dans le contexte de l'exercice.
\item Montrer que la probabilité que la fiche choisie corresponde à une formule \emph{Burger} est égale à $0.79$.
\item On a prélevé une fiche correspondant à la formule \emph{Burger}. Quelle est la probabilité, arrondie au millième, que la vente ait eu lieu le soir?
\end{enumerate}
\end{exercise}
\begin{solution}
\begin{enumerate}
\item
\begin{center}
\begin{tikzpicture}[sloped]
\node {.}
child {node {$M$}
child {node {$W$}
edge from parent
node[above] {$0.75$}
}
child {node {$B$}
edge from parent
node[above] {$0.25$}
}
edge from parent
node[above] {$0.0$}
}
child[missing] {}
child { node {$S$}
child {node {$W$}
edge from parent
node[above] {$0.21$}
}
child {node {$B$}
edge from parent
node[above] {$0.79$}
}
edge from parent
node[above] {$1.0$}
} ;
\end{tikzpicture}
\end{center}
\item On calcule la probabilité que la vente soit un wok et ait eu lieu à midi
\[ P(M\cap W) = P(M) \times P_M(W) = 0.0 \times 0.75 = 0.0 \]
\item Probabilité que la vente soit un burger.
\[
P(B) = P(M\cap B) + P(S\cap B) = 0.0 \times 0.75 + 1.0 \times 0.21 = 0.79
\]
\item On cherche à calculer la quantité $P_B(S)$. Pour cela on utilise la formule de Bayes
\[
P_B(S) = \frac{P(B\cap S)}{P(B)} = \frac{P_S(B) \times P(S)}{P(B)} = \frac{0.79\times 1.0}{0.79} = 1.0 \approx 1.0
\]
\end{enumerate}
\end{solution}
\begin{exercise}[subtitle={Continent plastique}]
\textit{Les quantités évoqués dans cette exercice sont générés au hasard et sont donc complètement farfelus.}
\medskip
Le \og continent de plastique\fg{} est la plus grande des plaques de déchets plastiques évoluant sur les océans. Elle occupe actuellement dans l'océan Pacifique une surface dont l'aire est évaluée à plus de $1,6$ million de km$^2$, entre Hawaï et la Californie.
En 2017, des scientifiques ont estimé qu'il y avait $11$ millions de tonnes de déchets plastiques qui était déversé chaque année dans les océans et que cette quantité augmentait de $16\n\%$ par chaque année.
On modélise l'évolution de la masse de ces déchets plastiques déversée chaque année, si rien n'est fait pour la réduire, par une suite géométrique $\left(u_n\right)$. L'arrondi au centième du terme $u_n$ représente la masse de ces déchets déversée chaque année, exprimée en million de tonnes, pour l'année $(2017 + n)$.
\medskip
\begin{enumerate}
\item Expliquer pourquoi la suite $u_n$ est géométrique?
\item Calculer $u_1$ et $u_2$.
\item Exprimer $u_n$ en fonction de $n$.
\item Au début de l'année 2017, il y avait $300$ millions de tonnes de déchets plastique. Calculer la quantité totale de déchets plastiques en 2030.
\item On souhaite déterminer en quelle année la masse totale de ces déchets plastiques aura pour la première fois augmenté de $50$\,\% par rapport à sa valeur de 2017.
\begin{enumerate}
\item Recopier et compléter l'algorithme ci-dessous pour que la variable $N$ contienne la réponse au problème posé.
\begin{center}
\begin{tabularx}{0.4\linewidth}{|X|}\hline
$N = 2017$\\
$U = 11$ \\
$S = 300 + U$ \\
while $S < 450$: \\
\hspace{1cm} $N = \ldots$\\
\hspace{1cm} $U = \ldots$\\
\hspace{1cm} $S = \ldots$\\
\hline
\end{tabularx}
\end{center}
\item Que contiennent les variables $S$, $U$ et $N$ après exécution de cet algorithme ?
Interpréter les résultats dans le contexte de l'exercice.
\end{enumerate}
\end{enumerate}
\end{exercise}
\begin{solution}
\begin{enumerate}
\item Une augmentation de $16\,\%$ revient à multiplier la quantité par $1.16$. La suite est donc bien géométrique. Son premier terme est $u_0 = 11$ et sa raison est $q = 1.16$
\item
\[
u_1 = u_0 * 1.16 = 12.76
\]
\[
u_2 = u_0 * 1.16^2 = 14.8016
\]
\item
\[
u_n = u_0 \times q^n = 11 \times 1.16^n
\]
\item On calcule la quantité totale déversée entre 2017 et 2030.
\[
\sum_{n = 0}^{13} u_n = u_0 \times \frac{1-q^{13}}{1-q} = 11 \times \frac{1 - 1.16^{13}}{1 - 1.16} = 404.65
\]
On en déduit la quantité totale de déchets en 2030
\[
300 + 404.65 = 704.65
\]
\item
\begin{enumerate}
\item ~
\begin{center}
\begin{tabularx}{0.4\linewidth}{|X|}\hline
$N \gets 2017$\\
$U \gets 11$ \\
$S \gets 300 + U$ \\
Tant que $S < 450$ \\
\hspace{1cm} $N \gets N + 1$\\
\hspace{1cm} $U \gets U * 1.16$\\
\hspace{1cm} $S \gets S + u$\\
Fin Tant que\\\hline
\end{tabularx}
\end{center}
\item \textit{Pas de correction automatisé}
\end{enumerate}
\end{enumerate}
\end{solution}
\end{document}
%%% Local Variables:
%%% mode: latex
%%% TeX-master: "master"
%%% End:

View File

@ -0,0 +1,262 @@
\documentclass[a4paper,10pt]{article}
\usepackage{myXsim}
% Title Page
\title{DS8 \hfill VECCHIO Léa}
\tribe{TST}
\date{\hfillÀ render pour le Mercredi 7 avril}
\xsimsetup{
solution/print = true
}
\begin{document}
\maketitle
\begin{exercise}[subtitle={Automatismes}]
\textit{Toutes les questions de cette exercice sont indépendantes et peuvent être répondus séparément}
\begin{enumerate}
\item De janvier à septembre, une quantité a augmenté de $28\,\%$. Faire un schéma pour représenter la situation puis calculer le taux d'évolution moyen mensuel.
\item Une quantité augmente de $28\,\%$ par ans. En 2020, elle est de 113\euro. Quelle était sa valeur en 2019? Faire un schéma pour représenter la situation.
\item Déterminer l'équation de la droite \\
\begin{tikzpicture}[xscale=0.8, yscale=0.5]
\tkzInit[xmin=-5,xmax=5,xstep=1,
ymin=-5,ymax=5,ystep=1]
\tkzGrid
\tkzAxeXY
\tkzFct[domain=-5:5,color=red,very thick]%
{2.0*\x -2};
\end{tikzpicture}
\item Résoudre l'équation $8 \times 0.84^x = 26$
\end{enumerate}
\end{exercise}
\begin{solution}
\begin{enumerate}
\item On veut partager cette évolution en 8 évolutions.
\[
\left(1 + \frac{28}{100}\right)^{\frac{1}{8}} = 1.0313
\]
Donc le taux d'évolution moyen est
\[
t_m = 1.0313 - 1 = 0.031300000000000106
\]
\item Coefficient multiplicateur pour revenir en arrière
\[
CM = (1 + \frac{28}{100})^{-1} = 0.7812
\]
On en déduit la quantité en 2019
\[
113 * 0.7812 = 88.2756
\]
\item L'équation de la droite est
\[
y = 2.0 x -2
\]
\item Il faut penser à faire la division à par $8$ avant d'utiliser le log car sinon, on ne peut pas utiliser la formule $\log(a^n) = n\times \log(a)$.
\[x = \frac{\log(3.25)}{\log(0.84)}\]
\end{enumerate}
\end{solution}
\begin{exercise}[subtitle={Restaurant}]
Un \emph{food truck}, ouvert le midi et le soir, propose deux types de formules :
\setlength\parindent{10mm}
\begin{itemize}
\item la formule \emph{Burger} ;
\item la formule \emph{Wok}.
\end{itemize}
\setlength\parindent{0mm}
\medskip
Le gérant a remarqué que 18\,\% de ses ventes ont lieu le midi. Le quart des ventes du midi correspondent à la formule \emph{Burger}, alors que 16\,\% des ventes du soir correspondent à la formule \emph{Wok}.
Le gérant se constitue un fichier en notant, pour chaque vente, la formule choisie et le moment de cette vente (midi ou soir).
On prélève une fiche de façon équiprobable. On définit les quatre évènements suivants:
\begin{enumerate}
\item $M$ : \og la fiche correspond à une vente du midi\fg{} ;
\item $S$ : \og la fiche correspond à une vente du soir\fg {};
\item $W$ : \og la fiche correspond à une formule \emph{Wok} \fg{} ;
\item $B$ : \og la fiche correspond à une formule \emph{Burger} \fg.
\end{enumerate}
\setlength\parindent{0mm}
\medskip
\begin{enumerate}
\item Recopier puis compléter l'arbre pondéré
\begin{center}
\begin{tikzpicture}[sloped]
\node {.}
child {node {$M$}
child {node {$W$}
edge from parent
node[above] {...}
}
child {node {$B$}
edge from parent
node[above] {...}
}
edge from parent
node[above] {...}
}
child[missing] {}
child { node {$S$}
child {node {$W$}
edge from parent
node[above] {...}
}
child {node {$B$}
edge from parent
node[above] {...}
}
edge from parent
node[above] {...}
} ;
\end{tikzpicture}
\end{center}
\item Calculer la probabilité de l'évènement $M \cap W$. Interpréter ce résultat dans le contexte de l'exercice.
\item Montrer que la probabilité que la fiche choisie corresponde à une formule \emph{Burger} est égale à $0.7338$.
\item On a prélevé une fiche correspondant à la formule \emph{Burger}. Quelle est la probabilité, arrondie au millième, que la vente ait eu lieu le soir?
\end{enumerate}
\end{exercise}
\begin{solution}
\begin{enumerate}
\item
\begin{center}
\begin{tikzpicture}[sloped]
\node {.}
child {node {$M$}
child {node {$W$}
edge from parent
node[above] {$0.75$}
}
child {node {$B$}
edge from parent
node[above] {$0.25$}
}
edge from parent
node[above] {$0.18$}
}
child[missing] {}
child { node {$S$}
child {node {$W$}
edge from parent
node[above] {$0.16$}
}
child {node {$B$}
edge from parent
node[above] {$0.84$}
}
edge from parent
node[above] {$0.82$}
} ;
\end{tikzpicture}
\end{center}
\item On calcule la probabilité que la vente soit un wok et ait eu lieu à midi
\[ P(M\cap W) = P(M) \times P_M(W) = 0.18 \times 0.75 = 0.135 \]
\item Probabilité que la vente soit un burger.
\[
P(B) = P(M\cap B) + P(S\cap B) = 0.18 \times 0.75 + 0.82 \times 0.16 = 0.7338
\]
\item On cherche à calculer la quantité $P_B(S)$. Pour cela on utilise la formule de Bayes
\[
P_B(S) = \frac{P(B\cap S)}{P(B)} = \frac{P_S(B) \times P(S)}{P(B)} = \frac{0.84\times 0.82}{0.7338} = 0.9386753883892068 \approx 0.939
\]
\end{enumerate}
\end{solution}
\begin{exercise}[subtitle={Continent plastique}]
\textit{Les quantités évoqués dans cette exercice sont générés au hasard et sont donc complètement farfelus.}
\medskip
Le \og continent de plastique\fg{} est la plus grande des plaques de déchets plastiques évoluant sur les océans. Elle occupe actuellement dans l'océan Pacifique une surface dont l'aire est évaluée à plus de $1,6$ million de km$^2$, entre Hawaï et la Californie.
En 2017, des scientifiques ont estimé qu'il y avait $2$ millions de tonnes de déchets plastiques qui était déversé chaque année dans les océans et que cette quantité augmentait de $24\n\%$ par chaque année.
On modélise l'évolution de la masse de ces déchets plastiques déversée chaque année, si rien n'est fait pour la réduire, par une suite géométrique $\left(u_n\right)$. L'arrondi au centième du terme $u_n$ représente la masse de ces déchets déversée chaque année, exprimée en million de tonnes, pour l'année $(2017 + n)$.
\medskip
\begin{enumerate}
\item Expliquer pourquoi la suite $u_n$ est géométrique?
\item Calculer $u_1$ et $u_2$.
\item Exprimer $u_n$ en fonction de $n$.
\item Au début de l'année 2017, il y avait $300$ millions de tonnes de déchets plastique. Calculer la quantité totale de déchets plastiques en 2030.
\item On souhaite déterminer en quelle année la masse totale de ces déchets plastiques aura pour la première fois augmenté de $50$\,\% par rapport à sa valeur de 2017.
\begin{enumerate}
\item Recopier et compléter l'algorithme ci-dessous pour que la variable $N$ contienne la réponse au problème posé.
\begin{center}
\begin{tabularx}{0.4\linewidth}{|X|}\hline
$N = 2017$\\
$U = 2$ \\
$S = 300 + U$ \\
while $S < 450$: \\
\hspace{1cm} $N = \ldots$\\
\hspace{1cm} $U = \ldots$\\
\hspace{1cm} $S = \ldots$\\
\hline
\end{tabularx}
\end{center}
\item Que contiennent les variables $S$, $U$ et $N$ après exécution de cet algorithme ?
Interpréter les résultats dans le contexte de l'exercice.
\end{enumerate}
\end{enumerate}
\end{exercise}
\begin{solution}
\begin{enumerate}
\item Une augmentation de $24\,\%$ revient à multiplier la quantité par $1.24$. La suite est donc bien géométrique. Son premier terme est $u_0 = 2$ et sa raison est $q = 1.24$
\item
\[
u_1 = u_0 * 1.24 = 2.48
\]
\[
u_2 = u_0 * 1.24^2 = 3.0752
\]
\item
\[
u_n = u_0 \times q^n = 2 \times 1.24^n
\]
\item On calcule la quantité totale déversée entre 2017 et 2030.
\[
\sum_{n = 0}^{13} u_n = u_0 \times \frac{1-q^{13}}{1-q} = 2 \times \frac{1 - 1.24^{13}}{1 - 1.24} = 128.22
\]
On en déduit la quantité totale de déchets en 2030
\[
300 + 128.22 = 428.22
\]
\item
\begin{enumerate}
\item ~
\begin{center}
\begin{tabularx}{0.4\linewidth}{|X|}\hline
$N \gets 2017$\\
$U \gets 2$ \\
$S \gets 300 + U$ \\
Tant que $S < 450$ \\
\hspace{1cm} $N \gets N + 1$\\
\hspace{1cm} $U \gets U * 1.24$\\
\hspace{1cm} $S \gets S + u$\\
Fin Tant que\\\hline
\end{tabularx}
\end{center}
\item \textit{Pas de correction automatisé}
\end{enumerate}
\end{enumerate}
\end{solution}
\end{document}
%%% Local Variables:
%%% mode: latex
%%% TeX-master: "master"
%%% End:

Binary file not shown.

View File

@ -0,0 +1,262 @@
\documentclass[a4paper,10pt]{article}
\usepackage{myXsim}
% Title Page
\title{DS8 \hfill BELARBI Samira}
\tribe{TST}
\date{\hfillÀ render pour le Mercredi 7 avril}
\xsimsetup{
solution/print = false
}
\begin{document}
\maketitle
\begin{exercise}[subtitle={Automatismes}]
\textit{Toutes les questions de cette exercice sont indépendantes et peuvent être répondus séparément}
\begin{enumerate}
\item De janvier à septembre, une quantité a augmenté de $21\,\%$. Faire un schéma pour représenter la situation puis calculer le taux d'évolution moyen mensuel.
\item Une quantité augmente de $21\,\%$ par ans. En 2020, elle est de 143\euro. Quelle était sa valeur en 2019? Faire un schéma pour représenter la situation.
\item Déterminer l'équation de la droite \\
\begin{tikzpicture}[xscale=0.8, yscale=0.5]
\tkzInit[xmin=-5,xmax=5,xstep=1,
ymin=-5,ymax=5,ystep=1]
\tkzGrid
\tkzAxeXY
\tkzFct[domain=-5:5,color=red,very thick]%
{2.6666666666666665*\x -4};
\end{tikzpicture}
\item Résoudre l'équation $8 \times 0.76^x = 26$
\end{enumerate}
\end{exercise}
\begin{solution}
\begin{enumerate}
\item On veut partager cette évolution en 8 évolutions.
\[
\left(1 + \frac{21}{100}\right)^{\frac{1}{8}} = 1.0241
\]
Donc le taux d'évolution moyen est
\[
t_m = 1.0241 - 1 = 0.02410000000000001
\]
\item Coefficient multiplicateur pour revenir en arrière
\[
CM = (1 + \frac{21}{100})^{-1} = 0.8264
\]
On en déduit la quantité en 2019
\[
143 * 0.8264 = 118.1752
\]
\item L'équation de la droite est
\[
y = 2.6666666666666665 x -4
\]
\item Il faut penser à faire la division à par $8$ avant d'utiliser le log car sinon, on ne peut pas utiliser la formule $\log(a^n) = n\times \log(a)$.
\[x = \frac{\log(3.25)}{\log(0.76)}\]
\end{enumerate}
\end{solution}
\begin{exercise}[subtitle={Restaurant}]
Un \emph{food truck}, ouvert le midi et le soir, propose deux types de formules :
\setlength\parindent{10mm}
\begin{itemize}
\item la formule \emph{Burger} ;
\item la formule \emph{Wok}.
\end{itemize}
\setlength\parindent{0mm}
\medskip
Le gérant a remarqué que 21\,\% de ses ventes ont lieu le midi. Le quart des ventes du midi correspondent à la formule \emph{Burger}, alors que 27\,\% des ventes du soir correspondent à la formule \emph{Wok}.
Le gérant se constitue un fichier en notant, pour chaque vente, la formule choisie et le moment de cette vente (midi ou soir).
On prélève une fiche de façon équiprobable. On définit les quatre évènements suivants:
\begin{enumerate}
\item $M$ : \og la fiche correspond à une vente du midi\fg{} ;
\item $S$ : \og la fiche correspond à une vente du soir\fg {};
\item $W$ : \og la fiche correspond à une formule \emph{Wok} \fg{} ;
\item $B$ : \og la fiche correspond à une formule \emph{Burger} \fg.
\end{enumerate}
\setlength\parindent{0mm}
\medskip
\begin{enumerate}
\item Recopier puis compléter l'arbre pondéré
\begin{center}
\begin{tikzpicture}[sloped]
\node {.}
child {node {$M$}
child {node {$W$}
edge from parent
node[above] {...}
}
child {node {$B$}
edge from parent
node[above] {...}
}
edge from parent
node[above] {...}
}
child[missing] {}
child { node {$S$}
child {node {$W$}
edge from parent
node[above] {...}
}
child {node {$B$}
edge from parent
node[above] {...}
}
edge from parent
node[above] {...}
} ;
\end{tikzpicture}
\end{center}
\item Calculer la probabilité de l'évènement $M \cap W$. Interpréter ce résultat dans le contexte de l'exercice.
\item Montrer que la probabilité que la fiche choisie corresponde à une formule \emph{Burger} est égale à $0.6292$.
\item On a prélevé une fiche correspondant à la formule \emph{Burger}. Quelle est la probabilité, arrondie au millième, que la vente ait eu lieu le soir?
\end{enumerate}
\end{exercise}
\begin{solution}
\begin{enumerate}
\item
\begin{center}
\begin{tikzpicture}[sloped]
\node {.}
child {node {$M$}
child {node {$W$}
edge from parent
node[above] {$0.75$}
}
child {node {$B$}
edge from parent
node[above] {$0.25$}
}
edge from parent
node[above] {$0.21$}
}
child[missing] {}
child { node {$S$}
child {node {$W$}
edge from parent
node[above] {$0.27$}
}
child {node {$B$}
edge from parent
node[above] {$0.73$}
}
edge from parent
node[above] {$0.79$}
} ;
\end{tikzpicture}
\end{center}
\item On calcule la probabilité que la vente soit un wok et ait eu lieu à midi
\[ P(M\cap W) = P(M) \times P_M(W) = 0.21 \times 0.75 = 0.1575 \]
\item Probabilité que la vente soit un burger.
\[
P(B) = P(M\cap B) + P(S\cap B) = 0.21 \times 0.75 + 0.79 \times 0.27 = 0.6292
\]
\item On cherche à calculer la quantité $P_B(S)$. Pour cela on utilise la formule de Bayes
\[
P_B(S) = \frac{P(B\cap S)}{P(B)} = \frac{P_S(B) \times P(S)}{P(B)} = \frac{0.73\times 0.79}{0.6292} = 0.9165607120152575 \approx 0.917
\]
\end{enumerate}
\end{solution}
\begin{exercise}[subtitle={Continent plastique}]
\textit{Les quantités évoqués dans cette exercice sont générés au hasard et sont donc complètement farfelus.}
\medskip
Le \og continent de plastique\fg{} est la plus grande des plaques de déchets plastiques évoluant sur les océans. Elle occupe actuellement dans l'océan Pacifique une surface dont l'aire est évaluée à plus de $1,6$ million de km$^2$, entre Hawaï et la Californie.
En 2017, des scientifiques ont estimé qu'il y avait $20$ millions de tonnes de déchets plastiques qui était déversé chaque année dans les océans et que cette quantité augmentait de $28\n\%$ par chaque année.
On modélise l'évolution de la masse de ces déchets plastiques déversée chaque année, si rien n'est fait pour la réduire, par une suite géométrique $\left(u_n\right)$. L'arrondi au centième du terme $u_n$ représente la masse de ces déchets déversée chaque année, exprimée en million de tonnes, pour l'année $(2017 + n)$.
\medskip
\begin{enumerate}
\item Expliquer pourquoi la suite $u_n$ est géométrique?
\item Calculer $u_1$ et $u_2$.
\item Exprimer $u_n$ en fonction de $n$.
\item Au début de l'année 2017, il y avait $300$ millions de tonnes de déchets plastique. Calculer la quantité totale de déchets plastiques en 2030.
\item On souhaite déterminer en quelle année la masse totale de ces déchets plastiques aura pour la première fois augmenté de $50$\,\% par rapport à sa valeur de 2017.
\begin{enumerate}
\item Recopier et compléter l'algorithme ci-dessous pour que la variable $N$ contienne la réponse au problème posé.
\begin{center}
\begin{tabularx}{0.4\linewidth}{|X|}\hline
$N = 2017$\\
$U = 20$ \\
$S = 300 + U$ \\
while $S < 450$: \\
\hspace{1cm} $N = \ldots$\\
\hspace{1cm} $U = \ldots$\\
\hspace{1cm} $S = \ldots$\\
\hline
\end{tabularx}
\end{center}
\item Que contiennent les variables $S$, $U$ et $N$ après exécution de cet algorithme ?
Interpréter les résultats dans le contexte de l'exercice.
\end{enumerate}
\end{enumerate}
\end{exercise}
\begin{solution}
\begin{enumerate}
\item Une augmentation de $28\,\%$ revient à multiplier la quantité par $1.28$. La suite est donc bien géométrique. Son premier terme est $u_0 = 20$ et sa raison est $q = 1.28$
\item
\[
u_1 = u_0 * 1.28 = 25.6
\]
\[
u_2 = u_0 * 1.28^2 = 32.768
\]
\item
\[
u_n = u_0 \times q^n = 20 \times 1.28^n
\]
\item On calcule la quantité totale déversée entre 2017 et 2030.
\[
\sum_{n = 0}^{13} u_n = u_0 \times \frac{1-q^{13}}{1-q} = 20 \times \frac{1 - 1.28^{13}}{1 - 1.28} = 1697.06
\]
On en déduit la quantité totale de déchets en 2030
\[
300 + 1697.06 = 1997.06
\]
\item
\begin{enumerate}
\item ~
\begin{center}
\begin{tabularx}{0.4\linewidth}{|X|}\hline
$N \gets 2017$\\
$U \gets 20$ \\
$S \gets 300 + U$ \\
Tant que $S < 450$ \\
\hspace{1cm} $N \gets N + 1$\\
\hspace{1cm} $U \gets U * 1.28$\\
\hspace{1cm} $S \gets S + u$\\
Fin Tant que\\\hline
\end{tabularx}
\end{center}
\item \textit{Pas de correction automatisé}
\end{enumerate}
\end{enumerate}
\end{solution}
\end{document}
%%% Local Variables:
%%% mode: latex
%%% TeX-master: "master"
%%% End:

View File

@ -0,0 +1,262 @@
\documentclass[a4paper,10pt]{article}
\usepackage{myXsim}
% Title Page
\title{DS8 \hfill BERTAN Ufuk}
\tribe{TST}
\date{\hfillÀ render pour le Mercredi 7 avril}
\xsimsetup{
solution/print = false
}
\begin{document}
\maketitle
\begin{exercise}[subtitle={Automatismes}]
\textit{Toutes les questions de cette exercice sont indépendantes et peuvent être répondus séparément}
\begin{enumerate}
\item De janvier à septembre, une quantité a augmenté de $22\,\%$. Faire un schéma pour représenter la situation puis calculer le taux d'évolution moyen mensuel.
\item Une quantité augmente de $22\,\%$ par ans. En 2020, elle est de 134\euro. Quelle était sa valeur en 2019? Faire un schéma pour représenter la situation.
\item Déterminer l'équation de la droite \\
\begin{tikzpicture}[xscale=0.8, yscale=0.5]
\tkzInit[xmin=-5,xmax=5,xstep=1,
ymin=-5,ymax=5,ystep=1]
\tkzGrid
\tkzAxeXY
\tkzFct[domain=-5:5,color=red,very thick]%
{2.0*\x -2};
\end{tikzpicture}
\item Résoudre l'équation $5 \times 0.99^x = 21$
\end{enumerate}
\end{exercise}
\begin{solution}
\begin{enumerate}
\item On veut partager cette évolution en 8 évolutions.
\[
\left(1 + \frac{22}{100}\right)^{\frac{1}{8}} = 1.0252
\]
Donc le taux d'évolution moyen est
\[
t_m = 1.0252 - 1 = 0.02519999999999989
\]
\item Coefficient multiplicateur pour revenir en arrière
\[
CM = (1 + \frac{22}{100})^{-1} = 0.8197
\]
On en déduit la quantité en 2019
\[
134 * 0.8197 = 109.8398
\]
\item L'équation de la droite est
\[
y = 2.0 x -2
\]
\item Il faut penser à faire la division à par $5$ avant d'utiliser le log car sinon, on ne peut pas utiliser la formule $\log(a^n) = n\times \log(a)$.
\[x = \frac{\log(4.2)}{\log(0.99)}\]
\end{enumerate}
\end{solution}
\begin{exercise}[subtitle={Restaurant}]
Un \emph{food truck}, ouvert le midi et le soir, propose deux types de formules :
\setlength\parindent{10mm}
\begin{itemize}
\item la formule \emph{Burger} ;
\item la formule \emph{Wok}.
\end{itemize}
\setlength\parindent{0mm}
\medskip
Le gérant a remarqué que 64\,\% de ses ventes ont lieu le midi. Le quart des ventes du midi correspondent à la formule \emph{Burger}, alors que 100\,\% des ventes du soir correspondent à la formule \emph{Wok}.
Le gérant se constitue un fichier en notant, pour chaque vente, la formule choisie et le moment de cette vente (midi ou soir).
On prélève une fiche de façon équiprobable. On définit les quatre évènements suivants:
\begin{enumerate}
\item $M$ : \og la fiche correspond à une vente du midi\fg{} ;
\item $S$ : \og la fiche correspond à une vente du soir\fg {};
\item $W$ : \og la fiche correspond à une formule \emph{Wok} \fg{} ;
\item $B$ : \og la fiche correspond à une formule \emph{Burger} \fg.
\end{enumerate}
\setlength\parindent{0mm}
\medskip
\begin{enumerate}
\item Recopier puis compléter l'arbre pondéré
\begin{center}
\begin{tikzpicture}[sloped]
\node {.}
child {node {$M$}
child {node {$W$}
edge from parent
node[above] {...}
}
child {node {$B$}
edge from parent
node[above] {...}
}
edge from parent
node[above] {...}
}
child[missing] {}
child { node {$S$}
child {node {$W$}
edge from parent
node[above] {...}
}
child {node {$B$}
edge from parent
node[above] {...}
}
edge from parent
node[above] {...}
} ;
\end{tikzpicture}
\end{center}
\item Calculer la probabilité de l'évènement $M \cap W$. Interpréter ce résultat dans le contexte de l'exercice.
\item Montrer que la probabilité que la fiche choisie corresponde à une formule \emph{Burger} est égale à $0.16$.
\item On a prélevé une fiche correspondant à la formule \emph{Burger}. Quelle est la probabilité, arrondie au millième, que la vente ait eu lieu le soir?
\end{enumerate}
\end{exercise}
\begin{solution}
\begin{enumerate}
\item
\begin{center}
\begin{tikzpicture}[sloped]
\node {.}
child {node {$M$}
child {node {$W$}
edge from parent
node[above] {$0.75$}
}
child {node {$B$}
edge from parent
node[above] {$0.25$}
}
edge from parent
node[above] {$0.64$}
}
child[missing] {}
child { node {$S$}
child {node {$W$}
edge from parent
node[above] {$1.0$}
}
child {node {$B$}
edge from parent
node[above] {$0.0$}
}
edge from parent
node[above] {$0.36$}
} ;
\end{tikzpicture}
\end{center}
\item On calcule la probabilité que la vente soit un wok et ait eu lieu à midi
\[ P(M\cap W) = P(M) \times P_M(W) = 0.64 \times 0.75 = 0.48 \]
\item Probabilité que la vente soit un burger.
\[
P(B) = P(M\cap B) + P(S\cap B) = 0.64 \times 0.75 + 0.36 \times 1.0 = 0.16
\]
\item On cherche à calculer la quantité $P_B(S)$. Pour cela on utilise la formule de Bayes
\[
P_B(S) = \frac{P(B\cap S)}{P(B)} = \frac{P_S(B) \times P(S)}{P(B)} = \frac{0.0\times 0.36}{0.16} = 0.0 \approx 0.0
\]
\end{enumerate}
\end{solution}
\begin{exercise}[subtitle={Continent plastique}]
\textit{Les quantités évoqués dans cette exercice sont générés au hasard et sont donc complètement farfelus.}
\medskip
Le \og continent de plastique\fg{} est la plus grande des plaques de déchets plastiques évoluant sur les océans. Elle occupe actuellement dans l'océan Pacifique une surface dont l'aire est évaluée à plus de $1,6$ million de km$^2$, entre Hawaï et la Californie.
En 2017, des scientifiques ont estimé qu'il y avait $2$ millions de tonnes de déchets plastiques qui était déversé chaque année dans les océans et que cette quantité augmentait de $27\n\%$ par chaque année.
On modélise l'évolution de la masse de ces déchets plastiques déversée chaque année, si rien n'est fait pour la réduire, par une suite géométrique $\left(u_n\right)$. L'arrondi au centième du terme $u_n$ représente la masse de ces déchets déversée chaque année, exprimée en million de tonnes, pour l'année $(2017 + n)$.
\medskip
\begin{enumerate}
\item Expliquer pourquoi la suite $u_n$ est géométrique?
\item Calculer $u_1$ et $u_2$.
\item Exprimer $u_n$ en fonction de $n$.
\item Au début de l'année 2017, il y avait $300$ millions de tonnes de déchets plastique. Calculer la quantité totale de déchets plastiques en 2030.
\item On souhaite déterminer en quelle année la masse totale de ces déchets plastiques aura pour la première fois augmenté de $50$\,\% par rapport à sa valeur de 2017.
\begin{enumerate}
\item Recopier et compléter l'algorithme ci-dessous pour que la variable $N$ contienne la réponse au problème posé.
\begin{center}
\begin{tabularx}{0.4\linewidth}{|X|}\hline
$N = 2017$\\
$U = 2$ \\
$S = 300 + U$ \\
while $S < 450$: \\
\hspace{1cm} $N = \ldots$\\
\hspace{1cm} $U = \ldots$\\
\hspace{1cm} $S = \ldots$\\
\hline
\end{tabularx}
\end{center}
\item Que contiennent les variables $S$, $U$ et $N$ après exécution de cet algorithme ?
Interpréter les résultats dans le contexte de l'exercice.
\end{enumerate}
\end{enumerate}
\end{exercise}
\begin{solution}
\begin{enumerate}
\item Une augmentation de $27\,\%$ revient à multiplier la quantité par $1.27$. La suite est donc bien géométrique. Son premier terme est $u_0 = 2$ et sa raison est $q = 1.27$
\item
\[
u_1 = u_0 * 1.27 = 2.54
\]
\[
u_2 = u_0 * 1.27^2 = 3.2258
\]
\item
\[
u_n = u_0 \times q^n = 2 \times 1.27^n
\]
\item On calcule la quantité totale déversée entre 2017 et 2030.
\[
\sum_{n = 0}^{13} u_n = u_0 \times \frac{1-q^{13}}{1-q} = 2 \times \frac{1 - 1.27^{13}}{1 - 1.27} = 158.21
\]
On en déduit la quantité totale de déchets en 2030
\[
300 + 158.21 = 458.21000000000004
\]
\item
\begin{enumerate}
\item ~
\begin{center}
\begin{tabularx}{0.4\linewidth}{|X|}\hline
$N \gets 2017$\\
$U \gets 2$ \\
$S \gets 300 + U$ \\
Tant que $S < 450$ \\
\hspace{1cm} $N \gets N + 1$\\
\hspace{1cm} $U \gets U * 1.27$\\
\hspace{1cm} $S \gets S + u$\\
Fin Tant que\\\hline
\end{tabularx}
\end{center}
\item \textit{Pas de correction automatisé}
\end{enumerate}
\end{enumerate}
\end{solution}
\end{document}
%%% Local Variables:
%%% mode: latex
%%% TeX-master: "master"
%%% End:

View File

@ -0,0 +1,262 @@
\documentclass[a4paper,10pt]{article}
\usepackage{myXsim}
% Title Page
\title{DS8 \hfill BOUALIA Bilel}
\tribe{TST}
\date{\hfillÀ render pour le Mercredi 7 avril}
\xsimsetup{
solution/print = false
}
\begin{document}
\maketitle
\begin{exercise}[subtitle={Automatismes}]
\textit{Toutes les questions de cette exercice sont indépendantes et peuvent être répondus séparément}
\begin{enumerate}
\item De janvier à septembre, une quantité a augmenté de $17\,\%$. Faire un schéma pour représenter la situation puis calculer le taux d'évolution moyen mensuel.
\item Une quantité augmente de $17\,\%$ par ans. En 2020, elle est de 135\euro. Quelle était sa valeur en 2019? Faire un schéma pour représenter la situation.
\item Déterminer l'équation de la droite \\
\begin{tikzpicture}[xscale=0.8, yscale=0.5]
\tkzInit[xmin=-5,xmax=5,xstep=1,
ymin=-5,ymax=5,ystep=1]
\tkzGrid
\tkzAxeXY
\tkzFct[domain=-5:5,color=red,very thick]%
{2.0*\x -4};
\end{tikzpicture}
\item Résoudre l'équation $3 \times 0.2^x = 2$
\end{enumerate}
\end{exercise}
\begin{solution}
\begin{enumerate}
\item On veut partager cette évolution en 8 évolutions.
\[
\left(1 + \frac{17}{100}\right)^{\frac{1}{8}} = 1.0198
\]
Donc le taux d'évolution moyen est
\[
t_m = 1.0198 - 1 = 0.01980000000000004
\]
\item Coefficient multiplicateur pour revenir en arrière
\[
CM = (1 + \frac{17}{100})^{-1} = 0.8547
\]
On en déduit la quantité en 2019
\[
135 * 0.8547 = 115.3845
\]
\item L'équation de la droite est
\[
y = 2.0 x -4
\]
\item Il faut penser à faire la division à par $3$ avant d'utiliser le log car sinon, on ne peut pas utiliser la formule $\log(a^n) = n\times \log(a)$.
\[x = \frac{\log(0.67)}{\log(0.2)}\]
\end{enumerate}
\end{solution}
\begin{exercise}[subtitle={Restaurant}]
Un \emph{food truck}, ouvert le midi et le soir, propose deux types de formules :
\setlength\parindent{10mm}
\begin{itemize}
\item la formule \emph{Burger} ;
\item la formule \emph{Wok}.
\end{itemize}
\setlength\parindent{0mm}
\medskip
Le gérant a remarqué que 89\,\% de ses ventes ont lieu le midi. Le quart des ventes du midi correspondent à la formule \emph{Burger}, alors que 12\,\% des ventes du soir correspondent à la formule \emph{Wok}.
Le gérant se constitue un fichier en notant, pour chaque vente, la formule choisie et le moment de cette vente (midi ou soir).
On prélève une fiche de façon équiprobable. On définit les quatre évènements suivants:
\begin{enumerate}
\item $M$ : \og la fiche correspond à une vente du midi\fg{} ;
\item $S$ : \og la fiche correspond à une vente du soir\fg {};
\item $W$ : \og la fiche correspond à une formule \emph{Wok} \fg{} ;
\item $B$ : \og la fiche correspond à une formule \emph{Burger} \fg.
\end{enumerate}
\setlength\parindent{0mm}
\medskip
\begin{enumerate}
\item Recopier puis compléter l'arbre pondéré
\begin{center}
\begin{tikzpicture}[sloped]
\node {.}
child {node {$M$}
child {node {$W$}
edge from parent
node[above] {...}
}
child {node {$B$}
edge from parent
node[above] {...}
}
edge from parent
node[above] {...}
}
child[missing] {}
child { node {$S$}
child {node {$W$}
edge from parent
node[above] {...}
}
child {node {$B$}
edge from parent
node[above] {...}
}
edge from parent
node[above] {...}
} ;
\end{tikzpicture}
\end{center}
\item Calculer la probabilité de l'évènement $M \cap W$. Interpréter ce résultat dans le contexte de l'exercice.
\item Montrer que la probabilité que la fiche choisie corresponde à une formule \emph{Burger} est égale à $0.3193$.
\item On a prélevé une fiche correspondant à la formule \emph{Burger}. Quelle est la probabilité, arrondie au millième, que la vente ait eu lieu le soir?
\end{enumerate}
\end{exercise}
\begin{solution}
\begin{enumerate}
\item
\begin{center}
\begin{tikzpicture}[sloped]
\node {.}
child {node {$M$}
child {node {$W$}
edge from parent
node[above] {$0.75$}
}
child {node {$B$}
edge from parent
node[above] {$0.25$}
}
edge from parent
node[above] {$0.89$}
}
child[missing] {}
child { node {$S$}
child {node {$W$}
edge from parent
node[above] {$0.12$}
}
child {node {$B$}
edge from parent
node[above] {$0.88$}
}
edge from parent
node[above] {$0.11$}
} ;
\end{tikzpicture}
\end{center}
\item On calcule la probabilité que la vente soit un wok et ait eu lieu à midi
\[ P(M\cap W) = P(M) \times P_M(W) = 0.89 \times 0.75 = 0.6675 \]
\item Probabilité que la vente soit un burger.
\[
P(B) = P(M\cap B) + P(S\cap B) = 0.89 \times 0.75 + 0.11 \times 0.12 = 0.3193
\]
\item On cherche à calculer la quantité $P_B(S)$. Pour cela on utilise la formule de Bayes
\[
P_B(S) = \frac{P(B\cap S)}{P(B)} = \frac{P_S(B) \times P(S)}{P(B)} = \frac{0.88\times 0.11}{0.3193} = 0.303163169433135 \approx 0.303
\]
\end{enumerate}
\end{solution}
\begin{exercise}[subtitle={Continent plastique}]
\textit{Les quantités évoqués dans cette exercice sont générés au hasard et sont donc complètement farfelus.}
\medskip
Le \og continent de plastique\fg{} est la plus grande des plaques de déchets plastiques évoluant sur les océans. Elle occupe actuellement dans l'océan Pacifique une surface dont l'aire est évaluée à plus de $1,6$ million de km$^2$, entre Hawaï et la Californie.
En 2017, des scientifiques ont estimé qu'il y avait $10$ millions de tonnes de déchets plastiques qui était déversé chaque année dans les océans et que cette quantité augmentait de $27\n\%$ par chaque année.
On modélise l'évolution de la masse de ces déchets plastiques déversée chaque année, si rien n'est fait pour la réduire, par une suite géométrique $\left(u_n\right)$. L'arrondi au centième du terme $u_n$ représente la masse de ces déchets déversée chaque année, exprimée en million de tonnes, pour l'année $(2017 + n)$.
\medskip
\begin{enumerate}
\item Expliquer pourquoi la suite $u_n$ est géométrique?
\item Calculer $u_1$ et $u_2$.
\item Exprimer $u_n$ en fonction de $n$.
\item Au début de l'année 2017, il y avait $300$ millions de tonnes de déchets plastique. Calculer la quantité totale de déchets plastiques en 2030.
\item On souhaite déterminer en quelle année la masse totale de ces déchets plastiques aura pour la première fois augmenté de $50$\,\% par rapport à sa valeur de 2017.
\begin{enumerate}
\item Recopier et compléter l'algorithme ci-dessous pour que la variable $N$ contienne la réponse au problème posé.
\begin{center}
\begin{tabularx}{0.4\linewidth}{|X|}\hline
$N = 2017$\\
$U = 10$ \\
$S = 300 + U$ \\
while $S < 450$: \\
\hspace{1cm} $N = \ldots$\\
\hspace{1cm} $U = \ldots$\\
\hspace{1cm} $S = \ldots$\\
\hline
\end{tabularx}
\end{center}
\item Que contiennent les variables $S$, $U$ et $N$ après exécution de cet algorithme ?
Interpréter les résultats dans le contexte de l'exercice.
\end{enumerate}
\end{enumerate}
\end{exercise}
\begin{solution}
\begin{enumerate}
\item Une augmentation de $27\,\%$ revient à multiplier la quantité par $1.27$. La suite est donc bien géométrique. Son premier terme est $u_0 = 10$ et sa raison est $q = 1.27$
\item
\[
u_1 = u_0 * 1.27 = 12.7
\]
\[
u_2 = u_0 * 1.27^2 = 16.129
\]
\item
\[
u_n = u_0 \times q^n = 10 \times 1.27^n
\]
\item On calcule la quantité totale déversée entre 2017 et 2030.
\[
\sum_{n = 0}^{13} u_n = u_0 \times \frac{1-q^{13}}{1-q} = 10 \times \frac{1 - 1.27^{13}}{1 - 1.27} = 791.07
\]
On en déduit la quantité totale de déchets en 2030
\[
300 + 791.07 = 1091.0700000000002
\]
\item
\begin{enumerate}
\item ~
\begin{center}
\begin{tabularx}{0.4\linewidth}{|X|}\hline
$N \gets 2017$\\
$U \gets 10$ \\
$S \gets 300 + U$ \\
Tant que $S < 450$ \\
\hspace{1cm} $N \gets N + 1$\\
\hspace{1cm} $U \gets U * 1.27$\\
\hspace{1cm} $S \gets S + u$\\
Fin Tant que\\\hline
\end{tabularx}
\end{center}
\item \textit{Pas de correction automatisé}
\end{enumerate}
\end{enumerate}
\end{solution}
\end{document}
%%% Local Variables:
%%% mode: latex
%%% TeX-master: "master"
%%% End:

View File

@ -0,0 +1,262 @@
\documentclass[a4paper,10pt]{article}
\usepackage{myXsim}
% Title Page
\title{DS8 \hfill BOUCHOUX Kevin}
\tribe{TST}
\date{\hfillÀ render pour le Mercredi 7 avril}
\xsimsetup{
solution/print = false
}
\begin{document}
\maketitle
\begin{exercise}[subtitle={Automatismes}]
\textit{Toutes les questions de cette exercice sont indépendantes et peuvent être répondus séparément}
\begin{enumerate}
\item De janvier à septembre, une quantité a augmenté de $10\,\%$. Faire un schéma pour représenter la situation puis calculer le taux d'évolution moyen mensuel.
\item Une quantité augmente de $10\,\%$ par ans. En 2020, elle est de 118\euro. Quelle était sa valeur en 2019? Faire un schéma pour représenter la situation.
\item Déterminer l'équation de la droite \\
\begin{tikzpicture}[xscale=0.8, yscale=0.5]
\tkzInit[xmin=-5,xmax=5,xstep=1,
ymin=-5,ymax=5,ystep=1]
\tkzGrid
\tkzAxeXY
\tkzFct[domain=-5:5,color=red,very thick]%
{3.0*\x -3};
\end{tikzpicture}
\item Résoudre l'équation $2 \times 0.93^x = 44$
\end{enumerate}
\end{exercise}
\begin{solution}
\begin{enumerate}
\item On veut partager cette évolution en 8 évolutions.
\[
\left(1 + \frac{10}{100}\right)^{\frac{1}{8}} = 1.012
\]
Donc le taux d'évolution moyen est
\[
t_m = 1.012 - 1 = 0.01200000000000001
\]
\item Coefficient multiplicateur pour revenir en arrière
\[
CM = (1 + \frac{10}{100})^{-1} = 0.9091
\]
On en déduit la quantité en 2019
\[
118 * 0.9091 = 107.27380000000001
\]
\item L'équation de la droite est
\[
y = 3.0 x -3
\]
\item Il faut penser à faire la division à par $2$ avant d'utiliser le log car sinon, on ne peut pas utiliser la formule $\log(a^n) = n\times \log(a)$.
\[x = \frac{\log(22.0)}{\log(0.93)}\]
\end{enumerate}
\end{solution}
\begin{exercise}[subtitle={Restaurant}]
Un \emph{food truck}, ouvert le midi et le soir, propose deux types de formules :
\setlength\parindent{10mm}
\begin{itemize}
\item la formule \emph{Burger} ;
\item la formule \emph{Wok}.
\end{itemize}
\setlength\parindent{0mm}
\medskip
Le gérant a remarqué que 83\,\% de ses ventes ont lieu le midi. Le quart des ventes du midi correspondent à la formule \emph{Burger}, alors que 26\,\% des ventes du soir correspondent à la formule \emph{Wok}.
Le gérant se constitue un fichier en notant, pour chaque vente, la formule choisie et le moment de cette vente (midi ou soir).
On prélève une fiche de façon équiprobable. On définit les quatre évènements suivants:
\begin{enumerate}
\item $M$ : \og la fiche correspond à une vente du midi\fg{} ;
\item $S$ : \og la fiche correspond à une vente du soir\fg {};
\item $W$ : \og la fiche correspond à une formule \emph{Wok} \fg{} ;
\item $B$ : \og la fiche correspond à une formule \emph{Burger} \fg.
\end{enumerate}
\setlength\parindent{0mm}
\medskip
\begin{enumerate}
\item Recopier puis compléter l'arbre pondéré
\begin{center}
\begin{tikzpicture}[sloped]
\node {.}
child {node {$M$}
child {node {$W$}
edge from parent
node[above] {...}
}
child {node {$B$}
edge from parent
node[above] {...}
}
edge from parent
node[above] {...}
}
child[missing] {}
child { node {$S$}
child {node {$W$}
edge from parent
node[above] {...}
}
child {node {$B$}
edge from parent
node[above] {...}
}
edge from parent
node[above] {...}
} ;
\end{tikzpicture}
\end{center}
\item Calculer la probabilité de l'évènement $M \cap W$. Interpréter ce résultat dans le contexte de l'exercice.
\item Montrer que la probabilité que la fiche choisie corresponde à une formule \emph{Burger} est égale à $0.3333$.
\item On a prélevé une fiche correspondant à la formule \emph{Burger}. Quelle est la probabilité, arrondie au millième, que la vente ait eu lieu le soir?
\end{enumerate}
\end{exercise}
\begin{solution}
\begin{enumerate}
\item
\begin{center}
\begin{tikzpicture}[sloped]
\node {.}
child {node {$M$}
child {node {$W$}
edge from parent
node[above] {$0.75$}
}
child {node {$B$}
edge from parent
node[above] {$0.25$}
}
edge from parent
node[above] {$0.83$}
}
child[missing] {}
child { node {$S$}
child {node {$W$}
edge from parent
node[above] {$0.26$}
}
child {node {$B$}
edge from parent
node[above] {$0.74$}
}
edge from parent
node[above] {$0.17$}
} ;
\end{tikzpicture}
\end{center}
\item On calcule la probabilité que la vente soit un wok et ait eu lieu à midi
\[ P(M\cap W) = P(M) \times P_M(W) = 0.83 \times 0.75 = 0.6225 \]
\item Probabilité que la vente soit un burger.
\[
P(B) = P(M\cap B) + P(S\cap B) = 0.83 \times 0.75 + 0.17 \times 0.26 = 0.3333
\]
\item On cherche à calculer la quantité $P_B(S)$. Pour cela on utilise la formule de Bayes
\[
P_B(S) = \frac{P(B\cap S)}{P(B)} = \frac{P_S(B) \times P(S)}{P(B)} = \frac{0.74\times 0.17}{0.3333} = 0.37743774377437744 \approx 0.377
\]
\end{enumerate}
\end{solution}
\begin{exercise}[subtitle={Continent plastique}]
\textit{Les quantités évoqués dans cette exercice sont générés au hasard et sont donc complètement farfelus.}
\medskip
Le \og continent de plastique\fg{} est la plus grande des plaques de déchets plastiques évoluant sur les océans. Elle occupe actuellement dans l'océan Pacifique une surface dont l'aire est évaluée à plus de $1,6$ million de km$^2$, entre Hawaï et la Californie.
En 2017, des scientifiques ont estimé qu'il y avait $18$ millions de tonnes de déchets plastiques qui était déversé chaque année dans les océans et que cette quantité augmentait de $14\n\%$ par chaque année.
On modélise l'évolution de la masse de ces déchets plastiques déversée chaque année, si rien n'est fait pour la réduire, par une suite géométrique $\left(u_n\right)$. L'arrondi au centième du terme $u_n$ représente la masse de ces déchets déversée chaque année, exprimée en million de tonnes, pour l'année $(2017 + n)$.
\medskip
\begin{enumerate}
\item Expliquer pourquoi la suite $u_n$ est géométrique?
\item Calculer $u_1$ et $u_2$.
\item Exprimer $u_n$ en fonction de $n$.
\item Au début de l'année 2017, il y avait $300$ millions de tonnes de déchets plastique. Calculer la quantité totale de déchets plastiques en 2030.
\item On souhaite déterminer en quelle année la masse totale de ces déchets plastiques aura pour la première fois augmenté de $50$\,\% par rapport à sa valeur de 2017.
\begin{enumerate}
\item Recopier et compléter l'algorithme ci-dessous pour que la variable $N$ contienne la réponse au problème posé.
\begin{center}
\begin{tabularx}{0.4\linewidth}{|X|}\hline
$N = 2017$\\
$U = 18$ \\
$S = 300 + U$ \\
while $S < 450$: \\
\hspace{1cm} $N = \ldots$\\
\hspace{1cm} $U = \ldots$\\
\hspace{1cm} $S = \ldots$\\
\hline
\end{tabularx}
\end{center}
\item Que contiennent les variables $S$, $U$ et $N$ après exécution de cet algorithme ?
Interpréter les résultats dans le contexte de l'exercice.
\end{enumerate}
\end{enumerate}
\end{exercise}
\begin{solution}
\begin{enumerate}
\item Une augmentation de $14\,\%$ revient à multiplier la quantité par $1.1400000000000001$. La suite est donc bien géométrique. Son premier terme est $u_0 = 18$ et sa raison est $q = 1.1400000000000001$
\item
\[
u_1 = u_0 * 1.1400000000000001 = 20.520000000000003
\]
\[
u_2 = u_0 * 1.1400000000000001^2 = 23.3928
\]
\item
\[
u_n = u_0 \times q^n = 18 \times 1.1400000000000001^n
\]
\item On calcule la quantité totale déversée entre 2017 et 2030.
\[
\sum_{n = 0}^{13} u_n = u_0 \times \frac{1-q^{13}}{1-q} = 18 \times \frac{1 - 1.1400000000000001^{13}}{1 - 1.1400000000000001} = 577.6
\]
On en déduit la quantité totale de déchets en 2030
\[
300 + 577.6 = 877.6
\]
\item
\begin{enumerate}
\item ~
\begin{center}
\begin{tabularx}{0.4\linewidth}{|X|}\hline
$N \gets 2017$\\
$U \gets 18$ \\
$S \gets 300 + U$ \\
Tant que $S < 450$ \\
\hspace{1cm} $N \gets N + 1$\\
\hspace{1cm} $U \gets U * 1.1400000000000001$\\
\hspace{1cm} $S \gets S + u$\\
Fin Tant que\\\hline
\end{tabularx}
\end{center}
\item \textit{Pas de correction automatisé}
\end{enumerate}
\end{enumerate}
\end{solution}
\end{document}
%%% Local Variables:
%%% mode: latex
%%% TeX-master: "master"
%%% End:

View File

@ -0,0 +1,262 @@
\documentclass[a4paper,10pt]{article}
\usepackage{myXsim}
% Title Page
\title{DS8 \hfill BUDIN Nathan}
\tribe{TST}
\date{\hfillÀ render pour le Mercredi 7 avril}
\xsimsetup{
solution/print = false
}
\begin{document}
\maketitle
\begin{exercise}[subtitle={Automatismes}]
\textit{Toutes les questions de cette exercice sont indépendantes et peuvent être répondus séparément}
\begin{enumerate}
\item De janvier à septembre, une quantité a augmenté de $16\,\%$. Faire un schéma pour représenter la situation puis calculer le taux d'évolution moyen mensuel.
\item Une quantité augmente de $16\,\%$ par ans. En 2020, elle est de 112\euro. Quelle était sa valeur en 2019? Faire un schéma pour représenter la situation.
\item Déterminer l'équation de la droite \\
\begin{tikzpicture}[xscale=0.8, yscale=0.5]
\tkzInit[xmin=-5,xmax=5,xstep=1,
ymin=-5,ymax=5,ystep=1]
\tkzGrid
\tkzAxeXY
\tkzFct[domain=-5:5,color=red,very thick]%
{1.0*\x -2};
\end{tikzpicture}
\item Résoudre l'équation $7 \times 0.22^x = 49$
\end{enumerate}
\end{exercise}
\begin{solution}
\begin{enumerate}
\item On veut partager cette évolution en 8 évolutions.
\[
\left(1 + \frac{16}{100}\right)^{\frac{1}{8}} = 1.0187
\]
Donc le taux d'évolution moyen est
\[
t_m = 1.0187 - 1 = 0.01869999999999994
\]
\item Coefficient multiplicateur pour revenir en arrière
\[
CM = (1 + \frac{16}{100})^{-1} = 0.8621
\]
On en déduit la quantité en 2019
\[
112 * 0.8621 = 96.5552
\]
\item L'équation de la droite est
\[
y = 1.0 x -2
\]
\item Il faut penser à faire la division à par $7$ avant d'utiliser le log car sinon, on ne peut pas utiliser la formule $\log(a^n) = n\times \log(a)$.
\[x = \frac{\log(7.0)}{\log(0.22)}\]
\end{enumerate}
\end{solution}
\begin{exercise}[subtitle={Restaurant}]
Un \emph{food truck}, ouvert le midi et le soir, propose deux types de formules :
\setlength\parindent{10mm}
\begin{itemize}
\item la formule \emph{Burger} ;
\item la formule \emph{Wok}.
\end{itemize}
\setlength\parindent{0mm}
\medskip
Le gérant a remarqué que 75\,\% de ses ventes ont lieu le midi. Le quart des ventes du midi correspondent à la formule \emph{Burger}, alors que 71\,\% des ventes du soir correspondent à la formule \emph{Wok}.
Le gérant se constitue un fichier en notant, pour chaque vente, la formule choisie et le moment de cette vente (midi ou soir).
On prélève une fiche de façon équiprobable. On définit les quatre évènements suivants:
\begin{enumerate}
\item $M$ : \og la fiche correspond à une vente du midi\fg{} ;
\item $S$ : \og la fiche correspond à une vente du soir\fg {};
\item $W$ : \og la fiche correspond à une formule \emph{Wok} \fg{} ;
\item $B$ : \og la fiche correspond à une formule \emph{Burger} \fg.
\end{enumerate}
\setlength\parindent{0mm}
\medskip
\begin{enumerate}
\item Recopier puis compléter l'arbre pondéré
\begin{center}
\begin{tikzpicture}[sloped]
\node {.}
child {node {$M$}
child {node {$W$}
edge from parent
node[above] {...}
}
child {node {$B$}
edge from parent
node[above] {...}
}
edge from parent
node[above] {...}
}
child[missing] {}
child { node {$S$}
child {node {$W$}
edge from parent
node[above] {...}
}
child {node {$B$}
edge from parent
node[above] {...}
}
edge from parent
node[above] {...}
} ;
\end{tikzpicture}
\end{center}
\item Calculer la probabilité de l'évènement $M \cap W$. Interpréter ce résultat dans le contexte de l'exercice.
\item Montrer que la probabilité que la fiche choisie corresponde à une formule \emph{Burger} est égale à $0.26$.
\item On a prélevé une fiche correspondant à la formule \emph{Burger}. Quelle est la probabilité, arrondie au millième, que la vente ait eu lieu le soir?
\end{enumerate}
\end{exercise}
\begin{solution}
\begin{enumerate}
\item
\begin{center}
\begin{tikzpicture}[sloped]
\node {.}
child {node {$M$}
child {node {$W$}
edge from parent
node[above] {$0.75$}
}
child {node {$B$}
edge from parent
node[above] {$0.25$}
}
edge from parent
node[above] {$0.75$}
}
child[missing] {}
child { node {$S$}
child {node {$W$}
edge from parent
node[above] {$0.71$}
}
child {node {$B$}
edge from parent
node[above] {$0.29$}
}
edge from parent
node[above] {$0.25$}
} ;
\end{tikzpicture}
\end{center}
\item On calcule la probabilité que la vente soit un wok et ait eu lieu à midi
\[ P(M\cap W) = P(M) \times P_M(W) = 0.75 \times 0.75 = 0.5625 \]
\item Probabilité que la vente soit un burger.
\[
P(B) = P(M\cap B) + P(S\cap B) = 0.75 \times 0.75 + 0.25 \times 0.71 = 0.26
\]
\item On cherche à calculer la quantité $P_B(S)$. Pour cela on utilise la formule de Bayes
\[
P_B(S) = \frac{P(B\cap S)}{P(B)} = \frac{P_S(B) \times P(S)}{P(B)} = \frac{0.29\times 0.25}{0.26} = 0.2788461538461538 \approx 0.279
\]
\end{enumerate}
\end{solution}
\begin{exercise}[subtitle={Continent plastique}]
\textit{Les quantités évoqués dans cette exercice sont générés au hasard et sont donc complètement farfelus.}
\medskip
Le \og continent de plastique\fg{} est la plus grande des plaques de déchets plastiques évoluant sur les océans. Elle occupe actuellement dans l'océan Pacifique une surface dont l'aire est évaluée à plus de $1,6$ million de km$^2$, entre Hawaï et la Californie.
En 2017, des scientifiques ont estimé qu'il y avait $12$ millions de tonnes de déchets plastiques qui était déversé chaque année dans les océans et que cette quantité augmentait de $25\n\%$ par chaque année.
On modélise l'évolution de la masse de ces déchets plastiques déversée chaque année, si rien n'est fait pour la réduire, par une suite géométrique $\left(u_n\right)$. L'arrondi au centième du terme $u_n$ représente la masse de ces déchets déversée chaque année, exprimée en million de tonnes, pour l'année $(2017 + n)$.
\medskip
\begin{enumerate}
\item Expliquer pourquoi la suite $u_n$ est géométrique?
\item Calculer $u_1$ et $u_2$.
\item Exprimer $u_n$ en fonction de $n$.
\item Au début de l'année 2017, il y avait $300$ millions de tonnes de déchets plastique. Calculer la quantité totale de déchets plastiques en 2030.
\item On souhaite déterminer en quelle année la masse totale de ces déchets plastiques aura pour la première fois augmenté de $50$\,\% par rapport à sa valeur de 2017.
\begin{enumerate}
\item Recopier et compléter l'algorithme ci-dessous pour que la variable $N$ contienne la réponse au problème posé.
\begin{center}
\begin{tabularx}{0.4\linewidth}{|X|}\hline
$N = 2017$\\
$U = 12$ \\
$S = 300 + U$ \\
while $S < 450$: \\
\hspace{1cm} $N = \ldots$\\
\hspace{1cm} $U = \ldots$\\
\hspace{1cm} $S = \ldots$\\
\hline
\end{tabularx}
\end{center}
\item Que contiennent les variables $S$, $U$ et $N$ après exécution de cet algorithme ?
Interpréter les résultats dans le contexte de l'exercice.
\end{enumerate}
\end{enumerate}
\end{exercise}
\begin{solution}
\begin{enumerate}
\item Une augmentation de $25\,\%$ revient à multiplier la quantité par $1.25$. La suite est donc bien géométrique. Son premier terme est $u_0 = 12$ et sa raison est $q = 1.25$
\item
\[
u_1 = u_0 * 1.25 = 15.0
\]
\[
u_2 = u_0 * 1.25^2 = 18.75
\]
\item
\[
u_n = u_0 \times q^n = 12 \times 1.25^n
\]
\item On calcule la quantité totale déversée entre 2017 et 2030.
\[
\sum_{n = 0}^{13} u_n = u_0 \times \frac{1-q^{13}}{1-q} = 12 \times \frac{1 - 1.25^{13}}{1 - 1.25} = 825.11
\]
On en déduit la quantité totale de déchets en 2030
\[
300 + 825.11 = 1125.1100000000001
\]
\item
\begin{enumerate}
\item ~
\begin{center}
\begin{tabularx}{0.4\linewidth}{|X|}\hline
$N \gets 2017$\\
$U \gets 12$ \\
$S \gets 300 + U$ \\
Tant que $S < 450$ \\
\hspace{1cm} $N \gets N + 1$\\
\hspace{1cm} $U \gets U * 1.25$\\
\hspace{1cm} $S \gets S + u$\\
Fin Tant que\\\hline
\end{tabularx}
\end{center}
\item \textit{Pas de correction automatisé}
\end{enumerate}
\end{enumerate}
\end{solution}
\end{document}
%%% Local Variables:
%%% mode: latex
%%% TeX-master: "master"
%%% End:

View File

@ -0,0 +1,262 @@
\documentclass[a4paper,10pt]{article}
\usepackage{myXsim}
% Title Page
\title{DS8 \hfill DARICHE Kaïs}
\tribe{TST}
\date{\hfillÀ render pour le Mercredi 7 avril}
\xsimsetup{
solution/print = false
}
\begin{document}
\maketitle
\begin{exercise}[subtitle={Automatismes}]
\textit{Toutes les questions de cette exercice sont indépendantes et peuvent être répondus séparément}
\begin{enumerate}
\item De janvier à septembre, une quantité a augmenté de $27\,\%$. Faire un schéma pour représenter la situation puis calculer le taux d'évolution moyen mensuel.
\item Une quantité augmente de $27\,\%$ par ans. En 2020, elle est de 143\euro. Quelle était sa valeur en 2019? Faire un schéma pour représenter la situation.
\item Déterminer l'équation de la droite \\
\begin{tikzpicture}[xscale=0.8, yscale=0.5]
\tkzInit[xmin=-5,xmax=5,xstep=1,
ymin=-5,ymax=5,ystep=1]
\tkzGrid
\tkzAxeXY
\tkzFct[domain=-5:5,color=red,very thick]%
{3.0*\x -3};
\end{tikzpicture}
\item Résoudre l'équation $7 \times 0.19^x = 21$
\end{enumerate}
\end{exercise}
\begin{solution}
\begin{enumerate}
\item On veut partager cette évolution en 8 évolutions.
\[
\left(1 + \frac{27}{100}\right)^{\frac{1}{8}} = 1.0303
\]
Donc le taux d'évolution moyen est
\[
t_m = 1.0303 - 1 = 0.030299999999999994
\]
\item Coefficient multiplicateur pour revenir en arrière
\[
CM = (1 + \frac{27}{100})^{-1} = 0.7874
\]
On en déduit la quantité en 2019
\[
143 * 0.7874 = 112.59819999999999
\]
\item L'équation de la droite est
\[
y = 3.0 x -3
\]
\item Il faut penser à faire la division à par $7$ avant d'utiliser le log car sinon, on ne peut pas utiliser la formule $\log(a^n) = n\times \log(a)$.
\[x = \frac{\log(3.0)}{\log(0.19)}\]
\end{enumerate}
\end{solution}
\begin{exercise}[subtitle={Restaurant}]
Un \emph{food truck}, ouvert le midi et le soir, propose deux types de formules :
\setlength\parindent{10mm}
\begin{itemize}
\item la formule \emph{Burger} ;
\item la formule \emph{Wok}.
\end{itemize}
\setlength\parindent{0mm}
\medskip
Le gérant a remarqué que 15\,\% de ses ventes ont lieu le midi. Le quart des ventes du midi correspondent à la formule \emph{Burger}, alors que 13\,\% des ventes du soir correspondent à la formule \emph{Wok}.
Le gérant se constitue un fichier en notant, pour chaque vente, la formule choisie et le moment de cette vente (midi ou soir).
On prélève une fiche de façon équiprobable. On définit les quatre évènements suivants:
\begin{enumerate}
\item $M$ : \og la fiche correspond à une vente du midi\fg{} ;
\item $S$ : \og la fiche correspond à une vente du soir\fg {};
\item $W$ : \og la fiche correspond à une formule \emph{Wok} \fg{} ;
\item $B$ : \og la fiche correspond à une formule \emph{Burger} \fg.
\end{enumerate}
\setlength\parindent{0mm}
\medskip
\begin{enumerate}
\item Recopier puis compléter l'arbre pondéré
\begin{center}
\begin{tikzpicture}[sloped]
\node {.}
child {node {$M$}
child {node {$W$}
edge from parent
node[above] {...}
}
child {node {$B$}
edge from parent
node[above] {...}
}
edge from parent
node[above] {...}
}
child[missing] {}
child { node {$S$}
child {node {$W$}
edge from parent
node[above] {...}
}
child {node {$B$}
edge from parent
node[above] {...}
}
edge from parent
node[above] {...}
} ;
\end{tikzpicture}
\end{center}
\item Calculer la probabilité de l'évènement $M \cap W$. Interpréter ce résultat dans le contexte de l'exercice.
\item Montrer que la probabilité que la fiche choisie corresponde à une formule \emph{Burger} est égale à $0.777$.
\item On a prélevé une fiche correspondant à la formule \emph{Burger}. Quelle est la probabilité, arrondie au millième, que la vente ait eu lieu le soir?
\end{enumerate}
\end{exercise}
\begin{solution}
\begin{enumerate}
\item
\begin{center}
\begin{tikzpicture}[sloped]
\node {.}
child {node {$M$}
child {node {$W$}
edge from parent
node[above] {$0.75$}
}
child {node {$B$}
edge from parent
node[above] {$0.25$}
}
edge from parent
node[above] {$0.15$}
}
child[missing] {}
child { node {$S$}
child {node {$W$}
edge from parent
node[above] {$0.13$}
}
child {node {$B$}
edge from parent
node[above] {$0.87$}
}
edge from parent
node[above] {$0.85$}
} ;
\end{tikzpicture}
\end{center}
\item On calcule la probabilité que la vente soit un wok et ait eu lieu à midi
\[ P(M\cap W) = P(M) \times P_M(W) = 0.15 \times 0.75 = 0.1125 \]
\item Probabilité que la vente soit un burger.
\[
P(B) = P(M\cap B) + P(S\cap B) = 0.15 \times 0.75 + 0.85 \times 0.13 = 0.777
\]
\item On cherche à calculer la quantité $P_B(S)$. Pour cela on utilise la formule de Bayes
\[
P_B(S) = \frac{P(B\cap S)}{P(B)} = \frac{P_S(B) \times P(S)}{P(B)} = \frac{0.87\times 0.85}{0.777} = 0.9517374517374516 \approx 0.952
\]
\end{enumerate}
\end{solution}
\begin{exercise}[subtitle={Continent plastique}]
\textit{Les quantités évoqués dans cette exercice sont générés au hasard et sont donc complètement farfelus.}
\medskip
Le \og continent de plastique\fg{} est la plus grande des plaques de déchets plastiques évoluant sur les océans. Elle occupe actuellement dans l'océan Pacifique une surface dont l'aire est évaluée à plus de $1,6$ million de km$^2$, entre Hawaï et la Californie.
En 2017, des scientifiques ont estimé qu'il y avait $2$ millions de tonnes de déchets plastiques qui était déversé chaque année dans les océans et que cette quantité augmentait de $30\n\%$ par chaque année.
On modélise l'évolution de la masse de ces déchets plastiques déversée chaque année, si rien n'est fait pour la réduire, par une suite géométrique $\left(u_n\right)$. L'arrondi au centième du terme $u_n$ représente la masse de ces déchets déversée chaque année, exprimée en million de tonnes, pour l'année $(2017 + n)$.
\medskip
\begin{enumerate}
\item Expliquer pourquoi la suite $u_n$ est géométrique?
\item Calculer $u_1$ et $u_2$.
\item Exprimer $u_n$ en fonction de $n$.
\item Au début de l'année 2017, il y avait $300$ millions de tonnes de déchets plastique. Calculer la quantité totale de déchets plastiques en 2030.
\item On souhaite déterminer en quelle année la masse totale de ces déchets plastiques aura pour la première fois augmenté de $50$\,\% par rapport à sa valeur de 2017.
\begin{enumerate}
\item Recopier et compléter l'algorithme ci-dessous pour que la variable $N$ contienne la réponse au problème posé.
\begin{center}
\begin{tabularx}{0.4\linewidth}{|X|}\hline
$N = 2017$\\
$U = 2$ \\
$S = 300 + U$ \\
while $S < 450$: \\
\hspace{1cm} $N = \ldots$\\
\hspace{1cm} $U = \ldots$\\
\hspace{1cm} $S = \ldots$\\
\hline
\end{tabularx}
\end{center}
\item Que contiennent les variables $S$, $U$ et $N$ après exécution de cet algorithme ?
Interpréter les résultats dans le contexte de l'exercice.
\end{enumerate}
\end{enumerate}
\end{exercise}
\begin{solution}
\begin{enumerate}
\item Une augmentation de $30\,\%$ revient à multiplier la quantité par $1.3$. La suite est donc bien géométrique. Son premier terme est $u_0 = 2$ et sa raison est $q = 1.3$
\item
\[
u_1 = u_0 * 1.3 = 2.6
\]
\[
u_2 = u_0 * 1.3^2 = 3.38
\]
\item
\[
u_n = u_0 \times q^n = 2 \times 1.3^n
\]
\item On calcule la quantité totale déversée entre 2017 et 2030.
\[
\sum_{n = 0}^{13} u_n = u_0 \times \frac{1-q^{13}}{1-q} = 2 \times \frac{1 - 1.3^{13}}{1 - 1.3} = 195.25
\]
On en déduit la quantité totale de déchets en 2030
\[
300 + 195.25 = 495.25
\]
\item
\begin{enumerate}
\item ~
\begin{center}
\begin{tabularx}{0.4\linewidth}{|X|}\hline
$N \gets 2017$\\
$U \gets 2$ \\
$S \gets 300 + U$ \\
Tant que $S < 450$ \\
\hspace{1cm} $N \gets N + 1$\\
\hspace{1cm} $U \gets U * 1.3$\\
\hspace{1cm} $S \gets S + u$\\
Fin Tant que\\\hline
\end{tabularx}
\end{center}
\item \textit{Pas de correction automatisé}
\end{enumerate}
\end{enumerate}
\end{solution}
\end{document}
%%% Local Variables:
%%% mode: latex
%%% TeX-master: "master"
%%% End:

View File

@ -0,0 +1,262 @@
\documentclass[a4paper,10pt]{article}
\usepackage{myXsim}
% Title Page
\title{DS8 \hfill DEBRAS Noémie}
\tribe{TST}
\date{\hfillÀ render pour le Mercredi 7 avril}
\xsimsetup{
solution/print = false
}
\begin{document}
\maketitle
\begin{exercise}[subtitle={Automatismes}]
\textit{Toutes les questions de cette exercice sont indépendantes et peuvent être répondus séparément}
\begin{enumerate}
\item De janvier à septembre, une quantité a augmenté de $24\,\%$. Faire un schéma pour représenter la situation puis calculer le taux d'évolution moyen mensuel.
\item Une quantité augmente de $24\,\%$ par ans. En 2020, elle est de 110\euro. Quelle était sa valeur en 2019? Faire un schéma pour représenter la situation.
\item Déterminer l'équation de la droite \\
\begin{tikzpicture}[xscale=0.8, yscale=0.5]
\tkzInit[xmin=-5,xmax=5,xstep=1,
ymin=-5,ymax=5,ystep=1]
\tkzGrid
\tkzAxeXY
\tkzFct[domain=-5:5,color=red,very thick]%
{0.6666666666666666*\x -1};
\end{tikzpicture}
\item Résoudre l'équation $8 \times 0.49^x = 9$
\end{enumerate}
\end{exercise}
\begin{solution}
\begin{enumerate}
\item On veut partager cette évolution en 8 évolutions.
\[
\left(1 + \frac{24}{100}\right)^{\frac{1}{8}} = 1.0273
\]
Donc le taux d'évolution moyen est
\[
t_m = 1.0273 - 1 = 0.027300000000000102
\]
\item Coefficient multiplicateur pour revenir en arrière
\[
CM = (1 + \frac{24}{100})^{-1} = 0.8065
\]
On en déduit la quantité en 2019
\[
110 * 0.8065 = 88.715
\]
\item L'équation de la droite est
\[
y = 0.6666666666666666 x -1
\]
\item Il faut penser à faire la division à par $8$ avant d'utiliser le log car sinon, on ne peut pas utiliser la formule $\log(a^n) = n\times \log(a)$.
\[x = \frac{\log(1.12)}{\log(0.49)}\]
\end{enumerate}
\end{solution}
\begin{exercise}[subtitle={Restaurant}]
Un \emph{food truck}, ouvert le midi et le soir, propose deux types de formules :
\setlength\parindent{10mm}
\begin{itemize}
\item la formule \emph{Burger} ;
\item la formule \emph{Wok}.
\end{itemize}
\setlength\parindent{0mm}
\medskip
Le gérant a remarqué que 49\,\% de ses ventes ont lieu le midi. Le quart des ventes du midi correspondent à la formule \emph{Burger}, alors que 61\,\% des ventes du soir correspondent à la formule \emph{Wok}.
Le gérant se constitue un fichier en notant, pour chaque vente, la formule choisie et le moment de cette vente (midi ou soir).
On prélève une fiche de façon équiprobable. On définit les quatre évènements suivants:
\begin{enumerate}
\item $M$ : \og la fiche correspond à une vente du midi\fg{} ;
\item $S$ : \og la fiche correspond à une vente du soir\fg {};
\item $W$ : \og la fiche correspond à une formule \emph{Wok} \fg{} ;
\item $B$ : \og la fiche correspond à une formule \emph{Burger} \fg.
\end{enumerate}
\setlength\parindent{0mm}
\medskip
\begin{enumerate}
\item Recopier puis compléter l'arbre pondéré
\begin{center}
\begin{tikzpicture}[sloped]
\node {.}
child {node {$M$}
child {node {$W$}
edge from parent
node[above] {...}
}
child {node {$B$}
edge from parent
node[above] {...}
}
edge from parent
node[above] {...}
}
child[missing] {}
child { node {$S$}
child {node {$W$}
edge from parent
node[above] {...}
}
child {node {$B$}
edge from parent
node[above] {...}
}
edge from parent
node[above] {...}
} ;
\end{tikzpicture}
\end{center}
\item Calculer la probabilité de l'évènement $M \cap W$. Interpréter ce résultat dans le contexte de l'exercice.
\item Montrer que la probabilité que la fiche choisie corresponde à une formule \emph{Burger} est égale à $0.3214$.
\item On a prélevé une fiche correspondant à la formule \emph{Burger}. Quelle est la probabilité, arrondie au millième, que la vente ait eu lieu le soir?
\end{enumerate}
\end{exercise}
\begin{solution}
\begin{enumerate}
\item
\begin{center}
\begin{tikzpicture}[sloped]
\node {.}
child {node {$M$}
child {node {$W$}
edge from parent
node[above] {$0.75$}
}
child {node {$B$}
edge from parent
node[above] {$0.25$}
}
edge from parent
node[above] {$0.49$}
}
child[missing] {}
child { node {$S$}
child {node {$W$}
edge from parent
node[above] {$0.61$}
}
child {node {$B$}
edge from parent
node[above] {$0.39$}
}
edge from parent
node[above] {$0.51$}
} ;
\end{tikzpicture}
\end{center}
\item On calcule la probabilité que la vente soit un wok et ait eu lieu à midi
\[ P(M\cap W) = P(M) \times P_M(W) = 0.49 \times 0.75 = 0.3675 \]
\item Probabilité que la vente soit un burger.
\[
P(B) = P(M\cap B) + P(S\cap B) = 0.49 \times 0.75 + 0.51 \times 0.61 = 0.3214
\]
\item On cherche à calculer la quantité $P_B(S)$. Pour cela on utilise la formule de Bayes
\[
P_B(S) = \frac{P(B\cap S)}{P(B)} = \frac{P_S(B) \times P(S)}{P(B)} = \frac{0.39\times 0.51}{0.3214} = 0.618855009334163 \approx 0.619
\]
\end{enumerate}
\end{solution}
\begin{exercise}[subtitle={Continent plastique}]
\textit{Les quantités évoqués dans cette exercice sont générés au hasard et sont donc complètement farfelus.}
\medskip
Le \og continent de plastique\fg{} est la plus grande des plaques de déchets plastiques évoluant sur les océans. Elle occupe actuellement dans l'océan Pacifique une surface dont l'aire est évaluée à plus de $1,6$ million de km$^2$, entre Hawaï et la Californie.
En 2017, des scientifiques ont estimé qu'il y avait $15$ millions de tonnes de déchets plastiques qui était déversé chaque année dans les océans et que cette quantité augmentait de $21\n\%$ par chaque année.
On modélise l'évolution de la masse de ces déchets plastiques déversée chaque année, si rien n'est fait pour la réduire, par une suite géométrique $\left(u_n\right)$. L'arrondi au centième du terme $u_n$ représente la masse de ces déchets déversée chaque année, exprimée en million de tonnes, pour l'année $(2017 + n)$.
\medskip
\begin{enumerate}
\item Expliquer pourquoi la suite $u_n$ est géométrique?
\item Calculer $u_1$ et $u_2$.
\item Exprimer $u_n$ en fonction de $n$.
\item Au début de l'année 2017, il y avait $300$ millions de tonnes de déchets plastique. Calculer la quantité totale de déchets plastiques en 2030.
\item On souhaite déterminer en quelle année la masse totale de ces déchets plastiques aura pour la première fois augmenté de $50$\,\% par rapport à sa valeur de 2017.
\begin{enumerate}
\item Recopier et compléter l'algorithme ci-dessous pour que la variable $N$ contienne la réponse au problème posé.
\begin{center}
\begin{tabularx}{0.4\linewidth}{|X|}\hline
$N = 2017$\\
$U = 15$ \\
$S = 300 + U$ \\
while $S < 450$: \\
\hspace{1cm} $N = \ldots$\\
\hspace{1cm} $U = \ldots$\\
\hspace{1cm} $S = \ldots$\\
\hline
\end{tabularx}
\end{center}
\item Que contiennent les variables $S$, $U$ et $N$ après exécution de cet algorithme ?
Interpréter les résultats dans le contexte de l'exercice.
\end{enumerate}
\end{enumerate}
\end{exercise}
\begin{solution}
\begin{enumerate}
\item Une augmentation de $21\,\%$ revient à multiplier la quantité par $1.21$. La suite est donc bien géométrique. Son premier terme est $u_0 = 15$ et sa raison est $q = 1.21$
\item
\[
u_1 = u_0 * 1.21 = 18.15
\]
\[
u_2 = u_0 * 1.21^2 = 21.9615
\]
\item
\[
u_n = u_0 \times q^n = 15 \times 1.21^n
\]
\item On calcule la quantité totale déversée entre 2017 et 2030.
\[
\sum_{n = 0}^{13} u_n = u_0 \times \frac{1-q^{13}}{1-q} = 15 \times \frac{1 - 1.21^{13}}{1 - 1.21} = 779.87
\]
On en déduit la quantité totale de déchets en 2030
\[
300 + 779.87 = 1079.87
\]
\item
\begin{enumerate}
\item ~
\begin{center}
\begin{tabularx}{0.4\linewidth}{|X|}\hline
$N \gets 2017$\\
$U \gets 15$ \\
$S \gets 300 + U$ \\
Tant que $S < 450$ \\
\hspace{1cm} $N \gets N + 1$\\
\hspace{1cm} $U \gets U * 1.21$\\
\hspace{1cm} $S \gets S + u$\\
Fin Tant que\\\hline
\end{tabularx}
\end{center}
\item \textit{Pas de correction automatisé}
\end{enumerate}
\end{enumerate}
\end{solution}
\end{document}
%%% Local Variables:
%%% mode: latex
%%% TeX-master: "master"
%%% End:

View File

@ -0,0 +1,262 @@
\documentclass[a4paper,10pt]{article}
\usepackage{myXsim}
% Title Page
\title{DS8 \hfill GERMAIN Anaïs}
\tribe{TST}
\date{\hfillÀ render pour le Mercredi 7 avril}
\xsimsetup{
solution/print = false
}
\begin{document}
\maketitle
\begin{exercise}[subtitle={Automatismes}]
\textit{Toutes les questions de cette exercice sont indépendantes et peuvent être répondus séparément}
\begin{enumerate}
\item De janvier à septembre, une quantité a augmenté de $27\,\%$. Faire un schéma pour représenter la situation puis calculer le taux d'évolution moyen mensuel.
\item Une quantité augmente de $27\,\%$ par ans. En 2020, elle est de 132\euro. Quelle était sa valeur en 2019? Faire un schéma pour représenter la situation.
\item Déterminer l'équation de la droite \\
\begin{tikzpicture}[xscale=0.8, yscale=0.5]
\tkzInit[xmin=-5,xmax=5,xstep=1,
ymin=-5,ymax=5,ystep=1]
\tkzGrid
\tkzAxeXY
\tkzFct[domain=-5:5,color=red,very thick]%
{2.0*\x -4};
\end{tikzpicture}
\item Résoudre l'équation $7 \times 0.05^x = 18$
\end{enumerate}
\end{exercise}
\begin{solution}
\begin{enumerate}
\item On veut partager cette évolution en 8 évolutions.
\[
\left(1 + \frac{27}{100}\right)^{\frac{1}{8}} = 1.0303
\]
Donc le taux d'évolution moyen est
\[
t_m = 1.0303 - 1 = 0.030299999999999994
\]
\item Coefficient multiplicateur pour revenir en arrière
\[
CM = (1 + \frac{27}{100})^{-1} = 0.7874
\]
On en déduit la quantité en 2019
\[
132 * 0.7874 = 103.9368
\]
\item L'équation de la droite est
\[
y = 2.0 x -4
\]
\item Il faut penser à faire la division à par $7$ avant d'utiliser le log car sinon, on ne peut pas utiliser la formule $\log(a^n) = n\times \log(a)$.
\[x = \frac{\log(2.57)}{\log(0.05)}\]
\end{enumerate}
\end{solution}
\begin{exercise}[subtitle={Restaurant}]
Un \emph{food truck}, ouvert le midi et le soir, propose deux types de formules :
\setlength\parindent{10mm}
\begin{itemize}
\item la formule \emph{Burger} ;
\item la formule \emph{Wok}.
\end{itemize}
\setlength\parindent{0mm}
\medskip
Le gérant a remarqué que 38\,\% de ses ventes ont lieu le midi. Le quart des ventes du midi correspondent à la formule \emph{Burger}, alors que 63\,\% des ventes du soir correspondent à la formule \emph{Wok}.
Le gérant se constitue un fichier en notant, pour chaque vente, la formule choisie et le moment de cette vente (midi ou soir).
On prélève une fiche de façon équiprobable. On définit les quatre évènements suivants:
\begin{enumerate}
\item $M$ : \og la fiche correspond à une vente du midi\fg{} ;
\item $S$ : \og la fiche correspond à une vente du soir\fg {};
\item $W$ : \og la fiche correspond à une formule \emph{Wok} \fg{} ;
\item $B$ : \og la fiche correspond à une formule \emph{Burger} \fg.
\end{enumerate}
\setlength\parindent{0mm}
\medskip
\begin{enumerate}
\item Recopier puis compléter l'arbre pondéré
\begin{center}
\begin{tikzpicture}[sloped]
\node {.}
child {node {$M$}
child {node {$W$}
edge from parent
node[above] {...}
}
child {node {$B$}
edge from parent
node[above] {...}
}
edge from parent
node[above] {...}
}
child[missing] {}
child { node {$S$}
child {node {$W$}
edge from parent
node[above] {...}
}
child {node {$B$}
edge from parent
node[above] {...}
}
edge from parent
node[above] {...}
} ;
\end{tikzpicture}
\end{center}
\item Calculer la probabilité de l'évènement $M \cap W$. Interpréter ce résultat dans le contexte de l'exercice.
\item Montrer que la probabilité que la fiche choisie corresponde à une formule \emph{Burger} est égale à $0.3244$.
\item On a prélevé une fiche correspondant à la formule \emph{Burger}. Quelle est la probabilité, arrondie au millième, que la vente ait eu lieu le soir?
\end{enumerate}
\end{exercise}
\begin{solution}
\begin{enumerate}
\item
\begin{center}
\begin{tikzpicture}[sloped]
\node {.}
child {node {$M$}
child {node {$W$}
edge from parent
node[above] {$0.75$}
}
child {node {$B$}
edge from parent
node[above] {$0.25$}
}
edge from parent
node[above] {$0.38$}
}
child[missing] {}
child { node {$S$}
child {node {$W$}
edge from parent
node[above] {$0.63$}
}
child {node {$B$}
edge from parent
node[above] {$0.37$}
}
edge from parent
node[above] {$0.62$}
} ;
\end{tikzpicture}
\end{center}
\item On calcule la probabilité que la vente soit un wok et ait eu lieu à midi
\[ P(M\cap W) = P(M) \times P_M(W) = 0.38 \times 0.75 = 0.285 \]
\item Probabilité que la vente soit un burger.
\[
P(B) = P(M\cap B) + P(S\cap B) = 0.38 \times 0.75 + 0.62 \times 0.63 = 0.3244
\]
\item On cherche à calculer la quantité $P_B(S)$. Pour cela on utilise la formule de Bayes
\[
P_B(S) = \frac{P(B\cap S)}{P(B)} = \frac{P_S(B) \times P(S)}{P(B)} = \frac{0.37\times 0.62}{0.3244} = 0.7071516646115905 \approx 0.707
\]
\end{enumerate}
\end{solution}
\begin{exercise}[subtitle={Continent plastique}]
\textit{Les quantités évoqués dans cette exercice sont générés au hasard et sont donc complètement farfelus.}
\medskip
Le \og continent de plastique\fg{} est la plus grande des plaques de déchets plastiques évoluant sur les océans. Elle occupe actuellement dans l'océan Pacifique une surface dont l'aire est évaluée à plus de $1,6$ million de km$^2$, entre Hawaï et la Californie.
En 2017, des scientifiques ont estimé qu'il y avait $18$ millions de tonnes de déchets plastiques qui était déversé chaque année dans les océans et que cette quantité augmentait de $28\n\%$ par chaque année.
On modélise l'évolution de la masse de ces déchets plastiques déversée chaque année, si rien n'est fait pour la réduire, par une suite géométrique $\left(u_n\right)$. L'arrondi au centième du terme $u_n$ représente la masse de ces déchets déversée chaque année, exprimée en million de tonnes, pour l'année $(2017 + n)$.
\medskip
\begin{enumerate}
\item Expliquer pourquoi la suite $u_n$ est géométrique?
\item Calculer $u_1$ et $u_2$.
\item Exprimer $u_n$ en fonction de $n$.
\item Au début de l'année 2017, il y avait $300$ millions de tonnes de déchets plastique. Calculer la quantité totale de déchets plastiques en 2030.
\item On souhaite déterminer en quelle année la masse totale de ces déchets plastiques aura pour la première fois augmenté de $50$\,\% par rapport à sa valeur de 2017.
\begin{enumerate}
\item Recopier et compléter l'algorithme ci-dessous pour que la variable $N$ contienne la réponse au problème posé.
\begin{center}
\begin{tabularx}{0.4\linewidth}{|X|}\hline
$N = 2017$\\
$U = 18$ \\
$S = 300 + U$ \\
while $S < 450$: \\
\hspace{1cm} $N = \ldots$\\
\hspace{1cm} $U = \ldots$\\
\hspace{1cm} $S = \ldots$\\
\hline
\end{tabularx}
\end{center}
\item Que contiennent les variables $S$, $U$ et $N$ après exécution de cet algorithme ?
Interpréter les résultats dans le contexte de l'exercice.
\end{enumerate}
\end{enumerate}
\end{exercise}
\begin{solution}
\begin{enumerate}
\item Une augmentation de $28\,\%$ revient à multiplier la quantité par $1.28$. La suite est donc bien géométrique. Son premier terme est $u_0 = 18$ et sa raison est $q = 1.28$
\item
\[
u_1 = u_0 * 1.28 = 23.04
\]
\[
u_2 = u_0 * 1.28^2 = 29.4912
\]
\item
\[
u_n = u_0 \times q^n = 18 \times 1.28^n
\]
\item On calcule la quantité totale déversée entre 2017 et 2030.
\[
\sum_{n = 0}^{13} u_n = u_0 \times \frac{1-q^{13}}{1-q} = 18 \times \frac{1 - 1.28^{13}}{1 - 1.28} = 1527.35
\]
On en déduit la quantité totale de déchets en 2030
\[
300 + 1527.35 = 1827.35
\]
\item
\begin{enumerate}
\item ~
\begin{center}
\begin{tabularx}{0.4\linewidth}{|X|}\hline
$N \gets 2017$\\
$U \gets 18$ \\
$S \gets 300 + U$ \\
Tant que $S < 450$ \\
\hspace{1cm} $N \gets N + 1$\\
\hspace{1cm} $U \gets U * 1.28$\\
\hspace{1cm} $S \gets S + u$\\
Fin Tant que\\\hline
\end{tabularx}
\end{center}
\item \textit{Pas de correction automatisé}
\end{enumerate}
\end{enumerate}
\end{solution}
\end{document}
%%% Local Variables:
%%% mode: latex
%%% TeX-master: "master"
%%% End:

View File

@ -0,0 +1,262 @@
\documentclass[a4paper,10pt]{article}
\usepackage{myXsim}
% Title Page
\title{DS8 \hfill HADJRAS Mohcine}
\tribe{TST}
\date{\hfillÀ render pour le Mercredi 7 avril}
\xsimsetup{
solution/print = false
}
\begin{document}
\maketitle
\begin{exercise}[subtitle={Automatismes}]
\textit{Toutes les questions de cette exercice sont indépendantes et peuvent être répondus séparément}
\begin{enumerate}
\item De janvier à septembre, une quantité a augmenté de $19\,\%$. Faire un schéma pour représenter la situation puis calculer le taux d'évolution moyen mensuel.
\item Une quantité augmente de $19\,\%$ par ans. En 2020, elle est de 113\euro. Quelle était sa valeur en 2019? Faire un schéma pour représenter la situation.
\item Déterminer l'équation de la droite \\
\begin{tikzpicture}[xscale=0.8, yscale=0.5]
\tkzInit[xmin=-5,xmax=5,xstep=1,
ymin=-5,ymax=5,ystep=1]
\tkzGrid
\tkzAxeXY
\tkzFct[domain=-5:5,color=red,very thick]%
{0.5*\x -1};
\end{tikzpicture}
\item Résoudre l'équation $4 \times 0.54^x = 6$
\end{enumerate}
\end{exercise}
\begin{solution}
\begin{enumerate}
\item On veut partager cette évolution en 8 évolutions.
\[
\left(1 + \frac{19}{100}\right)^{\frac{1}{8}} = 1.022
\]
Donc le taux d'évolution moyen est
\[
t_m = 1.022 - 1 = 0.02200000000000002
\]
\item Coefficient multiplicateur pour revenir en arrière
\[
CM = (1 + \frac{19}{100})^{-1} = 0.8403
\]
On en déduit la quantité en 2019
\[
113 * 0.8403 = 94.9539
\]
\item L'équation de la droite est
\[
y = 0.5 x -1
\]
\item Il faut penser à faire la division à par $4$ avant d'utiliser le log car sinon, on ne peut pas utiliser la formule $\log(a^n) = n\times \log(a)$.
\[x = \frac{\log(1.5)}{\log(0.54)}\]
\end{enumerate}
\end{solution}
\begin{exercise}[subtitle={Restaurant}]
Un \emph{food truck}, ouvert le midi et le soir, propose deux types de formules :
\setlength\parindent{10mm}
\begin{itemize}
\item la formule \emph{Burger} ;
\item la formule \emph{Wok}.
\end{itemize}
\setlength\parindent{0mm}
\medskip
Le gérant a remarqué que 68\,\% de ses ventes ont lieu le midi. Le quart des ventes du midi correspondent à la formule \emph{Burger}, alors que 43\,\% des ventes du soir correspondent à la formule \emph{Wok}.
Le gérant se constitue un fichier en notant, pour chaque vente, la formule choisie et le moment de cette vente (midi ou soir).
On prélève une fiche de façon équiprobable. On définit les quatre évènements suivants:
\begin{enumerate}
\item $M$ : \og la fiche correspond à une vente du midi\fg{} ;
\item $S$ : \og la fiche correspond à une vente du soir\fg {};
\item $W$ : \og la fiche correspond à une formule \emph{Wok} \fg{} ;
\item $B$ : \og la fiche correspond à une formule \emph{Burger} \fg.
\end{enumerate}
\setlength\parindent{0mm}
\medskip
\begin{enumerate}
\item Recopier puis compléter l'arbre pondéré
\begin{center}
\begin{tikzpicture}[sloped]
\node {.}
child {node {$M$}
child {node {$W$}
edge from parent
node[above] {...}
}
child {node {$B$}
edge from parent
node[above] {...}
}
edge from parent
node[above] {...}
}
child[missing] {}
child { node {$S$}
child {node {$W$}
edge from parent
node[above] {...}
}
child {node {$B$}
edge from parent
node[above] {...}
}
edge from parent
node[above] {...}
} ;
\end{tikzpicture}
\end{center}
\item Calculer la probabilité de l'évènement $M \cap W$. Interpréter ce résultat dans le contexte de l'exercice.
\item Montrer que la probabilité que la fiche choisie corresponde à une formule \emph{Burger} est égale à $0.3524$.
\item On a prélevé une fiche correspondant à la formule \emph{Burger}. Quelle est la probabilité, arrondie au millième, que la vente ait eu lieu le soir?
\end{enumerate}
\end{exercise}
\begin{solution}
\begin{enumerate}
\item
\begin{center}
\begin{tikzpicture}[sloped]
\node {.}
child {node {$M$}
child {node {$W$}
edge from parent
node[above] {$0.75$}
}
child {node {$B$}
edge from parent
node[above] {$0.25$}
}
edge from parent
node[above] {$0.68$}
}
child[missing] {}
child { node {$S$}
child {node {$W$}
edge from parent
node[above] {$0.43$}
}
child {node {$B$}
edge from parent
node[above] {$0.57$}
}
edge from parent
node[above] {$0.32$}
} ;
\end{tikzpicture}
\end{center}
\item On calcule la probabilité que la vente soit un wok et ait eu lieu à midi
\[ P(M\cap W) = P(M) \times P_M(W) = 0.68 \times 0.75 = 0.51 \]
\item Probabilité que la vente soit un burger.
\[
P(B) = P(M\cap B) + P(S\cap B) = 0.68 \times 0.75 + 0.32 \times 0.43 = 0.3524
\]
\item On cherche à calculer la quantité $P_B(S)$. Pour cela on utilise la formule de Bayes
\[
P_B(S) = \frac{P(B\cap S)}{P(B)} = \frac{P_S(B) \times P(S)}{P(B)} = \frac{0.57\times 0.32}{0.3524} = 0.5175936435868331 \approx 0.518
\]
\end{enumerate}
\end{solution}
\begin{exercise}[subtitle={Continent plastique}]
\textit{Les quantités évoqués dans cette exercice sont générés au hasard et sont donc complètement farfelus.}
\medskip
Le \og continent de plastique\fg{} est la plus grande des plaques de déchets plastiques évoluant sur les océans. Elle occupe actuellement dans l'océan Pacifique une surface dont l'aire est évaluée à plus de $1,6$ million de km$^2$, entre Hawaï et la Californie.
En 2017, des scientifiques ont estimé qu'il y avait $4$ millions de tonnes de déchets plastiques qui était déversé chaque année dans les océans et que cette quantité augmentait de $10\n\%$ par chaque année.
On modélise l'évolution de la masse de ces déchets plastiques déversée chaque année, si rien n'est fait pour la réduire, par une suite géométrique $\left(u_n\right)$. L'arrondi au centième du terme $u_n$ représente la masse de ces déchets déversée chaque année, exprimée en million de tonnes, pour l'année $(2017 + n)$.
\medskip
\begin{enumerate}
\item Expliquer pourquoi la suite $u_n$ est géométrique?
\item Calculer $u_1$ et $u_2$.
\item Exprimer $u_n$ en fonction de $n$.
\item Au début de l'année 2017, il y avait $300$ millions de tonnes de déchets plastique. Calculer la quantité totale de déchets plastiques en 2030.
\item On souhaite déterminer en quelle année la masse totale de ces déchets plastiques aura pour la première fois augmenté de $50$\,\% par rapport à sa valeur de 2017.
\begin{enumerate}
\item Recopier et compléter l'algorithme ci-dessous pour que la variable $N$ contienne la réponse au problème posé.
\begin{center}
\begin{tabularx}{0.4\linewidth}{|X|}\hline
$N = 2017$\\
$U = 4$ \\
$S = 300 + U$ \\
while $S < 450$: \\
\hspace{1cm} $N = \ldots$\\
\hspace{1cm} $U = \ldots$\\
\hspace{1cm} $S = \ldots$\\
\hline
\end{tabularx}
\end{center}
\item Que contiennent les variables $S$, $U$ et $N$ après exécution de cet algorithme ?
Interpréter les résultats dans le contexte de l'exercice.
\end{enumerate}
\end{enumerate}
\end{exercise}
\begin{solution}
\begin{enumerate}
\item Une augmentation de $10\,\%$ revient à multiplier la quantité par $1.1$. La suite est donc bien géométrique. Son premier terme est $u_0 = 4$ et sa raison est $q = 1.1$
\item
\[
u_1 = u_0 * 1.1 = 4.4
\]
\[
u_2 = u_0 * 1.1^2 = 4.84
\]
\item
\[
u_n = u_0 \times q^n = 4 \times 1.1^n
\]
\item On calcule la quantité totale déversée entre 2017 et 2030.
\[
\sum_{n = 0}^{13} u_n = u_0 \times \frac{1-q^{13}}{1-q} = 4 \times \frac{1 - 1.1^{13}}{1 - 1.1} = 98.09
\]
On en déduit la quantité totale de déchets en 2030
\[
300 + 98.09 = 398.09000000000003
\]
\item
\begin{enumerate}
\item ~
\begin{center}
\begin{tabularx}{0.4\linewidth}{|X|}\hline
$N \gets 2017$\\
$U \gets 4$ \\
$S \gets 300 + U$ \\
Tant que $S < 450$ \\
\hspace{1cm} $N \gets N + 1$\\
\hspace{1cm} $U \gets U * 1.1$\\
\hspace{1cm} $S \gets S + u$\\
Fin Tant que\\\hline
\end{tabularx}
\end{center}
\item \textit{Pas de correction automatisé}
\end{enumerate}
\end{enumerate}
\end{solution}
\end{document}
%%% Local Variables:
%%% mode: latex
%%% TeX-master: "master"
%%% End:

View File

@ -0,0 +1,262 @@
\documentclass[a4paper,10pt]{article}
\usepackage{myXsim}
% Title Page
\title{DS8 \hfill HENRIST Maxime}
\tribe{TST}
\date{\hfillÀ render pour le Mercredi 7 avril}
\xsimsetup{
solution/print = false
}
\begin{document}
\maketitle
\begin{exercise}[subtitle={Automatismes}]
\textit{Toutes les questions de cette exercice sont indépendantes et peuvent être répondus séparément}
\begin{enumerate}
\item De janvier à septembre, une quantité a augmenté de $19\,\%$. Faire un schéma pour représenter la situation puis calculer le taux d'évolution moyen mensuel.
\item Une quantité augmente de $19\,\%$ par ans. En 2020, elle est de 133\euro. Quelle était sa valeur en 2019? Faire un schéma pour représenter la situation.
\item Déterminer l'équation de la droite \\
\begin{tikzpicture}[xscale=0.8, yscale=0.5]
\tkzInit[xmin=-5,xmax=5,xstep=1,
ymin=-5,ymax=5,ystep=1]
\tkzGrid
\tkzAxeXY
\tkzFct[domain=-5:5,color=red,very thick]%
{1.0*\x -2};
\end{tikzpicture}
\item Résoudre l'équation $7 \times 0.82^x = 19$
\end{enumerate}
\end{exercise}
\begin{solution}
\begin{enumerate}
\item On veut partager cette évolution en 8 évolutions.
\[
\left(1 + \frac{19}{100}\right)^{\frac{1}{8}} = 1.022
\]
Donc le taux d'évolution moyen est
\[
t_m = 1.022 - 1 = 0.02200000000000002
\]
\item Coefficient multiplicateur pour revenir en arrière
\[
CM = (1 + \frac{19}{100})^{-1} = 0.8403
\]
On en déduit la quantité en 2019
\[
133 * 0.8403 = 111.7599
\]
\item L'équation de la droite est
\[
y = 1.0 x -2
\]
\item Il faut penser à faire la division à par $7$ avant d'utiliser le log car sinon, on ne peut pas utiliser la formule $\log(a^n) = n\times \log(a)$.
\[x = \frac{\log(2.71)}{\log(0.82)}\]
\end{enumerate}
\end{solution}
\begin{exercise}[subtitle={Restaurant}]
Un \emph{food truck}, ouvert le midi et le soir, propose deux types de formules :
\setlength\parindent{10mm}
\begin{itemize}
\item la formule \emph{Burger} ;
\item la formule \emph{Wok}.
\end{itemize}
\setlength\parindent{0mm}
\medskip
Le gérant a remarqué que 20\,\% de ses ventes ont lieu le midi. Le quart des ventes du midi correspondent à la formule \emph{Burger}, alors que 56\,\% des ventes du soir correspondent à la formule \emph{Wok}.
Le gérant se constitue un fichier en notant, pour chaque vente, la formule choisie et le moment de cette vente (midi ou soir).
On prélève une fiche de façon équiprobable. On définit les quatre évènements suivants:
\begin{enumerate}
\item $M$ : \og la fiche correspond à une vente du midi\fg{} ;
\item $S$ : \og la fiche correspond à une vente du soir\fg {};
\item $W$ : \og la fiche correspond à une formule \emph{Wok} \fg{} ;
\item $B$ : \og la fiche correspond à une formule \emph{Burger} \fg.
\end{enumerate}
\setlength\parindent{0mm}
\medskip
\begin{enumerate}
\item Recopier puis compléter l'arbre pondéré
\begin{center}
\begin{tikzpicture}[sloped]
\node {.}
child {node {$M$}
child {node {$W$}
edge from parent
node[above] {...}
}
child {node {$B$}
edge from parent
node[above] {...}
}
edge from parent
node[above] {...}
}
child[missing] {}
child { node {$S$}
child {node {$W$}
edge from parent
node[above] {...}
}
child {node {$B$}
edge from parent
node[above] {...}
}
edge from parent
node[above] {...}
} ;
\end{tikzpicture}
\end{center}
\item Calculer la probabilité de l'évènement $M \cap W$. Interpréter ce résultat dans le contexte de l'exercice.
\item Montrer que la probabilité que la fiche choisie corresponde à une formule \emph{Burger} est égale à $0.394$.
\item On a prélevé une fiche correspondant à la formule \emph{Burger}. Quelle est la probabilité, arrondie au millième, que la vente ait eu lieu le soir?
\end{enumerate}
\end{exercise}
\begin{solution}
\begin{enumerate}
\item
\begin{center}
\begin{tikzpicture}[sloped]
\node {.}
child {node {$M$}
child {node {$W$}
edge from parent
node[above] {$0.75$}
}
child {node {$B$}
edge from parent
node[above] {$0.25$}
}
edge from parent
node[above] {$0.2$}
}
child[missing] {}
child { node {$S$}
child {node {$W$}
edge from parent
node[above] {$0.57$}
}
child {node {$B$}
edge from parent
node[above] {$0.43$}
}
edge from parent
node[above] {$0.8$}
} ;
\end{tikzpicture}
\end{center}
\item On calcule la probabilité que la vente soit un wok et ait eu lieu à midi
\[ P(M\cap W) = P(M) \times P_M(W) = 0.2 \times 0.75 = 0.15 \]
\item Probabilité que la vente soit un burger.
\[
P(B) = P(M\cap B) + P(S\cap B) = 0.2 \times 0.75 + 0.8 \times 0.57 = 0.394
\]
\item On cherche à calculer la quantité $P_B(S)$. Pour cela on utilise la formule de Bayes
\[
P_B(S) = \frac{P(B\cap S)}{P(B)} = \frac{P_S(B) \times P(S)}{P(B)} = \frac{0.43\times 0.8}{0.394} = 0.8730964467005077 \approx 0.873
\]
\end{enumerate}
\end{solution}
\begin{exercise}[subtitle={Continent plastique}]
\textit{Les quantités évoqués dans cette exercice sont générés au hasard et sont donc complètement farfelus.}
\medskip
Le \og continent de plastique\fg{} est la plus grande des plaques de déchets plastiques évoluant sur les océans. Elle occupe actuellement dans l'océan Pacifique une surface dont l'aire est évaluée à plus de $1,6$ million de km$^2$, entre Hawaï et la Californie.
En 2017, des scientifiques ont estimé qu'il y avait $5$ millions de tonnes de déchets plastiques qui était déversé chaque année dans les océans et que cette quantité augmentait de $16\n\%$ par chaque année.
On modélise l'évolution de la masse de ces déchets plastiques déversée chaque année, si rien n'est fait pour la réduire, par une suite géométrique $\left(u_n\right)$. L'arrondi au centième du terme $u_n$ représente la masse de ces déchets déversée chaque année, exprimée en million de tonnes, pour l'année $(2017 + n)$.
\medskip
\begin{enumerate}
\item Expliquer pourquoi la suite $u_n$ est géométrique?
\item Calculer $u_1$ et $u_2$.
\item Exprimer $u_n$ en fonction de $n$.
\item Au début de l'année 2017, il y avait $300$ millions de tonnes de déchets plastique. Calculer la quantité totale de déchets plastiques en 2030.
\item On souhaite déterminer en quelle année la masse totale de ces déchets plastiques aura pour la première fois augmenté de $50$\,\% par rapport à sa valeur de 2017.
\begin{enumerate}
\item Recopier et compléter l'algorithme ci-dessous pour que la variable $N$ contienne la réponse au problème posé.
\begin{center}
\begin{tabularx}{0.4\linewidth}{|X|}\hline
$N = 2017$\\
$U = 5$ \\
$S = 300 + U$ \\
while $S < 450$: \\
\hspace{1cm} $N = \ldots$\\
\hspace{1cm} $U = \ldots$\\
\hspace{1cm} $S = \ldots$\\
\hline
\end{tabularx}
\end{center}
\item Que contiennent les variables $S$, $U$ et $N$ après exécution de cet algorithme ?
Interpréter les résultats dans le contexte de l'exercice.
\end{enumerate}
\end{enumerate}
\end{exercise}
\begin{solution}
\begin{enumerate}
\item Une augmentation de $16\,\%$ revient à multiplier la quantité par $1.16$. La suite est donc bien géométrique. Son premier terme est $u_0 = 5$ et sa raison est $q = 1.16$
\item
\[
u_1 = u_0 * 1.16 = 5.8
\]
\[
u_2 = u_0 * 1.16^2 = 6.728
\]
\item
\[
u_n = u_0 \times q^n = 5 \times 1.16^n
\]
\item On calcule la quantité totale déversée entre 2017 et 2030.
\[
\sum_{n = 0}^{13} u_n = u_0 \times \frac{1-q^{13}}{1-q} = 5 \times \frac{1 - 1.16^{13}}{1 - 1.16} = 183.93
\]
On en déduit la quantité totale de déchets en 2030
\[
300 + 183.93 = 483.93
\]
\item
\begin{enumerate}
\item ~
\begin{center}
\begin{tabularx}{0.4\linewidth}{|X|}\hline
$N \gets 2017$\\
$U \gets 5$ \\
$S \gets 300 + U$ \\
Tant que $S < 450$ \\
\hspace{1cm} $N \gets N + 1$\\
\hspace{1cm} $U \gets U * 1.16$\\
\hspace{1cm} $S \gets S + u$\\
Fin Tant que\\\hline
\end{tabularx}
\end{center}
\item \textit{Pas de correction automatisé}
\end{enumerate}
\end{enumerate}
\end{solution}
\end{document}
%%% Local Variables:
%%% mode: latex
%%% TeX-master: "master"
%%% End:

View File

@ -0,0 +1,262 @@
\documentclass[a4paper,10pt]{article}
\usepackage{myXsim}
% Title Page
\title{DS8 \hfill INFANTES Antoine}
\tribe{TST}
\date{\hfillÀ render pour le Mercredi 7 avril}
\xsimsetup{
solution/print = false
}
\begin{document}
\maketitle
\begin{exercise}[subtitle={Automatismes}]
\textit{Toutes les questions de cette exercice sont indépendantes et peuvent être répondus séparément}
\begin{enumerate}
\item De janvier à septembre, une quantité a augmenté de $18\,\%$. Faire un schéma pour représenter la situation puis calculer le taux d'évolution moyen mensuel.
\item Une quantité augmente de $18\,\%$ par ans. En 2020, elle est de 130\euro. Quelle était sa valeur en 2019? Faire un schéma pour représenter la situation.
\item Déterminer l'équation de la droite \\
\begin{tikzpicture}[xscale=0.8, yscale=0.5]
\tkzInit[xmin=-5,xmax=5,xstep=1,
ymin=-5,ymax=5,ystep=1]
\tkzGrid
\tkzAxeXY
\tkzFct[domain=-5:5,color=red,very thick]%
{2.6666666666666665*\x -4};
\end{tikzpicture}
\item Résoudre l'équation $3 \times 0.31^x = 15$
\end{enumerate}
\end{exercise}
\begin{solution}
\begin{enumerate}
\item On veut partager cette évolution en 8 évolutions.
\[
\left(1 + \frac{18}{100}\right)^{\frac{1}{8}} = 1.0209
\]
Donc le taux d'évolution moyen est
\[
t_m = 1.0209 - 1 = 0.02089999999999992
\]
\item Coefficient multiplicateur pour revenir en arrière
\[
CM = (1 + \frac{18}{100})^{-1} = 0.8475
\]
On en déduit la quantité en 2019
\[
130 * 0.8475 = 110.175
\]
\item L'équation de la droite est
\[
y = 2.6666666666666665 x -4
\]
\item Il faut penser à faire la division à par $3$ avant d'utiliser le log car sinon, on ne peut pas utiliser la formule $\log(a^n) = n\times \log(a)$.
\[x = \frac{\log(5.0)}{\log(0.31)}\]
\end{enumerate}
\end{solution}
\begin{exercise}[subtitle={Restaurant}]
Un \emph{food truck}, ouvert le midi et le soir, propose deux types de formules :
\setlength\parindent{10mm}
\begin{itemize}
\item la formule \emph{Burger} ;
\item la formule \emph{Wok}.
\end{itemize}
\setlength\parindent{0mm}
\medskip
Le gérant a remarqué que 33\,\% de ses ventes ont lieu le midi. Le quart des ventes du midi correspondent à la formule \emph{Burger}, alors que 13\,\% des ventes du soir correspondent à la formule \emph{Wok}.
Le gérant se constitue un fichier en notant, pour chaque vente, la formule choisie et le moment de cette vente (midi ou soir).
On prélève une fiche de façon équiprobable. On définit les quatre évènements suivants:
\begin{enumerate}
\item $M$ : \og la fiche correspond à une vente du midi\fg{} ;
\item $S$ : \og la fiche correspond à une vente du soir\fg {};
\item $W$ : \og la fiche correspond à une formule \emph{Wok} \fg{} ;
\item $B$ : \og la fiche correspond à une formule \emph{Burger} \fg.
\end{enumerate}
\setlength\parindent{0mm}
\medskip
\begin{enumerate}
\item Recopier puis compléter l'arbre pondéré
\begin{center}
\begin{tikzpicture}[sloped]
\node {.}
child {node {$M$}
child {node {$W$}
edge from parent
node[above] {...}
}
child {node {$B$}
edge from parent
node[above] {...}
}
edge from parent
node[above] {...}
}
child[missing] {}
child { node {$S$}
child {node {$W$}
edge from parent
node[above] {...}
}
child {node {$B$}
edge from parent
node[above] {...}
}
edge from parent
node[above] {...}
} ;
\end{tikzpicture}
\end{center}
\item Calculer la probabilité de l'évènement $M \cap W$. Interpréter ce résultat dans le contexte de l'exercice.
\item Montrer que la probabilité que la fiche choisie corresponde à une formule \emph{Burger} est égale à $0.6654$.
\item On a prélevé une fiche correspondant à la formule \emph{Burger}. Quelle est la probabilité, arrondie au millième, que la vente ait eu lieu le soir?
\end{enumerate}
\end{exercise}
\begin{solution}
\begin{enumerate}
\item
\begin{center}
\begin{tikzpicture}[sloped]
\node {.}
child {node {$M$}
child {node {$W$}
edge from parent
node[above] {$0.75$}
}
child {node {$B$}
edge from parent
node[above] {$0.25$}
}
edge from parent
node[above] {$0.33$}
}
child[missing] {}
child { node {$S$}
child {node {$W$}
edge from parent
node[above] {$0.13$}
}
child {node {$B$}
edge from parent
node[above] {$0.87$}
}
edge from parent
node[above] {$0.67$}
} ;
\end{tikzpicture}
\end{center}
\item On calcule la probabilité que la vente soit un wok et ait eu lieu à midi
\[ P(M\cap W) = P(M) \times P_M(W) = 0.33 \times 0.75 = 0.2475 \]
\item Probabilité que la vente soit un burger.
\[
P(B) = P(M\cap B) + P(S\cap B) = 0.33 \times 0.75 + 0.67 \times 0.13 = 0.6654
\]
\item On cherche à calculer la quantité $P_B(S)$. Pour cela on utilise la formule de Bayes
\[
P_B(S) = \frac{P(B\cap S)}{P(B)} = \frac{P_S(B) \times P(S)}{P(B)} = \frac{0.87\times 0.67}{0.6654} = 0.8760144274120831 \approx 0.876
\]
\end{enumerate}
\end{solution}
\begin{exercise}[subtitle={Continent plastique}]
\textit{Les quantités évoqués dans cette exercice sont générés au hasard et sont donc complètement farfelus.}
\medskip
Le \og continent de plastique\fg{} est la plus grande des plaques de déchets plastiques évoluant sur les océans. Elle occupe actuellement dans l'océan Pacifique une surface dont l'aire est évaluée à plus de $1,6$ million de km$^2$, entre Hawaï et la Californie.
En 2017, des scientifiques ont estimé qu'il y avait $12$ millions de tonnes de déchets plastiques qui était déversé chaque année dans les océans et que cette quantité augmentait de $26\n\%$ par chaque année.
On modélise l'évolution de la masse de ces déchets plastiques déversée chaque année, si rien n'est fait pour la réduire, par une suite géométrique $\left(u_n\right)$. L'arrondi au centième du terme $u_n$ représente la masse de ces déchets déversée chaque année, exprimée en million de tonnes, pour l'année $(2017 + n)$.
\medskip
\begin{enumerate}
\item Expliquer pourquoi la suite $u_n$ est géométrique?
\item Calculer $u_1$ et $u_2$.
\item Exprimer $u_n$ en fonction de $n$.
\item Au début de l'année 2017, il y avait $300$ millions de tonnes de déchets plastique. Calculer la quantité totale de déchets plastiques en 2030.
\item On souhaite déterminer en quelle année la masse totale de ces déchets plastiques aura pour la première fois augmenté de $50$\,\% par rapport à sa valeur de 2017.
\begin{enumerate}
\item Recopier et compléter l'algorithme ci-dessous pour que la variable $N$ contienne la réponse au problème posé.
\begin{center}
\begin{tabularx}{0.4\linewidth}{|X|}\hline
$N = 2017$\\
$U = 12$ \\
$S = 300 + U$ \\
while $S < 450$: \\
\hspace{1cm} $N = \ldots$\\
\hspace{1cm} $U = \ldots$\\
\hspace{1cm} $S = \ldots$\\
\hline
\end{tabularx}
\end{center}
\item Que contiennent les variables $S$, $U$ et $N$ après exécution de cet algorithme ?
Interpréter les résultats dans le contexte de l'exercice.
\end{enumerate}
\end{enumerate}
\end{exercise}
\begin{solution}
\begin{enumerate}
\item Une augmentation de $26\,\%$ revient à multiplier la quantité par $1.26$. La suite est donc bien géométrique. Son premier terme est $u_0 = 12$ et sa raison est $q = 1.26$
\item
\[
u_1 = u_0 * 1.26 = 15.120000000000001
\]
\[
u_2 = u_0 * 1.26^2 = 19.0512
\]
\item
\[
u_n = u_0 \times q^n = 12 \times 1.26^n
\]
\item On calcule la quantité totale déversée entre 2017 et 2030.
\[
\sum_{n = 0}^{13} u_n = u_0 \times \frac{1-q^{13}}{1-q} = 12 \times \frac{1 - 1.26^{13}}{1 - 1.26} = 885.01
\]
On en déduit la quantité totale de déchets en 2030
\[
300 + 885.01 = 1185.01
\]
\item
\begin{enumerate}
\item ~
\begin{center}
\begin{tabularx}{0.4\linewidth}{|X|}\hline
$N \gets 2017$\\
$U \gets 12$ \\
$S \gets 300 + U$ \\
Tant que $S < 450$ \\
\hspace{1cm} $N \gets N + 1$\\
\hspace{1cm} $U \gets U * 1.26$\\
\hspace{1cm} $S \gets S + u$\\
Fin Tant que\\\hline
\end{tabularx}
\end{center}
\item \textit{Pas de correction automatisé}
\end{enumerate}
\end{enumerate}
\end{solution}
\end{document}
%%% Local Variables:
%%% mode: latex
%%% TeX-master: "master"
%%% End:

View File

@ -0,0 +1,262 @@
\documentclass[a4paper,10pt]{article}
\usepackage{myXsim}
% Title Page
\title{DS8 \hfill MAGRO Robin}
\tribe{TST}
\date{\hfillÀ render pour le Mercredi 7 avril}
\xsimsetup{
solution/print = false
}
\begin{document}
\maketitle
\begin{exercise}[subtitle={Automatismes}]
\textit{Toutes les questions de cette exercice sont indépendantes et peuvent être répondus séparément}
\begin{enumerate}
\item De janvier à septembre, une quantité a augmenté de $11\,\%$. Faire un schéma pour représenter la situation puis calculer le taux d'évolution moyen mensuel.
\item Une quantité augmente de $11\,\%$ par ans. En 2020, elle est de 143\euro. Quelle était sa valeur en 2019? Faire un schéma pour représenter la situation.
\item Déterminer l'équation de la droite \\
\begin{tikzpicture}[xscale=0.8, yscale=0.5]
\tkzInit[xmin=-5,xmax=5,xstep=1,
ymin=-5,ymax=5,ystep=1]
\tkzGrid
\tkzAxeXY
\tkzFct[domain=-5:5,color=red,very thick]%
{3.0*\x -3};
\end{tikzpicture}
\item Résoudre l'équation $5 \times 0.12^x = 40$
\end{enumerate}
\end{exercise}
\begin{solution}
\begin{enumerate}
\item On veut partager cette évolution en 8 évolutions.
\[
\left(1 + \frac{11}{100}\right)^{\frac{1}{8}} = 1.0131
\]
Donc le taux d'évolution moyen est
\[
t_m = 1.0131 - 1 = 0.01309999999999989
\]
\item Coefficient multiplicateur pour revenir en arrière
\[
CM = (1 + \frac{11}{100})^{-1} = 0.9009
\]
On en déduit la quantité en 2019
\[
143 * 0.9009 = 128.8287
\]
\item L'équation de la droite est
\[
y = 3.0 x -3
\]
\item Il faut penser à faire la division à par $5$ avant d'utiliser le log car sinon, on ne peut pas utiliser la formule $\log(a^n) = n\times \log(a)$.
\[x = \frac{\log(8.0)}{\log(0.12)}\]
\end{enumerate}
\end{solution}
\begin{exercise}[subtitle={Restaurant}]
Un \emph{food truck}, ouvert le midi et le soir, propose deux types de formules :
\setlength\parindent{10mm}
\begin{itemize}
\item la formule \emph{Burger} ;
\item la formule \emph{Wok}.
\end{itemize}
\setlength\parindent{0mm}
\medskip
Le gérant a remarqué que 37\,\% de ses ventes ont lieu le midi. Le quart des ventes du midi correspondent à la formule \emph{Burger}, alors que 32\,\% des ventes du soir correspondent à la formule \emph{Wok}.
Le gérant se constitue un fichier en notant, pour chaque vente, la formule choisie et le moment de cette vente (midi ou soir).
On prélève une fiche de façon équiprobable. On définit les quatre évènements suivants:
\begin{enumerate}
\item $M$ : \og la fiche correspond à une vente du midi\fg{} ;
\item $S$ : \og la fiche correspond à une vente du soir\fg {};
\item $W$ : \og la fiche correspond à une formule \emph{Wok} \fg{} ;
\item $B$ : \og la fiche correspond à une formule \emph{Burger} \fg.
\end{enumerate}
\setlength\parindent{0mm}
\medskip
\begin{enumerate}
\item Recopier puis compléter l'arbre pondéré
\begin{center}
\begin{tikzpicture}[sloped]
\node {.}
child {node {$M$}
child {node {$W$}
edge from parent
node[above] {...}
}
child {node {$B$}
edge from parent
node[above] {...}
}
edge from parent
node[above] {...}
}
child[missing] {}
child { node {$S$}
child {node {$W$}
edge from parent
node[above] {...}
}
child {node {$B$}
edge from parent
node[above] {...}
}
edge from parent
node[above] {...}
} ;
\end{tikzpicture}
\end{center}
\item Calculer la probabilité de l'évènement $M \cap W$. Interpréter ce résultat dans le contexte de l'exercice.
\item Montrer que la probabilité que la fiche choisie corresponde à une formule \emph{Burger} est égale à $0.5209$.
\item On a prélevé une fiche correspondant à la formule \emph{Burger}. Quelle est la probabilité, arrondie au millième, que la vente ait eu lieu le soir?
\end{enumerate}
\end{exercise}
\begin{solution}
\begin{enumerate}
\item
\begin{center}
\begin{tikzpicture}[sloped]
\node {.}
child {node {$M$}
child {node {$W$}
edge from parent
node[above] {$0.75$}
}
child {node {$B$}
edge from parent
node[above] {$0.25$}
}
edge from parent
node[above] {$0.37$}
}
child[missing] {}
child { node {$S$}
child {node {$W$}
edge from parent
node[above] {$0.32$}
}
child {node {$B$}
edge from parent
node[above] {$0.68$}
}
edge from parent
node[above] {$0.63$}
} ;
\end{tikzpicture}
\end{center}
\item On calcule la probabilité que la vente soit un wok et ait eu lieu à midi
\[ P(M\cap W) = P(M) \times P_M(W) = 0.37 \times 0.75 = 0.2775 \]
\item Probabilité que la vente soit un burger.
\[
P(B) = P(M\cap B) + P(S\cap B) = 0.37 \times 0.75 + 0.63 \times 0.32 = 0.5209
\]
\item On cherche à calculer la quantité $P_B(S)$. Pour cela on utilise la formule de Bayes
\[
P_B(S) = \frac{P(B\cap S)}{P(B)} = \frac{P_S(B) \times P(S)}{P(B)} = \frac{0.68\times 0.63}{0.5209} = 0.8224227298905741 \approx 0.822
\]
\end{enumerate}
\end{solution}
\begin{exercise}[subtitle={Continent plastique}]
\textit{Les quantités évoqués dans cette exercice sont générés au hasard et sont donc complètement farfelus.}
\medskip
Le \og continent de plastique\fg{} est la plus grande des plaques de déchets plastiques évoluant sur les océans. Elle occupe actuellement dans l'océan Pacifique une surface dont l'aire est évaluée à plus de $1,6$ million de km$^2$, entre Hawaï et la Californie.
En 2017, des scientifiques ont estimé qu'il y avait $16$ millions de tonnes de déchets plastiques qui était déversé chaque année dans les océans et que cette quantité augmentait de $17\n\%$ par chaque année.
On modélise l'évolution de la masse de ces déchets plastiques déversée chaque année, si rien n'est fait pour la réduire, par une suite géométrique $\left(u_n\right)$. L'arrondi au centième du terme $u_n$ représente la masse de ces déchets déversée chaque année, exprimée en million de tonnes, pour l'année $(2017 + n)$.
\medskip
\begin{enumerate}
\item Expliquer pourquoi la suite $u_n$ est géométrique?
\item Calculer $u_1$ et $u_2$.
\item Exprimer $u_n$ en fonction de $n$.
\item Au début de l'année 2017, il y avait $300$ millions de tonnes de déchets plastique. Calculer la quantité totale de déchets plastiques en 2030.
\item On souhaite déterminer en quelle année la masse totale de ces déchets plastiques aura pour la première fois augmenté de $50$\,\% par rapport à sa valeur de 2017.
\begin{enumerate}
\item Recopier et compléter l'algorithme ci-dessous pour que la variable $N$ contienne la réponse au problème posé.
\begin{center}
\begin{tabularx}{0.4\linewidth}{|X|}\hline
$N = 2017$\\
$U = 16$ \\
$S = 300 + U$ \\
while $S < 450$: \\
\hspace{1cm} $N = \ldots$\\
\hspace{1cm} $U = \ldots$\\
\hspace{1cm} $S = \ldots$\\
\hline
\end{tabularx}
\end{center}
\item Que contiennent les variables $S$, $U$ et $N$ après exécution de cet algorithme ?
Interpréter les résultats dans le contexte de l'exercice.
\end{enumerate}
\end{enumerate}
\end{exercise}
\begin{solution}
\begin{enumerate}
\item Une augmentation de $17\,\%$ revient à multiplier la quantité par $1.17$. La suite est donc bien géométrique. Son premier terme est $u_0 = 16$ et sa raison est $q = 1.17$
\item
\[
u_1 = u_0 * 1.17 = 18.72
\]
\[
u_2 = u_0 * 1.17^2 = 21.9024
\]
\item
\[
u_n = u_0 \times q^n = 16 \times 1.17^n
\]
\item On calcule la quantité totale déversée entre 2017 et 2030.
\[
\sum_{n = 0}^{13} u_n = u_0 \times \frac{1-q^{13}}{1-q} = 16 \times \frac{1 - 1.17^{13}}{1 - 1.17} = 630.46
\]
On en déduit la quantité totale de déchets en 2030
\[
300 + 630.46 = 930.46
\]
\item
\begin{enumerate}
\item ~
\begin{center}
\begin{tabularx}{0.4\linewidth}{|X|}\hline
$N \gets 2017$\\
$U \gets 16$ \\
$S \gets 300 + U$ \\
Tant que $S < 450$ \\
\hspace{1cm} $N \gets N + 1$\\
\hspace{1cm} $U \gets U * 1.17$\\
\hspace{1cm} $S \gets S + u$\\
Fin Tant que\\\hline
\end{tabularx}
\end{center}
\item \textit{Pas de correction automatisé}
\end{enumerate}
\end{enumerate}
\end{solution}
\end{document}
%%% Local Variables:
%%% mode: latex
%%% TeX-master: "master"
%%% End:

View File

@ -0,0 +1,262 @@
\documentclass[a4paper,10pt]{article}
\usepackage{myXsim}
% Title Page
\title{DS8 \hfill MORFIN Chloé}
\tribe{TST}
\date{\hfillÀ render pour le Mercredi 7 avril}
\xsimsetup{
solution/print = false
}
\begin{document}
\maketitle
\begin{exercise}[subtitle={Automatismes}]
\textit{Toutes les questions de cette exercice sont indépendantes et peuvent être répondus séparément}
\begin{enumerate}
\item De janvier à septembre, une quantité a augmenté de $18\,\%$. Faire un schéma pour représenter la situation puis calculer le taux d'évolution moyen mensuel.
\item Une quantité augmente de $18\,\%$ par ans. En 2020, elle est de 119\euro. Quelle était sa valeur en 2019? Faire un schéma pour représenter la situation.
\item Déterminer l'équation de la droite \\
\begin{tikzpicture}[xscale=0.8, yscale=0.5]
\tkzInit[xmin=-5,xmax=5,xstep=1,
ymin=-5,ymax=5,ystep=1]
\tkzGrid
\tkzAxeXY
\tkzFct[domain=-5:5,color=red,very thick]%
{4.0*\x -4};
\end{tikzpicture}
\item Résoudre l'équation $3 \times 1.0^x = 32$
\end{enumerate}
\end{exercise}
\begin{solution}
\begin{enumerate}
\item On veut partager cette évolution en 8 évolutions.
\[
\left(1 + \frac{18}{100}\right)^{\frac{1}{8}} = 1.0209
\]
Donc le taux d'évolution moyen est
\[
t_m = 1.0209 - 1 = 0.02089999999999992
\]
\item Coefficient multiplicateur pour revenir en arrière
\[
CM = (1 + \frac{18}{100})^{-1} = 0.8475
\]
On en déduit la quantité en 2019
\[
119 * 0.8475 = 100.8525
\]
\item L'équation de la droite est
\[
y = 4.0 x -4
\]
\item Il faut penser à faire la division à par $3$ avant d'utiliser le log car sinon, on ne peut pas utiliser la formule $\log(a^n) = n\times \log(a)$.
\[x = \frac{\log(10.67)}{\log(1.0)}\]
\end{enumerate}
\end{solution}
\begin{exercise}[subtitle={Restaurant}]
Un \emph{food truck}, ouvert le midi et le soir, propose deux types de formules :
\setlength\parindent{10mm}
\begin{itemize}
\item la formule \emph{Burger} ;
\item la formule \emph{Wok}.
\end{itemize}
\setlength\parindent{0mm}
\medskip
Le gérant a remarqué que 24\,\% de ses ventes ont lieu le midi. Le quart des ventes du midi correspondent à la formule \emph{Burger}, alors que 57\,\% des ventes du soir correspondent à la formule \emph{Wok}.
Le gérant se constitue un fichier en notant, pour chaque vente, la formule choisie et le moment de cette vente (midi ou soir).
On prélève une fiche de façon équiprobable. On définit les quatre évènements suivants:
\begin{enumerate}
\item $M$ : \og la fiche correspond à une vente du midi\fg{} ;
\item $S$ : \og la fiche correspond à une vente du soir\fg {};
\item $W$ : \og la fiche correspond à une formule \emph{Wok} \fg{} ;
\item $B$ : \og la fiche correspond à une formule \emph{Burger} \fg.
\end{enumerate}
\setlength\parindent{0mm}
\medskip
\begin{enumerate}
\item Recopier puis compléter l'arbre pondéré
\begin{center}
\begin{tikzpicture}[sloped]
\node {.}
child {node {$M$}
child {node {$W$}
edge from parent
node[above] {...}
}
child {node {$B$}
edge from parent
node[above] {...}
}
edge from parent
node[above] {...}
}
child[missing] {}
child { node {$S$}
child {node {$W$}
edge from parent
node[above] {...}
}
child {node {$B$}
edge from parent
node[above] {...}
}
edge from parent
node[above] {...}
} ;
\end{tikzpicture}
\end{center}
\item Calculer la probabilité de l'évènement $M \cap W$. Interpréter ce résultat dans le contexte de l'exercice.
\item Montrer que la probabilité que la fiche choisie corresponde à une formule \emph{Burger} est égale à $0.3792$.
\item On a prélevé une fiche correspondant à la formule \emph{Burger}. Quelle est la probabilité, arrondie au millième, que la vente ait eu lieu le soir?
\end{enumerate}
\end{exercise}
\begin{solution}
\begin{enumerate}
\item
\begin{center}
\begin{tikzpicture}[sloped]
\node {.}
child {node {$M$}
child {node {$W$}
edge from parent
node[above] {$0.75$}
}
child {node {$B$}
edge from parent
node[above] {$0.25$}
}
edge from parent
node[above] {$0.24$}
}
child[missing] {}
child { node {$S$}
child {node {$W$}
edge from parent
node[above] {$0.58$}
}
child {node {$B$}
edge from parent
node[above] {$0.42$}
}
edge from parent
node[above] {$0.76$}
} ;
\end{tikzpicture}
\end{center}
\item On calcule la probabilité que la vente soit un wok et ait eu lieu à midi
\[ P(M\cap W) = P(M) \times P_M(W) = 0.24 \times 0.75 = 0.18 \]
\item Probabilité que la vente soit un burger.
\[
P(B) = P(M\cap B) + P(S\cap B) = 0.24 \times 0.75 + 0.76 \times 0.58 = 0.3792
\]
\item On cherche à calculer la quantité $P_B(S)$. Pour cela on utilise la formule de Bayes
\[
P_B(S) = \frac{P(B\cap S)}{P(B)} = \frac{P_S(B) \times P(S)}{P(B)} = \frac{0.42\times 0.76}{0.3792} = 0.8417721518987342 \approx 0.842
\]
\end{enumerate}
\end{solution}
\begin{exercise}[subtitle={Continent plastique}]
\textit{Les quantités évoqués dans cette exercice sont générés au hasard et sont donc complètement farfelus.}
\medskip
Le \og continent de plastique\fg{} est la plus grande des plaques de déchets plastiques évoluant sur les océans. Elle occupe actuellement dans l'océan Pacifique une surface dont l'aire est évaluée à plus de $1,6$ million de km$^2$, entre Hawaï et la Californie.
En 2017, des scientifiques ont estimé qu'il y avait $17$ millions de tonnes de déchets plastiques qui était déversé chaque année dans les océans et que cette quantité augmentait de $16\n\%$ par chaque année.
On modélise l'évolution de la masse de ces déchets plastiques déversée chaque année, si rien n'est fait pour la réduire, par une suite géométrique $\left(u_n\right)$. L'arrondi au centième du terme $u_n$ représente la masse de ces déchets déversée chaque année, exprimée en million de tonnes, pour l'année $(2017 + n)$.
\medskip
\begin{enumerate}
\item Expliquer pourquoi la suite $u_n$ est géométrique?
\item Calculer $u_1$ et $u_2$.
\item Exprimer $u_n$ en fonction de $n$.
\item Au début de l'année 2017, il y avait $300$ millions de tonnes de déchets plastique. Calculer la quantité totale de déchets plastiques en 2030.
\item On souhaite déterminer en quelle année la masse totale de ces déchets plastiques aura pour la première fois augmenté de $50$\,\% par rapport à sa valeur de 2017.
\begin{enumerate}
\item Recopier et compléter l'algorithme ci-dessous pour que la variable $N$ contienne la réponse au problème posé.
\begin{center}
\begin{tabularx}{0.4\linewidth}{|X|}\hline
$N = 2017$\\
$U = 17$ \\
$S = 300 + U$ \\
while $S < 450$: \\
\hspace{1cm} $N = \ldots$\\
\hspace{1cm} $U = \ldots$\\
\hspace{1cm} $S = \ldots$\\
\hline
\end{tabularx}
\end{center}
\item Que contiennent les variables $S$, $U$ et $N$ après exécution de cet algorithme ?
Interpréter les résultats dans le contexte de l'exercice.
\end{enumerate}
\end{enumerate}
\end{exercise}
\begin{solution}
\begin{enumerate}
\item Une augmentation de $16\,\%$ revient à multiplier la quantité par $1.16$. La suite est donc bien géométrique. Son premier terme est $u_0 = 17$ et sa raison est $q = 1.16$
\item
\[
u_1 = u_0 * 1.16 = 19.72
\]
\[
u_2 = u_0 * 1.16^2 = 22.8752
\]
\item
\[
u_n = u_0 \times q^n = 17 \times 1.16^n
\]
\item On calcule la quantité totale déversée entre 2017 et 2030.
\[
\sum_{n = 0}^{13} u_n = u_0 \times \frac{1-q^{13}}{1-q} = 17 \times \frac{1 - 1.16^{13}}{1 - 1.16} = 625.37
\]
On en déduit la quantité totale de déchets en 2030
\[
300 + 625.37 = 925.37
\]
\item
\begin{enumerate}
\item ~
\begin{center}
\begin{tabularx}{0.4\linewidth}{|X|}\hline
$N \gets 2017$\\
$U \gets 17$ \\
$S \gets 300 + U$ \\
Tant que $S < 450$ \\
\hspace{1cm} $N \gets N + 1$\\
\hspace{1cm} $U \gets U * 1.16$\\
\hspace{1cm} $S \gets S + u$\\
Fin Tant que\\\hline
\end{tabularx}
\end{center}
\item \textit{Pas de correction automatisé}
\end{enumerate}
\end{enumerate}
\end{solution}
\end{document}
%%% Local Variables:
%%% mode: latex
%%% TeX-master: "master"
%%% End:

View File

@ -0,0 +1,262 @@
\documentclass[a4paper,10pt]{article}
\usepackage{myXsim}
% Title Page
\title{DS8 \hfill PERES RAMALHO Emeric}
\tribe{TST}
\date{\hfillÀ render pour le Mercredi 7 avril}
\xsimsetup{
solution/print = false
}
\begin{document}
\maketitle
\begin{exercise}[subtitle={Automatismes}]
\textit{Toutes les questions de cette exercice sont indépendantes et peuvent être répondus séparément}
\begin{enumerate}
\item De janvier à septembre, une quantité a augmenté de $23\,\%$. Faire un schéma pour représenter la situation puis calculer le taux d'évolution moyen mensuel.
\item Une quantité augmente de $23\,\%$ par ans. En 2020, elle est de 118\euro. Quelle était sa valeur en 2019? Faire un schéma pour représenter la situation.
\item Déterminer l'équation de la droite \\
\begin{tikzpicture}[xscale=0.8, yscale=0.5]
\tkzInit[xmin=-5,xmax=5,xstep=1,
ymin=-5,ymax=5,ystep=1]
\tkzGrid
\tkzAxeXY
\tkzFct[domain=-5:5,color=red,very thick]%
{1.3333333333333333*\x -2};
\end{tikzpicture}
\item Résoudre l'équation $5 \times 0.46^x = 2$
\end{enumerate}
\end{exercise}
\begin{solution}
\begin{enumerate}
\item On veut partager cette évolution en 8 évolutions.
\[
\left(1 + \frac{23}{100}\right)^{\frac{1}{8}} = 1.0262
\]
Donc le taux d'évolution moyen est
\[
t_m = 1.0262 - 1 = 0.0262
\]
\item Coefficient multiplicateur pour revenir en arrière
\[
CM = (1 + \frac{23}{100})^{-1} = 0.813
\]
On en déduit la quantité en 2019
\[
118 * 0.813 = 95.934
\]
\item L'équation de la droite est
\[
y = 1.3333333333333333 x -2
\]
\item Il faut penser à faire la division à par $5$ avant d'utiliser le log car sinon, on ne peut pas utiliser la formule $\log(a^n) = n\times \log(a)$.
\[x = \frac{\log(0.4)}{\log(0.46)}\]
\end{enumerate}
\end{solution}
\begin{exercise}[subtitle={Restaurant}]
Un \emph{food truck}, ouvert le midi et le soir, propose deux types de formules :
\setlength\parindent{10mm}
\begin{itemize}
\item la formule \emph{Burger} ;
\item la formule \emph{Wok}.
\end{itemize}
\setlength\parindent{0mm}
\medskip
Le gérant a remarqué que 11\,\% de ses ventes ont lieu le midi. Le quart des ventes du midi correspondent à la formule \emph{Burger}, alors que 18\,\% des ventes du soir correspondent à la formule \emph{Wok}.
Le gérant se constitue un fichier en notant, pour chaque vente, la formule choisie et le moment de cette vente (midi ou soir).
On prélève une fiche de façon équiprobable. On définit les quatre évènements suivants:
\begin{enumerate}
\item $M$ : \og la fiche correspond à une vente du midi\fg{} ;
\item $S$ : \og la fiche correspond à une vente du soir\fg {};
\item $W$ : \og la fiche correspond à une formule \emph{Wok} \fg{} ;
\item $B$ : \og la fiche correspond à une formule \emph{Burger} \fg.
\end{enumerate}
\setlength\parindent{0mm}
\medskip
\begin{enumerate}
\item Recopier puis compléter l'arbre pondéré
\begin{center}
\begin{tikzpicture}[sloped]
\node {.}
child {node {$M$}
child {node {$W$}
edge from parent
node[above] {...}
}
child {node {$B$}
edge from parent
node[above] {...}
}
edge from parent
node[above] {...}
}
child[missing] {}
child { node {$S$}
child {node {$W$}
edge from parent
node[above] {...}
}
child {node {$B$}
edge from parent
node[above] {...}
}
edge from parent
node[above] {...}
} ;
\end{tikzpicture}
\end{center}
\item Calculer la probabilité de l'évènement $M \cap W$. Interpréter ce résultat dans le contexte de l'exercice.
\item Montrer que la probabilité que la fiche choisie corresponde à une formule \emph{Burger} est égale à $0.7573$.
\item On a prélevé une fiche correspondant à la formule \emph{Burger}. Quelle est la probabilité, arrondie au millième, que la vente ait eu lieu le soir?
\end{enumerate}
\end{exercise}
\begin{solution}
\begin{enumerate}
\item
\begin{center}
\begin{tikzpicture}[sloped]
\node {.}
child {node {$M$}
child {node {$W$}
edge from parent
node[above] {$0.75$}
}
child {node {$B$}
edge from parent
node[above] {$0.25$}
}
edge from parent
node[above] {$0.11$}
}
child[missing] {}
child { node {$S$}
child {node {$W$}
edge from parent
node[above] {$0.18$}
}
child {node {$B$}
edge from parent
node[above] {$0.82$}
}
edge from parent
node[above] {$0.89$}
} ;
\end{tikzpicture}
\end{center}
\item On calcule la probabilité que la vente soit un wok et ait eu lieu à midi
\[ P(M\cap W) = P(M) \times P_M(W) = 0.11 \times 0.75 = 0.0825 \]
\item Probabilité que la vente soit un burger.
\[
P(B) = P(M\cap B) + P(S\cap B) = 0.11 \times 0.75 + 0.89 \times 0.18 = 0.7573
\]
\item On cherche à calculer la quantité $P_B(S)$. Pour cela on utilise la formule de Bayes
\[
P_B(S) = \frac{P(B\cap S)}{P(B)} = \frac{P_S(B) \times P(S)}{P(B)} = \frac{0.82\times 0.89}{0.7573} = 0.9636867819886439 \approx 0.964
\]
\end{enumerate}
\end{solution}
\begin{exercise}[subtitle={Continent plastique}]
\textit{Les quantités évoqués dans cette exercice sont générés au hasard et sont donc complètement farfelus.}
\medskip
Le \og continent de plastique\fg{} est la plus grande des plaques de déchets plastiques évoluant sur les océans. Elle occupe actuellement dans l'océan Pacifique une surface dont l'aire est évaluée à plus de $1,6$ million de km$^2$, entre Hawaï et la Californie.
En 2017, des scientifiques ont estimé qu'il y avait $19$ millions de tonnes de déchets plastiques qui était déversé chaque année dans les océans et que cette quantité augmentait de $22\n\%$ par chaque année.
On modélise l'évolution de la masse de ces déchets plastiques déversée chaque année, si rien n'est fait pour la réduire, par une suite géométrique $\left(u_n\right)$. L'arrondi au centième du terme $u_n$ représente la masse de ces déchets déversée chaque année, exprimée en million de tonnes, pour l'année $(2017 + n)$.
\medskip
\begin{enumerate}
\item Expliquer pourquoi la suite $u_n$ est géométrique?
\item Calculer $u_1$ et $u_2$.
\item Exprimer $u_n$ en fonction de $n$.
\item Au début de l'année 2017, il y avait $300$ millions de tonnes de déchets plastique. Calculer la quantité totale de déchets plastiques en 2030.
\item On souhaite déterminer en quelle année la masse totale de ces déchets plastiques aura pour la première fois augmenté de $50$\,\% par rapport à sa valeur de 2017.
\begin{enumerate}
\item Recopier et compléter l'algorithme ci-dessous pour que la variable $N$ contienne la réponse au problème posé.
\begin{center}
\begin{tabularx}{0.4\linewidth}{|X|}\hline
$N = 2017$\\
$U = 19$ \\
$S = 300 + U$ \\
while $S < 450$: \\
\hspace{1cm} $N = \ldots$\\
\hspace{1cm} $U = \ldots$\\
\hspace{1cm} $S = \ldots$\\
\hline
\end{tabularx}
\end{center}
\item Que contiennent les variables $S$, $U$ et $N$ après exécution de cet algorithme ?
Interpréter les résultats dans le contexte de l'exercice.
\end{enumerate}
\end{enumerate}
\end{exercise}
\begin{solution}
\begin{enumerate}
\item Une augmentation de $22\,\%$ revient à multiplier la quantité par $1.22$. La suite est donc bien géométrique. Son premier terme est $u_0 = 19$ et sa raison est $q = 1.22$
\item
\[
u_1 = u_0 * 1.22 = 23.18
\]
\[
u_2 = u_0 * 1.22^2 = 28.2796
\]
\item
\[
u_n = u_0 \times q^n = 19 \times 1.22^n
\]
\item On calcule la quantité totale déversée entre 2017 et 2030.
\[
\sum_{n = 0}^{13} u_n = u_0 \times \frac{1-q^{13}}{1-q} = 19 \times \frac{1 - 1.22^{13}}{1 - 1.22} = 1059.17
\]
On en déduit la quantité totale de déchets en 2030
\[
300 + 1059.17 = 1359.17
\]
\item
\begin{enumerate}
\item ~
\begin{center}
\begin{tabularx}{0.4\linewidth}{|X|}\hline
$N \gets 2017$\\
$U \gets 19$ \\
$S \gets 300 + U$ \\
Tant que $S < 450$ \\
\hspace{1cm} $N \gets N + 1$\\
\hspace{1cm} $U \gets U * 1.22$\\
\hspace{1cm} $S \gets S + u$\\
Fin Tant que\\\hline
\end{tabularx}
\end{center}
\item \textit{Pas de correction automatisé}
\end{enumerate}
\end{enumerate}
\end{solution}
\end{document}
%%% Local Variables:
%%% mode: latex
%%% TeX-master: "master"
%%% End:

View File

@ -0,0 +1,262 @@
\documentclass[a4paper,10pt]{article}
\usepackage{myXsim}
% Title Page
\title{DS8 \hfill RADOUAA Saleh}
\tribe{TST}
\date{\hfillÀ render pour le Mercredi 7 avril}
\xsimsetup{
solution/print = false
}
\begin{document}
\maketitle
\begin{exercise}[subtitle={Automatismes}]
\textit{Toutes les questions de cette exercice sont indépendantes et peuvent être répondus séparément}
\begin{enumerate}
\item De janvier à septembre, une quantité a augmenté de $27\,\%$. Faire un schéma pour représenter la situation puis calculer le taux d'évolution moyen mensuel.
\item Une quantité augmente de $27\,\%$ par ans. En 2020, elle est de 141\euro. Quelle était sa valeur en 2019? Faire un schéma pour représenter la situation.
\item Déterminer l'équation de la droite \\
\begin{tikzpicture}[xscale=0.8, yscale=0.5]
\tkzInit[xmin=-5,xmax=5,xstep=1,
ymin=-5,ymax=5,ystep=1]
\tkzGrid
\tkzAxeXY
\tkzFct[domain=-5:5,color=red,very thick]%
{2.0*\x -4};
\end{tikzpicture}
\item Résoudre l'équation $2 \times 0.88^x = 8$
\end{enumerate}
\end{exercise}
\begin{solution}
\begin{enumerate}
\item On veut partager cette évolution en 8 évolutions.
\[
\left(1 + \frac{27}{100}\right)^{\frac{1}{8}} = 1.0303
\]
Donc le taux d'évolution moyen est
\[
t_m = 1.0303 - 1 = 0.030299999999999994
\]
\item Coefficient multiplicateur pour revenir en arrière
\[
CM = (1 + \frac{27}{100})^{-1} = 0.7874
\]
On en déduit la quantité en 2019
\[
141 * 0.7874 = 111.0234
\]
\item L'équation de la droite est
\[
y = 2.0 x -4
\]
\item Il faut penser à faire la division à par $2$ avant d'utiliser le log car sinon, on ne peut pas utiliser la formule $\log(a^n) = n\times \log(a)$.
\[x = \frac{\log(4.0)}{\log(0.88)}\]
\end{enumerate}
\end{solution}
\begin{exercise}[subtitle={Restaurant}]
Un \emph{food truck}, ouvert le midi et le soir, propose deux types de formules :
\setlength\parindent{10mm}
\begin{itemize}
\item la formule \emph{Burger} ;
\item la formule \emph{Wok}.
\end{itemize}
\setlength\parindent{0mm}
\medskip
Le gérant a remarqué que 1\,\% de ses ventes ont lieu le midi. Le quart des ventes du midi correspondent à la formule \emph{Burger}, alors que 99\,\% des ventes du soir correspondent à la formule \emph{Wok}.
Le gérant se constitue un fichier en notant, pour chaque vente, la formule choisie et le moment de cette vente (midi ou soir).
On prélève une fiche de façon équiprobable. On définit les quatre évènements suivants:
\begin{enumerate}
\item $M$ : \og la fiche correspond à une vente du midi\fg{} ;
\item $S$ : \og la fiche correspond à une vente du soir\fg {};
\item $W$ : \og la fiche correspond à une formule \emph{Wok} \fg{} ;
\item $B$ : \og la fiche correspond à une formule \emph{Burger} \fg.
\end{enumerate}
\setlength\parindent{0mm}
\medskip
\begin{enumerate}
\item Recopier puis compléter l'arbre pondéré
\begin{center}
\begin{tikzpicture}[sloped]
\node {.}
child {node {$M$}
child {node {$W$}
edge from parent
node[above] {...}
}
child {node {$B$}
edge from parent
node[above] {...}
}
edge from parent
node[above] {...}
}
child[missing] {}
child { node {$S$}
child {node {$W$}
edge from parent
node[above] {...}
}
child {node {$B$}
edge from parent
node[above] {...}
}
edge from parent
node[above] {...}
} ;
\end{tikzpicture}
\end{center}
\item Calculer la probabilité de l'évènement $M \cap W$. Interpréter ce résultat dans le contexte de l'exercice.
\item Montrer que la probabilité que la fiche choisie corresponde à une formule \emph{Burger} est égale à $0.0124$.
\item On a prélevé une fiche correspondant à la formule \emph{Burger}. Quelle est la probabilité, arrondie au millième, que la vente ait eu lieu le soir?
\end{enumerate}
\end{exercise}
\begin{solution}
\begin{enumerate}
\item
\begin{center}
\begin{tikzpicture}[sloped]
\node {.}
child {node {$M$}
child {node {$W$}
edge from parent
node[above] {$0.75$}
}
child {node {$B$}
edge from parent
node[above] {$0.25$}
}
edge from parent
node[above] {$0.01$}
}
child[missing] {}
child { node {$S$}
child {node {$W$}
edge from parent
node[above] {$0.99$}
}
child {node {$B$}
edge from parent
node[above] {$0.01$}
}
edge from parent
node[above] {$0.99$}
} ;
\end{tikzpicture}
\end{center}
\item On calcule la probabilité que la vente soit un wok et ait eu lieu à midi
\[ P(M\cap W) = P(M) \times P_M(W) = 0.01 \times 0.75 = 0.0075 \]
\item Probabilité que la vente soit un burger.
\[
P(B) = P(M\cap B) + P(S\cap B) = 0.01 \times 0.75 + 0.99 \times 0.99 = 0.0124
\]
\item On cherche à calculer la quantité $P_B(S)$. Pour cela on utilise la formule de Bayes
\[
P_B(S) = \frac{P(B\cap S)}{P(B)} = \frac{P_S(B) \times P(S)}{P(B)} = \frac{0.01\times 0.99}{0.0124} = 0.7983870967741936 \approx 0.798
\]
\end{enumerate}
\end{solution}
\begin{exercise}[subtitle={Continent plastique}]
\textit{Les quantités évoqués dans cette exercice sont générés au hasard et sont donc complètement farfelus.}
\medskip
Le \og continent de plastique\fg{} est la plus grande des plaques de déchets plastiques évoluant sur les océans. Elle occupe actuellement dans l'océan Pacifique une surface dont l'aire est évaluée à plus de $1,6$ million de km$^2$, entre Hawaï et la Californie.
En 2017, des scientifiques ont estimé qu'il y avait $5$ millions de tonnes de déchets plastiques qui était déversé chaque année dans les océans et que cette quantité augmentait de $17\n\%$ par chaque année.
On modélise l'évolution de la masse de ces déchets plastiques déversée chaque année, si rien n'est fait pour la réduire, par une suite géométrique $\left(u_n\right)$. L'arrondi au centième du terme $u_n$ représente la masse de ces déchets déversée chaque année, exprimée en million de tonnes, pour l'année $(2017 + n)$.
\medskip
\begin{enumerate}
\item Expliquer pourquoi la suite $u_n$ est géométrique?
\item Calculer $u_1$ et $u_2$.
\item Exprimer $u_n$ en fonction de $n$.
\item Au début de l'année 2017, il y avait $300$ millions de tonnes de déchets plastique. Calculer la quantité totale de déchets plastiques en 2030.
\item On souhaite déterminer en quelle année la masse totale de ces déchets plastiques aura pour la première fois augmenté de $50$\,\% par rapport à sa valeur de 2017.
\begin{enumerate}
\item Recopier et compléter l'algorithme ci-dessous pour que la variable $N$ contienne la réponse au problème posé.
\begin{center}
\begin{tabularx}{0.4\linewidth}{|X|}\hline
$N = 2017$\\
$U = 5$ \\
$S = 300 + U$ \\
while $S < 450$: \\
\hspace{1cm} $N = \ldots$\\
\hspace{1cm} $U = \ldots$\\
\hspace{1cm} $S = \ldots$\\
\hline
\end{tabularx}
\end{center}
\item Que contiennent les variables $S$, $U$ et $N$ après exécution de cet algorithme ?
Interpréter les résultats dans le contexte de l'exercice.
\end{enumerate}
\end{enumerate}
\end{exercise}
\begin{solution}
\begin{enumerate}
\item Une augmentation de $17\,\%$ revient à multiplier la quantité par $1.17$. La suite est donc bien géométrique. Son premier terme est $u_0 = 5$ et sa raison est $q = 1.17$
\item
\[
u_1 = u_0 * 1.17 = 5.85
\]
\[
u_2 = u_0 * 1.17^2 = 6.8445
\]
\item
\[
u_n = u_0 \times q^n = 5 \times 1.17^n
\]
\item On calcule la quantité totale déversée entre 2017 et 2030.
\[
\sum_{n = 0}^{13} u_n = u_0 \times \frac{1-q^{13}}{1-q} = 5 \times \frac{1 - 1.17^{13}}{1 - 1.17} = 197.02
\]
On en déduit la quantité totale de déchets en 2030
\[
300 + 197.02 = 497.02
\]
\item
\begin{enumerate}
\item ~
\begin{center}
\begin{tabularx}{0.4\linewidth}{|X|}\hline
$N \gets 2017$\\
$U \gets 5$ \\
$S \gets 300 + U$ \\
Tant que $S < 450$ \\
\hspace{1cm} $N \gets N + 1$\\
\hspace{1cm} $U \gets U * 1.17$\\
\hspace{1cm} $S \gets S + u$\\
Fin Tant que\\\hline
\end{tabularx}
\end{center}
\item \textit{Pas de correction automatisé}
\end{enumerate}
\end{enumerate}
\end{solution}
\end{document}
%%% Local Variables:
%%% mode: latex
%%% TeX-master: "master"
%%% End:

View File

@ -0,0 +1,262 @@
\documentclass[a4paper,10pt]{article}
\usepackage{myXsim}
% Title Page
\title{DS8 \hfill TAY Ummuhan}
\tribe{TST}
\date{\hfillÀ render pour le Mercredi 7 avril}
\xsimsetup{
solution/print = false
}
\begin{document}
\maketitle
\begin{exercise}[subtitle={Automatismes}]
\textit{Toutes les questions de cette exercice sont indépendantes et peuvent être répondus séparément}
\begin{enumerate}
\item De janvier à septembre, une quantité a augmenté de $14\,\%$. Faire un schéma pour représenter la situation puis calculer le taux d'évolution moyen mensuel.
\item Une quantité augmente de $14\,\%$ par ans. En 2020, elle est de 116\euro. Quelle était sa valeur en 2019? Faire un schéma pour représenter la situation.
\item Déterminer l'équation de la droite \\
\begin{tikzpicture}[xscale=0.8, yscale=0.5]
\tkzInit[xmin=-5,xmax=5,xstep=1,
ymin=-5,ymax=5,ystep=1]
\tkzGrid
\tkzAxeXY
\tkzFct[domain=-5:5,color=red,very thick]%
{1.3333333333333333*\x -2};
\end{tikzpicture}
\item Résoudre l'équation $6 \times 0.14^x = 14$
\end{enumerate}
\end{exercise}
\begin{solution}
\begin{enumerate}
\item On veut partager cette évolution en 8 évolutions.
\[
\left(1 + \frac{14}{100}\right)^{\frac{1}{8}} = 1.0165
\]
Donc le taux d'évolution moyen est
\[
t_m = 1.0165 - 1 = 0.01649999999999996
\]
\item Coefficient multiplicateur pour revenir en arrière
\[
CM = (1 + \frac{14}{100})^{-1} = 0.8772
\]
On en déduit la quantité en 2019
\[
116 * 0.8772 = 101.7552
\]
\item L'équation de la droite est
\[
y = 1.3333333333333333 x -2
\]
\item Il faut penser à faire la division à par $6$ avant d'utiliser le log car sinon, on ne peut pas utiliser la formule $\log(a^n) = n\times \log(a)$.
\[x = \frac{\log(2.33)}{\log(0.14)}\]
\end{enumerate}
\end{solution}
\begin{exercise}[subtitle={Restaurant}]
Un \emph{food truck}, ouvert le midi et le soir, propose deux types de formules :
\setlength\parindent{10mm}
\begin{itemize}
\item la formule \emph{Burger} ;
\item la formule \emph{Wok}.
\end{itemize}
\setlength\parindent{0mm}
\medskip
Le gérant a remarqué que 28\,\% de ses ventes ont lieu le midi. Le quart des ventes du midi correspondent à la formule \emph{Burger}, alors que 80\,\% des ventes du soir correspondent à la formule \emph{Wok}.
Le gérant se constitue un fichier en notant, pour chaque vente, la formule choisie et le moment de cette vente (midi ou soir).
On prélève une fiche de façon équiprobable. On définit les quatre évènements suivants:
\begin{enumerate}
\item $M$ : \og la fiche correspond à une vente du midi\fg{} ;
\item $S$ : \og la fiche correspond à une vente du soir\fg {};
\item $W$ : \og la fiche correspond à une formule \emph{Wok} \fg{} ;
\item $B$ : \og la fiche correspond à une formule \emph{Burger} \fg.
\end{enumerate}
\setlength\parindent{0mm}
\medskip
\begin{enumerate}
\item Recopier puis compléter l'arbre pondéré
\begin{center}
\begin{tikzpicture}[sloped]
\node {.}
child {node {$M$}
child {node {$W$}
edge from parent
node[above] {...}
}
child {node {$B$}
edge from parent
node[above] {...}
}
edge from parent
node[above] {...}
}
child[missing] {}
child { node {$S$}
child {node {$W$}
edge from parent
node[above] {...}
}
child {node {$B$}
edge from parent
node[above] {...}
}
edge from parent
node[above] {...}
} ;
\end{tikzpicture}
\end{center}
\item Calculer la probabilité de l'évènement $M \cap W$. Interpréter ce résultat dans le contexte de l'exercice.
\item Montrer que la probabilité que la fiche choisie corresponde à une formule \emph{Burger} est égale à $0.214$.
\item On a prélevé une fiche correspondant à la formule \emph{Burger}. Quelle est la probabilité, arrondie au millième, que la vente ait eu lieu le soir?
\end{enumerate}
\end{exercise}
\begin{solution}
\begin{enumerate}
\item
\begin{center}
\begin{tikzpicture}[sloped]
\node {.}
child {node {$M$}
child {node {$W$}
edge from parent
node[above] {$0.75$}
}
child {node {$B$}
edge from parent
node[above] {$0.25$}
}
edge from parent
node[above] {$0.28$}
}
child[missing] {}
child { node {$S$}
child {node {$W$}
edge from parent
node[above] {$0.8$}
}
child {node {$B$}
edge from parent
node[above] {$0.2$}
}
edge from parent
node[above] {$0.72$}
} ;
\end{tikzpicture}
\end{center}
\item On calcule la probabilité que la vente soit un wok et ait eu lieu à midi
\[ P(M\cap W) = P(M) \times P_M(W) = 0.28 \times 0.75 = 0.21 \]
\item Probabilité que la vente soit un burger.
\[
P(B) = P(M\cap B) + P(S\cap B) = 0.28 \times 0.75 + 0.72 \times 0.8 = 0.214
\]
\item On cherche à calculer la quantité $P_B(S)$. Pour cela on utilise la formule de Bayes
\[
P_B(S) = \frac{P(B\cap S)}{P(B)} = \frac{P_S(B) \times P(S)}{P(B)} = \frac{0.2\times 0.72}{0.214} = 0.6728971962616822 \approx 0.673
\]
\end{enumerate}
\end{solution}
\begin{exercise}[subtitle={Continent plastique}]
\textit{Les quantités évoqués dans cette exercice sont générés au hasard et sont donc complètement farfelus.}
\medskip
Le \og continent de plastique\fg{} est la plus grande des plaques de déchets plastiques évoluant sur les océans. Elle occupe actuellement dans l'océan Pacifique une surface dont l'aire est évaluée à plus de $1,6$ million de km$^2$, entre Hawaï et la Californie.
En 2017, des scientifiques ont estimé qu'il y avait $7$ millions de tonnes de déchets plastiques qui était déversé chaque année dans les océans et que cette quantité augmentait de $26\n\%$ par chaque année.
On modélise l'évolution de la masse de ces déchets plastiques déversée chaque année, si rien n'est fait pour la réduire, par une suite géométrique $\left(u_n\right)$. L'arrondi au centième du terme $u_n$ représente la masse de ces déchets déversée chaque année, exprimée en million de tonnes, pour l'année $(2017 + n)$.
\medskip
\begin{enumerate}
\item Expliquer pourquoi la suite $u_n$ est géométrique?
\item Calculer $u_1$ et $u_2$.
\item Exprimer $u_n$ en fonction de $n$.
\item Au début de l'année 2017, il y avait $300$ millions de tonnes de déchets plastique. Calculer la quantité totale de déchets plastiques en 2030.
\item On souhaite déterminer en quelle année la masse totale de ces déchets plastiques aura pour la première fois augmenté de $50$\,\% par rapport à sa valeur de 2017.
\begin{enumerate}
\item Recopier et compléter l'algorithme ci-dessous pour que la variable $N$ contienne la réponse au problème posé.
\begin{center}
\begin{tabularx}{0.4\linewidth}{|X|}\hline
$N = 2017$\\
$U = 7$ \\
$S = 300 + U$ \\
while $S < 450$: \\
\hspace{1cm} $N = \ldots$\\
\hspace{1cm} $U = \ldots$\\
\hspace{1cm} $S = \ldots$\\
\hline
\end{tabularx}
\end{center}
\item Que contiennent les variables $S$, $U$ et $N$ après exécution de cet algorithme ?
Interpréter les résultats dans le contexte de l'exercice.
\end{enumerate}
\end{enumerate}
\end{exercise}
\begin{solution}
\begin{enumerate}
\item Une augmentation de $26\,\%$ revient à multiplier la quantité par $1.26$. La suite est donc bien géométrique. Son premier terme est $u_0 = 7$ et sa raison est $q = 1.26$
\item
\[
u_1 = u_0 * 1.26 = 8.82
\]
\[
u_2 = u_0 * 1.26^2 = 11.1132
\]
\item
\[
u_n = u_0 \times q^n = 7 \times 1.26^n
\]
\item On calcule la quantité totale déversée entre 2017 et 2030.
\[
\sum_{n = 0}^{13} u_n = u_0 \times \frac{1-q^{13}}{1-q} = 7 \times \frac{1 - 1.26^{13}}{1 - 1.26} = 516.25
\]
On en déduit la quantité totale de déchets en 2030
\[
300 + 516.25 = 816.25
\]
\item
\begin{enumerate}
\item ~
\begin{center}
\begin{tabularx}{0.4\linewidth}{|X|}\hline
$N \gets 2017$\\
$U \gets 7$ \\
$S \gets 300 + U$ \\
Tant que $S < 450$ \\
\hspace{1cm} $N \gets N + 1$\\
\hspace{1cm} $U \gets U * 1.26$\\
\hspace{1cm} $S \gets S + u$\\
Fin Tant que\\\hline
\end{tabularx}
\end{center}
\item \textit{Pas de correction automatisé}
\end{enumerate}
\end{enumerate}
\end{solution}
\end{document}
%%% Local Variables:
%%% mode: latex
%%% TeX-master: "master"
%%% End:

View File

@ -0,0 +1,262 @@
\documentclass[a4paper,10pt]{article}
\usepackage{myXsim}
% Title Page
\title{DS8 \hfill VIALON-DUPERRON Victorien}
\tribe{TST}
\date{\hfillÀ render pour le Mercredi 7 avril}
\xsimsetup{
solution/print = false
}
\begin{document}
\maketitle
\begin{exercise}[subtitle={Automatismes}]
\textit{Toutes les questions de cette exercice sont indépendantes et peuvent être répondus séparément}
\begin{enumerate}
\item De janvier à septembre, une quantité a augmenté de $20\,\%$. Faire un schéma pour représenter la situation puis calculer le taux d'évolution moyen mensuel.
\item Une quantité augmente de $20\,\%$ par ans. En 2020, elle est de 137\euro. Quelle était sa valeur en 2019? Faire un schéma pour représenter la situation.
\item Déterminer l'équation de la droite \\
\begin{tikzpicture}[xscale=0.8, yscale=0.5]
\tkzInit[xmin=-5,xmax=5,xstep=1,
ymin=-5,ymax=5,ystep=1]
\tkzGrid
\tkzAxeXY
\tkzFct[domain=-5:5,color=red,very thick]%
{2.6666666666666665*\x -4};
\end{tikzpicture}
\item Résoudre l'équation $10 \times 0.26^x = 42$
\end{enumerate}
\end{exercise}
\begin{solution}
\begin{enumerate}
\item On veut partager cette évolution en 8 évolutions.
\[
\left(1 + \frac{20}{100}\right)^{\frac{1}{8}} = 1.0231
\]
Donc le taux d'évolution moyen est
\[
t_m = 1.0231 - 1 = 0.0230999999999999
\]
\item Coefficient multiplicateur pour revenir en arrière
\[
CM = (1 + \frac{20}{100})^{-1} = 0.8333
\]
On en déduit la quantité en 2019
\[
137 * 0.8333 = 114.16210000000001
\]
\item L'équation de la droite est
\[
y = 2.6666666666666665 x -4
\]
\item Il faut penser à faire la division à par $10$ avant d'utiliser le log car sinon, on ne peut pas utiliser la formule $\log(a^n) = n\times \log(a)$.
\[x = \frac{\log(4.2)}{\log(0.26)}\]
\end{enumerate}
\end{solution}
\begin{exercise}[subtitle={Restaurant}]
Un \emph{food truck}, ouvert le midi et le soir, propose deux types de formules :
\setlength\parindent{10mm}
\begin{itemize}
\item la formule \emph{Burger} ;
\item la formule \emph{Wok}.
\end{itemize}
\setlength\parindent{0mm}
\medskip
Le gérant a remarqué que 87\,\% de ses ventes ont lieu le midi. Le quart des ventes du midi correspondent à la formule \emph{Burger}, alors que 47\,\% des ventes du soir correspondent à la formule \emph{Wok}.
Le gérant se constitue un fichier en notant, pour chaque vente, la formule choisie et le moment de cette vente (midi ou soir).
On prélève une fiche de façon équiprobable. On définit les quatre évènements suivants:
\begin{enumerate}
\item $M$ : \og la fiche correspond à une vente du midi\fg{} ;
\item $S$ : \og la fiche correspond à une vente du soir\fg {};
\item $W$ : \og la fiche correspond à une formule \emph{Wok} \fg{} ;
\item $B$ : \og la fiche correspond à une formule \emph{Burger} \fg.
\end{enumerate}
\setlength\parindent{0mm}
\medskip
\begin{enumerate}
\item Recopier puis compléter l'arbre pondéré
\begin{center}
\begin{tikzpicture}[sloped]
\node {.}
child {node {$M$}
child {node {$W$}
edge from parent
node[above] {...}
}
child {node {$B$}
edge from parent
node[above] {...}
}
edge from parent
node[above] {...}
}
child[missing] {}
child { node {$S$}
child {node {$W$}
edge from parent
node[above] {...}
}
child {node {$B$}
edge from parent
node[above] {...}
}
edge from parent
node[above] {...}
} ;
\end{tikzpicture}
\end{center}
\item Calculer la probabilité de l'évènement $M \cap W$. Interpréter ce résultat dans le contexte de l'exercice.
\item Montrer que la probabilité que la fiche choisie corresponde à une formule \emph{Burger} est égale à $0.2864$.
\item On a prélevé une fiche correspondant à la formule \emph{Burger}. Quelle est la probabilité, arrondie au millième, que la vente ait eu lieu le soir?
\end{enumerate}
\end{exercise}
\begin{solution}
\begin{enumerate}
\item
\begin{center}
\begin{tikzpicture}[sloped]
\node {.}
child {node {$M$}
child {node {$W$}
edge from parent
node[above] {$0.75$}
}
child {node {$B$}
edge from parent
node[above] {$0.25$}
}
edge from parent
node[above] {$0.87$}
}
child[missing] {}
child { node {$S$}
child {node {$W$}
edge from parent
node[above] {$0.47$}
}
child {node {$B$}
edge from parent
node[above] {$0.53$}
}
edge from parent
node[above] {$0.13$}
} ;
\end{tikzpicture}
\end{center}
\item On calcule la probabilité que la vente soit un wok et ait eu lieu à midi
\[ P(M\cap W) = P(M) \times P_M(W) = 0.87 \times 0.75 = 0.6525 \]
\item Probabilité que la vente soit un burger.
\[
P(B) = P(M\cap B) + P(S\cap B) = 0.87 \times 0.75 + 0.13 \times 0.47 = 0.2864
\]
\item On cherche à calculer la quantité $P_B(S)$. Pour cela on utilise la formule de Bayes
\[
P_B(S) = \frac{P(B\cap S)}{P(B)} = \frac{P_S(B) \times P(S)}{P(B)} = \frac{0.53\times 0.13}{0.2864} = 0.24057262569832405 \approx 0.241
\]
\end{enumerate}
\end{solution}
\begin{exercise}[subtitle={Continent plastique}]
\textit{Les quantités évoqués dans cette exercice sont générés au hasard et sont donc complètement farfelus.}
\medskip
Le \og continent de plastique\fg{} est la plus grande des plaques de déchets plastiques évoluant sur les océans. Elle occupe actuellement dans l'océan Pacifique une surface dont l'aire est évaluée à plus de $1,6$ million de km$^2$, entre Hawaï et la Californie.
En 2017, des scientifiques ont estimé qu'il y avait $14$ millions de tonnes de déchets plastiques qui était déversé chaque année dans les océans et que cette quantité augmentait de $17\n\%$ par chaque année.
On modélise l'évolution de la masse de ces déchets plastiques déversée chaque année, si rien n'est fait pour la réduire, par une suite géométrique $\left(u_n\right)$. L'arrondi au centième du terme $u_n$ représente la masse de ces déchets déversée chaque année, exprimée en million de tonnes, pour l'année $(2017 + n)$.
\medskip
\begin{enumerate}
\item Expliquer pourquoi la suite $u_n$ est géométrique?
\item Calculer $u_1$ et $u_2$.
\item Exprimer $u_n$ en fonction de $n$.
\item Au début de l'année 2017, il y avait $300$ millions de tonnes de déchets plastique. Calculer la quantité totale de déchets plastiques en 2030.
\item On souhaite déterminer en quelle année la masse totale de ces déchets plastiques aura pour la première fois augmenté de $50$\,\% par rapport à sa valeur de 2017.
\begin{enumerate}
\item Recopier et compléter l'algorithme ci-dessous pour que la variable $N$ contienne la réponse au problème posé.
\begin{center}
\begin{tabularx}{0.4\linewidth}{|X|}\hline
$N = 2017$\\
$U = 14$ \\
$S = 300 + U$ \\
while $S < 450$: \\
\hspace{1cm} $N = \ldots$\\
\hspace{1cm} $U = \ldots$\\
\hspace{1cm} $S = \ldots$\\
\hline
\end{tabularx}
\end{center}
\item Que contiennent les variables $S$, $U$ et $N$ après exécution de cet algorithme ?
Interpréter les résultats dans le contexte de l'exercice.
\end{enumerate}
\end{enumerate}
\end{exercise}
\begin{solution}
\begin{enumerate}
\item Une augmentation de $17\,\%$ revient à multiplier la quantité par $1.17$. La suite est donc bien géométrique. Son premier terme est $u_0 = 14$ et sa raison est $q = 1.17$
\item
\[
u_1 = u_0 * 1.17 = 16.38
\]
\[
u_2 = u_0 * 1.17^2 = 19.1646
\]
\item
\[
u_n = u_0 \times q^n = 14 \times 1.17^n
\]
\item On calcule la quantité totale déversée entre 2017 et 2030.
\[
\sum_{n = 0}^{13} u_n = u_0 \times \frac{1-q^{13}}{1-q} = 14 \times \frac{1 - 1.17^{13}}{1 - 1.17} = 551.66
\]
On en déduit la quantité totale de déchets en 2030
\[
300 + 551.66 = 851.66
\]
\item
\begin{enumerate}
\item ~
\begin{center}
\begin{tabularx}{0.4\linewidth}{|X|}\hline
$N \gets 2017$\\
$U \gets 14$ \\
$S \gets 300 + U$ \\
Tant que $S < 450$ \\
\hspace{1cm} $N \gets N + 1$\\
\hspace{1cm} $U \gets U * 1.17$\\
\hspace{1cm} $S \gets S + u$\\
Fin Tant que\\\hline
\end{tabularx}
\end{center}
\item \textit{Pas de correction automatisé}
\end{enumerate}
\end{enumerate}
\end{solution}
\end{document}
%%% Local Variables:
%%% mode: latex
%%% TeX-master: "master"
%%% End:

View File

@ -0,0 +1,262 @@
\documentclass[a4paper,10pt]{article}
\usepackage{myXsim}
% Title Page
\title{DS8 \hfill ZENAGUI Yanis}
\tribe{TST}
\date{\hfillÀ render pour le Mercredi 7 avril}
\xsimsetup{
solution/print = false
}
\begin{document}
\maketitle
\begin{exercise}[subtitle={Automatismes}]
\textit{Toutes les questions de cette exercice sont indépendantes et peuvent être répondus séparément}
\begin{enumerate}
\item De janvier à septembre, une quantité a augmenté de $29\,\%$. Faire un schéma pour représenter la situation puis calculer le taux d'évolution moyen mensuel.
\item Une quantité augmente de $29\,\%$ par ans. En 2020, elle est de 142\euro. Quelle était sa valeur en 2019? Faire un schéma pour représenter la situation.
\item Déterminer l'équation de la droite \\
\begin{tikzpicture}[xscale=0.8, yscale=0.5]
\tkzInit[xmin=-5,xmax=5,xstep=1,
ymin=-5,ymax=5,ystep=1]
\tkzGrid
\tkzAxeXY
\tkzFct[domain=-5:5,color=red,very thick]%
{2.0*\x -4};
\end{tikzpicture}
\item Résoudre l'équation $6 \times 0.41^x = 34$
\end{enumerate}
\end{exercise}
\begin{solution}
\begin{enumerate}
\item On veut partager cette évolution en 8 évolutions.
\[
\left(1 + \frac{29}{100}\right)^{\frac{1}{8}} = 1.0323
\]
Donc le taux d'évolution moyen est
\[
t_m = 1.0323 - 1 = 0.032299999999999995
\]
\item Coefficient multiplicateur pour revenir en arrière
\[
CM = (1 + \frac{29}{100})^{-1} = 0.7752
\]
On en déduit la quantité en 2019
\[
142 * 0.7752 = 110.0784
\]
\item L'équation de la droite est
\[
y = 2.0 x -4
\]
\item Il faut penser à faire la division à par $6$ avant d'utiliser le log car sinon, on ne peut pas utiliser la formule $\log(a^n) = n\times \log(a)$.
\[x = \frac{\log(5.67)}{\log(0.41)}\]
\end{enumerate}
\end{solution}
\begin{exercise}[subtitle={Restaurant}]
Un \emph{food truck}, ouvert le midi et le soir, propose deux types de formules :
\setlength\parindent{10mm}
\begin{itemize}
\item la formule \emph{Burger} ;
\item la formule \emph{Wok}.
\end{itemize}
\setlength\parindent{0mm}
\medskip
Le gérant a remarqué que 30\,\% de ses ventes ont lieu le midi. Le quart des ventes du midi correspondent à la formule \emph{Burger}, alors que 33\,\% des ventes du soir correspondent à la formule \emph{Wok}.
Le gérant se constitue un fichier en notant, pour chaque vente, la formule choisie et le moment de cette vente (midi ou soir).
On prélève une fiche de façon équiprobable. On définit les quatre évènements suivants:
\begin{enumerate}
\item $M$ : \og la fiche correspond à une vente du midi\fg{} ;
\item $S$ : \og la fiche correspond à une vente du soir\fg {};
\item $W$ : \og la fiche correspond à une formule \emph{Wok} \fg{} ;
\item $B$ : \og la fiche correspond à une formule \emph{Burger} \fg.
\end{enumerate}
\setlength\parindent{0mm}
\medskip
\begin{enumerate}
\item Recopier puis compléter l'arbre pondéré
\begin{center}
\begin{tikzpicture}[sloped]
\node {.}
child {node {$M$}
child {node {$W$}
edge from parent
node[above] {...}
}
child {node {$B$}
edge from parent
node[above] {...}
}
edge from parent
node[above] {...}
}
child[missing] {}
child { node {$S$}
child {node {$W$}
edge from parent
node[above] {...}
}
child {node {$B$}
edge from parent
node[above] {...}
}
edge from parent
node[above] {...}
} ;
\end{tikzpicture}
\end{center}
\item Calculer la probabilité de l'évènement $M \cap W$. Interpréter ce résultat dans le contexte de l'exercice.
\item Montrer que la probabilité que la fiche choisie corresponde à une formule \emph{Burger} est égale à $0.544$.
\item On a prélevé une fiche correspondant à la formule \emph{Burger}. Quelle est la probabilité, arrondie au millième, que la vente ait eu lieu le soir?
\end{enumerate}
\end{exercise}
\begin{solution}
\begin{enumerate}
\item
\begin{center}
\begin{tikzpicture}[sloped]
\node {.}
child {node {$M$}
child {node {$W$}
edge from parent
node[above] {$0.75$}
}
child {node {$B$}
edge from parent
node[above] {$0.25$}
}
edge from parent
node[above] {$0.3$}
}
child[missing] {}
child { node {$S$}
child {node {$W$}
edge from parent
node[above] {$0.33$}
}
child {node {$B$}
edge from parent
node[above] {$0.67$}
}
edge from parent
node[above] {$0.7$}
} ;
\end{tikzpicture}
\end{center}
\item On calcule la probabilité que la vente soit un wok et ait eu lieu à midi
\[ P(M\cap W) = P(M) \times P_M(W) = 0.3 \times 0.75 = 0.225 \]
\item Probabilité que la vente soit un burger.
\[
P(B) = P(M\cap B) + P(S\cap B) = 0.3 \times 0.75 + 0.7 \times 0.33 = 0.544
\]
\item On cherche à calculer la quantité $P_B(S)$. Pour cela on utilise la formule de Bayes
\[
P_B(S) = \frac{P(B\cap S)}{P(B)} = \frac{P_S(B) \times P(S)}{P(B)} = \frac{0.67\times 0.7}{0.544} = 0.8621323529411764 \approx 0.862
\]
\end{enumerate}
\end{solution}
\begin{exercise}[subtitle={Continent plastique}]
\textit{Les quantités évoqués dans cette exercice sont générés au hasard et sont donc complètement farfelus.}
\medskip
Le \og continent de plastique\fg{} est la plus grande des plaques de déchets plastiques évoluant sur les océans. Elle occupe actuellement dans l'océan Pacifique une surface dont l'aire est évaluée à plus de $1,6$ million de km$^2$, entre Hawaï et la Californie.
En 2017, des scientifiques ont estimé qu'il y avait $4$ millions de tonnes de déchets plastiques qui était déversé chaque année dans les océans et que cette quantité augmentait de $18\n\%$ par chaque année.
On modélise l'évolution de la masse de ces déchets plastiques déversée chaque année, si rien n'est fait pour la réduire, par une suite géométrique $\left(u_n\right)$. L'arrondi au centième du terme $u_n$ représente la masse de ces déchets déversée chaque année, exprimée en million de tonnes, pour l'année $(2017 + n)$.
\medskip
\begin{enumerate}
\item Expliquer pourquoi la suite $u_n$ est géométrique?
\item Calculer $u_1$ et $u_2$.
\item Exprimer $u_n$ en fonction de $n$.
\item Au début de l'année 2017, il y avait $300$ millions de tonnes de déchets plastique. Calculer la quantité totale de déchets plastiques en 2030.
\item On souhaite déterminer en quelle année la masse totale de ces déchets plastiques aura pour la première fois augmenté de $50$\,\% par rapport à sa valeur de 2017.
\begin{enumerate}
\item Recopier et compléter l'algorithme ci-dessous pour que la variable $N$ contienne la réponse au problème posé.
\begin{center}
\begin{tabularx}{0.4\linewidth}{|X|}\hline
$N = 2017$\\
$U = 4$ \\
$S = 300 + U$ \\
while $S < 450$: \\
\hspace{1cm} $N = \ldots$\\
\hspace{1cm} $U = \ldots$\\
\hspace{1cm} $S = \ldots$\\
\hline
\end{tabularx}
\end{center}
\item Que contiennent les variables $S$, $U$ et $N$ après exécution de cet algorithme ?
Interpréter les résultats dans le contexte de l'exercice.
\end{enumerate}
\end{enumerate}
\end{exercise}
\begin{solution}
\begin{enumerate}
\item Une augmentation de $18\,\%$ revient à multiplier la quantité par $1.18$. La suite est donc bien géométrique. Son premier terme est $u_0 = 4$ et sa raison est $q = 1.18$
\item
\[
u_1 = u_0 * 1.18 = 4.72
\]
\[
u_2 = u_0 * 1.18^2 = 5.5696
\]
\item
\[
u_n = u_0 \times q^n = 4 \times 1.18^n
\]
\item On calcule la quantité totale déversée entre 2017 et 2030.
\[
\sum_{n = 0}^{13} u_n = u_0 \times \frac{1-q^{13}}{1-q} = 4 \times \frac{1 - 1.18^{13}}{1 - 1.18} = 168.87
\]
On en déduit la quantité totale de déchets en 2030
\[
300 + 168.87 = 468.87
\]
\item
\begin{enumerate}
\item ~
\begin{center}
\begin{tabularx}{0.4\linewidth}{|X|}\hline
$N \gets 2017$\\
$U \gets 4$ \\
$S \gets 300 + U$ \\
Tant que $S < 450$ \\
\hspace{1cm} $N \gets N + 1$\\
\hspace{1cm} $U \gets U * 1.18$\\
\hspace{1cm} $S \gets S + u$\\
Fin Tant que\\\hline
\end{tabularx}
\end{center}
\item \textit{Pas de correction automatisé}
\end{enumerate}
\end{enumerate}
\end{solution}
\end{document}
%%% Local Variables:
%%% mode: latex
%%% TeX-master: "master"
%%% End:

View File

@ -0,0 +1,262 @@
\documentclass[a4paper,10pt]{article}
\usepackage{myXsim}
% Title Page
\title{DS8 \hfill VIALON-DUPERRON Victorien}
\tribe{TST}
\date{\hfillÀ render pour le Mercredi 7 avril}
\xsimsetup{
solution/print = false
}
\begin{document}
\maketitle
\begin{exercise}[subtitle={Automatismes}]
\textit{Toutes les questions de cette exercice sont indépendantes et peuvent être répondus séparément}
\begin{enumerate}
\item De janvier à septembre, une quantité a augmenté de $10\,\%$. Faire un schéma pour représenter la situation puis calculer le taux d'évolution moyen mensuel.
\item Une quantité augmente de $10\,\%$ par ans. En 2020, elle est de 120\euro. Quelle était sa valeur en 2019? Faire un schéma pour représenter la situation.
\item Déterminer l'équation de la droite \\
\begin{tikzpicture}[xscale=0.8, yscale=0.5]
\tkzInit[xmin=-5,xmax=5,xstep=1,
ymin=-5,ymax=5,ystep=1]
\tkzGrid
\tkzAxeXY
\tkzFct[domain=-5:5,color=red,very thick]%
{2.0*\x -3};
\end{tikzpicture}
\item Résoudre l'équation $6 \times 0.69^x = 39$
\end{enumerate}
\end{exercise}
\begin{solution}
\begin{enumerate}
\item On veut partager cette évolution en 8 évolutions.
\[
\left(1 + \frac{10}{100}\right)^{\frac{1}{8}} = 1.012
\]
Donc le taux d'évolution moyen est
\[
t_m = 1.012 - 1 = 0.01200000000000001
\]
\item Coefficient multiplicateur pour revenir en arrière
\[
CM = (1 + \frac{10}{100})^{-1} = 0.9091
\]
On en déduit la quantité en 2019
\[
120 * 0.9091 = 109.092
\]
\item L'équation de la droite est
\[
y = 2.0 x -3
\]
\item Il faut penser à faire la division à par $6$ avant d'utiliser le log car sinon, on ne peut pas utiliser la formule $\log(a^n) = n\times \log(a)$.
\[x = \frac{\log(6.5)}{\log(0.69)}\]
\end{enumerate}
\end{solution}
\begin{exercise}[subtitle={Restaurant}]
Un \emph{food truck}, ouvert le midi et le soir, propose deux types de formules :
\setlength\parindent{10mm}
\begin{itemize}
\item la formule \emph{Burger} ;
\item la formule \emph{Wok}.
\end{itemize}
\setlength\parindent{0mm}
\medskip
Le gérant a remarqué que 43\,\% de ses ventes ont lieu le midi. Le quart des ventes du midi correspondent à la formule \emph{Burger}, alors que 68\,\% des ventes du soir correspondent à la formule \emph{Wok}.
Le gérant se constitue un fichier en notant, pour chaque vente, la formule choisie et le moment de cette vente (midi ou soir).
On prélève une fiche de façon équiprobable. On définit les quatre évènements suivants:
\begin{enumerate}
\item $M$ : \og la fiche correspond à une vente du midi\fg{} ;
\item $S$ : \og la fiche correspond à une vente du soir\fg {};
\item $W$ : \og la fiche correspond à une formule \emph{Wok} \fg{} ;
\item $B$ : \og la fiche correspond à une formule \emph{Burger} \fg.
\end{enumerate}
\setlength\parindent{0mm}
\medskip
\begin{enumerate}
\item Recopier puis compléter l'arbre pondéré
\begin{center}
\begin{tikzpicture}[sloped]
\node {.}
child {node {$M$}
child {node {$W$}
edge from parent
node[above] {...}
}
child {node {$B$}
edge from parent
node[above] {...}
}
edge from parent
node[above] {...}
}
child[missing] {}
child { node {$S$}
child {node {$W$}
edge from parent
node[above] {...}
}
child {node {$B$}
edge from parent
node[above] {...}
}
edge from parent
node[above] {...}
} ;
\end{tikzpicture}
\end{center}
\item Calculer la probabilité de l'évènement $M \cap W$. Interpréter ce résultat dans le contexte de l'exercice.
\item Montrer que la probabilité que la fiche choisie corresponde à une formule \emph{Burger} est égale à $0.2899$.
\item On a prélevé une fiche correspondant à la formule \emph{Burger}. Quelle est la probabilité, arrondie au millième, que la vente ait eu lieu le soir?
\end{enumerate}
\end{exercise}
\begin{solution}
\begin{enumerate}
\item
\begin{center}
\begin{tikzpicture}[sloped]
\node {.}
child {node {$M$}
child {node {$W$}
edge from parent
node[above] {$0.75$}
}
child {node {$B$}
edge from parent
node[above] {$0.25$}
}
edge from parent
node[above] {$0.43$}
}
child[missing] {}
child { node {$S$}
child {node {$W$}
edge from parent
node[above] {$0.68$}
}
child {node {$B$}
edge from parent
node[above] {$0.32$}
}
edge from parent
node[above] {$0.57$}
} ;
\end{tikzpicture}
\end{center}
\item On calcule la probabilité que la vente soit un wok et ait eu lieu à midi
\[ P(M\cap W) = P(M) \times P_M(W) = 0.43 \times 0.75 = 0.3225 \]
\item Probabilité que la vente soit un burger.
\[
P(B) = P(M\cap B) + P(S\cap B) = 0.43 \times 0.75 + 0.57 \times 0.68 = 0.2899
\]
\item On cherche à calculer la quantité $P_B(S)$. Pour cela on utilise la formule de Bayes
\[
P_B(S) = \frac{P(B\cap S)}{P(B)} = \frac{P_S(B) \times P(S)}{P(B)} = \frac{0.32\times 0.57}{0.2899} = 0.6291824767161089 \approx 0.629
\]
\end{enumerate}
\end{solution}
\begin{exercise}[subtitle={Continent plastique}]
\textit{Les quantités évoqués dans cette exercice sont générés au hasard et sont donc complètement farfelus.}
\medskip
Le \og continent de plastique\fg{} est la plus grande des plaques de déchets plastiques évoluant sur les océans. Elle occupe actuellement dans l'océan Pacifique une surface dont l'aire est évaluée à plus de $1,6$ million de km$^2$, entre Hawaï et la Californie.
En 2017, des scientifiques ont estimé qu'il y avait $7$ millions de tonnes de déchets plastiques qui était déversé chaque année dans les océans et que cette quantité augmentait de $11\n\%$ par chaque année.
On modélise l'évolution de la masse de ces déchets plastiques déversée chaque année, si rien n'est fait pour la réduire, par une suite géométrique $\left(u_n\right)$. L'arrondi au centième du terme $u_n$ représente la masse de ces déchets déversée chaque année, exprimée en million de tonnes, pour l'année $(2017 + n)$.
\medskip
\begin{enumerate}
\item Expliquer pourquoi la suite $u_n$ est géométrique?
\item Calculer $u_1$ et $u_2$.
\item Exprimer $u_n$ en fonction de $n$.
\item Au début de l'année 2017, il y avait $300$ millions de tonnes de déchets plastique. Calculer la quantité totale de déchets plastiques en 2030.
\item On souhaite déterminer en quelle année la masse totale de ces déchets plastiques aura pour la première fois augmenté de $50$\,\% par rapport à sa valeur de 2017.
\begin{enumerate}
\item Recopier et compléter l'algorithme ci-dessous pour que la variable $N$ contienne la réponse au problème posé.
\begin{center}
\begin{tabularx}{0.4\linewidth}{|X|}\hline
$N = 2017$\\
$U = 7$ \\
$S = 300 + U$ \\
while $S < 450$: \\
\hspace{1cm} $N = \ldots$\\
\hspace{1cm} $U = \ldots$\\
\hspace{1cm} $S = \ldots$\\
\hline
\end{tabularx}
\end{center}
\item Que contiennent les variables $S$, $U$ et $N$ après exécution de cet algorithme ?
Interpréter les résultats dans le contexte de l'exercice.
\end{enumerate}
\end{enumerate}
\end{exercise}
\begin{solution}
\begin{enumerate}
\item Une augmentation de $11\,\%$ revient à multiplier la quantité par $1.11$. La suite est donc bien géométrique. Son premier terme est $u_0 = 7$ et sa raison est $q = 1.11$
\item
\[
u_1 = u_0 * 1.11 = 7.7700000000000005
\]
\[
u_2 = u_0 * 1.11^2 = 8.6247
\]
\item
\[
u_n = u_0 \times q^n = 7 \times 1.11^n
\]
\item On calcule la quantité totale déversée entre 2017 et 2030.
\[
\sum_{n = 0}^{13} u_n = u_0 \times \frac{1-q^{13}}{1-q} = 7 \times \frac{1 - 1.11^{13}}{1 - 1.11} = 183.48
\]
On en déduit la quantité totale de déchets en 2030
\[
300 + 183.48 = 483.48
\]
\item
\begin{enumerate}
\item ~
\begin{center}
\begin{tabularx}{0.4\linewidth}{|X|}\hline
$N \gets 2017$\\
$U \gets 7$ \\
$S \gets 300 + U$ \\
Tant que $S < 450$ \\
\hspace{1cm} $N \gets N + 1$\\
\hspace{1cm} $U \gets U * 1.11$\\
\hspace{1cm} $S \gets S + u$\\
Fin Tant que\\\hline
\end{tabularx}
\end{center}
\item \textit{Pas de correction automatisé}
\end{enumerate}
\end{enumerate}
\end{solution}
\end{document}
%%% Local Variables:
%%% mode: latex
%%% TeX-master: "master"
%%% End:

View File

@ -0,0 +1,262 @@
\documentclass[a4paper,10pt]{article}
\usepackage{myXsim}
% Title Page
\title{DS8 \hfill ZENAGUI Yanis}
\tribe{TST}
\date{\hfillÀ render pour le Mercredi 7 avril}
\xsimsetup{
solution/print = false
}
\begin{document}
\maketitle
\begin{exercise}[subtitle={Automatismes}]
\textit{Toutes les questions de cette exercice sont indépendantes et peuvent être répondus séparément}
\begin{enumerate}
\item De janvier à septembre, une quantité a augmenté de $23\,\%$. Faire un schéma pour représenter la situation puis calculer le taux d'évolution moyen mensuel.
\item Une quantité augmente de $23\,\%$ par ans. En 2020, elle est de 135\euro. Quelle était sa valeur en 2019? Faire un schéma pour représenter la situation.
\item Déterminer l'équation de la droite \\
\begin{tikzpicture}[xscale=0.8, yscale=0.5]
\tkzInit[xmin=-5,xmax=5,xstep=1,
ymin=-5,ymax=5,ystep=1]
\tkzGrid
\tkzAxeXY
\tkzFct[domain=-5:5,color=red,very thick]%
{0.5*\x -1};
\end{tikzpicture}
\item Résoudre l'équation $3 \times 0.55^x = 39$
\end{enumerate}
\end{exercise}
\begin{solution}
\begin{enumerate}
\item On veut partager cette évolution en 8 évolutions.
\[
\left(1 + \frac{23}{100}\right)^{\frac{1}{8}} = 1.0262
\]
Donc le taux d'évolution moyen est
\[
t_m = 1.0262 - 1 = 0.0262
\]
\item Coefficient multiplicateur pour revenir en arrière
\[
CM = (1 + \frac{23}{100})^{-1} = 0.813
\]
On en déduit la quantité en 2019
\[
135 * 0.813 = 109.755
\]
\item L'équation de la droite est
\[
y = 0.5 x -1
\]
\item Il faut penser à faire la division à par $3$ avant d'utiliser le log car sinon, on ne peut pas utiliser la formule $\log(a^n) = n\times \log(a)$.
\[x = \frac{\log(13.0)}{\log(0.55)}\]
\end{enumerate}
\end{solution}
\begin{exercise}[subtitle={Restaurant}]
Un \emph{food truck}, ouvert le midi et le soir, propose deux types de formules :
\setlength\parindent{10mm}
\begin{itemize}
\item la formule \emph{Burger} ;
\item la formule \emph{Wok}.
\end{itemize}
\setlength\parindent{0mm}
\medskip
Le gérant a remarqué que 9\,\% de ses ventes ont lieu le midi. Le quart des ventes du midi correspondent à la formule \emph{Burger}, alors que 11\,\% des ventes du soir correspondent à la formule \emph{Wok}.
Le gérant se constitue un fichier en notant, pour chaque vente, la formule choisie et le moment de cette vente (midi ou soir).
On prélève une fiche de façon équiprobable. On définit les quatre évènements suivants:
\begin{enumerate}
\item $M$ : \og la fiche correspond à une vente du midi\fg{} ;
\item $S$ : \og la fiche correspond à une vente du soir\fg {};
\item $W$ : \og la fiche correspond à une formule \emph{Wok} \fg{} ;
\item $B$ : \og la fiche correspond à une formule \emph{Burger} \fg.
\end{enumerate}
\setlength\parindent{0mm}
\medskip
\begin{enumerate}
\item Recopier puis compléter l'arbre pondéré
\begin{center}
\begin{tikzpicture}[sloped]
\node {.}
child {node {$M$}
child {node {$W$}
edge from parent
node[above] {...}
}
child {node {$B$}
edge from parent
node[above] {...}
}
edge from parent
node[above] {...}
}
child[missing] {}
child { node {$S$}
child {node {$W$}
edge from parent
node[above] {...}
}
child {node {$B$}
edge from parent
node[above] {...}
}
edge from parent
node[above] {...}
} ;
\end{tikzpicture}
\end{center}
\item Calculer la probabilité de l'évènement $M \cap W$. Interpréter ce résultat dans le contexte de l'exercice.
\item Montrer que la probabilité que la fiche choisie corresponde à une formule \emph{Burger} est égale à $0.8324$.
\item On a prélevé une fiche correspondant à la formule \emph{Burger}. Quelle est la probabilité, arrondie au millième, que la vente ait eu lieu le soir?
\end{enumerate}
\end{exercise}
\begin{solution}
\begin{enumerate}
\item
\begin{center}
\begin{tikzpicture}[sloped]
\node {.}
child {node {$M$}
child {node {$W$}
edge from parent
node[above] {$0.75$}
}
child {node {$B$}
edge from parent
node[above] {$0.25$}
}
edge from parent
node[above] {$0.09$}
}
child[missing] {}
child { node {$S$}
child {node {$W$}
edge from parent
node[above] {$0.11$}
}
child {node {$B$}
edge from parent
node[above] {$0.89$}
}
edge from parent
node[above] {$0.91$}
} ;
\end{tikzpicture}
\end{center}
\item On calcule la probabilité que la vente soit un wok et ait eu lieu à midi
\[ P(M\cap W) = P(M) \times P_M(W) = 0.09 \times 0.75 = 0.0675 \]
\item Probabilité que la vente soit un burger.
\[
P(B) = P(M\cap B) + P(S\cap B) = 0.09 \times 0.75 + 0.91 \times 0.11 = 0.8324
\]
\item On cherche à calculer la quantité $P_B(S)$. Pour cela on utilise la formule de Bayes
\[
P_B(S) = \frac{P(B\cap S)}{P(B)} = \frac{P_S(B) \times P(S)}{P(B)} = \frac{0.89\times 0.91}{0.8324} = 0.9729697260932244 \approx 0.973
\]
\end{enumerate}
\end{solution}
\begin{exercise}[subtitle={Continent plastique}]
\textit{Les quantités évoqués dans cette exercice sont générés au hasard et sont donc complètement farfelus.}
\medskip
Le \og continent de plastique\fg{} est la plus grande des plaques de déchets plastiques évoluant sur les océans. Elle occupe actuellement dans l'océan Pacifique une surface dont l'aire est évaluée à plus de $1,6$ million de km$^2$, entre Hawaï et la Californie.
En 2017, des scientifiques ont estimé qu'il y avait $4$ millions de tonnes de déchets plastiques qui était déversé chaque année dans les océans et que cette quantité augmentait de $19\n\%$ par chaque année.
On modélise l'évolution de la masse de ces déchets plastiques déversée chaque année, si rien n'est fait pour la réduire, par une suite géométrique $\left(u_n\right)$. L'arrondi au centième du terme $u_n$ représente la masse de ces déchets déversée chaque année, exprimée en million de tonnes, pour l'année $(2017 + n)$.
\medskip
\begin{enumerate}
\item Expliquer pourquoi la suite $u_n$ est géométrique?
\item Calculer $u_1$ et $u_2$.
\item Exprimer $u_n$ en fonction de $n$.
\item Au début de l'année 2017, il y avait $300$ millions de tonnes de déchets plastique. Calculer la quantité totale de déchets plastiques en 2030.
\item On souhaite déterminer en quelle année la masse totale de ces déchets plastiques aura pour la première fois augmenté de $50$\,\% par rapport à sa valeur de 2017.
\begin{enumerate}
\item Recopier et compléter l'algorithme ci-dessous pour que la variable $N$ contienne la réponse au problème posé.
\begin{center}
\begin{tabularx}{0.4\linewidth}{|X|}\hline
$N = 2017$\\
$U = 4$ \\
$S = 300 + U$ \\
while $S < 450$: \\
\hspace{1cm} $N = \ldots$\\
\hspace{1cm} $U = \ldots$\\
\hspace{1cm} $S = \ldots$\\
\hline
\end{tabularx}
\end{center}
\item Que contiennent les variables $S$, $U$ et $N$ après exécution de cet algorithme ?
Interpréter les résultats dans le contexte de l'exercice.
\end{enumerate}
\end{enumerate}
\end{exercise}
\begin{solution}
\begin{enumerate}
\item Une augmentation de $19\,\%$ revient à multiplier la quantité par $1.19$. La suite est donc bien géométrique. Son premier terme est $u_0 = 4$ et sa raison est $q = 1.19$
\item
\[
u_1 = u_0 * 1.19 = 4.76
\]
\[
u_2 = u_0 * 1.19^2 = 5.6644
\]
\item
\[
u_n = u_0 \times q^n = 4 \times 1.19^n
\]
\item On calcule la quantité totale déversée entre 2017 et 2030.
\[
\sum_{n = 0}^{13} u_n = u_0 \times \frac{1-q^{13}}{1-q} = 4 \times \frac{1 - 1.19^{13}}{1 - 1.19} = 180.98
\]
On en déduit la quantité totale de déchets en 2030
\[
300 + 180.98 = 480.98
\]
\item
\begin{enumerate}
\item ~
\begin{center}
\begin{tabularx}{0.4\linewidth}{|X|}\hline
$N \gets 2017$\\
$U \gets 4$ \\
$S \gets 300 + U$ \\
Tant que $S < 450$ \\
\hspace{1cm} $N \gets N + 1$\\
\hspace{1cm} $U \gets U * 1.19$\\
\hspace{1cm} $S \gets S + u$\\
Fin Tant que\\\hline
\end{tabularx}
\end{center}
\item \textit{Pas de correction automatisé}
\end{enumerate}
\end{enumerate}
\end{solution}
\end{document}
%%% Local Variables:
%%% mode: latex
%%% TeX-master: "master"
%%% End:

Binary file not shown.

View File

@ -0,0 +1,262 @@
\documentclass[a4paper,10pt]{article}
\usepackage{myXsim}
% Title Page
\title{DS8 \hfill BELARBI Samira}
\tribe{TST}
\date{\hfillÀ render pour le Mercredi 7 avril}
\xsimsetup{
solution/print = true
}
\begin{document}
\maketitle
\begin{exercise}[subtitle={Automatismes}]
\textit{Toutes les questions de cette exercice sont indépendantes et peuvent être répondus séparément}
\begin{enumerate}
\item De janvier à septembre, une quantité a augmenté de $21\,\%$. Faire un schéma pour représenter la situation puis calculer le taux d'évolution moyen mensuel.
\item Une quantité augmente de $21\,\%$ par ans. En 2020, elle est de 143\euro. Quelle était sa valeur en 2019? Faire un schéma pour représenter la situation.
\item Déterminer l'équation de la droite \\
\begin{tikzpicture}[xscale=0.8, yscale=0.5]
\tkzInit[xmin=-5,xmax=5,xstep=1,
ymin=-5,ymax=5,ystep=1]
\tkzGrid
\tkzAxeXY
\tkzFct[domain=-5:5,color=red,very thick]%
{2.6666666666666665*\x -4};
\end{tikzpicture}
\item Résoudre l'équation $8 \times 0.76^x = 26$
\end{enumerate}
\end{exercise}
\begin{solution}
\begin{enumerate}
\item On veut partager cette évolution en 8 évolutions.
\[
\left(1 + \frac{21}{100}\right)^{\frac{1}{8}} = 1.0241
\]
Donc le taux d'évolution moyen est
\[
t_m = 1.0241 - 1 = 0.02410000000000001
\]
\item Coefficient multiplicateur pour revenir en arrière
\[
CM = (1 + \frac{21}{100})^{-1} = 0.8264
\]
On en déduit la quantité en 2019
\[
143 * 0.8264 = 118.1752
\]
\item L'équation de la droite est
\[
y = 2.6666666666666665 x -4
\]
\item Il faut penser à faire la division à par $8$ avant d'utiliser le log car sinon, on ne peut pas utiliser la formule $\log(a^n) = n\times \log(a)$.
\[x = \frac{\log(3.25)}{\log(0.76)}\]
\end{enumerate}
\end{solution}
\begin{exercise}[subtitle={Restaurant}]
Un \emph{food truck}, ouvert le midi et le soir, propose deux types de formules :
\setlength\parindent{10mm}
\begin{itemize}
\item la formule \emph{Burger} ;
\item la formule \emph{Wok}.
\end{itemize}
\setlength\parindent{0mm}
\medskip
Le gérant a remarqué que 21\,\% de ses ventes ont lieu le midi. Le quart des ventes du midi correspondent à la formule \emph{Burger}, alors que 27\,\% des ventes du soir correspondent à la formule \emph{Wok}.
Le gérant se constitue un fichier en notant, pour chaque vente, la formule choisie et le moment de cette vente (midi ou soir).
On prélève une fiche de façon équiprobable. On définit les quatre évènements suivants:
\begin{enumerate}
\item $M$ : \og la fiche correspond à une vente du midi\fg{} ;
\item $S$ : \og la fiche correspond à une vente du soir\fg {};
\item $W$ : \og la fiche correspond à une formule \emph{Wok} \fg{} ;
\item $B$ : \og la fiche correspond à une formule \emph{Burger} \fg.
\end{enumerate}
\setlength\parindent{0mm}
\medskip
\begin{enumerate}
\item Recopier puis compléter l'arbre pondéré
\begin{center}
\begin{tikzpicture}[sloped]
\node {.}
child {node {$M$}
child {node {$W$}
edge from parent
node[above] {...}
}
child {node {$B$}
edge from parent
node[above] {...}
}
edge from parent
node[above] {...}
}
child[missing] {}
child { node {$S$}
child {node {$W$}
edge from parent
node[above] {...}
}
child {node {$B$}
edge from parent
node[above] {...}
}
edge from parent
node[above] {...}
} ;
\end{tikzpicture}
\end{center}
\item Calculer la probabilité de l'évènement $M \cap W$. Interpréter ce résultat dans le contexte de l'exercice.
\item Montrer que la probabilité que la fiche choisie corresponde à une formule \emph{Burger} est égale à $0.6292$.
\item On a prélevé une fiche correspondant à la formule \emph{Burger}. Quelle est la probabilité, arrondie au millième, que la vente ait eu lieu le soir?
\end{enumerate}
\end{exercise}
\begin{solution}
\begin{enumerate}
\item
\begin{center}
\begin{tikzpicture}[sloped]
\node {.}
child {node {$M$}
child {node {$W$}
edge from parent
node[above] {$0.75$}
}
child {node {$B$}
edge from parent
node[above] {$0.25$}
}
edge from parent
node[above] {$0.21$}
}
child[missing] {}
child { node {$S$}
child {node {$W$}
edge from parent
node[above] {$0.27$}
}
child {node {$B$}
edge from parent
node[above] {$0.73$}
}
edge from parent
node[above] {$0.79$}
} ;
\end{tikzpicture}
\end{center}
\item On calcule la probabilité que la vente soit un wok et ait eu lieu à midi
\[ P(M\cap W) = P(M) \times P_M(W) = 0.21 \times 0.75 = 0.1575 \]
\item Probabilité que la vente soit un burger.
\[
P(B) = P(M\cap B) + P(S\cap B) = 0.21 \times 0.75 + 0.79 \times 0.27 = 0.6292
\]
\item On cherche à calculer la quantité $P_B(S)$. Pour cela on utilise la formule de Bayes
\[
P_B(S) = \frac{P(B\cap S)}{P(B)} = \frac{P_S(B) \times P(S)}{P(B)} = \frac{0.73\times 0.79}{0.6292} = 0.9165607120152575 \approx 0.917
\]
\end{enumerate}
\end{solution}
\begin{exercise}[subtitle={Continent plastique}]
\textit{Les quantités évoqués dans cette exercice sont générés au hasard et sont donc complètement farfelus.}
\medskip
Le \og continent de plastique\fg{} est la plus grande des plaques de déchets plastiques évoluant sur les océans. Elle occupe actuellement dans l'océan Pacifique une surface dont l'aire est évaluée à plus de $1,6$ million de km$^2$, entre Hawaï et la Californie.
En 2017, des scientifiques ont estimé qu'il y avait $20$ millions de tonnes de déchets plastiques qui était déversé chaque année dans les océans et que cette quantité augmentait de $28\n\%$ par chaque année.
On modélise l'évolution de la masse de ces déchets plastiques déversée chaque année, si rien n'est fait pour la réduire, par une suite géométrique $\left(u_n\right)$. L'arrondi au centième du terme $u_n$ représente la masse de ces déchets déversée chaque année, exprimée en million de tonnes, pour l'année $(2017 + n)$.
\medskip
\begin{enumerate}
\item Expliquer pourquoi la suite $u_n$ est géométrique?
\item Calculer $u_1$ et $u_2$.
\item Exprimer $u_n$ en fonction de $n$.
\item Au début de l'année 2017, il y avait $300$ millions de tonnes de déchets plastique. Calculer la quantité totale de déchets plastiques en 2030.
\item On souhaite déterminer en quelle année la masse totale de ces déchets plastiques aura pour la première fois augmenté de $50$\,\% par rapport à sa valeur de 2017.
\begin{enumerate}
\item Recopier et compléter l'algorithme ci-dessous pour que la variable $N$ contienne la réponse au problème posé.
\begin{center}
\begin{tabularx}{0.4\linewidth}{|X|}\hline
$N = 2017$\\
$U = 20$ \\
$S = 300 + U$ \\
while $S < 450$: \\
\hspace{1cm} $N = \ldots$\\
\hspace{1cm} $U = \ldots$\\
\hspace{1cm} $S = \ldots$\\
\hline
\end{tabularx}
\end{center}
\item Que contiennent les variables $S$, $U$ et $N$ après exécution de cet algorithme ?
Interpréter les résultats dans le contexte de l'exercice.
\end{enumerate}
\end{enumerate}
\end{exercise}
\begin{solution}
\begin{enumerate}
\item Une augmentation de $28\,\%$ revient à multiplier la quantité par $1.28$. La suite est donc bien géométrique. Son premier terme est $u_0 = 20$ et sa raison est $q = 1.28$
\item
\[
u_1 = u_0 * 1.28 = 25.6
\]
\[
u_2 = u_0 * 1.28^2 = 32.768
\]
\item
\[
u_n = u_0 \times q^n = 20 \times 1.28^n
\]
\item On calcule la quantité totale déversée entre 2017 et 2030.
\[
\sum_{n = 0}^{13} u_n = u_0 \times \frac{1-q^{13}}{1-q} = 20 \times \frac{1 - 1.28^{13}}{1 - 1.28} = 1697.06
\]
On en déduit la quantité totale de déchets en 2030
\[
300 + 1697.06 = 1997.06
\]
\item
\begin{enumerate}
\item ~
\begin{center}
\begin{tabularx}{0.4\linewidth}{|X|}\hline
$N \gets 2017$\\
$U \gets 20$ \\
$S \gets 300 + U$ \\
Tant que $S < 450$ \\
\hspace{1cm} $N \gets N + 1$\\
\hspace{1cm} $U \gets U * 1.28$\\
\hspace{1cm} $S \gets S + u$\\
Fin Tant que\\\hline
\end{tabularx}
\end{center}
\item \textit{Pas de correction automatisé}
\end{enumerate}
\end{enumerate}
\end{solution}
\end{document}
%%% Local Variables:
%%% mode: latex
%%% TeX-master: "master"
%%% End:

View File

@ -0,0 +1,262 @@
\documentclass[a4paper,10pt]{article}
\usepackage{myXsim}
% Title Page
\title{DS8 \hfill BERTAN Ufuk}
\tribe{TST}
\date{\hfillÀ render pour le Mercredi 7 avril}
\xsimsetup{
solution/print = true
}
\begin{document}
\maketitle
\begin{exercise}[subtitle={Automatismes}]
\textit{Toutes les questions de cette exercice sont indépendantes et peuvent être répondus séparément}
\begin{enumerate}
\item De janvier à septembre, une quantité a augmenté de $22\,\%$. Faire un schéma pour représenter la situation puis calculer le taux d'évolution moyen mensuel.
\item Une quantité augmente de $22\,\%$ par ans. En 2020, elle est de 134\euro. Quelle était sa valeur en 2019? Faire un schéma pour représenter la situation.
\item Déterminer l'équation de la droite \\
\begin{tikzpicture}[xscale=0.8, yscale=0.5]
\tkzInit[xmin=-5,xmax=5,xstep=1,
ymin=-5,ymax=5,ystep=1]
\tkzGrid
\tkzAxeXY
\tkzFct[domain=-5:5,color=red,very thick]%
{2.0*\x -2};
\end{tikzpicture}
\item Résoudre l'équation $5 \times 0.99^x = 21$
\end{enumerate}
\end{exercise}
\begin{solution}
\begin{enumerate}
\item On veut partager cette évolution en 8 évolutions.
\[
\left(1 + \frac{22}{100}\right)^{\frac{1}{8}} = 1.0252
\]
Donc le taux d'évolution moyen est
\[
t_m = 1.0252 - 1 = 0.02519999999999989
\]
\item Coefficient multiplicateur pour revenir en arrière
\[
CM = (1 + \frac{22}{100})^{-1} = 0.8197
\]
On en déduit la quantité en 2019
\[
134 * 0.8197 = 109.8398
\]
\item L'équation de la droite est
\[
y = 2.0 x -2
\]
\item Il faut penser à faire la division à par $5$ avant d'utiliser le log car sinon, on ne peut pas utiliser la formule $\log(a^n) = n\times \log(a)$.
\[x = \frac{\log(4.2)}{\log(0.99)}\]
\end{enumerate}
\end{solution}
\begin{exercise}[subtitle={Restaurant}]
Un \emph{food truck}, ouvert le midi et le soir, propose deux types de formules :
\setlength\parindent{10mm}
\begin{itemize}
\item la formule \emph{Burger} ;
\item la formule \emph{Wok}.
\end{itemize}
\setlength\parindent{0mm}
\medskip
Le gérant a remarqué que 64\,\% de ses ventes ont lieu le midi. Le quart des ventes du midi correspondent à la formule \emph{Burger}, alors que 100\,\% des ventes du soir correspondent à la formule \emph{Wok}.
Le gérant se constitue un fichier en notant, pour chaque vente, la formule choisie et le moment de cette vente (midi ou soir).
On prélève une fiche de façon équiprobable. On définit les quatre évènements suivants:
\begin{enumerate}
\item $M$ : \og la fiche correspond à une vente du midi\fg{} ;
\item $S$ : \og la fiche correspond à une vente du soir\fg {};
\item $W$ : \og la fiche correspond à une formule \emph{Wok} \fg{} ;
\item $B$ : \og la fiche correspond à une formule \emph{Burger} \fg.
\end{enumerate}
\setlength\parindent{0mm}
\medskip
\begin{enumerate}
\item Recopier puis compléter l'arbre pondéré
\begin{center}
\begin{tikzpicture}[sloped]
\node {.}
child {node {$M$}
child {node {$W$}
edge from parent
node[above] {...}
}
child {node {$B$}
edge from parent
node[above] {...}
}
edge from parent
node[above] {...}
}
child[missing] {}
child { node {$S$}
child {node {$W$}
edge from parent
node[above] {...}
}
child {node {$B$}
edge from parent
node[above] {...}
}
edge from parent
node[above] {...}
} ;
\end{tikzpicture}
\end{center}
\item Calculer la probabilité de l'évènement $M \cap W$. Interpréter ce résultat dans le contexte de l'exercice.
\item Montrer que la probabilité que la fiche choisie corresponde à une formule \emph{Burger} est égale à $0.16$.
\item On a prélevé une fiche correspondant à la formule \emph{Burger}. Quelle est la probabilité, arrondie au millième, que la vente ait eu lieu le soir?
\end{enumerate}
\end{exercise}
\begin{solution}
\begin{enumerate}
\item
\begin{center}
\begin{tikzpicture}[sloped]
\node {.}
child {node {$M$}
child {node {$W$}
edge from parent
node[above] {$0.75$}
}
child {node {$B$}
edge from parent
node[above] {$0.25$}
}
edge from parent
node[above] {$0.64$}
}
child[missing] {}
child { node {$S$}
child {node {$W$}
edge from parent
node[above] {$1.0$}
}
child {node {$B$}
edge from parent
node[above] {$0.0$}
}
edge from parent
node[above] {$0.36$}
} ;
\end{tikzpicture}
\end{center}
\item On calcule la probabilité que la vente soit un wok et ait eu lieu à midi
\[ P(M\cap W) = P(M) \times P_M(W) = 0.64 \times 0.75 = 0.48 \]
\item Probabilité que la vente soit un burger.
\[
P(B) = P(M\cap B) + P(S\cap B) = 0.64 \times 0.75 + 0.36 \times 1.0 = 0.16
\]
\item On cherche à calculer la quantité $P_B(S)$. Pour cela on utilise la formule de Bayes
\[
P_B(S) = \frac{P(B\cap S)}{P(B)} = \frac{P_S(B) \times P(S)}{P(B)} = \frac{0.0\times 0.36}{0.16} = 0.0 \approx 0.0
\]
\end{enumerate}
\end{solution}
\begin{exercise}[subtitle={Continent plastique}]
\textit{Les quantités évoqués dans cette exercice sont générés au hasard et sont donc complètement farfelus.}
\medskip
Le \og continent de plastique\fg{} est la plus grande des plaques de déchets plastiques évoluant sur les océans. Elle occupe actuellement dans l'océan Pacifique une surface dont l'aire est évaluée à plus de $1,6$ million de km$^2$, entre Hawaï et la Californie.
En 2017, des scientifiques ont estimé qu'il y avait $2$ millions de tonnes de déchets plastiques qui était déversé chaque année dans les océans et que cette quantité augmentait de $27\n\%$ par chaque année.
On modélise l'évolution de la masse de ces déchets plastiques déversée chaque année, si rien n'est fait pour la réduire, par une suite géométrique $\left(u_n\right)$. L'arrondi au centième du terme $u_n$ représente la masse de ces déchets déversée chaque année, exprimée en million de tonnes, pour l'année $(2017 + n)$.
\medskip
\begin{enumerate}
\item Expliquer pourquoi la suite $u_n$ est géométrique?
\item Calculer $u_1$ et $u_2$.
\item Exprimer $u_n$ en fonction de $n$.
\item Au début de l'année 2017, il y avait $300$ millions de tonnes de déchets plastique. Calculer la quantité totale de déchets plastiques en 2030.
\item On souhaite déterminer en quelle année la masse totale de ces déchets plastiques aura pour la première fois augmenté de $50$\,\% par rapport à sa valeur de 2017.
\begin{enumerate}
\item Recopier et compléter l'algorithme ci-dessous pour que la variable $N$ contienne la réponse au problème posé.
\begin{center}
\begin{tabularx}{0.4\linewidth}{|X|}\hline
$N = 2017$\\
$U = 2$ \\
$S = 300 + U$ \\
while $S < 450$: \\
\hspace{1cm} $N = \ldots$\\
\hspace{1cm} $U = \ldots$\\
\hspace{1cm} $S = \ldots$\\
\hline
\end{tabularx}
\end{center}
\item Que contiennent les variables $S$, $U$ et $N$ après exécution de cet algorithme ?
Interpréter les résultats dans le contexte de l'exercice.
\end{enumerate}
\end{enumerate}
\end{exercise}
\begin{solution}
\begin{enumerate}
\item Une augmentation de $27\,\%$ revient à multiplier la quantité par $1.27$. La suite est donc bien géométrique. Son premier terme est $u_0 = 2$ et sa raison est $q = 1.27$
\item
\[
u_1 = u_0 * 1.27 = 2.54
\]
\[
u_2 = u_0 * 1.27^2 = 3.2258
\]
\item
\[
u_n = u_0 \times q^n = 2 \times 1.27^n
\]
\item On calcule la quantité totale déversée entre 2017 et 2030.
\[
\sum_{n = 0}^{13} u_n = u_0 \times \frac{1-q^{13}}{1-q} = 2 \times \frac{1 - 1.27^{13}}{1 - 1.27} = 158.21
\]
On en déduit la quantité totale de déchets en 2030
\[
300 + 158.21 = 458.21000000000004
\]
\item
\begin{enumerate}
\item ~
\begin{center}
\begin{tabularx}{0.4\linewidth}{|X|}\hline
$N \gets 2017$\\
$U \gets 2$ \\
$S \gets 300 + U$ \\
Tant que $S < 450$ \\
\hspace{1cm} $N \gets N + 1$\\
\hspace{1cm} $U \gets U * 1.27$\\
\hspace{1cm} $S \gets S + u$\\
Fin Tant que\\\hline
\end{tabularx}
\end{center}
\item \textit{Pas de correction automatisé}
\end{enumerate}
\end{enumerate}
\end{solution}
\end{document}
%%% Local Variables:
%%% mode: latex
%%% TeX-master: "master"
%%% End:

View File

@ -0,0 +1,262 @@
\documentclass[a4paper,10pt]{article}
\usepackage{myXsim}
% Title Page
\title{DS8 \hfill BOUALIA Bilel}
\tribe{TST}
\date{\hfillÀ render pour le Mercredi 7 avril}
\xsimsetup{
solution/print = true
}
\begin{document}
\maketitle
\begin{exercise}[subtitle={Automatismes}]
\textit{Toutes les questions de cette exercice sont indépendantes et peuvent être répondus séparément}
\begin{enumerate}
\item De janvier à septembre, une quantité a augmenté de $17\,\%$. Faire un schéma pour représenter la situation puis calculer le taux d'évolution moyen mensuel.
\item Une quantité augmente de $17\,\%$ par ans. En 2020, elle est de 135\euro. Quelle était sa valeur en 2019? Faire un schéma pour représenter la situation.
\item Déterminer l'équation de la droite \\
\begin{tikzpicture}[xscale=0.8, yscale=0.5]
\tkzInit[xmin=-5,xmax=5,xstep=1,
ymin=-5,ymax=5,ystep=1]
\tkzGrid
\tkzAxeXY
\tkzFct[domain=-5:5,color=red,very thick]%
{2.0*\x -4};
\end{tikzpicture}
\item Résoudre l'équation $3 \times 0.2^x = 2$
\end{enumerate}
\end{exercise}
\begin{solution}
\begin{enumerate}
\item On veut partager cette évolution en 8 évolutions.
\[
\left(1 + \frac{17}{100}\right)^{\frac{1}{8}} = 1.0198
\]
Donc le taux d'évolution moyen est
\[
t_m = 1.0198 - 1 = 0.01980000000000004
\]
\item Coefficient multiplicateur pour revenir en arrière
\[
CM = (1 + \frac{17}{100})^{-1} = 0.8547
\]
On en déduit la quantité en 2019
\[
135 * 0.8547 = 115.3845
\]
\item L'équation de la droite est
\[
y = 2.0 x -4
\]
\item Il faut penser à faire la division à par $3$ avant d'utiliser le log car sinon, on ne peut pas utiliser la formule $\log(a^n) = n\times \log(a)$.
\[x = \frac{\log(0.67)}{\log(0.2)}\]
\end{enumerate}
\end{solution}
\begin{exercise}[subtitle={Restaurant}]
Un \emph{food truck}, ouvert le midi et le soir, propose deux types de formules :
\setlength\parindent{10mm}
\begin{itemize}
\item la formule \emph{Burger} ;
\item la formule \emph{Wok}.
\end{itemize}
\setlength\parindent{0mm}
\medskip
Le gérant a remarqué que 89\,\% de ses ventes ont lieu le midi. Le quart des ventes du midi correspondent à la formule \emph{Burger}, alors que 12\,\% des ventes du soir correspondent à la formule \emph{Wok}.
Le gérant se constitue un fichier en notant, pour chaque vente, la formule choisie et le moment de cette vente (midi ou soir).
On prélève une fiche de façon équiprobable. On définit les quatre évènements suivants:
\begin{enumerate}
\item $M$ : \og la fiche correspond à une vente du midi\fg{} ;
\item $S$ : \og la fiche correspond à une vente du soir\fg {};
\item $W$ : \og la fiche correspond à une formule \emph{Wok} \fg{} ;
\item $B$ : \og la fiche correspond à une formule \emph{Burger} \fg.
\end{enumerate}
\setlength\parindent{0mm}
\medskip
\begin{enumerate}
\item Recopier puis compléter l'arbre pondéré
\begin{center}
\begin{tikzpicture}[sloped]
\node {.}
child {node {$M$}
child {node {$W$}
edge from parent
node[above] {...}
}
child {node {$B$}
edge from parent
node[above] {...}
}
edge from parent
node[above] {...}
}
child[missing] {}
child { node {$S$}
child {node {$W$}
edge from parent
node[above] {...}
}
child {node {$B$}
edge from parent
node[above] {...}
}
edge from parent
node[above] {...}
} ;
\end{tikzpicture}
\end{center}
\item Calculer la probabilité de l'évènement $M \cap W$. Interpréter ce résultat dans le contexte de l'exercice.
\item Montrer que la probabilité que la fiche choisie corresponde à une formule \emph{Burger} est égale à $0.3193$.
\item On a prélevé une fiche correspondant à la formule \emph{Burger}. Quelle est la probabilité, arrondie au millième, que la vente ait eu lieu le soir?
\end{enumerate}
\end{exercise}
\begin{solution}
\begin{enumerate}
\item
\begin{center}
\begin{tikzpicture}[sloped]
\node {.}
child {node {$M$}
child {node {$W$}
edge from parent
node[above] {$0.75$}
}
child {node {$B$}
edge from parent
node[above] {$0.25$}
}
edge from parent
node[above] {$0.89$}
}
child[missing] {}
child { node {$S$}
child {node {$W$}
edge from parent
node[above] {$0.12$}
}
child {node {$B$}
edge from parent
node[above] {$0.88$}
}
edge from parent
node[above] {$0.11$}
} ;
\end{tikzpicture}
\end{center}
\item On calcule la probabilité que la vente soit un wok et ait eu lieu à midi
\[ P(M\cap W) = P(M) \times P_M(W) = 0.89 \times 0.75 = 0.6675 \]
\item Probabilité que la vente soit un burger.
\[
P(B) = P(M\cap B) + P(S\cap B) = 0.89 \times 0.75 + 0.11 \times 0.12 = 0.3193
\]
\item On cherche à calculer la quantité $P_B(S)$. Pour cela on utilise la formule de Bayes
\[
P_B(S) = \frac{P(B\cap S)}{P(B)} = \frac{P_S(B) \times P(S)}{P(B)} = \frac{0.88\times 0.11}{0.3193} = 0.303163169433135 \approx 0.303
\]
\end{enumerate}
\end{solution}
\begin{exercise}[subtitle={Continent plastique}]
\textit{Les quantités évoqués dans cette exercice sont générés au hasard et sont donc complètement farfelus.}
\medskip
Le \og continent de plastique\fg{} est la plus grande des plaques de déchets plastiques évoluant sur les océans. Elle occupe actuellement dans l'océan Pacifique une surface dont l'aire est évaluée à plus de $1,6$ million de km$^2$, entre Hawaï et la Californie.
En 2017, des scientifiques ont estimé qu'il y avait $10$ millions de tonnes de déchets plastiques qui était déversé chaque année dans les océans et que cette quantité augmentait de $27\n\%$ par chaque année.
On modélise l'évolution de la masse de ces déchets plastiques déversée chaque année, si rien n'est fait pour la réduire, par une suite géométrique $\left(u_n\right)$. L'arrondi au centième du terme $u_n$ représente la masse de ces déchets déversée chaque année, exprimée en million de tonnes, pour l'année $(2017 + n)$.
\medskip
\begin{enumerate}
\item Expliquer pourquoi la suite $u_n$ est géométrique?
\item Calculer $u_1$ et $u_2$.
\item Exprimer $u_n$ en fonction de $n$.
\item Au début de l'année 2017, il y avait $300$ millions de tonnes de déchets plastique. Calculer la quantité totale de déchets plastiques en 2030.
\item On souhaite déterminer en quelle année la masse totale de ces déchets plastiques aura pour la première fois augmenté de $50$\,\% par rapport à sa valeur de 2017.
\begin{enumerate}
\item Recopier et compléter l'algorithme ci-dessous pour que la variable $N$ contienne la réponse au problème posé.
\begin{center}
\begin{tabularx}{0.4\linewidth}{|X|}\hline
$N = 2017$\\
$U = 10$ \\
$S = 300 + U$ \\
while $S < 450$: \\
\hspace{1cm} $N = \ldots$\\
\hspace{1cm} $U = \ldots$\\
\hspace{1cm} $S = \ldots$\\
\hline
\end{tabularx}
\end{center}
\item Que contiennent les variables $S$, $U$ et $N$ après exécution de cet algorithme ?
Interpréter les résultats dans le contexte de l'exercice.
\end{enumerate}
\end{enumerate}
\end{exercise}
\begin{solution}
\begin{enumerate}
\item Une augmentation de $27\,\%$ revient à multiplier la quantité par $1.27$. La suite est donc bien géométrique. Son premier terme est $u_0 = 10$ et sa raison est $q = 1.27$
\item
\[
u_1 = u_0 * 1.27 = 12.7
\]
\[
u_2 = u_0 * 1.27^2 = 16.129
\]
\item
\[
u_n = u_0 \times q^n = 10 \times 1.27^n
\]
\item On calcule la quantité totale déversée entre 2017 et 2030.
\[
\sum_{n = 0}^{13} u_n = u_0 \times \frac{1-q^{13}}{1-q} = 10 \times \frac{1 - 1.27^{13}}{1 - 1.27} = 791.07
\]
On en déduit la quantité totale de déchets en 2030
\[
300 + 791.07 = 1091.0700000000002
\]
\item
\begin{enumerate}
\item ~
\begin{center}
\begin{tabularx}{0.4\linewidth}{|X|}\hline
$N \gets 2017$\\
$U \gets 10$ \\
$S \gets 300 + U$ \\
Tant que $S < 450$ \\
\hspace{1cm} $N \gets N + 1$\\
\hspace{1cm} $U \gets U * 1.27$\\
\hspace{1cm} $S \gets S + u$\\
Fin Tant que\\\hline
\end{tabularx}
\end{center}
\item \textit{Pas de correction automatisé}
\end{enumerate}
\end{enumerate}
\end{solution}
\end{document}
%%% Local Variables:
%%% mode: latex
%%% TeX-master: "master"
%%% End:

View File

@ -0,0 +1,262 @@
\documentclass[a4paper,10pt]{article}
\usepackage{myXsim}
% Title Page
\title{DS8 \hfill BOUCHOUX Kevin}
\tribe{TST}
\date{\hfillÀ render pour le Mercredi 7 avril}
\xsimsetup{
solution/print = true
}
\begin{document}
\maketitle
\begin{exercise}[subtitle={Automatismes}]
\textit{Toutes les questions de cette exercice sont indépendantes et peuvent être répondus séparément}
\begin{enumerate}
\item De janvier à septembre, une quantité a augmenté de $10\,\%$. Faire un schéma pour représenter la situation puis calculer le taux d'évolution moyen mensuel.
\item Une quantité augmente de $10\,\%$ par ans. En 2020, elle est de 118\euro. Quelle était sa valeur en 2019? Faire un schéma pour représenter la situation.
\item Déterminer l'équation de la droite \\
\begin{tikzpicture}[xscale=0.8, yscale=0.5]
\tkzInit[xmin=-5,xmax=5,xstep=1,
ymin=-5,ymax=5,ystep=1]
\tkzGrid
\tkzAxeXY
\tkzFct[domain=-5:5,color=red,very thick]%
{3.0*\x -3};
\end{tikzpicture}
\item Résoudre l'équation $2 \times 0.93^x = 44$
\end{enumerate}
\end{exercise}
\begin{solution}
\begin{enumerate}
\item On veut partager cette évolution en 8 évolutions.
\[
\left(1 + \frac{10}{100}\right)^{\frac{1}{8}} = 1.012
\]
Donc le taux d'évolution moyen est
\[
t_m = 1.012 - 1 = 0.01200000000000001
\]
\item Coefficient multiplicateur pour revenir en arrière
\[
CM = (1 + \frac{10}{100})^{-1} = 0.9091
\]
On en déduit la quantité en 2019
\[
118 * 0.9091 = 107.27380000000001
\]
\item L'équation de la droite est
\[
y = 3.0 x -3
\]
\item Il faut penser à faire la division à par $2$ avant d'utiliser le log car sinon, on ne peut pas utiliser la formule $\log(a^n) = n\times \log(a)$.
\[x = \frac{\log(22.0)}{\log(0.93)}\]
\end{enumerate}
\end{solution}
\begin{exercise}[subtitle={Restaurant}]
Un \emph{food truck}, ouvert le midi et le soir, propose deux types de formules :
\setlength\parindent{10mm}
\begin{itemize}
\item la formule \emph{Burger} ;
\item la formule \emph{Wok}.
\end{itemize}
\setlength\parindent{0mm}
\medskip
Le gérant a remarqué que 83\,\% de ses ventes ont lieu le midi. Le quart des ventes du midi correspondent à la formule \emph{Burger}, alors que 26\,\% des ventes du soir correspondent à la formule \emph{Wok}.
Le gérant se constitue un fichier en notant, pour chaque vente, la formule choisie et le moment de cette vente (midi ou soir).
On prélève une fiche de façon équiprobable. On définit les quatre évènements suivants:
\begin{enumerate}
\item $M$ : \og la fiche correspond à une vente du midi\fg{} ;
\item $S$ : \og la fiche correspond à une vente du soir\fg {};
\item $W$ : \og la fiche correspond à une formule \emph{Wok} \fg{} ;
\item $B$ : \og la fiche correspond à une formule \emph{Burger} \fg.
\end{enumerate}
\setlength\parindent{0mm}
\medskip
\begin{enumerate}
\item Recopier puis compléter l'arbre pondéré
\begin{center}
\begin{tikzpicture}[sloped]
\node {.}
child {node {$M$}
child {node {$W$}
edge from parent
node[above] {...}
}
child {node {$B$}
edge from parent
node[above] {...}
}
edge from parent
node[above] {...}
}
child[missing] {}
child { node {$S$}
child {node {$W$}
edge from parent
node[above] {...}
}
child {node {$B$}
edge from parent
node[above] {...}
}
edge from parent
node[above] {...}
} ;
\end{tikzpicture}
\end{center}
\item Calculer la probabilité de l'évènement $M \cap W$. Interpréter ce résultat dans le contexte de l'exercice.
\item Montrer que la probabilité que la fiche choisie corresponde à une formule \emph{Burger} est égale à $0.3333$.
\item On a prélevé une fiche correspondant à la formule \emph{Burger}. Quelle est la probabilité, arrondie au millième, que la vente ait eu lieu le soir?
\end{enumerate}
\end{exercise}
\begin{solution}
\begin{enumerate}
\item
\begin{center}
\begin{tikzpicture}[sloped]
\node {.}
child {node {$M$}
child {node {$W$}
edge from parent
node[above] {$0.75$}
}
child {node {$B$}
edge from parent
node[above] {$0.25$}
}
edge from parent
node[above] {$0.83$}
}
child[missing] {}
child { node {$S$}
child {node {$W$}
edge from parent
node[above] {$0.26$}
}
child {node {$B$}
edge from parent
node[above] {$0.74$}
}
edge from parent
node[above] {$0.17$}
} ;
\end{tikzpicture}
\end{center}
\item On calcule la probabilité que la vente soit un wok et ait eu lieu à midi
\[ P(M\cap W) = P(M) \times P_M(W) = 0.83 \times 0.75 = 0.6225 \]
\item Probabilité que la vente soit un burger.
\[
P(B) = P(M\cap B) + P(S\cap B) = 0.83 \times 0.75 + 0.17 \times 0.26 = 0.3333
\]
\item On cherche à calculer la quantité $P_B(S)$. Pour cela on utilise la formule de Bayes
\[
P_B(S) = \frac{P(B\cap S)}{P(B)} = \frac{P_S(B) \times P(S)}{P(B)} = \frac{0.74\times 0.17}{0.3333} = 0.37743774377437744 \approx 0.377
\]
\end{enumerate}
\end{solution}
\begin{exercise}[subtitle={Continent plastique}]
\textit{Les quantités évoqués dans cette exercice sont générés au hasard et sont donc complètement farfelus.}
\medskip
Le \og continent de plastique\fg{} est la plus grande des plaques de déchets plastiques évoluant sur les océans. Elle occupe actuellement dans l'océan Pacifique une surface dont l'aire est évaluée à plus de $1,6$ million de km$^2$, entre Hawaï et la Californie.
En 2017, des scientifiques ont estimé qu'il y avait $18$ millions de tonnes de déchets plastiques qui était déversé chaque année dans les océans et que cette quantité augmentait de $14\n\%$ par chaque année.
On modélise l'évolution de la masse de ces déchets plastiques déversée chaque année, si rien n'est fait pour la réduire, par une suite géométrique $\left(u_n\right)$. L'arrondi au centième du terme $u_n$ représente la masse de ces déchets déversée chaque année, exprimée en million de tonnes, pour l'année $(2017 + n)$.
\medskip
\begin{enumerate}
\item Expliquer pourquoi la suite $u_n$ est géométrique?
\item Calculer $u_1$ et $u_2$.
\item Exprimer $u_n$ en fonction de $n$.
\item Au début de l'année 2017, il y avait $300$ millions de tonnes de déchets plastique. Calculer la quantité totale de déchets plastiques en 2030.
\item On souhaite déterminer en quelle année la masse totale de ces déchets plastiques aura pour la première fois augmenté de $50$\,\% par rapport à sa valeur de 2017.
\begin{enumerate}
\item Recopier et compléter l'algorithme ci-dessous pour que la variable $N$ contienne la réponse au problème posé.
\begin{center}
\begin{tabularx}{0.4\linewidth}{|X|}\hline
$N = 2017$\\
$U = 18$ \\
$S = 300 + U$ \\
while $S < 450$: \\
\hspace{1cm} $N = \ldots$\\
\hspace{1cm} $U = \ldots$\\
\hspace{1cm} $S = \ldots$\\
\hline
\end{tabularx}
\end{center}
\item Que contiennent les variables $S$, $U$ et $N$ après exécution de cet algorithme ?
Interpréter les résultats dans le contexte de l'exercice.
\end{enumerate}
\end{enumerate}
\end{exercise}
\begin{solution}
\begin{enumerate}
\item Une augmentation de $14\,\%$ revient à multiplier la quantité par $1.1400000000000001$. La suite est donc bien géométrique. Son premier terme est $u_0 = 18$ et sa raison est $q = 1.1400000000000001$
\item
\[
u_1 = u_0 * 1.1400000000000001 = 20.520000000000003
\]
\[
u_2 = u_0 * 1.1400000000000001^2 = 23.3928
\]
\item
\[
u_n = u_0 \times q^n = 18 \times 1.1400000000000001^n
\]
\item On calcule la quantité totale déversée entre 2017 et 2030.
\[
\sum_{n = 0}^{13} u_n = u_0 \times \frac{1-q^{13}}{1-q} = 18 \times \frac{1 - 1.1400000000000001^{13}}{1 - 1.1400000000000001} = 577.6
\]
On en déduit la quantité totale de déchets en 2030
\[
300 + 577.6 = 877.6
\]
\item
\begin{enumerate}
\item ~
\begin{center}
\begin{tabularx}{0.4\linewidth}{|X|}\hline
$N \gets 2017$\\
$U \gets 18$ \\
$S \gets 300 + U$ \\
Tant que $S < 450$ \\
\hspace{1cm} $N \gets N + 1$\\
\hspace{1cm} $U \gets U * 1.1400000000000001$\\
\hspace{1cm} $S \gets S + u$\\
Fin Tant que\\\hline
\end{tabularx}
\end{center}
\item \textit{Pas de correction automatisé}
\end{enumerate}
\end{enumerate}
\end{solution}
\end{document}
%%% Local Variables:
%%% mode: latex
%%% TeX-master: "master"
%%% End:

View File

@ -0,0 +1,262 @@
\documentclass[a4paper,10pt]{article}
\usepackage{myXsim}
% Title Page
\title{DS8 \hfill BUDIN Nathan}
\tribe{TST}
\date{\hfillÀ render pour le Mercredi 7 avril}
\xsimsetup{
solution/print = true
}
\begin{document}
\maketitle
\begin{exercise}[subtitle={Automatismes}]
\textit{Toutes les questions de cette exercice sont indépendantes et peuvent être répondus séparément}
\begin{enumerate}
\item De janvier à septembre, une quantité a augmenté de $16\,\%$. Faire un schéma pour représenter la situation puis calculer le taux d'évolution moyen mensuel.
\item Une quantité augmente de $16\,\%$ par ans. En 2020, elle est de 112\euro. Quelle était sa valeur en 2019? Faire un schéma pour représenter la situation.
\item Déterminer l'équation de la droite \\
\begin{tikzpicture}[xscale=0.8, yscale=0.5]
\tkzInit[xmin=-5,xmax=5,xstep=1,
ymin=-5,ymax=5,ystep=1]
\tkzGrid
\tkzAxeXY
\tkzFct[domain=-5:5,color=red,very thick]%
{1.0*\x -2};
\end{tikzpicture}
\item Résoudre l'équation $7 \times 0.22^x = 49$
\end{enumerate}
\end{exercise}
\begin{solution}
\begin{enumerate}
\item On veut partager cette évolution en 8 évolutions.
\[
\left(1 + \frac{16}{100}\right)^{\frac{1}{8}} = 1.0187
\]
Donc le taux d'évolution moyen est
\[
t_m = 1.0187 - 1 = 0.01869999999999994
\]
\item Coefficient multiplicateur pour revenir en arrière
\[
CM = (1 + \frac{16}{100})^{-1} = 0.8621
\]
On en déduit la quantité en 2019
\[
112 * 0.8621 = 96.5552
\]
\item L'équation de la droite est
\[
y = 1.0 x -2
\]
\item Il faut penser à faire la division à par $7$ avant d'utiliser le log car sinon, on ne peut pas utiliser la formule $\log(a^n) = n\times \log(a)$.
\[x = \frac{\log(7.0)}{\log(0.22)}\]
\end{enumerate}
\end{solution}
\begin{exercise}[subtitle={Restaurant}]
Un \emph{food truck}, ouvert le midi et le soir, propose deux types de formules :
\setlength\parindent{10mm}
\begin{itemize}
\item la formule \emph{Burger} ;
\item la formule \emph{Wok}.
\end{itemize}
\setlength\parindent{0mm}
\medskip
Le gérant a remarqué que 75\,\% de ses ventes ont lieu le midi. Le quart des ventes du midi correspondent à la formule \emph{Burger}, alors que 71\,\% des ventes du soir correspondent à la formule \emph{Wok}.
Le gérant se constitue un fichier en notant, pour chaque vente, la formule choisie et le moment de cette vente (midi ou soir).
On prélève une fiche de façon équiprobable. On définit les quatre évènements suivants:
\begin{enumerate}
\item $M$ : \og la fiche correspond à une vente du midi\fg{} ;
\item $S$ : \og la fiche correspond à une vente du soir\fg {};
\item $W$ : \og la fiche correspond à une formule \emph{Wok} \fg{} ;
\item $B$ : \og la fiche correspond à une formule \emph{Burger} \fg.
\end{enumerate}
\setlength\parindent{0mm}
\medskip
\begin{enumerate}
\item Recopier puis compléter l'arbre pondéré
\begin{center}
\begin{tikzpicture}[sloped]
\node {.}
child {node {$M$}
child {node {$W$}
edge from parent
node[above] {...}
}
child {node {$B$}
edge from parent
node[above] {...}
}
edge from parent
node[above] {...}
}
child[missing] {}
child { node {$S$}
child {node {$W$}
edge from parent
node[above] {...}
}
child {node {$B$}
edge from parent
node[above] {...}
}
edge from parent
node[above] {...}
} ;
\end{tikzpicture}
\end{center}
\item Calculer la probabilité de l'évènement $M \cap W$. Interpréter ce résultat dans le contexte de l'exercice.
\item Montrer que la probabilité que la fiche choisie corresponde à une formule \emph{Burger} est égale à $0.26$.
\item On a prélevé une fiche correspondant à la formule \emph{Burger}. Quelle est la probabilité, arrondie au millième, que la vente ait eu lieu le soir?
\end{enumerate}
\end{exercise}
\begin{solution}
\begin{enumerate}
\item
\begin{center}
\begin{tikzpicture}[sloped]
\node {.}
child {node {$M$}
child {node {$W$}
edge from parent
node[above] {$0.75$}
}
child {node {$B$}
edge from parent
node[above] {$0.25$}
}
edge from parent
node[above] {$0.75$}
}
child[missing] {}
child { node {$S$}
child {node {$W$}
edge from parent
node[above] {$0.71$}
}
child {node {$B$}
edge from parent
node[above] {$0.29$}
}
edge from parent
node[above] {$0.25$}
} ;
\end{tikzpicture}
\end{center}
\item On calcule la probabilité que la vente soit un wok et ait eu lieu à midi
\[ P(M\cap W) = P(M) \times P_M(W) = 0.75 \times 0.75 = 0.5625 \]
\item Probabilité que la vente soit un burger.
\[
P(B) = P(M\cap B) + P(S\cap B) = 0.75 \times 0.75 + 0.25 \times 0.71 = 0.26
\]
\item On cherche à calculer la quantité $P_B(S)$. Pour cela on utilise la formule de Bayes
\[
P_B(S) = \frac{P(B\cap S)}{P(B)} = \frac{P_S(B) \times P(S)}{P(B)} = \frac{0.29\times 0.25}{0.26} = 0.2788461538461538 \approx 0.279
\]
\end{enumerate}
\end{solution}
\begin{exercise}[subtitle={Continent plastique}]
\textit{Les quantités évoqués dans cette exercice sont générés au hasard et sont donc complètement farfelus.}
\medskip
Le \og continent de plastique\fg{} est la plus grande des plaques de déchets plastiques évoluant sur les océans. Elle occupe actuellement dans l'océan Pacifique une surface dont l'aire est évaluée à plus de $1,6$ million de km$^2$, entre Hawaï et la Californie.
En 2017, des scientifiques ont estimé qu'il y avait $12$ millions de tonnes de déchets plastiques qui était déversé chaque année dans les océans et que cette quantité augmentait de $25\n\%$ par chaque année.
On modélise l'évolution de la masse de ces déchets plastiques déversée chaque année, si rien n'est fait pour la réduire, par une suite géométrique $\left(u_n\right)$. L'arrondi au centième du terme $u_n$ représente la masse de ces déchets déversée chaque année, exprimée en million de tonnes, pour l'année $(2017 + n)$.
\medskip
\begin{enumerate}
\item Expliquer pourquoi la suite $u_n$ est géométrique?
\item Calculer $u_1$ et $u_2$.
\item Exprimer $u_n$ en fonction de $n$.
\item Au début de l'année 2017, il y avait $300$ millions de tonnes de déchets plastique. Calculer la quantité totale de déchets plastiques en 2030.
\item On souhaite déterminer en quelle année la masse totale de ces déchets plastiques aura pour la première fois augmenté de $50$\,\% par rapport à sa valeur de 2017.
\begin{enumerate}
\item Recopier et compléter l'algorithme ci-dessous pour que la variable $N$ contienne la réponse au problème posé.
\begin{center}
\begin{tabularx}{0.4\linewidth}{|X|}\hline
$N = 2017$\\
$U = 12$ \\
$S = 300 + U$ \\
while $S < 450$: \\
\hspace{1cm} $N = \ldots$\\
\hspace{1cm} $U = \ldots$\\
\hspace{1cm} $S = \ldots$\\
\hline
\end{tabularx}
\end{center}
\item Que contiennent les variables $S$, $U$ et $N$ après exécution de cet algorithme ?
Interpréter les résultats dans le contexte de l'exercice.
\end{enumerate}
\end{enumerate}
\end{exercise}
\begin{solution}
\begin{enumerate}
\item Une augmentation de $25\,\%$ revient à multiplier la quantité par $1.25$. La suite est donc bien géométrique. Son premier terme est $u_0 = 12$ et sa raison est $q = 1.25$
\item
\[
u_1 = u_0 * 1.25 = 15.0
\]
\[
u_2 = u_0 * 1.25^2 = 18.75
\]
\item
\[
u_n = u_0 \times q^n = 12 \times 1.25^n
\]
\item On calcule la quantité totale déversée entre 2017 et 2030.
\[
\sum_{n = 0}^{13} u_n = u_0 \times \frac{1-q^{13}}{1-q} = 12 \times \frac{1 - 1.25^{13}}{1 - 1.25} = 825.11
\]
On en déduit la quantité totale de déchets en 2030
\[
300 + 825.11 = 1125.1100000000001
\]
\item
\begin{enumerate}
\item ~
\begin{center}
\begin{tabularx}{0.4\linewidth}{|X|}\hline
$N \gets 2017$\\
$U \gets 12$ \\
$S \gets 300 + U$ \\
Tant que $S < 450$ \\
\hspace{1cm} $N \gets N + 1$\\
\hspace{1cm} $U \gets U * 1.25$\\
\hspace{1cm} $S \gets S + u$\\
Fin Tant que\\\hline
\end{tabularx}
\end{center}
\item \textit{Pas de correction automatisé}
\end{enumerate}
\end{enumerate}
\end{solution}
\end{document}
%%% Local Variables:
%%% mode: latex
%%% TeX-master: "master"
%%% End:

View File

@ -0,0 +1,262 @@
\documentclass[a4paper,10pt]{article}
\usepackage{myXsim}
% Title Page
\title{DS8 \hfill DARICHE Kaïs}
\tribe{TST}
\date{\hfillÀ render pour le Mercredi 7 avril}
\xsimsetup{
solution/print = true
}
\begin{document}
\maketitle
\begin{exercise}[subtitle={Automatismes}]
\textit{Toutes les questions de cette exercice sont indépendantes et peuvent être répondus séparément}
\begin{enumerate}
\item De janvier à septembre, une quantité a augmenté de $27\,\%$. Faire un schéma pour représenter la situation puis calculer le taux d'évolution moyen mensuel.
\item Une quantité augmente de $27\,\%$ par ans. En 2020, elle est de 143\euro. Quelle était sa valeur en 2019? Faire un schéma pour représenter la situation.
\item Déterminer l'équation de la droite \\
\begin{tikzpicture}[xscale=0.8, yscale=0.5]
\tkzInit[xmin=-5,xmax=5,xstep=1,
ymin=-5,ymax=5,ystep=1]
\tkzGrid
\tkzAxeXY
\tkzFct[domain=-5:5,color=red,very thick]%
{3.0*\x -3};
\end{tikzpicture}
\item Résoudre l'équation $7 \times 0.19^x = 21$
\end{enumerate}
\end{exercise}
\begin{solution}
\begin{enumerate}
\item On veut partager cette évolution en 8 évolutions.
\[
\left(1 + \frac{27}{100}\right)^{\frac{1}{8}} = 1.0303
\]
Donc le taux d'évolution moyen est
\[
t_m = 1.0303 - 1 = 0.030299999999999994
\]
\item Coefficient multiplicateur pour revenir en arrière
\[
CM = (1 + \frac{27}{100})^{-1} = 0.7874
\]
On en déduit la quantité en 2019
\[
143 * 0.7874 = 112.59819999999999
\]
\item L'équation de la droite est
\[
y = 3.0 x -3
\]
\item Il faut penser à faire la division à par $7$ avant d'utiliser le log car sinon, on ne peut pas utiliser la formule $\log(a^n) = n\times \log(a)$.
\[x = \frac{\log(3.0)}{\log(0.19)}\]
\end{enumerate}
\end{solution}
\begin{exercise}[subtitle={Restaurant}]
Un \emph{food truck}, ouvert le midi et le soir, propose deux types de formules :
\setlength\parindent{10mm}
\begin{itemize}
\item la formule \emph{Burger} ;
\item la formule \emph{Wok}.
\end{itemize}
\setlength\parindent{0mm}
\medskip
Le gérant a remarqué que 15\,\% de ses ventes ont lieu le midi. Le quart des ventes du midi correspondent à la formule \emph{Burger}, alors que 13\,\% des ventes du soir correspondent à la formule \emph{Wok}.
Le gérant se constitue un fichier en notant, pour chaque vente, la formule choisie et le moment de cette vente (midi ou soir).
On prélève une fiche de façon équiprobable. On définit les quatre évènements suivants:
\begin{enumerate}
\item $M$ : \og la fiche correspond à une vente du midi\fg{} ;
\item $S$ : \og la fiche correspond à une vente du soir\fg {};
\item $W$ : \og la fiche correspond à une formule \emph{Wok} \fg{} ;
\item $B$ : \og la fiche correspond à une formule \emph{Burger} \fg.
\end{enumerate}
\setlength\parindent{0mm}
\medskip
\begin{enumerate}
\item Recopier puis compléter l'arbre pondéré
\begin{center}
\begin{tikzpicture}[sloped]
\node {.}
child {node {$M$}
child {node {$W$}
edge from parent
node[above] {...}
}
child {node {$B$}
edge from parent
node[above] {...}
}
edge from parent
node[above] {...}
}
child[missing] {}
child { node {$S$}
child {node {$W$}
edge from parent
node[above] {...}
}
child {node {$B$}
edge from parent
node[above] {...}
}
edge from parent
node[above] {...}
} ;
\end{tikzpicture}
\end{center}
\item Calculer la probabilité de l'évènement $M \cap W$. Interpréter ce résultat dans le contexte de l'exercice.
\item Montrer que la probabilité que la fiche choisie corresponde à une formule \emph{Burger} est égale à $0.777$.
\item On a prélevé une fiche correspondant à la formule \emph{Burger}. Quelle est la probabilité, arrondie au millième, que la vente ait eu lieu le soir?
\end{enumerate}
\end{exercise}
\begin{solution}
\begin{enumerate}
\item
\begin{center}
\begin{tikzpicture}[sloped]
\node {.}
child {node {$M$}
child {node {$W$}
edge from parent
node[above] {$0.75$}
}
child {node {$B$}
edge from parent
node[above] {$0.25$}
}
edge from parent
node[above] {$0.15$}
}
child[missing] {}
child { node {$S$}
child {node {$W$}
edge from parent
node[above] {$0.13$}
}
child {node {$B$}
edge from parent
node[above] {$0.87$}
}
edge from parent
node[above] {$0.85$}
} ;
\end{tikzpicture}
\end{center}
\item On calcule la probabilité que la vente soit un wok et ait eu lieu à midi
\[ P(M\cap W) = P(M) \times P_M(W) = 0.15 \times 0.75 = 0.1125 \]
\item Probabilité que la vente soit un burger.
\[
P(B) = P(M\cap B) + P(S\cap B) = 0.15 \times 0.75 + 0.85 \times 0.13 = 0.777
\]
\item On cherche à calculer la quantité $P_B(S)$. Pour cela on utilise la formule de Bayes
\[
P_B(S) = \frac{P(B\cap S)}{P(B)} = \frac{P_S(B) \times P(S)}{P(B)} = \frac{0.87\times 0.85}{0.777} = 0.9517374517374516 \approx 0.952
\]
\end{enumerate}
\end{solution}
\begin{exercise}[subtitle={Continent plastique}]
\textit{Les quantités évoqués dans cette exercice sont générés au hasard et sont donc complètement farfelus.}
\medskip
Le \og continent de plastique\fg{} est la plus grande des plaques de déchets plastiques évoluant sur les océans. Elle occupe actuellement dans l'océan Pacifique une surface dont l'aire est évaluée à plus de $1,6$ million de km$^2$, entre Hawaï et la Californie.
En 2017, des scientifiques ont estimé qu'il y avait $2$ millions de tonnes de déchets plastiques qui était déversé chaque année dans les océans et que cette quantité augmentait de $30\n\%$ par chaque année.
On modélise l'évolution de la masse de ces déchets plastiques déversée chaque année, si rien n'est fait pour la réduire, par une suite géométrique $\left(u_n\right)$. L'arrondi au centième du terme $u_n$ représente la masse de ces déchets déversée chaque année, exprimée en million de tonnes, pour l'année $(2017 + n)$.
\medskip
\begin{enumerate}
\item Expliquer pourquoi la suite $u_n$ est géométrique?
\item Calculer $u_1$ et $u_2$.
\item Exprimer $u_n$ en fonction de $n$.
\item Au début de l'année 2017, il y avait $300$ millions de tonnes de déchets plastique. Calculer la quantité totale de déchets plastiques en 2030.
\item On souhaite déterminer en quelle année la masse totale de ces déchets plastiques aura pour la première fois augmenté de $50$\,\% par rapport à sa valeur de 2017.
\begin{enumerate}
\item Recopier et compléter l'algorithme ci-dessous pour que la variable $N$ contienne la réponse au problème posé.
\begin{center}
\begin{tabularx}{0.4\linewidth}{|X|}\hline
$N = 2017$\\
$U = 2$ \\
$S = 300 + U$ \\
while $S < 450$: \\
\hspace{1cm} $N = \ldots$\\
\hspace{1cm} $U = \ldots$\\
\hspace{1cm} $S = \ldots$\\
\hline
\end{tabularx}
\end{center}
\item Que contiennent les variables $S$, $U$ et $N$ après exécution de cet algorithme ?
Interpréter les résultats dans le contexte de l'exercice.
\end{enumerate}
\end{enumerate}
\end{exercise}
\begin{solution}
\begin{enumerate}
\item Une augmentation de $30\,\%$ revient à multiplier la quantité par $1.3$. La suite est donc bien géométrique. Son premier terme est $u_0 = 2$ et sa raison est $q = 1.3$
\item
\[
u_1 = u_0 * 1.3 = 2.6
\]
\[
u_2 = u_0 * 1.3^2 = 3.38
\]
\item
\[
u_n = u_0 \times q^n = 2 \times 1.3^n
\]
\item On calcule la quantité totale déversée entre 2017 et 2030.
\[
\sum_{n = 0}^{13} u_n = u_0 \times \frac{1-q^{13}}{1-q} = 2 \times \frac{1 - 1.3^{13}}{1 - 1.3} = 195.25
\]
On en déduit la quantité totale de déchets en 2030
\[
300 + 195.25 = 495.25
\]
\item
\begin{enumerate}
\item ~
\begin{center}
\begin{tabularx}{0.4\linewidth}{|X|}\hline
$N \gets 2017$\\
$U \gets 2$ \\
$S \gets 300 + U$ \\
Tant que $S < 450$ \\
\hspace{1cm} $N \gets N + 1$\\
\hspace{1cm} $U \gets U * 1.3$\\
\hspace{1cm} $S \gets S + u$\\
Fin Tant que\\\hline
\end{tabularx}
\end{center}
\item \textit{Pas de correction automatisé}
\end{enumerate}
\end{enumerate}
\end{solution}
\end{document}
%%% Local Variables:
%%% mode: latex
%%% TeX-master: "master"
%%% End:

View File

@ -0,0 +1,262 @@
\documentclass[a4paper,10pt]{article}
\usepackage{myXsim}
% Title Page
\title{DS8 \hfill DEBRAS Noémie}
\tribe{TST}
\date{\hfillÀ render pour le Mercredi 7 avril}
\xsimsetup{
solution/print = true
}
\begin{document}
\maketitle
\begin{exercise}[subtitle={Automatismes}]
\textit{Toutes les questions de cette exercice sont indépendantes et peuvent être répondus séparément}
\begin{enumerate}
\item De janvier à septembre, une quantité a augmenté de $24\,\%$. Faire un schéma pour représenter la situation puis calculer le taux d'évolution moyen mensuel.
\item Une quantité augmente de $24\,\%$ par ans. En 2020, elle est de 110\euro. Quelle était sa valeur en 2019? Faire un schéma pour représenter la situation.
\item Déterminer l'équation de la droite \\
\begin{tikzpicture}[xscale=0.8, yscale=0.5]
\tkzInit[xmin=-5,xmax=5,xstep=1,
ymin=-5,ymax=5,ystep=1]
\tkzGrid
\tkzAxeXY
\tkzFct[domain=-5:5,color=red,very thick]%
{0.6666666666666666*\x -1};
\end{tikzpicture}
\item Résoudre l'équation $8 \times 0.49^x = 9$
\end{enumerate}
\end{exercise}
\begin{solution}
\begin{enumerate}
\item On veut partager cette évolution en 8 évolutions.
\[
\left(1 + \frac{24}{100}\right)^{\frac{1}{8}} = 1.0273
\]
Donc le taux d'évolution moyen est
\[
t_m = 1.0273 - 1 = 0.027300000000000102
\]
\item Coefficient multiplicateur pour revenir en arrière
\[
CM = (1 + \frac{24}{100})^{-1} = 0.8065
\]
On en déduit la quantité en 2019
\[
110 * 0.8065 = 88.715
\]
\item L'équation de la droite est
\[
y = 0.6666666666666666 x -1
\]
\item Il faut penser à faire la division à par $8$ avant d'utiliser le log car sinon, on ne peut pas utiliser la formule $\log(a^n) = n\times \log(a)$.
\[x = \frac{\log(1.12)}{\log(0.49)}\]
\end{enumerate}
\end{solution}
\begin{exercise}[subtitle={Restaurant}]
Un \emph{food truck}, ouvert le midi et le soir, propose deux types de formules :
\setlength\parindent{10mm}
\begin{itemize}
\item la formule \emph{Burger} ;
\item la formule \emph{Wok}.
\end{itemize}
\setlength\parindent{0mm}
\medskip
Le gérant a remarqué que 49\,\% de ses ventes ont lieu le midi. Le quart des ventes du midi correspondent à la formule \emph{Burger}, alors que 61\,\% des ventes du soir correspondent à la formule \emph{Wok}.
Le gérant se constitue un fichier en notant, pour chaque vente, la formule choisie et le moment de cette vente (midi ou soir).
On prélève une fiche de façon équiprobable. On définit les quatre évènements suivants:
\begin{enumerate}
\item $M$ : \og la fiche correspond à une vente du midi\fg{} ;
\item $S$ : \og la fiche correspond à une vente du soir\fg {};
\item $W$ : \og la fiche correspond à une formule \emph{Wok} \fg{} ;
\item $B$ : \og la fiche correspond à une formule \emph{Burger} \fg.
\end{enumerate}
\setlength\parindent{0mm}
\medskip
\begin{enumerate}
\item Recopier puis compléter l'arbre pondéré
\begin{center}
\begin{tikzpicture}[sloped]
\node {.}
child {node {$M$}
child {node {$W$}
edge from parent
node[above] {...}
}
child {node {$B$}
edge from parent
node[above] {...}
}
edge from parent
node[above] {...}
}
child[missing] {}
child { node {$S$}
child {node {$W$}
edge from parent
node[above] {...}
}
child {node {$B$}
edge from parent
node[above] {...}
}
edge from parent
node[above] {...}
} ;
\end{tikzpicture}
\end{center}
\item Calculer la probabilité de l'évènement $M \cap W$. Interpréter ce résultat dans le contexte de l'exercice.
\item Montrer que la probabilité que la fiche choisie corresponde à une formule \emph{Burger} est égale à $0.3214$.
\item On a prélevé une fiche correspondant à la formule \emph{Burger}. Quelle est la probabilité, arrondie au millième, que la vente ait eu lieu le soir?
\end{enumerate}
\end{exercise}
\begin{solution}
\begin{enumerate}
\item
\begin{center}
\begin{tikzpicture}[sloped]
\node {.}
child {node {$M$}
child {node {$W$}
edge from parent
node[above] {$0.75$}
}
child {node {$B$}
edge from parent
node[above] {$0.25$}
}
edge from parent
node[above] {$0.49$}
}
child[missing] {}
child { node {$S$}
child {node {$W$}
edge from parent
node[above] {$0.61$}
}
child {node {$B$}
edge from parent
node[above] {$0.39$}
}
edge from parent
node[above] {$0.51$}
} ;
\end{tikzpicture}
\end{center}
\item On calcule la probabilité que la vente soit un wok et ait eu lieu à midi
\[ P(M\cap W) = P(M) \times P_M(W) = 0.49 \times 0.75 = 0.3675 \]
\item Probabilité que la vente soit un burger.
\[
P(B) = P(M\cap B) + P(S\cap B) = 0.49 \times 0.75 + 0.51 \times 0.61 = 0.3214
\]
\item On cherche à calculer la quantité $P_B(S)$. Pour cela on utilise la formule de Bayes
\[
P_B(S) = \frac{P(B\cap S)}{P(B)} = \frac{P_S(B) \times P(S)}{P(B)} = \frac{0.39\times 0.51}{0.3214} = 0.618855009334163 \approx 0.619
\]
\end{enumerate}
\end{solution}
\begin{exercise}[subtitle={Continent plastique}]
\textit{Les quantités évoqués dans cette exercice sont générés au hasard et sont donc complètement farfelus.}
\medskip
Le \og continent de plastique\fg{} est la plus grande des plaques de déchets plastiques évoluant sur les océans. Elle occupe actuellement dans l'océan Pacifique une surface dont l'aire est évaluée à plus de $1,6$ million de km$^2$, entre Hawaï et la Californie.
En 2017, des scientifiques ont estimé qu'il y avait $15$ millions de tonnes de déchets plastiques qui était déversé chaque année dans les océans et que cette quantité augmentait de $21\n\%$ par chaque année.
On modélise l'évolution de la masse de ces déchets plastiques déversée chaque année, si rien n'est fait pour la réduire, par une suite géométrique $\left(u_n\right)$. L'arrondi au centième du terme $u_n$ représente la masse de ces déchets déversée chaque année, exprimée en million de tonnes, pour l'année $(2017 + n)$.
\medskip
\begin{enumerate}
\item Expliquer pourquoi la suite $u_n$ est géométrique?
\item Calculer $u_1$ et $u_2$.
\item Exprimer $u_n$ en fonction de $n$.
\item Au début de l'année 2017, il y avait $300$ millions de tonnes de déchets plastique. Calculer la quantité totale de déchets plastiques en 2030.
\item On souhaite déterminer en quelle année la masse totale de ces déchets plastiques aura pour la première fois augmenté de $50$\,\% par rapport à sa valeur de 2017.
\begin{enumerate}
\item Recopier et compléter l'algorithme ci-dessous pour que la variable $N$ contienne la réponse au problème posé.
\begin{center}
\begin{tabularx}{0.4\linewidth}{|X|}\hline
$N = 2017$\\
$U = 15$ \\
$S = 300 + U$ \\
while $S < 450$: \\
\hspace{1cm} $N = \ldots$\\
\hspace{1cm} $U = \ldots$\\
\hspace{1cm} $S = \ldots$\\
\hline
\end{tabularx}
\end{center}
\item Que contiennent les variables $S$, $U$ et $N$ après exécution de cet algorithme ?
Interpréter les résultats dans le contexte de l'exercice.
\end{enumerate}
\end{enumerate}
\end{exercise}
\begin{solution}
\begin{enumerate}
\item Une augmentation de $21\,\%$ revient à multiplier la quantité par $1.21$. La suite est donc bien géométrique. Son premier terme est $u_0 = 15$ et sa raison est $q = 1.21$
\item
\[
u_1 = u_0 * 1.21 = 18.15
\]
\[
u_2 = u_0 * 1.21^2 = 21.9615
\]
\item
\[
u_n = u_0 \times q^n = 15 \times 1.21^n
\]
\item On calcule la quantité totale déversée entre 2017 et 2030.
\[
\sum_{n = 0}^{13} u_n = u_0 \times \frac{1-q^{13}}{1-q} = 15 \times \frac{1 - 1.21^{13}}{1 - 1.21} = 779.87
\]
On en déduit la quantité totale de déchets en 2030
\[
300 + 779.87 = 1079.87
\]
\item
\begin{enumerate}
\item ~
\begin{center}
\begin{tabularx}{0.4\linewidth}{|X|}\hline
$N \gets 2017$\\
$U \gets 15$ \\
$S \gets 300 + U$ \\
Tant que $S < 450$ \\
\hspace{1cm} $N \gets N + 1$\\
\hspace{1cm} $U \gets U * 1.21$\\
\hspace{1cm} $S \gets S + u$\\
Fin Tant que\\\hline
\end{tabularx}
\end{center}
\item \textit{Pas de correction automatisé}
\end{enumerate}
\end{enumerate}
\end{solution}
\end{document}
%%% Local Variables:
%%% mode: latex
%%% TeX-master: "master"
%%% End:

View File

@ -0,0 +1,262 @@
\documentclass[a4paper,10pt]{article}
\usepackage{myXsim}
% Title Page
\title{DS8 \hfill GERMAIN Anaïs}
\tribe{TST}
\date{\hfillÀ render pour le Mercredi 7 avril}
\xsimsetup{
solution/print = true
}
\begin{document}
\maketitle
\begin{exercise}[subtitle={Automatismes}]
\textit{Toutes les questions de cette exercice sont indépendantes et peuvent être répondus séparément}
\begin{enumerate}
\item De janvier à septembre, une quantité a augmenté de $27\,\%$. Faire un schéma pour représenter la situation puis calculer le taux d'évolution moyen mensuel.
\item Une quantité augmente de $27\,\%$ par ans. En 2020, elle est de 132\euro. Quelle était sa valeur en 2019? Faire un schéma pour représenter la situation.
\item Déterminer l'équation de la droite \\
\begin{tikzpicture}[xscale=0.8, yscale=0.5]
\tkzInit[xmin=-5,xmax=5,xstep=1,
ymin=-5,ymax=5,ystep=1]
\tkzGrid
\tkzAxeXY
\tkzFct[domain=-5:5,color=red,very thick]%
{2.0*\x -4};
\end{tikzpicture}
\item Résoudre l'équation $7 \times 0.05^x = 18$
\end{enumerate}
\end{exercise}
\begin{solution}
\begin{enumerate}
\item On veut partager cette évolution en 8 évolutions.
\[
\left(1 + \frac{27}{100}\right)^{\frac{1}{8}} = 1.0303
\]
Donc le taux d'évolution moyen est
\[
t_m = 1.0303 - 1 = 0.030299999999999994
\]
\item Coefficient multiplicateur pour revenir en arrière
\[
CM = (1 + \frac{27}{100})^{-1} = 0.7874
\]
On en déduit la quantité en 2019
\[
132 * 0.7874 = 103.9368
\]
\item L'équation de la droite est
\[
y = 2.0 x -4
\]
\item Il faut penser à faire la division à par $7$ avant d'utiliser le log car sinon, on ne peut pas utiliser la formule $\log(a^n) = n\times \log(a)$.
\[x = \frac{\log(2.57)}{\log(0.05)}\]
\end{enumerate}
\end{solution}
\begin{exercise}[subtitle={Restaurant}]
Un \emph{food truck}, ouvert le midi et le soir, propose deux types de formules :
\setlength\parindent{10mm}
\begin{itemize}
\item la formule \emph{Burger} ;
\item la formule \emph{Wok}.
\end{itemize}
\setlength\parindent{0mm}
\medskip
Le gérant a remarqué que 38\,\% de ses ventes ont lieu le midi. Le quart des ventes du midi correspondent à la formule \emph{Burger}, alors que 63\,\% des ventes du soir correspondent à la formule \emph{Wok}.
Le gérant se constitue un fichier en notant, pour chaque vente, la formule choisie et le moment de cette vente (midi ou soir).
On prélève une fiche de façon équiprobable. On définit les quatre évènements suivants:
\begin{enumerate}
\item $M$ : \og la fiche correspond à une vente du midi\fg{} ;
\item $S$ : \og la fiche correspond à une vente du soir\fg {};
\item $W$ : \og la fiche correspond à une formule \emph{Wok} \fg{} ;
\item $B$ : \og la fiche correspond à une formule \emph{Burger} \fg.
\end{enumerate}
\setlength\parindent{0mm}
\medskip
\begin{enumerate}
\item Recopier puis compléter l'arbre pondéré
\begin{center}
\begin{tikzpicture}[sloped]
\node {.}
child {node {$M$}
child {node {$W$}
edge from parent
node[above] {...}
}
child {node {$B$}
edge from parent
node[above] {...}
}
edge from parent
node[above] {...}
}
child[missing] {}
child { node {$S$}
child {node {$W$}
edge from parent
node[above] {...}
}
child {node {$B$}
edge from parent
node[above] {...}
}
edge from parent
node[above] {...}
} ;
\end{tikzpicture}
\end{center}
\item Calculer la probabilité de l'évènement $M \cap W$. Interpréter ce résultat dans le contexte de l'exercice.
\item Montrer que la probabilité que la fiche choisie corresponde à une formule \emph{Burger} est égale à $0.3244$.
\item On a prélevé une fiche correspondant à la formule \emph{Burger}. Quelle est la probabilité, arrondie au millième, que la vente ait eu lieu le soir?
\end{enumerate}
\end{exercise}
\begin{solution}
\begin{enumerate}
\item
\begin{center}
\begin{tikzpicture}[sloped]
\node {.}
child {node {$M$}
child {node {$W$}
edge from parent
node[above] {$0.75$}
}
child {node {$B$}
edge from parent
node[above] {$0.25$}
}
edge from parent
node[above] {$0.38$}
}
child[missing] {}
child { node {$S$}
child {node {$W$}
edge from parent
node[above] {$0.63$}
}
child {node {$B$}
edge from parent
node[above] {$0.37$}
}
edge from parent
node[above] {$0.62$}
} ;
\end{tikzpicture}
\end{center}
\item On calcule la probabilité que la vente soit un wok et ait eu lieu à midi
\[ P(M\cap W) = P(M) \times P_M(W) = 0.38 \times 0.75 = 0.285 \]
\item Probabilité que la vente soit un burger.
\[
P(B) = P(M\cap B) + P(S\cap B) = 0.38 \times 0.75 + 0.62 \times 0.63 = 0.3244
\]
\item On cherche à calculer la quantité $P_B(S)$. Pour cela on utilise la formule de Bayes
\[
P_B(S) = \frac{P(B\cap S)}{P(B)} = \frac{P_S(B) \times P(S)}{P(B)} = \frac{0.37\times 0.62}{0.3244} = 0.7071516646115905 \approx 0.707
\]
\end{enumerate}
\end{solution}
\begin{exercise}[subtitle={Continent plastique}]
\textit{Les quantités évoqués dans cette exercice sont générés au hasard et sont donc complètement farfelus.}
\medskip
Le \og continent de plastique\fg{} est la plus grande des plaques de déchets plastiques évoluant sur les océans. Elle occupe actuellement dans l'océan Pacifique une surface dont l'aire est évaluée à plus de $1,6$ million de km$^2$, entre Hawaï et la Californie.
En 2017, des scientifiques ont estimé qu'il y avait $18$ millions de tonnes de déchets plastiques qui était déversé chaque année dans les océans et que cette quantité augmentait de $28\n\%$ par chaque année.
On modélise l'évolution de la masse de ces déchets plastiques déversée chaque année, si rien n'est fait pour la réduire, par une suite géométrique $\left(u_n\right)$. L'arrondi au centième du terme $u_n$ représente la masse de ces déchets déversée chaque année, exprimée en million de tonnes, pour l'année $(2017 + n)$.
\medskip
\begin{enumerate}
\item Expliquer pourquoi la suite $u_n$ est géométrique?
\item Calculer $u_1$ et $u_2$.
\item Exprimer $u_n$ en fonction de $n$.
\item Au début de l'année 2017, il y avait $300$ millions de tonnes de déchets plastique. Calculer la quantité totale de déchets plastiques en 2030.
\item On souhaite déterminer en quelle année la masse totale de ces déchets plastiques aura pour la première fois augmenté de $50$\,\% par rapport à sa valeur de 2017.
\begin{enumerate}
\item Recopier et compléter l'algorithme ci-dessous pour que la variable $N$ contienne la réponse au problème posé.
\begin{center}
\begin{tabularx}{0.4\linewidth}{|X|}\hline
$N = 2017$\\
$U = 18$ \\
$S = 300 + U$ \\
while $S < 450$: \\
\hspace{1cm} $N = \ldots$\\
\hspace{1cm} $U = \ldots$\\
\hspace{1cm} $S = \ldots$\\
\hline
\end{tabularx}
\end{center}
\item Que contiennent les variables $S$, $U$ et $N$ après exécution de cet algorithme ?
Interpréter les résultats dans le contexte de l'exercice.
\end{enumerate}
\end{enumerate}
\end{exercise}
\begin{solution}
\begin{enumerate}
\item Une augmentation de $28\,\%$ revient à multiplier la quantité par $1.28$. La suite est donc bien géométrique. Son premier terme est $u_0 = 18$ et sa raison est $q = 1.28$
\item
\[
u_1 = u_0 * 1.28 = 23.04
\]
\[
u_2 = u_0 * 1.28^2 = 29.4912
\]
\item
\[
u_n = u_0 \times q^n = 18 \times 1.28^n
\]
\item On calcule la quantité totale déversée entre 2017 et 2030.
\[
\sum_{n = 0}^{13} u_n = u_0 \times \frac{1-q^{13}}{1-q} = 18 \times \frac{1 - 1.28^{13}}{1 - 1.28} = 1527.35
\]
On en déduit la quantité totale de déchets en 2030
\[
300 + 1527.35 = 1827.35
\]
\item
\begin{enumerate}
\item ~
\begin{center}
\begin{tabularx}{0.4\linewidth}{|X|}\hline
$N \gets 2017$\\
$U \gets 18$ \\
$S \gets 300 + U$ \\
Tant que $S < 450$ \\
\hspace{1cm} $N \gets N + 1$\\
\hspace{1cm} $U \gets U * 1.28$\\
\hspace{1cm} $S \gets S + u$\\
Fin Tant que\\\hline
\end{tabularx}
\end{center}
\item \textit{Pas de correction automatisé}
\end{enumerate}
\end{enumerate}
\end{solution}
\end{document}
%%% Local Variables:
%%% mode: latex
%%% TeX-master: "master"
%%% End:

View File

@ -0,0 +1,262 @@
\documentclass[a4paper,10pt]{article}
\usepackage{myXsim}
% Title Page
\title{DS8 \hfill HADJRAS Mohcine}
\tribe{TST}
\date{\hfillÀ render pour le Mercredi 7 avril}
\xsimsetup{
solution/print = true
}
\begin{document}
\maketitle
\begin{exercise}[subtitle={Automatismes}]
\textit{Toutes les questions de cette exercice sont indépendantes et peuvent être répondus séparément}
\begin{enumerate}
\item De janvier à septembre, une quantité a augmenté de $19\,\%$. Faire un schéma pour représenter la situation puis calculer le taux d'évolution moyen mensuel.
\item Une quantité augmente de $19\,\%$ par ans. En 2020, elle est de 113\euro. Quelle était sa valeur en 2019? Faire un schéma pour représenter la situation.
\item Déterminer l'équation de la droite \\
\begin{tikzpicture}[xscale=0.8, yscale=0.5]
\tkzInit[xmin=-5,xmax=5,xstep=1,
ymin=-5,ymax=5,ystep=1]
\tkzGrid
\tkzAxeXY
\tkzFct[domain=-5:5,color=red,very thick]%
{0.5*\x -1};
\end{tikzpicture}
\item Résoudre l'équation $4 \times 0.54^x = 6$
\end{enumerate}
\end{exercise}
\begin{solution}
\begin{enumerate}
\item On veut partager cette évolution en 8 évolutions.
\[
\left(1 + \frac{19}{100}\right)^{\frac{1}{8}} = 1.022
\]
Donc le taux d'évolution moyen est
\[
t_m = 1.022 - 1 = 0.02200000000000002
\]
\item Coefficient multiplicateur pour revenir en arrière
\[
CM = (1 + \frac{19}{100})^{-1} = 0.8403
\]
On en déduit la quantité en 2019
\[
113 * 0.8403 = 94.9539
\]
\item L'équation de la droite est
\[
y = 0.5 x -1
\]
\item Il faut penser à faire la division à par $4$ avant d'utiliser le log car sinon, on ne peut pas utiliser la formule $\log(a^n) = n\times \log(a)$.
\[x = \frac{\log(1.5)}{\log(0.54)}\]
\end{enumerate}
\end{solution}
\begin{exercise}[subtitle={Restaurant}]
Un \emph{food truck}, ouvert le midi et le soir, propose deux types de formules :
\setlength\parindent{10mm}
\begin{itemize}
\item la formule \emph{Burger} ;
\item la formule \emph{Wok}.
\end{itemize}
\setlength\parindent{0mm}
\medskip
Le gérant a remarqué que 68\,\% de ses ventes ont lieu le midi. Le quart des ventes du midi correspondent à la formule \emph{Burger}, alors que 43\,\% des ventes du soir correspondent à la formule \emph{Wok}.
Le gérant se constitue un fichier en notant, pour chaque vente, la formule choisie et le moment de cette vente (midi ou soir).
On prélève une fiche de façon équiprobable. On définit les quatre évènements suivants:
\begin{enumerate}
\item $M$ : \og la fiche correspond à une vente du midi\fg{} ;
\item $S$ : \og la fiche correspond à une vente du soir\fg {};
\item $W$ : \og la fiche correspond à une formule \emph{Wok} \fg{} ;
\item $B$ : \og la fiche correspond à une formule \emph{Burger} \fg.
\end{enumerate}
\setlength\parindent{0mm}
\medskip
\begin{enumerate}
\item Recopier puis compléter l'arbre pondéré
\begin{center}
\begin{tikzpicture}[sloped]
\node {.}
child {node {$M$}
child {node {$W$}
edge from parent
node[above] {...}
}
child {node {$B$}
edge from parent
node[above] {...}
}
edge from parent
node[above] {...}
}
child[missing] {}
child { node {$S$}
child {node {$W$}
edge from parent
node[above] {...}
}
child {node {$B$}
edge from parent
node[above] {...}
}
edge from parent
node[above] {...}
} ;
\end{tikzpicture}
\end{center}
\item Calculer la probabilité de l'évènement $M \cap W$. Interpréter ce résultat dans le contexte de l'exercice.
\item Montrer que la probabilité que la fiche choisie corresponde à une formule \emph{Burger} est égale à $0.3524$.
\item On a prélevé une fiche correspondant à la formule \emph{Burger}. Quelle est la probabilité, arrondie au millième, que la vente ait eu lieu le soir?
\end{enumerate}
\end{exercise}
\begin{solution}
\begin{enumerate}
\item
\begin{center}
\begin{tikzpicture}[sloped]
\node {.}
child {node {$M$}
child {node {$W$}
edge from parent
node[above] {$0.75$}
}
child {node {$B$}
edge from parent
node[above] {$0.25$}
}
edge from parent
node[above] {$0.68$}
}
child[missing] {}
child { node {$S$}
child {node {$W$}
edge from parent
node[above] {$0.43$}
}
child {node {$B$}
edge from parent
node[above] {$0.57$}
}
edge from parent
node[above] {$0.32$}
} ;
\end{tikzpicture}
\end{center}
\item On calcule la probabilité que la vente soit un wok et ait eu lieu à midi
\[ P(M\cap W) = P(M) \times P_M(W) = 0.68 \times 0.75 = 0.51 \]
\item Probabilité que la vente soit un burger.
\[
P(B) = P(M\cap B) + P(S\cap B) = 0.68 \times 0.75 + 0.32 \times 0.43 = 0.3524
\]
\item On cherche à calculer la quantité $P_B(S)$. Pour cela on utilise la formule de Bayes
\[
P_B(S) = \frac{P(B\cap S)}{P(B)} = \frac{P_S(B) \times P(S)}{P(B)} = \frac{0.57\times 0.32}{0.3524} = 0.5175936435868331 \approx 0.518
\]
\end{enumerate}
\end{solution}
\begin{exercise}[subtitle={Continent plastique}]
\textit{Les quantités évoqués dans cette exercice sont générés au hasard et sont donc complètement farfelus.}
\medskip
Le \og continent de plastique\fg{} est la plus grande des plaques de déchets plastiques évoluant sur les océans. Elle occupe actuellement dans l'océan Pacifique une surface dont l'aire est évaluée à plus de $1,6$ million de km$^2$, entre Hawaï et la Californie.
En 2017, des scientifiques ont estimé qu'il y avait $4$ millions de tonnes de déchets plastiques qui était déversé chaque année dans les océans et que cette quantité augmentait de $10\n\%$ par chaque année.
On modélise l'évolution de la masse de ces déchets plastiques déversée chaque année, si rien n'est fait pour la réduire, par une suite géométrique $\left(u_n\right)$. L'arrondi au centième du terme $u_n$ représente la masse de ces déchets déversée chaque année, exprimée en million de tonnes, pour l'année $(2017 + n)$.
\medskip
\begin{enumerate}
\item Expliquer pourquoi la suite $u_n$ est géométrique?
\item Calculer $u_1$ et $u_2$.
\item Exprimer $u_n$ en fonction de $n$.
\item Au début de l'année 2017, il y avait $300$ millions de tonnes de déchets plastique. Calculer la quantité totale de déchets plastiques en 2030.
\item On souhaite déterminer en quelle année la masse totale de ces déchets plastiques aura pour la première fois augmenté de $50$\,\% par rapport à sa valeur de 2017.
\begin{enumerate}
\item Recopier et compléter l'algorithme ci-dessous pour que la variable $N$ contienne la réponse au problème posé.
\begin{center}
\begin{tabularx}{0.4\linewidth}{|X|}\hline
$N = 2017$\\
$U = 4$ \\
$S = 300 + U$ \\
while $S < 450$: \\
\hspace{1cm} $N = \ldots$\\
\hspace{1cm} $U = \ldots$\\
\hspace{1cm} $S = \ldots$\\
\hline
\end{tabularx}
\end{center}
\item Que contiennent les variables $S$, $U$ et $N$ après exécution de cet algorithme ?
Interpréter les résultats dans le contexte de l'exercice.
\end{enumerate}
\end{enumerate}
\end{exercise}
\begin{solution}
\begin{enumerate}
\item Une augmentation de $10\,\%$ revient à multiplier la quantité par $1.1$. La suite est donc bien géométrique. Son premier terme est $u_0 = 4$ et sa raison est $q = 1.1$
\item
\[
u_1 = u_0 * 1.1 = 4.4
\]
\[
u_2 = u_0 * 1.1^2 = 4.84
\]
\item
\[
u_n = u_0 \times q^n = 4 \times 1.1^n
\]
\item On calcule la quantité totale déversée entre 2017 et 2030.
\[
\sum_{n = 0}^{13} u_n = u_0 \times \frac{1-q^{13}}{1-q} = 4 \times \frac{1 - 1.1^{13}}{1 - 1.1} = 98.09
\]
On en déduit la quantité totale de déchets en 2030
\[
300 + 98.09 = 398.09000000000003
\]
\item
\begin{enumerate}
\item ~
\begin{center}
\begin{tabularx}{0.4\linewidth}{|X|}\hline
$N \gets 2017$\\
$U \gets 4$ \\
$S \gets 300 + U$ \\
Tant que $S < 450$ \\
\hspace{1cm} $N \gets N + 1$\\
\hspace{1cm} $U \gets U * 1.1$\\
\hspace{1cm} $S \gets S + u$\\
Fin Tant que\\\hline
\end{tabularx}
\end{center}
\item \textit{Pas de correction automatisé}
\end{enumerate}
\end{enumerate}
\end{solution}
\end{document}
%%% Local Variables:
%%% mode: latex
%%% TeX-master: "master"
%%% End:

View File

@ -0,0 +1,262 @@
\documentclass[a4paper,10pt]{article}
\usepackage{myXsim}
% Title Page
\title{DS8 \hfill HENRIST Maxime}
\tribe{TST}
\date{\hfillÀ render pour le Mercredi 7 avril}
\xsimsetup{
solution/print = true
}
\begin{document}
\maketitle
\begin{exercise}[subtitle={Automatismes}]
\textit{Toutes les questions de cette exercice sont indépendantes et peuvent être répondus séparément}
\begin{enumerate}
\item De janvier à septembre, une quantité a augmenté de $19\,\%$. Faire un schéma pour représenter la situation puis calculer le taux d'évolution moyen mensuel.
\item Une quantité augmente de $19\,\%$ par ans. En 2020, elle est de 133\euro. Quelle était sa valeur en 2019? Faire un schéma pour représenter la situation.
\item Déterminer l'équation de la droite \\
\begin{tikzpicture}[xscale=0.8, yscale=0.5]
\tkzInit[xmin=-5,xmax=5,xstep=1,
ymin=-5,ymax=5,ystep=1]
\tkzGrid
\tkzAxeXY
\tkzFct[domain=-5:5,color=red,very thick]%
{1.0*\x -2};
\end{tikzpicture}
\item Résoudre l'équation $7 \times 0.82^x = 19$
\end{enumerate}
\end{exercise}
\begin{solution}
\begin{enumerate}
\item On veut partager cette évolution en 8 évolutions.
\[
\left(1 + \frac{19}{100}\right)^{\frac{1}{8}} = 1.022
\]
Donc le taux d'évolution moyen est
\[
t_m = 1.022 - 1 = 0.02200000000000002
\]
\item Coefficient multiplicateur pour revenir en arrière
\[
CM = (1 + \frac{19}{100})^{-1} = 0.8403
\]
On en déduit la quantité en 2019
\[
133 * 0.8403 = 111.7599
\]
\item L'équation de la droite est
\[
y = 1.0 x -2
\]
\item Il faut penser à faire la division à par $7$ avant d'utiliser le log car sinon, on ne peut pas utiliser la formule $\log(a^n) = n\times \log(a)$.
\[x = \frac{\log(2.71)}{\log(0.82)}\]
\end{enumerate}
\end{solution}
\begin{exercise}[subtitle={Restaurant}]
Un \emph{food truck}, ouvert le midi et le soir, propose deux types de formules :
\setlength\parindent{10mm}
\begin{itemize}
\item la formule \emph{Burger} ;
\item la formule \emph{Wok}.
\end{itemize}
\setlength\parindent{0mm}
\medskip
Le gérant a remarqué que 20\,\% de ses ventes ont lieu le midi. Le quart des ventes du midi correspondent à la formule \emph{Burger}, alors que 56\,\% des ventes du soir correspondent à la formule \emph{Wok}.
Le gérant se constitue un fichier en notant, pour chaque vente, la formule choisie et le moment de cette vente (midi ou soir).
On prélève une fiche de façon équiprobable. On définit les quatre évènements suivants:
\begin{enumerate}
\item $M$ : \og la fiche correspond à une vente du midi\fg{} ;
\item $S$ : \og la fiche correspond à une vente du soir\fg {};
\item $W$ : \og la fiche correspond à une formule \emph{Wok} \fg{} ;
\item $B$ : \og la fiche correspond à une formule \emph{Burger} \fg.
\end{enumerate}
\setlength\parindent{0mm}
\medskip
\begin{enumerate}
\item Recopier puis compléter l'arbre pondéré
\begin{center}
\begin{tikzpicture}[sloped]
\node {.}
child {node {$M$}
child {node {$W$}
edge from parent
node[above] {...}
}
child {node {$B$}
edge from parent
node[above] {...}
}
edge from parent
node[above] {...}
}
child[missing] {}
child { node {$S$}
child {node {$W$}
edge from parent
node[above] {...}
}
child {node {$B$}
edge from parent
node[above] {...}
}
edge from parent
node[above] {...}
} ;
\end{tikzpicture}
\end{center}
\item Calculer la probabilité de l'évènement $M \cap W$. Interpréter ce résultat dans le contexte de l'exercice.
\item Montrer que la probabilité que la fiche choisie corresponde à une formule \emph{Burger} est égale à $0.394$.
\item On a prélevé une fiche correspondant à la formule \emph{Burger}. Quelle est la probabilité, arrondie au millième, que la vente ait eu lieu le soir?
\end{enumerate}
\end{exercise}
\begin{solution}
\begin{enumerate}
\item
\begin{center}
\begin{tikzpicture}[sloped]
\node {.}
child {node {$M$}
child {node {$W$}
edge from parent
node[above] {$0.75$}
}
child {node {$B$}
edge from parent
node[above] {$0.25$}
}
edge from parent
node[above] {$0.2$}
}
child[missing] {}
child { node {$S$}
child {node {$W$}
edge from parent
node[above] {$0.57$}
}
child {node {$B$}
edge from parent
node[above] {$0.43$}
}
edge from parent
node[above] {$0.8$}
} ;
\end{tikzpicture}
\end{center}
\item On calcule la probabilité que la vente soit un wok et ait eu lieu à midi
\[ P(M\cap W) = P(M) \times P_M(W) = 0.2 \times 0.75 = 0.15 \]
\item Probabilité que la vente soit un burger.
\[
P(B) = P(M\cap B) + P(S\cap B) = 0.2 \times 0.75 + 0.8 \times 0.57 = 0.394
\]
\item On cherche à calculer la quantité $P_B(S)$. Pour cela on utilise la formule de Bayes
\[
P_B(S) = \frac{P(B\cap S)}{P(B)} = \frac{P_S(B) \times P(S)}{P(B)} = \frac{0.43\times 0.8}{0.394} = 0.8730964467005077 \approx 0.873
\]
\end{enumerate}
\end{solution}
\begin{exercise}[subtitle={Continent plastique}]
\textit{Les quantités évoqués dans cette exercice sont générés au hasard et sont donc complètement farfelus.}
\medskip
Le \og continent de plastique\fg{} est la plus grande des plaques de déchets plastiques évoluant sur les océans. Elle occupe actuellement dans l'océan Pacifique une surface dont l'aire est évaluée à plus de $1,6$ million de km$^2$, entre Hawaï et la Californie.
En 2017, des scientifiques ont estimé qu'il y avait $5$ millions de tonnes de déchets plastiques qui était déversé chaque année dans les océans et que cette quantité augmentait de $16\n\%$ par chaque année.
On modélise l'évolution de la masse de ces déchets plastiques déversée chaque année, si rien n'est fait pour la réduire, par une suite géométrique $\left(u_n\right)$. L'arrondi au centième du terme $u_n$ représente la masse de ces déchets déversée chaque année, exprimée en million de tonnes, pour l'année $(2017 + n)$.
\medskip
\begin{enumerate}
\item Expliquer pourquoi la suite $u_n$ est géométrique?
\item Calculer $u_1$ et $u_2$.
\item Exprimer $u_n$ en fonction de $n$.
\item Au début de l'année 2017, il y avait $300$ millions de tonnes de déchets plastique. Calculer la quantité totale de déchets plastiques en 2030.
\item On souhaite déterminer en quelle année la masse totale de ces déchets plastiques aura pour la première fois augmenté de $50$\,\% par rapport à sa valeur de 2017.
\begin{enumerate}
\item Recopier et compléter l'algorithme ci-dessous pour que la variable $N$ contienne la réponse au problème posé.
\begin{center}
\begin{tabularx}{0.4\linewidth}{|X|}\hline
$N = 2017$\\
$U = 5$ \\
$S = 300 + U$ \\
while $S < 450$: \\
\hspace{1cm} $N = \ldots$\\
\hspace{1cm} $U = \ldots$\\
\hspace{1cm} $S = \ldots$\\
\hline
\end{tabularx}
\end{center}
\item Que contiennent les variables $S$, $U$ et $N$ après exécution de cet algorithme ?
Interpréter les résultats dans le contexte de l'exercice.
\end{enumerate}
\end{enumerate}
\end{exercise}
\begin{solution}
\begin{enumerate}
\item Une augmentation de $16\,\%$ revient à multiplier la quantité par $1.16$. La suite est donc bien géométrique. Son premier terme est $u_0 = 5$ et sa raison est $q = 1.16$
\item
\[
u_1 = u_0 * 1.16 = 5.8
\]
\[
u_2 = u_0 * 1.16^2 = 6.728
\]
\item
\[
u_n = u_0 \times q^n = 5 \times 1.16^n
\]
\item On calcule la quantité totale déversée entre 2017 et 2030.
\[
\sum_{n = 0}^{13} u_n = u_0 \times \frac{1-q^{13}}{1-q} = 5 \times \frac{1 - 1.16^{13}}{1 - 1.16} = 183.93
\]
On en déduit la quantité totale de déchets en 2030
\[
300 + 183.93 = 483.93
\]
\item
\begin{enumerate}
\item ~
\begin{center}
\begin{tabularx}{0.4\linewidth}{|X|}\hline
$N \gets 2017$\\
$U \gets 5$ \\
$S \gets 300 + U$ \\
Tant que $S < 450$ \\
\hspace{1cm} $N \gets N + 1$\\
\hspace{1cm} $U \gets U * 1.16$\\
\hspace{1cm} $S \gets S + u$\\
Fin Tant que\\\hline
\end{tabularx}
\end{center}
\item \textit{Pas de correction automatisé}
\end{enumerate}
\end{enumerate}
\end{solution}
\end{document}
%%% Local Variables:
%%% mode: latex
%%% TeX-master: "master"
%%% End:

View File

@ -0,0 +1,262 @@
\documentclass[a4paper,10pt]{article}
\usepackage{myXsim}
% Title Page
\title{DS8 \hfill INFANTES Antoine}
\tribe{TST}
\date{\hfillÀ render pour le Mercredi 7 avril}
\xsimsetup{
solution/print = true
}
\begin{document}
\maketitle
\begin{exercise}[subtitle={Automatismes}]
\textit{Toutes les questions de cette exercice sont indépendantes et peuvent être répondus séparément}
\begin{enumerate}
\item De janvier à septembre, une quantité a augmenté de $18\,\%$. Faire un schéma pour représenter la situation puis calculer le taux d'évolution moyen mensuel.
\item Une quantité augmente de $18\,\%$ par ans. En 2020, elle est de 130\euro. Quelle était sa valeur en 2019? Faire un schéma pour représenter la situation.
\item Déterminer l'équation de la droite \\
\begin{tikzpicture}[xscale=0.8, yscale=0.5]
\tkzInit[xmin=-5,xmax=5,xstep=1,
ymin=-5,ymax=5,ystep=1]
\tkzGrid
\tkzAxeXY
\tkzFct[domain=-5:5,color=red,very thick]%
{2.6666666666666665*\x -4};
\end{tikzpicture}
\item Résoudre l'équation $3 \times 0.31^x = 15$
\end{enumerate}
\end{exercise}
\begin{solution}
\begin{enumerate}
\item On veut partager cette évolution en 8 évolutions.
\[
\left(1 + \frac{18}{100}\right)^{\frac{1}{8}} = 1.0209
\]
Donc le taux d'évolution moyen est
\[
t_m = 1.0209 - 1 = 0.02089999999999992
\]
\item Coefficient multiplicateur pour revenir en arrière
\[
CM = (1 + \frac{18}{100})^{-1} = 0.8475
\]
On en déduit la quantité en 2019
\[
130 * 0.8475 = 110.175
\]
\item L'équation de la droite est
\[
y = 2.6666666666666665 x -4
\]
\item Il faut penser à faire la division à par $3$ avant d'utiliser le log car sinon, on ne peut pas utiliser la formule $\log(a^n) = n\times \log(a)$.
\[x = \frac{\log(5.0)}{\log(0.31)}\]
\end{enumerate}
\end{solution}
\begin{exercise}[subtitle={Restaurant}]
Un \emph{food truck}, ouvert le midi et le soir, propose deux types de formules :
\setlength\parindent{10mm}
\begin{itemize}
\item la formule \emph{Burger} ;
\item la formule \emph{Wok}.
\end{itemize}
\setlength\parindent{0mm}
\medskip
Le gérant a remarqué que 33\,\% de ses ventes ont lieu le midi. Le quart des ventes du midi correspondent à la formule \emph{Burger}, alors que 13\,\% des ventes du soir correspondent à la formule \emph{Wok}.
Le gérant se constitue un fichier en notant, pour chaque vente, la formule choisie et le moment de cette vente (midi ou soir).
On prélève une fiche de façon équiprobable. On définit les quatre évènements suivants:
\begin{enumerate}
\item $M$ : \og la fiche correspond à une vente du midi\fg{} ;
\item $S$ : \og la fiche correspond à une vente du soir\fg {};
\item $W$ : \og la fiche correspond à une formule \emph{Wok} \fg{} ;
\item $B$ : \og la fiche correspond à une formule \emph{Burger} \fg.
\end{enumerate}
\setlength\parindent{0mm}
\medskip
\begin{enumerate}
\item Recopier puis compléter l'arbre pondéré
\begin{center}
\begin{tikzpicture}[sloped]
\node {.}
child {node {$M$}
child {node {$W$}
edge from parent
node[above] {...}
}
child {node {$B$}
edge from parent
node[above] {...}
}
edge from parent
node[above] {...}
}
child[missing] {}
child { node {$S$}
child {node {$W$}
edge from parent
node[above] {...}
}
child {node {$B$}
edge from parent
node[above] {...}
}
edge from parent
node[above] {...}
} ;
\end{tikzpicture}
\end{center}
\item Calculer la probabilité de l'évènement $M \cap W$. Interpréter ce résultat dans le contexte de l'exercice.
\item Montrer que la probabilité que la fiche choisie corresponde à une formule \emph{Burger} est égale à $0.6654$.
\item On a prélevé une fiche correspondant à la formule \emph{Burger}. Quelle est la probabilité, arrondie au millième, que la vente ait eu lieu le soir?
\end{enumerate}
\end{exercise}
\begin{solution}
\begin{enumerate}
\item
\begin{center}
\begin{tikzpicture}[sloped]
\node {.}
child {node {$M$}
child {node {$W$}
edge from parent
node[above] {$0.75$}
}
child {node {$B$}
edge from parent
node[above] {$0.25$}
}
edge from parent
node[above] {$0.33$}
}
child[missing] {}
child { node {$S$}
child {node {$W$}
edge from parent
node[above] {$0.13$}
}
child {node {$B$}
edge from parent
node[above] {$0.87$}
}
edge from parent
node[above] {$0.67$}
} ;
\end{tikzpicture}
\end{center}
\item On calcule la probabilité que la vente soit un wok et ait eu lieu à midi
\[ P(M\cap W) = P(M) \times P_M(W) = 0.33 \times 0.75 = 0.2475 \]
\item Probabilité que la vente soit un burger.
\[
P(B) = P(M\cap B) + P(S\cap B) = 0.33 \times 0.75 + 0.67 \times 0.13 = 0.6654
\]
\item On cherche à calculer la quantité $P_B(S)$. Pour cela on utilise la formule de Bayes
\[
P_B(S) = \frac{P(B\cap S)}{P(B)} = \frac{P_S(B) \times P(S)}{P(B)} = \frac{0.87\times 0.67}{0.6654} = 0.8760144274120831 \approx 0.876
\]
\end{enumerate}
\end{solution}
\begin{exercise}[subtitle={Continent plastique}]
\textit{Les quantités évoqués dans cette exercice sont générés au hasard et sont donc complètement farfelus.}
\medskip
Le \og continent de plastique\fg{} est la plus grande des plaques de déchets plastiques évoluant sur les océans. Elle occupe actuellement dans l'océan Pacifique une surface dont l'aire est évaluée à plus de $1,6$ million de km$^2$, entre Hawaï et la Californie.
En 2017, des scientifiques ont estimé qu'il y avait $12$ millions de tonnes de déchets plastiques qui était déversé chaque année dans les océans et que cette quantité augmentait de $26\n\%$ par chaque année.
On modélise l'évolution de la masse de ces déchets plastiques déversée chaque année, si rien n'est fait pour la réduire, par une suite géométrique $\left(u_n\right)$. L'arrondi au centième du terme $u_n$ représente la masse de ces déchets déversée chaque année, exprimée en million de tonnes, pour l'année $(2017 + n)$.
\medskip
\begin{enumerate}
\item Expliquer pourquoi la suite $u_n$ est géométrique?
\item Calculer $u_1$ et $u_2$.
\item Exprimer $u_n$ en fonction de $n$.
\item Au début de l'année 2017, il y avait $300$ millions de tonnes de déchets plastique. Calculer la quantité totale de déchets plastiques en 2030.
\item On souhaite déterminer en quelle année la masse totale de ces déchets plastiques aura pour la première fois augmenté de $50$\,\% par rapport à sa valeur de 2017.
\begin{enumerate}
\item Recopier et compléter l'algorithme ci-dessous pour que la variable $N$ contienne la réponse au problème posé.
\begin{center}
\begin{tabularx}{0.4\linewidth}{|X|}\hline
$N = 2017$\\
$U = 12$ \\
$S = 300 + U$ \\
while $S < 450$: \\
\hspace{1cm} $N = \ldots$\\
\hspace{1cm} $U = \ldots$\\
\hspace{1cm} $S = \ldots$\\
\hline
\end{tabularx}
\end{center}
\item Que contiennent les variables $S$, $U$ et $N$ après exécution de cet algorithme ?
Interpréter les résultats dans le contexte de l'exercice.
\end{enumerate}
\end{enumerate}
\end{exercise}
\begin{solution}
\begin{enumerate}
\item Une augmentation de $26\,\%$ revient à multiplier la quantité par $1.26$. La suite est donc bien géométrique. Son premier terme est $u_0 = 12$ et sa raison est $q = 1.26$
\item
\[
u_1 = u_0 * 1.26 = 15.120000000000001
\]
\[
u_2 = u_0 * 1.26^2 = 19.0512
\]
\item
\[
u_n = u_0 \times q^n = 12 \times 1.26^n
\]
\item On calcule la quantité totale déversée entre 2017 et 2030.
\[
\sum_{n = 0}^{13} u_n = u_0 \times \frac{1-q^{13}}{1-q} = 12 \times \frac{1 - 1.26^{13}}{1 - 1.26} = 885.01
\]
On en déduit la quantité totale de déchets en 2030
\[
300 + 885.01 = 1185.01
\]
\item
\begin{enumerate}
\item ~
\begin{center}
\begin{tabularx}{0.4\linewidth}{|X|}\hline
$N \gets 2017$\\
$U \gets 12$ \\
$S \gets 300 + U$ \\
Tant que $S < 450$ \\
\hspace{1cm} $N \gets N + 1$\\
\hspace{1cm} $U \gets U * 1.26$\\
\hspace{1cm} $S \gets S + u$\\
Fin Tant que\\\hline
\end{tabularx}
\end{center}
\item \textit{Pas de correction automatisé}
\end{enumerate}
\end{enumerate}
\end{solution}
\end{document}
%%% Local Variables:
%%% mode: latex
%%% TeX-master: "master"
%%% End:

View File

@ -0,0 +1,262 @@
\documentclass[a4paper,10pt]{article}
\usepackage{myXsim}
% Title Page
\title{DS8 \hfill MAGRO Robin}
\tribe{TST}
\date{\hfillÀ render pour le Mercredi 7 avril}
\xsimsetup{
solution/print = true
}
\begin{document}
\maketitle
\begin{exercise}[subtitle={Automatismes}]
\textit{Toutes les questions de cette exercice sont indépendantes et peuvent être répondus séparément}
\begin{enumerate}
\item De janvier à septembre, une quantité a augmenté de $11\,\%$. Faire un schéma pour représenter la situation puis calculer le taux d'évolution moyen mensuel.
\item Une quantité augmente de $11\,\%$ par ans. En 2020, elle est de 143\euro. Quelle était sa valeur en 2019? Faire un schéma pour représenter la situation.
\item Déterminer l'équation de la droite \\
\begin{tikzpicture}[xscale=0.8, yscale=0.5]
\tkzInit[xmin=-5,xmax=5,xstep=1,
ymin=-5,ymax=5,ystep=1]
\tkzGrid
\tkzAxeXY
\tkzFct[domain=-5:5,color=red,very thick]%
{3.0*\x -3};
\end{tikzpicture}
\item Résoudre l'équation $5 \times 0.12^x = 40$
\end{enumerate}
\end{exercise}
\begin{solution}
\begin{enumerate}
\item On veut partager cette évolution en 8 évolutions.
\[
\left(1 + \frac{11}{100}\right)^{\frac{1}{8}} = 1.0131
\]
Donc le taux d'évolution moyen est
\[
t_m = 1.0131 - 1 = 0.01309999999999989
\]
\item Coefficient multiplicateur pour revenir en arrière
\[
CM = (1 + \frac{11}{100})^{-1} = 0.9009
\]
On en déduit la quantité en 2019
\[
143 * 0.9009 = 128.8287
\]
\item L'équation de la droite est
\[
y = 3.0 x -3
\]
\item Il faut penser à faire la division à par $5$ avant d'utiliser le log car sinon, on ne peut pas utiliser la formule $\log(a^n) = n\times \log(a)$.
\[x = \frac{\log(8.0)}{\log(0.12)}\]
\end{enumerate}
\end{solution}
\begin{exercise}[subtitle={Restaurant}]
Un \emph{food truck}, ouvert le midi et le soir, propose deux types de formules :
\setlength\parindent{10mm}
\begin{itemize}
\item la formule \emph{Burger} ;
\item la formule \emph{Wok}.
\end{itemize}
\setlength\parindent{0mm}
\medskip
Le gérant a remarqué que 37\,\% de ses ventes ont lieu le midi. Le quart des ventes du midi correspondent à la formule \emph{Burger}, alors que 32\,\% des ventes du soir correspondent à la formule \emph{Wok}.
Le gérant se constitue un fichier en notant, pour chaque vente, la formule choisie et le moment de cette vente (midi ou soir).
On prélève une fiche de façon équiprobable. On définit les quatre évènements suivants:
\begin{enumerate}
\item $M$ : \og la fiche correspond à une vente du midi\fg{} ;
\item $S$ : \og la fiche correspond à une vente du soir\fg {};
\item $W$ : \og la fiche correspond à une formule \emph{Wok} \fg{} ;
\item $B$ : \og la fiche correspond à une formule \emph{Burger} \fg.
\end{enumerate}
\setlength\parindent{0mm}
\medskip
\begin{enumerate}
\item Recopier puis compléter l'arbre pondéré
\begin{center}
\begin{tikzpicture}[sloped]
\node {.}
child {node {$M$}
child {node {$W$}
edge from parent
node[above] {...}
}
child {node {$B$}
edge from parent
node[above] {...}
}
edge from parent
node[above] {...}
}
child[missing] {}
child { node {$S$}
child {node {$W$}
edge from parent
node[above] {...}
}
child {node {$B$}
edge from parent
node[above] {...}
}
edge from parent
node[above] {...}
} ;
\end{tikzpicture}
\end{center}
\item Calculer la probabilité de l'évènement $M \cap W$. Interpréter ce résultat dans le contexte de l'exercice.
\item Montrer que la probabilité que la fiche choisie corresponde à une formule \emph{Burger} est égale à $0.5209$.
\item On a prélevé une fiche correspondant à la formule \emph{Burger}. Quelle est la probabilité, arrondie au millième, que la vente ait eu lieu le soir?
\end{enumerate}
\end{exercise}
\begin{solution}
\begin{enumerate}
\item
\begin{center}
\begin{tikzpicture}[sloped]
\node {.}
child {node {$M$}
child {node {$W$}
edge from parent
node[above] {$0.75$}
}
child {node {$B$}
edge from parent
node[above] {$0.25$}
}
edge from parent
node[above] {$0.37$}
}
child[missing] {}
child { node {$S$}
child {node {$W$}
edge from parent
node[above] {$0.32$}
}
child {node {$B$}
edge from parent
node[above] {$0.68$}
}
edge from parent
node[above] {$0.63$}
} ;
\end{tikzpicture}
\end{center}
\item On calcule la probabilité que la vente soit un wok et ait eu lieu à midi
\[ P(M\cap W) = P(M) \times P_M(W) = 0.37 \times 0.75 = 0.2775 \]
\item Probabilité que la vente soit un burger.
\[
P(B) = P(M\cap B) + P(S\cap B) = 0.37 \times 0.75 + 0.63 \times 0.32 = 0.5209
\]
\item On cherche à calculer la quantité $P_B(S)$. Pour cela on utilise la formule de Bayes
\[
P_B(S) = \frac{P(B\cap S)}{P(B)} = \frac{P_S(B) \times P(S)}{P(B)} = \frac{0.68\times 0.63}{0.5209} = 0.8224227298905741 \approx 0.822
\]
\end{enumerate}
\end{solution}
\begin{exercise}[subtitle={Continent plastique}]
\textit{Les quantités évoqués dans cette exercice sont générés au hasard et sont donc complètement farfelus.}
\medskip
Le \og continent de plastique\fg{} est la plus grande des plaques de déchets plastiques évoluant sur les océans. Elle occupe actuellement dans l'océan Pacifique une surface dont l'aire est évaluée à plus de $1,6$ million de km$^2$, entre Hawaï et la Californie.
En 2017, des scientifiques ont estimé qu'il y avait $16$ millions de tonnes de déchets plastiques qui était déversé chaque année dans les océans et que cette quantité augmentait de $17\n\%$ par chaque année.
On modélise l'évolution de la masse de ces déchets plastiques déversée chaque année, si rien n'est fait pour la réduire, par une suite géométrique $\left(u_n\right)$. L'arrondi au centième du terme $u_n$ représente la masse de ces déchets déversée chaque année, exprimée en million de tonnes, pour l'année $(2017 + n)$.
\medskip
\begin{enumerate}
\item Expliquer pourquoi la suite $u_n$ est géométrique?
\item Calculer $u_1$ et $u_2$.
\item Exprimer $u_n$ en fonction de $n$.
\item Au début de l'année 2017, il y avait $300$ millions de tonnes de déchets plastique. Calculer la quantité totale de déchets plastiques en 2030.
\item On souhaite déterminer en quelle année la masse totale de ces déchets plastiques aura pour la première fois augmenté de $50$\,\% par rapport à sa valeur de 2017.
\begin{enumerate}
\item Recopier et compléter l'algorithme ci-dessous pour que la variable $N$ contienne la réponse au problème posé.
\begin{center}
\begin{tabularx}{0.4\linewidth}{|X|}\hline
$N = 2017$\\
$U = 16$ \\
$S = 300 + U$ \\
while $S < 450$: \\
\hspace{1cm} $N = \ldots$\\
\hspace{1cm} $U = \ldots$\\
\hspace{1cm} $S = \ldots$\\
\hline
\end{tabularx}
\end{center}
\item Que contiennent les variables $S$, $U$ et $N$ après exécution de cet algorithme ?
Interpréter les résultats dans le contexte de l'exercice.
\end{enumerate}
\end{enumerate}
\end{exercise}
\begin{solution}
\begin{enumerate}
\item Une augmentation de $17\,\%$ revient à multiplier la quantité par $1.17$. La suite est donc bien géométrique. Son premier terme est $u_0 = 16$ et sa raison est $q = 1.17$
\item
\[
u_1 = u_0 * 1.17 = 18.72
\]
\[
u_2 = u_0 * 1.17^2 = 21.9024
\]
\item
\[
u_n = u_0 \times q^n = 16 \times 1.17^n
\]
\item On calcule la quantité totale déversée entre 2017 et 2030.
\[
\sum_{n = 0}^{13} u_n = u_0 \times \frac{1-q^{13}}{1-q} = 16 \times \frac{1 - 1.17^{13}}{1 - 1.17} = 630.46
\]
On en déduit la quantité totale de déchets en 2030
\[
300 + 630.46 = 930.46
\]
\item
\begin{enumerate}
\item ~
\begin{center}
\begin{tabularx}{0.4\linewidth}{|X|}\hline
$N \gets 2017$\\
$U \gets 16$ \\
$S \gets 300 + U$ \\
Tant que $S < 450$ \\
\hspace{1cm} $N \gets N + 1$\\
\hspace{1cm} $U \gets U * 1.17$\\
\hspace{1cm} $S \gets S + u$\\
Fin Tant que\\\hline
\end{tabularx}
\end{center}
\item \textit{Pas de correction automatisé}
\end{enumerate}
\end{enumerate}
\end{solution}
\end{document}
%%% Local Variables:
%%% mode: latex
%%% TeX-master: "master"
%%% End:

View File

@ -0,0 +1,262 @@
\documentclass[a4paper,10pt]{article}
\usepackage{myXsim}
% Title Page
\title{DS8 \hfill MORFIN Chloé}
\tribe{TST}
\date{\hfillÀ render pour le Mercredi 7 avril}
\xsimsetup{
solution/print = true
}
\begin{document}
\maketitle
\begin{exercise}[subtitle={Automatismes}]
\textit{Toutes les questions de cette exercice sont indépendantes et peuvent être répondus séparément}
\begin{enumerate}
\item De janvier à septembre, une quantité a augmenté de $18\,\%$. Faire un schéma pour représenter la situation puis calculer le taux d'évolution moyen mensuel.
\item Une quantité augmente de $18\,\%$ par ans. En 2020, elle est de 119\euro. Quelle était sa valeur en 2019? Faire un schéma pour représenter la situation.
\item Déterminer l'équation de la droite \\
\begin{tikzpicture}[xscale=0.8, yscale=0.5]
\tkzInit[xmin=-5,xmax=5,xstep=1,
ymin=-5,ymax=5,ystep=1]
\tkzGrid
\tkzAxeXY
\tkzFct[domain=-5:5,color=red,very thick]%
{4.0*\x -4};
\end{tikzpicture}
\item Résoudre l'équation $3 \times 1.0^x = 32$
\end{enumerate}
\end{exercise}
\begin{solution}
\begin{enumerate}
\item On veut partager cette évolution en 8 évolutions.
\[
\left(1 + \frac{18}{100}\right)^{\frac{1}{8}} = 1.0209
\]
Donc le taux d'évolution moyen est
\[
t_m = 1.0209 - 1 = 0.02089999999999992
\]
\item Coefficient multiplicateur pour revenir en arrière
\[
CM = (1 + \frac{18}{100})^{-1} = 0.8475
\]
On en déduit la quantité en 2019
\[
119 * 0.8475 = 100.8525
\]
\item L'équation de la droite est
\[
y = 4.0 x -4
\]
\item Il faut penser à faire la division à par $3$ avant d'utiliser le log car sinon, on ne peut pas utiliser la formule $\log(a^n) = n\times \log(a)$.
\[x = \frac{\log(10.67)}{\log(1.0)}\]
\end{enumerate}
\end{solution}
\begin{exercise}[subtitle={Restaurant}]
Un \emph{food truck}, ouvert le midi et le soir, propose deux types de formules :
\setlength\parindent{10mm}
\begin{itemize}
\item la formule \emph{Burger} ;
\item la formule \emph{Wok}.
\end{itemize}
\setlength\parindent{0mm}
\medskip
Le gérant a remarqué que 24\,\% de ses ventes ont lieu le midi. Le quart des ventes du midi correspondent à la formule \emph{Burger}, alors que 57\,\% des ventes du soir correspondent à la formule \emph{Wok}.
Le gérant se constitue un fichier en notant, pour chaque vente, la formule choisie et le moment de cette vente (midi ou soir).
On prélève une fiche de façon équiprobable. On définit les quatre évènements suivants:
\begin{enumerate}
\item $M$ : \og la fiche correspond à une vente du midi\fg{} ;
\item $S$ : \og la fiche correspond à une vente du soir\fg {};
\item $W$ : \og la fiche correspond à une formule \emph{Wok} \fg{} ;
\item $B$ : \og la fiche correspond à une formule \emph{Burger} \fg.
\end{enumerate}
\setlength\parindent{0mm}
\medskip
\begin{enumerate}
\item Recopier puis compléter l'arbre pondéré
\begin{center}
\begin{tikzpicture}[sloped]
\node {.}
child {node {$M$}
child {node {$W$}
edge from parent
node[above] {...}
}
child {node {$B$}
edge from parent
node[above] {...}
}
edge from parent
node[above] {...}
}
child[missing] {}
child { node {$S$}
child {node {$W$}
edge from parent
node[above] {...}
}
child {node {$B$}
edge from parent
node[above] {...}
}
edge from parent
node[above] {...}
} ;
\end{tikzpicture}
\end{center}
\item Calculer la probabilité de l'évènement $M \cap W$. Interpréter ce résultat dans le contexte de l'exercice.
\item Montrer que la probabilité que la fiche choisie corresponde à une formule \emph{Burger} est égale à $0.3792$.
\item On a prélevé une fiche correspondant à la formule \emph{Burger}. Quelle est la probabilité, arrondie au millième, que la vente ait eu lieu le soir?
\end{enumerate}
\end{exercise}
\begin{solution}
\begin{enumerate}
\item
\begin{center}
\begin{tikzpicture}[sloped]
\node {.}
child {node {$M$}
child {node {$W$}
edge from parent
node[above] {$0.75$}
}
child {node {$B$}
edge from parent
node[above] {$0.25$}
}
edge from parent
node[above] {$0.24$}
}
child[missing] {}
child { node {$S$}
child {node {$W$}
edge from parent
node[above] {$0.58$}
}
child {node {$B$}
edge from parent
node[above] {$0.42$}
}
edge from parent
node[above] {$0.76$}
} ;
\end{tikzpicture}
\end{center}
\item On calcule la probabilité que la vente soit un wok et ait eu lieu à midi
\[ P(M\cap W) = P(M) \times P_M(W) = 0.24 \times 0.75 = 0.18 \]
\item Probabilité que la vente soit un burger.
\[
P(B) = P(M\cap B) + P(S\cap B) = 0.24 \times 0.75 + 0.76 \times 0.58 = 0.3792
\]
\item On cherche à calculer la quantité $P_B(S)$. Pour cela on utilise la formule de Bayes
\[
P_B(S) = \frac{P(B\cap S)}{P(B)} = \frac{P_S(B) \times P(S)}{P(B)} = \frac{0.42\times 0.76}{0.3792} = 0.8417721518987342 \approx 0.842
\]
\end{enumerate}
\end{solution}
\begin{exercise}[subtitle={Continent plastique}]
\textit{Les quantités évoqués dans cette exercice sont générés au hasard et sont donc complètement farfelus.}
\medskip
Le \og continent de plastique\fg{} est la plus grande des plaques de déchets plastiques évoluant sur les océans. Elle occupe actuellement dans l'océan Pacifique une surface dont l'aire est évaluée à plus de $1,6$ million de km$^2$, entre Hawaï et la Californie.
En 2017, des scientifiques ont estimé qu'il y avait $17$ millions de tonnes de déchets plastiques qui était déversé chaque année dans les océans et que cette quantité augmentait de $16\n\%$ par chaque année.
On modélise l'évolution de la masse de ces déchets plastiques déversée chaque année, si rien n'est fait pour la réduire, par une suite géométrique $\left(u_n\right)$. L'arrondi au centième du terme $u_n$ représente la masse de ces déchets déversée chaque année, exprimée en million de tonnes, pour l'année $(2017 + n)$.
\medskip
\begin{enumerate}
\item Expliquer pourquoi la suite $u_n$ est géométrique?
\item Calculer $u_1$ et $u_2$.
\item Exprimer $u_n$ en fonction de $n$.
\item Au début de l'année 2017, il y avait $300$ millions de tonnes de déchets plastique. Calculer la quantité totale de déchets plastiques en 2030.
\item On souhaite déterminer en quelle année la masse totale de ces déchets plastiques aura pour la première fois augmenté de $50$\,\% par rapport à sa valeur de 2017.
\begin{enumerate}
\item Recopier et compléter l'algorithme ci-dessous pour que la variable $N$ contienne la réponse au problème posé.
\begin{center}
\begin{tabularx}{0.4\linewidth}{|X|}\hline
$N = 2017$\\
$U = 17$ \\
$S = 300 + U$ \\
while $S < 450$: \\
\hspace{1cm} $N = \ldots$\\
\hspace{1cm} $U = \ldots$\\
\hspace{1cm} $S = \ldots$\\
\hline
\end{tabularx}
\end{center}
\item Que contiennent les variables $S$, $U$ et $N$ après exécution de cet algorithme ?
Interpréter les résultats dans le contexte de l'exercice.
\end{enumerate}
\end{enumerate}
\end{exercise}
\begin{solution}
\begin{enumerate}
\item Une augmentation de $16\,\%$ revient à multiplier la quantité par $1.16$. La suite est donc bien géométrique. Son premier terme est $u_0 = 17$ et sa raison est $q = 1.16$
\item
\[
u_1 = u_0 * 1.16 = 19.72
\]
\[
u_2 = u_0 * 1.16^2 = 22.8752
\]
\item
\[
u_n = u_0 \times q^n = 17 \times 1.16^n
\]
\item On calcule la quantité totale déversée entre 2017 et 2030.
\[
\sum_{n = 0}^{13} u_n = u_0 \times \frac{1-q^{13}}{1-q} = 17 \times \frac{1 - 1.16^{13}}{1 - 1.16} = 625.37
\]
On en déduit la quantité totale de déchets en 2030
\[
300 + 625.37 = 925.37
\]
\item
\begin{enumerate}
\item ~
\begin{center}
\begin{tabularx}{0.4\linewidth}{|X|}\hline
$N \gets 2017$\\
$U \gets 17$ \\
$S \gets 300 + U$ \\
Tant que $S < 450$ \\
\hspace{1cm} $N \gets N + 1$\\
\hspace{1cm} $U \gets U * 1.16$\\
\hspace{1cm} $S \gets S + u$\\
Fin Tant que\\\hline
\end{tabularx}
\end{center}
\item \textit{Pas de correction automatisé}
\end{enumerate}
\end{enumerate}
\end{solution}
\end{document}
%%% Local Variables:
%%% mode: latex
%%% TeX-master: "master"
%%% End:

View File

@ -0,0 +1,262 @@
\documentclass[a4paper,10pt]{article}
\usepackage{myXsim}
% Title Page
\title{DS8 \hfill PERES RAMALHO Emeric}
\tribe{TST}
\date{\hfillÀ render pour le Mercredi 7 avril}
\xsimsetup{
solution/print = true
}
\begin{document}
\maketitle
\begin{exercise}[subtitle={Automatismes}]
\textit{Toutes les questions de cette exercice sont indépendantes et peuvent être répondus séparément}
\begin{enumerate}
\item De janvier à septembre, une quantité a augmenté de $23\,\%$. Faire un schéma pour représenter la situation puis calculer le taux d'évolution moyen mensuel.
\item Une quantité augmente de $23\,\%$ par ans. En 2020, elle est de 118\euro. Quelle était sa valeur en 2019? Faire un schéma pour représenter la situation.
\item Déterminer l'équation de la droite \\
\begin{tikzpicture}[xscale=0.8, yscale=0.5]
\tkzInit[xmin=-5,xmax=5,xstep=1,
ymin=-5,ymax=5,ystep=1]
\tkzGrid
\tkzAxeXY
\tkzFct[domain=-5:5,color=red,very thick]%
{1.3333333333333333*\x -2};
\end{tikzpicture}
\item Résoudre l'équation $5 \times 0.46^x = 2$
\end{enumerate}
\end{exercise}
\begin{solution}
\begin{enumerate}
\item On veut partager cette évolution en 8 évolutions.
\[
\left(1 + \frac{23}{100}\right)^{\frac{1}{8}} = 1.0262
\]
Donc le taux d'évolution moyen est
\[
t_m = 1.0262 - 1 = 0.0262
\]
\item Coefficient multiplicateur pour revenir en arrière
\[
CM = (1 + \frac{23}{100})^{-1} = 0.813
\]
On en déduit la quantité en 2019
\[
118 * 0.813 = 95.934
\]
\item L'équation de la droite est
\[
y = 1.3333333333333333 x -2
\]
\item Il faut penser à faire la division à par $5$ avant d'utiliser le log car sinon, on ne peut pas utiliser la formule $\log(a^n) = n\times \log(a)$.
\[x = \frac{\log(0.4)}{\log(0.46)}\]
\end{enumerate}
\end{solution}
\begin{exercise}[subtitle={Restaurant}]
Un \emph{food truck}, ouvert le midi et le soir, propose deux types de formules :
\setlength\parindent{10mm}
\begin{itemize}
\item la formule \emph{Burger} ;
\item la formule \emph{Wok}.
\end{itemize}
\setlength\parindent{0mm}
\medskip
Le gérant a remarqué que 11\,\% de ses ventes ont lieu le midi. Le quart des ventes du midi correspondent à la formule \emph{Burger}, alors que 18\,\% des ventes du soir correspondent à la formule \emph{Wok}.
Le gérant se constitue un fichier en notant, pour chaque vente, la formule choisie et le moment de cette vente (midi ou soir).
On prélève une fiche de façon équiprobable. On définit les quatre évènements suivants:
\begin{enumerate}
\item $M$ : \og la fiche correspond à une vente du midi\fg{} ;
\item $S$ : \og la fiche correspond à une vente du soir\fg {};
\item $W$ : \og la fiche correspond à une formule \emph{Wok} \fg{} ;
\item $B$ : \og la fiche correspond à une formule \emph{Burger} \fg.
\end{enumerate}
\setlength\parindent{0mm}
\medskip
\begin{enumerate}
\item Recopier puis compléter l'arbre pondéré
\begin{center}
\begin{tikzpicture}[sloped]
\node {.}
child {node {$M$}
child {node {$W$}
edge from parent
node[above] {...}
}
child {node {$B$}
edge from parent
node[above] {...}
}
edge from parent
node[above] {...}
}
child[missing] {}
child { node {$S$}
child {node {$W$}
edge from parent
node[above] {...}
}
child {node {$B$}
edge from parent
node[above] {...}
}
edge from parent
node[above] {...}
} ;
\end{tikzpicture}
\end{center}
\item Calculer la probabilité de l'évènement $M \cap W$. Interpréter ce résultat dans le contexte de l'exercice.
\item Montrer que la probabilité que la fiche choisie corresponde à une formule \emph{Burger} est égale à $0.7573$.
\item On a prélevé une fiche correspondant à la formule \emph{Burger}. Quelle est la probabilité, arrondie au millième, que la vente ait eu lieu le soir?
\end{enumerate}
\end{exercise}
\begin{solution}
\begin{enumerate}
\item
\begin{center}
\begin{tikzpicture}[sloped]
\node {.}
child {node {$M$}
child {node {$W$}
edge from parent
node[above] {$0.75$}
}
child {node {$B$}
edge from parent
node[above] {$0.25$}
}
edge from parent
node[above] {$0.11$}
}
child[missing] {}
child { node {$S$}
child {node {$W$}
edge from parent
node[above] {$0.18$}
}
child {node {$B$}
edge from parent
node[above] {$0.82$}
}
edge from parent
node[above] {$0.89$}
} ;
\end{tikzpicture}
\end{center}
\item On calcule la probabilité que la vente soit un wok et ait eu lieu à midi
\[ P(M\cap W) = P(M) \times P_M(W) = 0.11 \times 0.75 = 0.0825 \]
\item Probabilité que la vente soit un burger.
\[
P(B) = P(M\cap B) + P(S\cap B) = 0.11 \times 0.75 + 0.89 \times 0.18 = 0.7573
\]
\item On cherche à calculer la quantité $P_B(S)$. Pour cela on utilise la formule de Bayes
\[
P_B(S) = \frac{P(B\cap S)}{P(B)} = \frac{P_S(B) \times P(S)}{P(B)} = \frac{0.82\times 0.89}{0.7573} = 0.9636867819886439 \approx 0.964
\]
\end{enumerate}
\end{solution}
\begin{exercise}[subtitle={Continent plastique}]
\textit{Les quantités évoqués dans cette exercice sont générés au hasard et sont donc complètement farfelus.}
\medskip
Le \og continent de plastique\fg{} est la plus grande des plaques de déchets plastiques évoluant sur les océans. Elle occupe actuellement dans l'océan Pacifique une surface dont l'aire est évaluée à plus de $1,6$ million de km$^2$, entre Hawaï et la Californie.
En 2017, des scientifiques ont estimé qu'il y avait $19$ millions de tonnes de déchets plastiques qui était déversé chaque année dans les océans et que cette quantité augmentait de $22\n\%$ par chaque année.
On modélise l'évolution de la masse de ces déchets plastiques déversée chaque année, si rien n'est fait pour la réduire, par une suite géométrique $\left(u_n\right)$. L'arrondi au centième du terme $u_n$ représente la masse de ces déchets déversée chaque année, exprimée en million de tonnes, pour l'année $(2017 + n)$.
\medskip
\begin{enumerate}
\item Expliquer pourquoi la suite $u_n$ est géométrique?
\item Calculer $u_1$ et $u_2$.
\item Exprimer $u_n$ en fonction de $n$.
\item Au début de l'année 2017, il y avait $300$ millions de tonnes de déchets plastique. Calculer la quantité totale de déchets plastiques en 2030.
\item On souhaite déterminer en quelle année la masse totale de ces déchets plastiques aura pour la première fois augmenté de $50$\,\% par rapport à sa valeur de 2017.
\begin{enumerate}
\item Recopier et compléter l'algorithme ci-dessous pour que la variable $N$ contienne la réponse au problème posé.
\begin{center}
\begin{tabularx}{0.4\linewidth}{|X|}\hline
$N = 2017$\\
$U = 19$ \\
$S = 300 + U$ \\
while $S < 450$: \\
\hspace{1cm} $N = \ldots$\\
\hspace{1cm} $U = \ldots$\\
\hspace{1cm} $S = \ldots$\\
\hline
\end{tabularx}
\end{center}
\item Que contiennent les variables $S$, $U$ et $N$ après exécution de cet algorithme ?
Interpréter les résultats dans le contexte de l'exercice.
\end{enumerate}
\end{enumerate}
\end{exercise}
\begin{solution}
\begin{enumerate}
\item Une augmentation de $22\,\%$ revient à multiplier la quantité par $1.22$. La suite est donc bien géométrique. Son premier terme est $u_0 = 19$ et sa raison est $q = 1.22$
\item
\[
u_1 = u_0 * 1.22 = 23.18
\]
\[
u_2 = u_0 * 1.22^2 = 28.2796
\]
\item
\[
u_n = u_0 \times q^n = 19 \times 1.22^n
\]
\item On calcule la quantité totale déversée entre 2017 et 2030.
\[
\sum_{n = 0}^{13} u_n = u_0 \times \frac{1-q^{13}}{1-q} = 19 \times \frac{1 - 1.22^{13}}{1 - 1.22} = 1059.17
\]
On en déduit la quantité totale de déchets en 2030
\[
300 + 1059.17 = 1359.17
\]
\item
\begin{enumerate}
\item ~
\begin{center}
\begin{tabularx}{0.4\linewidth}{|X|}\hline
$N \gets 2017$\\
$U \gets 19$ \\
$S \gets 300 + U$ \\
Tant que $S < 450$ \\
\hspace{1cm} $N \gets N + 1$\\
\hspace{1cm} $U \gets U * 1.22$\\
\hspace{1cm} $S \gets S + u$\\
Fin Tant que\\\hline
\end{tabularx}
\end{center}
\item \textit{Pas de correction automatisé}
\end{enumerate}
\end{enumerate}
\end{solution}
\end{document}
%%% Local Variables:
%%% mode: latex
%%% TeX-master: "master"
%%% End:

View File

@ -0,0 +1,262 @@
\documentclass[a4paper,10pt]{article}
\usepackage{myXsim}
% Title Page
\title{DS8 \hfill RADOUAA Saleh}
\tribe{TST}
\date{\hfillÀ render pour le Mercredi 7 avril}
\xsimsetup{
solution/print = true
}
\begin{document}
\maketitle
\begin{exercise}[subtitle={Automatismes}]
\textit{Toutes les questions de cette exercice sont indépendantes et peuvent être répondus séparément}
\begin{enumerate}
\item De janvier à septembre, une quantité a augmenté de $27\,\%$. Faire un schéma pour représenter la situation puis calculer le taux d'évolution moyen mensuel.
\item Une quantité augmente de $27\,\%$ par ans. En 2020, elle est de 141\euro. Quelle était sa valeur en 2019? Faire un schéma pour représenter la situation.
\item Déterminer l'équation de la droite \\
\begin{tikzpicture}[xscale=0.8, yscale=0.5]
\tkzInit[xmin=-5,xmax=5,xstep=1,
ymin=-5,ymax=5,ystep=1]
\tkzGrid
\tkzAxeXY
\tkzFct[domain=-5:5,color=red,very thick]%
{2.0*\x -4};
\end{tikzpicture}
\item Résoudre l'équation $2 \times 0.88^x = 8$
\end{enumerate}
\end{exercise}
\begin{solution}
\begin{enumerate}
\item On veut partager cette évolution en 8 évolutions.
\[
\left(1 + \frac{27}{100}\right)^{\frac{1}{8}} = 1.0303
\]
Donc le taux d'évolution moyen est
\[
t_m = 1.0303 - 1 = 0.030299999999999994
\]
\item Coefficient multiplicateur pour revenir en arrière
\[
CM = (1 + \frac{27}{100})^{-1} = 0.7874
\]
On en déduit la quantité en 2019
\[
141 * 0.7874 = 111.0234
\]
\item L'équation de la droite est
\[
y = 2.0 x -4
\]
\item Il faut penser à faire la division à par $2$ avant d'utiliser le log car sinon, on ne peut pas utiliser la formule $\log(a^n) = n\times \log(a)$.
\[x = \frac{\log(4.0)}{\log(0.88)}\]
\end{enumerate}
\end{solution}
\begin{exercise}[subtitle={Restaurant}]
Un \emph{food truck}, ouvert le midi et le soir, propose deux types de formules :
\setlength\parindent{10mm}
\begin{itemize}
\item la formule \emph{Burger} ;
\item la formule \emph{Wok}.
\end{itemize}
\setlength\parindent{0mm}
\medskip
Le gérant a remarqué que 1\,\% de ses ventes ont lieu le midi. Le quart des ventes du midi correspondent à la formule \emph{Burger}, alors que 99\,\% des ventes du soir correspondent à la formule \emph{Wok}.
Le gérant se constitue un fichier en notant, pour chaque vente, la formule choisie et le moment de cette vente (midi ou soir).
On prélève une fiche de façon équiprobable. On définit les quatre évènements suivants:
\begin{enumerate}
\item $M$ : \og la fiche correspond à une vente du midi\fg{} ;
\item $S$ : \og la fiche correspond à une vente du soir\fg {};
\item $W$ : \og la fiche correspond à une formule \emph{Wok} \fg{} ;
\item $B$ : \og la fiche correspond à une formule \emph{Burger} \fg.
\end{enumerate}
\setlength\parindent{0mm}
\medskip
\begin{enumerate}
\item Recopier puis compléter l'arbre pondéré
\begin{center}
\begin{tikzpicture}[sloped]
\node {.}
child {node {$M$}
child {node {$W$}
edge from parent
node[above] {...}
}
child {node {$B$}
edge from parent
node[above] {...}
}
edge from parent
node[above] {...}
}
child[missing] {}
child { node {$S$}
child {node {$W$}
edge from parent
node[above] {...}
}
child {node {$B$}
edge from parent
node[above] {...}
}
edge from parent
node[above] {...}
} ;
\end{tikzpicture}
\end{center}
\item Calculer la probabilité de l'évènement $M \cap W$. Interpréter ce résultat dans le contexte de l'exercice.
\item Montrer que la probabilité que la fiche choisie corresponde à une formule \emph{Burger} est égale à $0.0124$.
\item On a prélevé une fiche correspondant à la formule \emph{Burger}. Quelle est la probabilité, arrondie au millième, que la vente ait eu lieu le soir?
\end{enumerate}
\end{exercise}
\begin{solution}
\begin{enumerate}
\item
\begin{center}
\begin{tikzpicture}[sloped]
\node {.}
child {node {$M$}
child {node {$W$}
edge from parent
node[above] {$0.75$}
}
child {node {$B$}
edge from parent
node[above] {$0.25$}
}
edge from parent
node[above] {$0.01$}
}
child[missing] {}
child { node {$S$}
child {node {$W$}
edge from parent
node[above] {$0.99$}
}
child {node {$B$}
edge from parent
node[above] {$0.01$}
}
edge from parent
node[above] {$0.99$}
} ;
\end{tikzpicture}
\end{center}
\item On calcule la probabilité que la vente soit un wok et ait eu lieu à midi
\[ P(M\cap W) = P(M) \times P_M(W) = 0.01 \times 0.75 = 0.0075 \]
\item Probabilité que la vente soit un burger.
\[
P(B) = P(M\cap B) + P(S\cap B) = 0.01 \times 0.75 + 0.99 \times 0.99 = 0.0124
\]
\item On cherche à calculer la quantité $P_B(S)$. Pour cela on utilise la formule de Bayes
\[
P_B(S) = \frac{P(B\cap S)}{P(B)} = \frac{P_S(B) \times P(S)}{P(B)} = \frac{0.01\times 0.99}{0.0124} = 0.7983870967741936 \approx 0.798
\]
\end{enumerate}
\end{solution}
\begin{exercise}[subtitle={Continent plastique}]
\textit{Les quantités évoqués dans cette exercice sont générés au hasard et sont donc complètement farfelus.}
\medskip
Le \og continent de plastique\fg{} est la plus grande des plaques de déchets plastiques évoluant sur les océans. Elle occupe actuellement dans l'océan Pacifique une surface dont l'aire est évaluée à plus de $1,6$ million de km$^2$, entre Hawaï et la Californie.
En 2017, des scientifiques ont estimé qu'il y avait $5$ millions de tonnes de déchets plastiques qui était déversé chaque année dans les océans et que cette quantité augmentait de $17\n\%$ par chaque année.
On modélise l'évolution de la masse de ces déchets plastiques déversée chaque année, si rien n'est fait pour la réduire, par une suite géométrique $\left(u_n\right)$. L'arrondi au centième du terme $u_n$ représente la masse de ces déchets déversée chaque année, exprimée en million de tonnes, pour l'année $(2017 + n)$.
\medskip
\begin{enumerate}
\item Expliquer pourquoi la suite $u_n$ est géométrique?
\item Calculer $u_1$ et $u_2$.
\item Exprimer $u_n$ en fonction de $n$.
\item Au début de l'année 2017, il y avait $300$ millions de tonnes de déchets plastique. Calculer la quantité totale de déchets plastiques en 2030.
\item On souhaite déterminer en quelle année la masse totale de ces déchets plastiques aura pour la première fois augmenté de $50$\,\% par rapport à sa valeur de 2017.
\begin{enumerate}
\item Recopier et compléter l'algorithme ci-dessous pour que la variable $N$ contienne la réponse au problème posé.
\begin{center}
\begin{tabularx}{0.4\linewidth}{|X|}\hline
$N = 2017$\\
$U = 5$ \\
$S = 300 + U$ \\
while $S < 450$: \\
\hspace{1cm} $N = \ldots$\\
\hspace{1cm} $U = \ldots$\\
\hspace{1cm} $S = \ldots$\\
\hline
\end{tabularx}
\end{center}
\item Que contiennent les variables $S$, $U$ et $N$ après exécution de cet algorithme ?
Interpréter les résultats dans le contexte de l'exercice.
\end{enumerate}
\end{enumerate}
\end{exercise}
\begin{solution}
\begin{enumerate}
\item Une augmentation de $17\,\%$ revient à multiplier la quantité par $1.17$. La suite est donc bien géométrique. Son premier terme est $u_0 = 5$ et sa raison est $q = 1.17$
\item
\[
u_1 = u_0 * 1.17 = 5.85
\]
\[
u_2 = u_0 * 1.17^2 = 6.8445
\]
\item
\[
u_n = u_0 \times q^n = 5 \times 1.17^n
\]
\item On calcule la quantité totale déversée entre 2017 et 2030.
\[
\sum_{n = 0}^{13} u_n = u_0 \times \frac{1-q^{13}}{1-q} = 5 \times \frac{1 - 1.17^{13}}{1 - 1.17} = 197.02
\]
On en déduit la quantité totale de déchets en 2030
\[
300 + 197.02 = 497.02
\]
\item
\begin{enumerate}
\item ~
\begin{center}
\begin{tabularx}{0.4\linewidth}{|X|}\hline
$N \gets 2017$\\
$U \gets 5$ \\
$S \gets 300 + U$ \\
Tant que $S < 450$ \\
\hspace{1cm} $N \gets N + 1$\\
\hspace{1cm} $U \gets U * 1.17$\\
\hspace{1cm} $S \gets S + u$\\
Fin Tant que\\\hline
\end{tabularx}
\end{center}
\item \textit{Pas de correction automatisé}
\end{enumerate}
\end{enumerate}
\end{solution}
\end{document}
%%% Local Variables:
%%% mode: latex
%%% TeX-master: "master"
%%% End:

View File

@ -0,0 +1,262 @@
\documentclass[a4paper,10pt]{article}
\usepackage{myXsim}
% Title Page
\title{DS8 \hfill TAY Ummuhan}
\tribe{TST}
\date{\hfillÀ render pour le Mercredi 7 avril}
\xsimsetup{
solution/print = true
}
\begin{document}
\maketitle
\begin{exercise}[subtitle={Automatismes}]
\textit{Toutes les questions de cette exercice sont indépendantes et peuvent être répondus séparément}
\begin{enumerate}
\item De janvier à septembre, une quantité a augmenté de $14\,\%$. Faire un schéma pour représenter la situation puis calculer le taux d'évolution moyen mensuel.
\item Une quantité augmente de $14\,\%$ par ans. En 2020, elle est de 116\euro. Quelle était sa valeur en 2019? Faire un schéma pour représenter la situation.
\item Déterminer l'équation de la droite \\
\begin{tikzpicture}[xscale=0.8, yscale=0.5]
\tkzInit[xmin=-5,xmax=5,xstep=1,
ymin=-5,ymax=5,ystep=1]
\tkzGrid
\tkzAxeXY
\tkzFct[domain=-5:5,color=red,very thick]%
{1.3333333333333333*\x -2};
\end{tikzpicture}
\item Résoudre l'équation $6 \times 0.14^x = 14$
\end{enumerate}
\end{exercise}
\begin{solution}
\begin{enumerate}
\item On veut partager cette évolution en 8 évolutions.
\[
\left(1 + \frac{14}{100}\right)^{\frac{1}{8}} = 1.0165
\]
Donc le taux d'évolution moyen est
\[
t_m = 1.0165 - 1 = 0.01649999999999996
\]
\item Coefficient multiplicateur pour revenir en arrière
\[
CM = (1 + \frac{14}{100})^{-1} = 0.8772
\]
On en déduit la quantité en 2019
\[
116 * 0.8772 = 101.7552
\]
\item L'équation de la droite est
\[
y = 1.3333333333333333 x -2
\]
\item Il faut penser à faire la division à par $6$ avant d'utiliser le log car sinon, on ne peut pas utiliser la formule $\log(a^n) = n\times \log(a)$.
\[x = \frac{\log(2.33)}{\log(0.14)}\]
\end{enumerate}
\end{solution}
\begin{exercise}[subtitle={Restaurant}]
Un \emph{food truck}, ouvert le midi et le soir, propose deux types de formules :
\setlength\parindent{10mm}
\begin{itemize}
\item la formule \emph{Burger} ;
\item la formule \emph{Wok}.
\end{itemize}
\setlength\parindent{0mm}
\medskip
Le gérant a remarqué que 28\,\% de ses ventes ont lieu le midi. Le quart des ventes du midi correspondent à la formule \emph{Burger}, alors que 80\,\% des ventes du soir correspondent à la formule \emph{Wok}.
Le gérant se constitue un fichier en notant, pour chaque vente, la formule choisie et le moment de cette vente (midi ou soir).
On prélève une fiche de façon équiprobable. On définit les quatre évènements suivants:
\begin{enumerate}
\item $M$ : \og la fiche correspond à une vente du midi\fg{} ;
\item $S$ : \og la fiche correspond à une vente du soir\fg {};
\item $W$ : \og la fiche correspond à une formule \emph{Wok} \fg{} ;
\item $B$ : \og la fiche correspond à une formule \emph{Burger} \fg.
\end{enumerate}
\setlength\parindent{0mm}
\medskip
\begin{enumerate}
\item Recopier puis compléter l'arbre pondéré
\begin{center}
\begin{tikzpicture}[sloped]
\node {.}
child {node {$M$}
child {node {$W$}
edge from parent
node[above] {...}
}
child {node {$B$}
edge from parent
node[above] {...}
}
edge from parent
node[above] {...}
}
child[missing] {}
child { node {$S$}
child {node {$W$}
edge from parent
node[above] {...}
}
child {node {$B$}
edge from parent
node[above] {...}
}
edge from parent
node[above] {...}
} ;
\end{tikzpicture}
\end{center}
\item Calculer la probabilité de l'évènement $M \cap W$. Interpréter ce résultat dans le contexte de l'exercice.
\item Montrer que la probabilité que la fiche choisie corresponde à une formule \emph{Burger} est égale à $0.214$.
\item On a prélevé une fiche correspondant à la formule \emph{Burger}. Quelle est la probabilité, arrondie au millième, que la vente ait eu lieu le soir?
\end{enumerate}
\end{exercise}
\begin{solution}
\begin{enumerate}
\item
\begin{center}
\begin{tikzpicture}[sloped]
\node {.}
child {node {$M$}
child {node {$W$}
edge from parent
node[above] {$0.75$}
}
child {node {$B$}
edge from parent
node[above] {$0.25$}
}
edge from parent
node[above] {$0.28$}
}
child[missing] {}
child { node {$S$}
child {node {$W$}
edge from parent
node[above] {$0.8$}
}
child {node {$B$}
edge from parent
node[above] {$0.2$}
}
edge from parent
node[above] {$0.72$}
} ;
\end{tikzpicture}
\end{center}
\item On calcule la probabilité que la vente soit un wok et ait eu lieu à midi
\[ P(M\cap W) = P(M) \times P_M(W) = 0.28 \times 0.75 = 0.21 \]
\item Probabilité que la vente soit un burger.
\[
P(B) = P(M\cap B) + P(S\cap B) = 0.28 \times 0.75 + 0.72 \times 0.8 = 0.214
\]
\item On cherche à calculer la quantité $P_B(S)$. Pour cela on utilise la formule de Bayes
\[
P_B(S) = \frac{P(B\cap S)}{P(B)} = \frac{P_S(B) \times P(S)}{P(B)} = \frac{0.2\times 0.72}{0.214} = 0.6728971962616822 \approx 0.673
\]
\end{enumerate}
\end{solution}
\begin{exercise}[subtitle={Continent plastique}]
\textit{Les quantités évoqués dans cette exercice sont générés au hasard et sont donc complètement farfelus.}
\medskip
Le \og continent de plastique\fg{} est la plus grande des plaques de déchets plastiques évoluant sur les océans. Elle occupe actuellement dans l'océan Pacifique une surface dont l'aire est évaluée à plus de $1,6$ million de km$^2$, entre Hawaï et la Californie.
En 2017, des scientifiques ont estimé qu'il y avait $7$ millions de tonnes de déchets plastiques qui était déversé chaque année dans les océans et que cette quantité augmentait de $26\n\%$ par chaque année.
On modélise l'évolution de la masse de ces déchets plastiques déversée chaque année, si rien n'est fait pour la réduire, par une suite géométrique $\left(u_n\right)$. L'arrondi au centième du terme $u_n$ représente la masse de ces déchets déversée chaque année, exprimée en million de tonnes, pour l'année $(2017 + n)$.
\medskip
\begin{enumerate}
\item Expliquer pourquoi la suite $u_n$ est géométrique?
\item Calculer $u_1$ et $u_2$.
\item Exprimer $u_n$ en fonction de $n$.
\item Au début de l'année 2017, il y avait $300$ millions de tonnes de déchets plastique. Calculer la quantité totale de déchets plastiques en 2030.
\item On souhaite déterminer en quelle année la masse totale de ces déchets plastiques aura pour la première fois augmenté de $50$\,\% par rapport à sa valeur de 2017.
\begin{enumerate}
\item Recopier et compléter l'algorithme ci-dessous pour que la variable $N$ contienne la réponse au problème posé.
\begin{center}
\begin{tabularx}{0.4\linewidth}{|X|}\hline
$N = 2017$\\
$U = 7$ \\
$S = 300 + U$ \\
while $S < 450$: \\
\hspace{1cm} $N = \ldots$\\
\hspace{1cm} $U = \ldots$\\
\hspace{1cm} $S = \ldots$\\
\hline
\end{tabularx}
\end{center}
\item Que contiennent les variables $S$, $U$ et $N$ après exécution de cet algorithme ?
Interpréter les résultats dans le contexte de l'exercice.
\end{enumerate}
\end{enumerate}
\end{exercise}
\begin{solution}
\begin{enumerate}
\item Une augmentation de $26\,\%$ revient à multiplier la quantité par $1.26$. La suite est donc bien géométrique. Son premier terme est $u_0 = 7$ et sa raison est $q = 1.26$
\item
\[
u_1 = u_0 * 1.26 = 8.82
\]
\[
u_2 = u_0 * 1.26^2 = 11.1132
\]
\item
\[
u_n = u_0 \times q^n = 7 \times 1.26^n
\]
\item On calcule la quantité totale déversée entre 2017 et 2030.
\[
\sum_{n = 0}^{13} u_n = u_0 \times \frac{1-q^{13}}{1-q} = 7 \times \frac{1 - 1.26^{13}}{1 - 1.26} = 516.25
\]
On en déduit la quantité totale de déchets en 2030
\[
300 + 516.25 = 816.25
\]
\item
\begin{enumerate}
\item ~
\begin{center}
\begin{tabularx}{0.4\linewidth}{|X|}\hline
$N \gets 2017$\\
$U \gets 7$ \\
$S \gets 300 + U$ \\
Tant que $S < 450$ \\
\hspace{1cm} $N \gets N + 1$\\
\hspace{1cm} $U \gets U * 1.26$\\
\hspace{1cm} $S \gets S + u$\\
Fin Tant que\\\hline
\end{tabularx}
\end{center}
\item \textit{Pas de correction automatisé}
\end{enumerate}
\end{enumerate}
\end{solution}
\end{document}
%%% Local Variables:
%%% mode: latex
%%% TeX-master: "master"
%%% End:

View File

@ -0,0 +1,262 @@
\documentclass[a4paper,10pt]{article}
\usepackage{myXsim}
% Title Page
\title{DS8 \hfill VIALON-DUPERRON Victorien}
\tribe{TST}
\date{\hfillÀ render pour le Mercredi 7 avril}
\xsimsetup{
solution/print = true
}
\begin{document}
\maketitle
\begin{exercise}[subtitle={Automatismes}]
\textit{Toutes les questions de cette exercice sont indépendantes et peuvent être répondus séparément}
\begin{enumerate}
\item De janvier à septembre, une quantité a augmenté de $20\,\%$. Faire un schéma pour représenter la situation puis calculer le taux d'évolution moyen mensuel.
\item Une quantité augmente de $20\,\%$ par ans. En 2020, elle est de 137\euro. Quelle était sa valeur en 2019? Faire un schéma pour représenter la situation.
\item Déterminer l'équation de la droite \\
\begin{tikzpicture}[xscale=0.8, yscale=0.5]
\tkzInit[xmin=-5,xmax=5,xstep=1,
ymin=-5,ymax=5,ystep=1]
\tkzGrid
\tkzAxeXY
\tkzFct[domain=-5:5,color=red,very thick]%
{2.6666666666666665*\x -4};
\end{tikzpicture}
\item Résoudre l'équation $10 \times 0.26^x = 42$
\end{enumerate}
\end{exercise}
\begin{solution}
\begin{enumerate}
\item On veut partager cette évolution en 8 évolutions.
\[
\left(1 + \frac{20}{100}\right)^{\frac{1}{8}} = 1.0231
\]
Donc le taux d'évolution moyen est
\[
t_m = 1.0231 - 1 = 0.0230999999999999
\]
\item Coefficient multiplicateur pour revenir en arrière
\[
CM = (1 + \frac{20}{100})^{-1} = 0.8333
\]
On en déduit la quantité en 2019
\[
137 * 0.8333 = 114.16210000000001
\]
\item L'équation de la droite est
\[
y = 2.6666666666666665 x -4
\]
\item Il faut penser à faire la division à par $10$ avant d'utiliser le log car sinon, on ne peut pas utiliser la formule $\log(a^n) = n\times \log(a)$.
\[x = \frac{\log(4.2)}{\log(0.26)}\]
\end{enumerate}
\end{solution}
\begin{exercise}[subtitle={Restaurant}]
Un \emph{food truck}, ouvert le midi et le soir, propose deux types de formules :
\setlength\parindent{10mm}
\begin{itemize}
\item la formule \emph{Burger} ;
\item la formule \emph{Wok}.
\end{itemize}
\setlength\parindent{0mm}
\medskip
Le gérant a remarqué que 87\,\% de ses ventes ont lieu le midi. Le quart des ventes du midi correspondent à la formule \emph{Burger}, alors que 47\,\% des ventes du soir correspondent à la formule \emph{Wok}.
Le gérant se constitue un fichier en notant, pour chaque vente, la formule choisie et le moment de cette vente (midi ou soir).
On prélève une fiche de façon équiprobable. On définit les quatre évènements suivants:
\begin{enumerate}
\item $M$ : \og la fiche correspond à une vente du midi\fg{} ;
\item $S$ : \og la fiche correspond à une vente du soir\fg {};
\item $W$ : \og la fiche correspond à une formule \emph{Wok} \fg{} ;
\item $B$ : \og la fiche correspond à une formule \emph{Burger} \fg.
\end{enumerate}
\setlength\parindent{0mm}
\medskip
\begin{enumerate}
\item Recopier puis compléter l'arbre pondéré
\begin{center}
\begin{tikzpicture}[sloped]
\node {.}
child {node {$M$}
child {node {$W$}
edge from parent
node[above] {...}
}
child {node {$B$}
edge from parent
node[above] {...}
}
edge from parent
node[above] {...}
}
child[missing] {}
child { node {$S$}
child {node {$W$}
edge from parent
node[above] {...}
}
child {node {$B$}
edge from parent
node[above] {...}
}
edge from parent
node[above] {...}
} ;
\end{tikzpicture}
\end{center}
\item Calculer la probabilité de l'évènement $M \cap W$. Interpréter ce résultat dans le contexte de l'exercice.
\item Montrer que la probabilité que la fiche choisie corresponde à une formule \emph{Burger} est égale à $0.2864$.
\item On a prélevé une fiche correspondant à la formule \emph{Burger}. Quelle est la probabilité, arrondie au millième, que la vente ait eu lieu le soir?
\end{enumerate}
\end{exercise}
\begin{solution}
\begin{enumerate}
\item
\begin{center}
\begin{tikzpicture}[sloped]
\node {.}
child {node {$M$}
child {node {$W$}
edge from parent
node[above] {$0.75$}
}
child {node {$B$}
edge from parent
node[above] {$0.25$}
}
edge from parent
node[above] {$0.87$}
}
child[missing] {}
child { node {$S$}
child {node {$W$}
edge from parent
node[above] {$0.47$}
}
child {node {$B$}
edge from parent
node[above] {$0.53$}
}
edge from parent
node[above] {$0.13$}
} ;
\end{tikzpicture}
\end{center}
\item On calcule la probabilité que la vente soit un wok et ait eu lieu à midi
\[ P(M\cap W) = P(M) \times P_M(W) = 0.87 \times 0.75 = 0.6525 \]
\item Probabilité que la vente soit un burger.
\[
P(B) = P(M\cap B) + P(S\cap B) = 0.87 \times 0.75 + 0.13 \times 0.47 = 0.2864
\]
\item On cherche à calculer la quantité $P_B(S)$. Pour cela on utilise la formule de Bayes
\[
P_B(S) = \frac{P(B\cap S)}{P(B)} = \frac{P_S(B) \times P(S)}{P(B)} = \frac{0.53\times 0.13}{0.2864} = 0.24057262569832405 \approx 0.241
\]
\end{enumerate}
\end{solution}
\begin{exercise}[subtitle={Continent plastique}]
\textit{Les quantités évoqués dans cette exercice sont générés au hasard et sont donc complètement farfelus.}
\medskip
Le \og continent de plastique\fg{} est la plus grande des plaques de déchets plastiques évoluant sur les océans. Elle occupe actuellement dans l'océan Pacifique une surface dont l'aire est évaluée à plus de $1,6$ million de km$^2$, entre Hawaï et la Californie.
En 2017, des scientifiques ont estimé qu'il y avait $14$ millions de tonnes de déchets plastiques qui était déversé chaque année dans les océans et que cette quantité augmentait de $17\n\%$ par chaque année.
On modélise l'évolution de la masse de ces déchets plastiques déversée chaque année, si rien n'est fait pour la réduire, par une suite géométrique $\left(u_n\right)$. L'arrondi au centième du terme $u_n$ représente la masse de ces déchets déversée chaque année, exprimée en million de tonnes, pour l'année $(2017 + n)$.
\medskip
\begin{enumerate}
\item Expliquer pourquoi la suite $u_n$ est géométrique?
\item Calculer $u_1$ et $u_2$.
\item Exprimer $u_n$ en fonction de $n$.
\item Au début de l'année 2017, il y avait $300$ millions de tonnes de déchets plastique. Calculer la quantité totale de déchets plastiques en 2030.
\item On souhaite déterminer en quelle année la masse totale de ces déchets plastiques aura pour la première fois augmenté de $50$\,\% par rapport à sa valeur de 2017.
\begin{enumerate}
\item Recopier et compléter l'algorithme ci-dessous pour que la variable $N$ contienne la réponse au problème posé.
\begin{center}
\begin{tabularx}{0.4\linewidth}{|X|}\hline
$N = 2017$\\
$U = 14$ \\
$S = 300 + U$ \\
while $S < 450$: \\
\hspace{1cm} $N = \ldots$\\
\hspace{1cm} $U = \ldots$\\
\hspace{1cm} $S = \ldots$\\
\hline
\end{tabularx}
\end{center}
\item Que contiennent les variables $S$, $U$ et $N$ après exécution de cet algorithme ?
Interpréter les résultats dans le contexte de l'exercice.
\end{enumerate}
\end{enumerate}
\end{exercise}
\begin{solution}
\begin{enumerate}
\item Une augmentation de $17\,\%$ revient à multiplier la quantité par $1.17$. La suite est donc bien géométrique. Son premier terme est $u_0 = 14$ et sa raison est $q = 1.17$
\item
\[
u_1 = u_0 * 1.17 = 16.38
\]
\[
u_2 = u_0 * 1.17^2 = 19.1646
\]
\item
\[
u_n = u_0 \times q^n = 14 \times 1.17^n
\]
\item On calcule la quantité totale déversée entre 2017 et 2030.
\[
\sum_{n = 0}^{13} u_n = u_0 \times \frac{1-q^{13}}{1-q} = 14 \times \frac{1 - 1.17^{13}}{1 - 1.17} = 551.66
\]
On en déduit la quantité totale de déchets en 2030
\[
300 + 551.66 = 851.66
\]
\item
\begin{enumerate}
\item ~
\begin{center}
\begin{tabularx}{0.4\linewidth}{|X|}\hline
$N \gets 2017$\\
$U \gets 14$ \\
$S \gets 300 + U$ \\
Tant que $S < 450$ \\
\hspace{1cm} $N \gets N + 1$\\
\hspace{1cm} $U \gets U * 1.17$\\
\hspace{1cm} $S \gets S + u$\\
Fin Tant que\\\hline
\end{tabularx}
\end{center}
\item \textit{Pas de correction automatisé}
\end{enumerate}
\end{enumerate}
\end{solution}
\end{document}
%%% Local Variables:
%%% mode: latex
%%% TeX-master: "master"
%%% End:

View File

@ -0,0 +1,262 @@
\documentclass[a4paper,10pt]{article}
\usepackage{myXsim}
% Title Page
\title{DS8 \hfill ZENAGUI Yanis}
\tribe{TST}
\date{\hfillÀ render pour le Mercredi 7 avril}
\xsimsetup{
solution/print = true
}
\begin{document}
\maketitle
\begin{exercise}[subtitle={Automatismes}]
\textit{Toutes les questions de cette exercice sont indépendantes et peuvent être répondus séparément}
\begin{enumerate}
\item De janvier à septembre, une quantité a augmenté de $29\,\%$. Faire un schéma pour représenter la situation puis calculer le taux d'évolution moyen mensuel.
\item Une quantité augmente de $29\,\%$ par ans. En 2020, elle est de 142\euro. Quelle était sa valeur en 2019? Faire un schéma pour représenter la situation.
\item Déterminer l'équation de la droite \\
\begin{tikzpicture}[xscale=0.8, yscale=0.5]
\tkzInit[xmin=-5,xmax=5,xstep=1,
ymin=-5,ymax=5,ystep=1]
\tkzGrid
\tkzAxeXY
\tkzFct[domain=-5:5,color=red,very thick]%
{2.0*\x -4};
\end{tikzpicture}
\item Résoudre l'équation $6 \times 0.41^x = 34$
\end{enumerate}
\end{exercise}
\begin{solution}
\begin{enumerate}
\item On veut partager cette évolution en 8 évolutions.
\[
\left(1 + \frac{29}{100}\right)^{\frac{1}{8}} = 1.0323
\]
Donc le taux d'évolution moyen est
\[
t_m = 1.0323 - 1 = 0.032299999999999995
\]
\item Coefficient multiplicateur pour revenir en arrière
\[
CM = (1 + \frac{29}{100})^{-1} = 0.7752
\]
On en déduit la quantité en 2019
\[
142 * 0.7752 = 110.0784
\]
\item L'équation de la droite est
\[
y = 2.0 x -4
\]
\item Il faut penser à faire la division à par $6$ avant d'utiliser le log car sinon, on ne peut pas utiliser la formule $\log(a^n) = n\times \log(a)$.
\[x = \frac{\log(5.67)}{\log(0.41)}\]
\end{enumerate}
\end{solution}
\begin{exercise}[subtitle={Restaurant}]
Un \emph{food truck}, ouvert le midi et le soir, propose deux types de formules :
\setlength\parindent{10mm}
\begin{itemize}
\item la formule \emph{Burger} ;
\item la formule \emph{Wok}.
\end{itemize}
\setlength\parindent{0mm}
\medskip
Le gérant a remarqué que 30\,\% de ses ventes ont lieu le midi. Le quart des ventes du midi correspondent à la formule \emph{Burger}, alors que 33\,\% des ventes du soir correspondent à la formule \emph{Wok}.
Le gérant se constitue un fichier en notant, pour chaque vente, la formule choisie et le moment de cette vente (midi ou soir).
On prélève une fiche de façon équiprobable. On définit les quatre évènements suivants:
\begin{enumerate}
\item $M$ : \og la fiche correspond à une vente du midi\fg{} ;
\item $S$ : \og la fiche correspond à une vente du soir\fg {};
\item $W$ : \og la fiche correspond à une formule \emph{Wok} \fg{} ;
\item $B$ : \og la fiche correspond à une formule \emph{Burger} \fg.
\end{enumerate}
\setlength\parindent{0mm}
\medskip
\begin{enumerate}
\item Recopier puis compléter l'arbre pondéré
\begin{center}
\begin{tikzpicture}[sloped]
\node {.}
child {node {$M$}
child {node {$W$}
edge from parent
node[above] {...}
}
child {node {$B$}
edge from parent
node[above] {...}
}
edge from parent
node[above] {...}
}
child[missing] {}
child { node {$S$}
child {node {$W$}
edge from parent
node[above] {...}
}
child {node {$B$}
edge from parent
node[above] {...}
}
edge from parent
node[above] {...}
} ;
\end{tikzpicture}
\end{center}
\item Calculer la probabilité de l'évènement $M \cap W$. Interpréter ce résultat dans le contexte de l'exercice.
\item Montrer que la probabilité que la fiche choisie corresponde à une formule \emph{Burger} est égale à $0.544$.
\item On a prélevé une fiche correspondant à la formule \emph{Burger}. Quelle est la probabilité, arrondie au millième, que la vente ait eu lieu le soir?
\end{enumerate}
\end{exercise}
\begin{solution}
\begin{enumerate}
\item
\begin{center}
\begin{tikzpicture}[sloped]
\node {.}
child {node {$M$}
child {node {$W$}
edge from parent
node[above] {$0.75$}
}
child {node {$B$}
edge from parent
node[above] {$0.25$}
}
edge from parent
node[above] {$0.3$}
}
child[missing] {}
child { node {$S$}
child {node {$W$}
edge from parent
node[above] {$0.33$}
}
child {node {$B$}
edge from parent
node[above] {$0.67$}
}
edge from parent
node[above] {$0.7$}
} ;
\end{tikzpicture}
\end{center}
\item On calcule la probabilité que la vente soit un wok et ait eu lieu à midi
\[ P(M\cap W) = P(M) \times P_M(W) = 0.3 \times 0.75 = 0.225 \]
\item Probabilité que la vente soit un burger.
\[
P(B) = P(M\cap B) + P(S\cap B) = 0.3 \times 0.75 + 0.7 \times 0.33 = 0.544
\]
\item On cherche à calculer la quantité $P_B(S)$. Pour cela on utilise la formule de Bayes
\[
P_B(S) = \frac{P(B\cap S)}{P(B)} = \frac{P_S(B) \times P(S)}{P(B)} = \frac{0.67\times 0.7}{0.544} = 0.8621323529411764 \approx 0.862
\]
\end{enumerate}
\end{solution}
\begin{exercise}[subtitle={Continent plastique}]
\textit{Les quantités évoqués dans cette exercice sont générés au hasard et sont donc complètement farfelus.}
\medskip
Le \og continent de plastique\fg{} est la plus grande des plaques de déchets plastiques évoluant sur les océans. Elle occupe actuellement dans l'océan Pacifique une surface dont l'aire est évaluée à plus de $1,6$ million de km$^2$, entre Hawaï et la Californie.
En 2017, des scientifiques ont estimé qu'il y avait $4$ millions de tonnes de déchets plastiques qui était déversé chaque année dans les océans et que cette quantité augmentait de $18\n\%$ par chaque année.
On modélise l'évolution de la masse de ces déchets plastiques déversée chaque année, si rien n'est fait pour la réduire, par une suite géométrique $\left(u_n\right)$. L'arrondi au centième du terme $u_n$ représente la masse de ces déchets déversée chaque année, exprimée en million de tonnes, pour l'année $(2017 + n)$.
\medskip
\begin{enumerate}
\item Expliquer pourquoi la suite $u_n$ est géométrique?
\item Calculer $u_1$ et $u_2$.
\item Exprimer $u_n$ en fonction de $n$.
\item Au début de l'année 2017, il y avait $300$ millions de tonnes de déchets plastique. Calculer la quantité totale de déchets plastiques en 2030.
\item On souhaite déterminer en quelle année la masse totale de ces déchets plastiques aura pour la première fois augmenté de $50$\,\% par rapport à sa valeur de 2017.
\begin{enumerate}
\item Recopier et compléter l'algorithme ci-dessous pour que la variable $N$ contienne la réponse au problème posé.
\begin{center}
\begin{tabularx}{0.4\linewidth}{|X|}\hline
$N = 2017$\\
$U = 4$ \\
$S = 300 + U$ \\
while $S < 450$: \\
\hspace{1cm} $N = \ldots$\\
\hspace{1cm} $U = \ldots$\\
\hspace{1cm} $S = \ldots$\\
\hline
\end{tabularx}
\end{center}
\item Que contiennent les variables $S$, $U$ et $N$ après exécution de cet algorithme ?
Interpréter les résultats dans le contexte de l'exercice.
\end{enumerate}
\end{enumerate}
\end{exercise}
\begin{solution}
\begin{enumerate}
\item Une augmentation de $18\,\%$ revient à multiplier la quantité par $1.18$. La suite est donc bien géométrique. Son premier terme est $u_0 = 4$ et sa raison est $q = 1.18$
\item
\[
u_1 = u_0 * 1.18 = 4.72
\]
\[
u_2 = u_0 * 1.18^2 = 5.5696
\]
\item
\[
u_n = u_0 \times q^n = 4 \times 1.18^n
\]
\item On calcule la quantité totale déversée entre 2017 et 2030.
\[
\sum_{n = 0}^{13} u_n = u_0 \times \frac{1-q^{13}}{1-q} = 4 \times \frac{1 - 1.18^{13}}{1 - 1.18} = 168.87
\]
On en déduit la quantité totale de déchets en 2030
\[
300 + 168.87 = 468.87
\]
\item
\begin{enumerate}
\item ~
\begin{center}
\begin{tabularx}{0.4\linewidth}{|X|}\hline
$N \gets 2017$\\
$U \gets 4$ \\
$S \gets 300 + U$ \\
Tant que $S < 450$ \\
\hspace{1cm} $N \gets N + 1$\\
\hspace{1cm} $U \gets U * 1.18$\\
\hspace{1cm} $S \gets S + u$\\
Fin Tant que\\\hline
\end{tabularx}
\end{center}
\item \textit{Pas de correction automatisé}
\end{enumerate}
\end{enumerate}
\end{solution}
\end{document}
%%% Local Variables:
%%% mode: latex
%%% TeX-master: "master"
%%% End:

View File

@ -0,0 +1,262 @@
\documentclass[a4paper,10pt]{article}
\usepackage{myXsim}
% Title Page
\title{DS8 \hfill VIALON-DUPERRON Victorien}
\tribe{TST}
\date{\hfillÀ render pour le Mercredi 7 avril}
\xsimsetup{
solution/print = true
}
\begin{document}
\maketitle
\begin{exercise}[subtitle={Automatismes}]
\textit{Toutes les questions de cette exercice sont indépendantes et peuvent être répondus séparément}
\begin{enumerate}
\item De janvier à septembre, une quantité a augmenté de $10\,\%$. Faire un schéma pour représenter la situation puis calculer le taux d'évolution moyen mensuel.
\item Une quantité augmente de $10\,\%$ par ans. En 2020, elle est de 120\euro. Quelle était sa valeur en 2019? Faire un schéma pour représenter la situation.
\item Déterminer l'équation de la droite \\
\begin{tikzpicture}[xscale=0.8, yscale=0.5]
\tkzInit[xmin=-5,xmax=5,xstep=1,
ymin=-5,ymax=5,ystep=1]
\tkzGrid
\tkzAxeXY
\tkzFct[domain=-5:5,color=red,very thick]%
{2.0*\x -3};
\end{tikzpicture}
\item Résoudre l'équation $6 \times 0.69^x = 39$
\end{enumerate}
\end{exercise}
\begin{solution}
\begin{enumerate}
\item On veut partager cette évolution en 8 évolutions.
\[
\left(1 + \frac{10}{100}\right)^{\frac{1}{8}} = 1.012
\]
Donc le taux d'évolution moyen est
\[
t_m = 1.012 - 1 = 0.01200000000000001
\]
\item Coefficient multiplicateur pour revenir en arrière
\[
CM = (1 + \frac{10}{100})^{-1} = 0.9091
\]
On en déduit la quantité en 2019
\[
120 * 0.9091 = 109.092
\]
\item L'équation de la droite est
\[
y = 2.0 x -3
\]
\item Il faut penser à faire la division à par $6$ avant d'utiliser le log car sinon, on ne peut pas utiliser la formule $\log(a^n) = n\times \log(a)$.
\[x = \frac{\log(6.5)}{\log(0.69)}\]
\end{enumerate}
\end{solution}
\begin{exercise}[subtitle={Restaurant}]
Un \emph{food truck}, ouvert le midi et le soir, propose deux types de formules :
\setlength\parindent{10mm}
\begin{itemize}
\item la formule \emph{Burger} ;
\item la formule \emph{Wok}.
\end{itemize}
\setlength\parindent{0mm}
\medskip
Le gérant a remarqué que 43\,\% de ses ventes ont lieu le midi. Le quart des ventes du midi correspondent à la formule \emph{Burger}, alors que 68\,\% des ventes du soir correspondent à la formule \emph{Wok}.
Le gérant se constitue un fichier en notant, pour chaque vente, la formule choisie et le moment de cette vente (midi ou soir).
On prélève une fiche de façon équiprobable. On définit les quatre évènements suivants:
\begin{enumerate}
\item $M$ : \og la fiche correspond à une vente du midi\fg{} ;
\item $S$ : \og la fiche correspond à une vente du soir\fg {};
\item $W$ : \og la fiche correspond à une formule \emph{Wok} \fg{} ;
\item $B$ : \og la fiche correspond à une formule \emph{Burger} \fg.
\end{enumerate}
\setlength\parindent{0mm}
\medskip
\begin{enumerate}
\item Recopier puis compléter l'arbre pondéré
\begin{center}
\begin{tikzpicture}[sloped]
\node {.}
child {node {$M$}
child {node {$W$}
edge from parent
node[above] {...}
}
child {node {$B$}
edge from parent
node[above] {...}
}
edge from parent
node[above] {...}
}
child[missing] {}
child { node {$S$}
child {node {$W$}
edge from parent
node[above] {...}
}
child {node {$B$}
edge from parent
node[above] {...}
}
edge from parent
node[above] {...}
} ;
\end{tikzpicture}
\end{center}
\item Calculer la probabilité de l'évènement $M \cap W$. Interpréter ce résultat dans le contexte de l'exercice.
\item Montrer que la probabilité que la fiche choisie corresponde à une formule \emph{Burger} est égale à $0.2899$.
\item On a prélevé une fiche correspondant à la formule \emph{Burger}. Quelle est la probabilité, arrondie au millième, que la vente ait eu lieu le soir?
\end{enumerate}
\end{exercise}
\begin{solution}
\begin{enumerate}
\item
\begin{center}
\begin{tikzpicture}[sloped]
\node {.}
child {node {$M$}
child {node {$W$}
edge from parent
node[above] {$0.75$}
}
child {node {$B$}
edge from parent
node[above] {$0.25$}
}
edge from parent
node[above] {$0.43$}
}
child[missing] {}
child { node {$S$}
child {node {$W$}
edge from parent
node[above] {$0.68$}
}
child {node {$B$}
edge from parent
node[above] {$0.32$}
}
edge from parent
node[above] {$0.57$}
} ;
\end{tikzpicture}
\end{center}
\item On calcule la probabilité que la vente soit un wok et ait eu lieu à midi
\[ P(M\cap W) = P(M) \times P_M(W) = 0.43 \times 0.75 = 0.3225 \]
\item Probabilité que la vente soit un burger.
\[
P(B) = P(M\cap B) + P(S\cap B) = 0.43 \times 0.75 + 0.57 \times 0.68 = 0.2899
\]
\item On cherche à calculer la quantité $P_B(S)$. Pour cela on utilise la formule de Bayes
\[
P_B(S) = \frac{P(B\cap S)}{P(B)} = \frac{P_S(B) \times P(S)}{P(B)} = \frac{0.32\times 0.57}{0.2899} = 0.6291824767161089 \approx 0.629
\]
\end{enumerate}
\end{solution}
\begin{exercise}[subtitle={Continent plastique}]
\textit{Les quantités évoqués dans cette exercice sont générés au hasard et sont donc complètement farfelus.}
\medskip
Le \og continent de plastique\fg{} est la plus grande des plaques de déchets plastiques évoluant sur les océans. Elle occupe actuellement dans l'océan Pacifique une surface dont l'aire est évaluée à plus de $1,6$ million de km$^2$, entre Hawaï et la Californie.
En 2017, des scientifiques ont estimé qu'il y avait $7$ millions de tonnes de déchets plastiques qui était déversé chaque année dans les océans et que cette quantité augmentait de $11\n\%$ par chaque année.
On modélise l'évolution de la masse de ces déchets plastiques déversée chaque année, si rien n'est fait pour la réduire, par une suite géométrique $\left(u_n\right)$. L'arrondi au centième du terme $u_n$ représente la masse de ces déchets déversée chaque année, exprimée en million de tonnes, pour l'année $(2017 + n)$.
\medskip
\begin{enumerate}
\item Expliquer pourquoi la suite $u_n$ est géométrique?
\item Calculer $u_1$ et $u_2$.
\item Exprimer $u_n$ en fonction de $n$.
\item Au début de l'année 2017, il y avait $300$ millions de tonnes de déchets plastique. Calculer la quantité totale de déchets plastiques en 2030.
\item On souhaite déterminer en quelle année la masse totale de ces déchets plastiques aura pour la première fois augmenté de $50$\,\% par rapport à sa valeur de 2017.
\begin{enumerate}
\item Recopier et compléter l'algorithme ci-dessous pour que la variable $N$ contienne la réponse au problème posé.
\begin{center}
\begin{tabularx}{0.4\linewidth}{|X|}\hline
$N = 2017$\\
$U = 7$ \\
$S = 300 + U$ \\
while $S < 450$: \\
\hspace{1cm} $N = \ldots$\\
\hspace{1cm} $U = \ldots$\\
\hspace{1cm} $S = \ldots$\\
\hline
\end{tabularx}
\end{center}
\item Que contiennent les variables $S$, $U$ et $N$ après exécution de cet algorithme ?
Interpréter les résultats dans le contexte de l'exercice.
\end{enumerate}
\end{enumerate}
\end{exercise}
\begin{solution}
\begin{enumerate}
\item Une augmentation de $11\,\%$ revient à multiplier la quantité par $1.11$. La suite est donc bien géométrique. Son premier terme est $u_0 = 7$ et sa raison est $q = 1.11$
\item
\[
u_1 = u_0 * 1.11 = 7.7700000000000005
\]
\[
u_2 = u_0 * 1.11^2 = 8.6247
\]
\item
\[
u_n = u_0 \times q^n = 7 \times 1.11^n
\]
\item On calcule la quantité totale déversée entre 2017 et 2030.
\[
\sum_{n = 0}^{13} u_n = u_0 \times \frac{1-q^{13}}{1-q} = 7 \times \frac{1 - 1.11^{13}}{1 - 1.11} = 183.48
\]
On en déduit la quantité totale de déchets en 2030
\[
300 + 183.48 = 483.48
\]
\item
\begin{enumerate}
\item ~
\begin{center}
\begin{tabularx}{0.4\linewidth}{|X|}\hline
$N \gets 2017$\\
$U \gets 7$ \\
$S \gets 300 + U$ \\
Tant que $S < 450$ \\
\hspace{1cm} $N \gets N + 1$\\
\hspace{1cm} $U \gets U * 1.11$\\
\hspace{1cm} $S \gets S + u$\\
Fin Tant que\\\hline
\end{tabularx}
\end{center}
\item \textit{Pas de correction automatisé}
\end{enumerate}
\end{enumerate}
\end{solution}
\end{document}
%%% Local Variables:
%%% mode: latex
%%% TeX-master: "master"
%%% End:

View File

@ -0,0 +1,262 @@
\documentclass[a4paper,10pt]{article}
\usepackage{myXsim}
% Title Page
\title{DS8 \hfill ZENAGUI Yanis}
\tribe{TST}
\date{\hfillÀ render pour le Mercredi 7 avril}
\xsimsetup{
solution/print = true
}
\begin{document}
\maketitle
\begin{exercise}[subtitle={Automatismes}]
\textit{Toutes les questions de cette exercice sont indépendantes et peuvent être répondus séparément}
\begin{enumerate}
\item De janvier à septembre, une quantité a augmenté de $23\,\%$. Faire un schéma pour représenter la situation puis calculer le taux d'évolution moyen mensuel.
\item Une quantité augmente de $23\,\%$ par ans. En 2020, elle est de 135\euro. Quelle était sa valeur en 2019? Faire un schéma pour représenter la situation.
\item Déterminer l'équation de la droite \\
\begin{tikzpicture}[xscale=0.8, yscale=0.5]
\tkzInit[xmin=-5,xmax=5,xstep=1,
ymin=-5,ymax=5,ystep=1]
\tkzGrid
\tkzAxeXY
\tkzFct[domain=-5:5,color=red,very thick]%
{0.5*\x -1};
\end{tikzpicture}
\item Résoudre l'équation $3 \times 0.55^x = 39$
\end{enumerate}
\end{exercise}
\begin{solution}
\begin{enumerate}
\item On veut partager cette évolution en 8 évolutions.
\[
\left(1 + \frac{23}{100}\right)^{\frac{1}{8}} = 1.0262
\]
Donc le taux d'évolution moyen est
\[
t_m = 1.0262 - 1 = 0.0262
\]
\item Coefficient multiplicateur pour revenir en arrière
\[
CM = (1 + \frac{23}{100})^{-1} = 0.813
\]
On en déduit la quantité en 2019
\[
135 * 0.813 = 109.755
\]
\item L'équation de la droite est
\[
y = 0.5 x -1
\]
\item Il faut penser à faire la division à par $3$ avant d'utiliser le log car sinon, on ne peut pas utiliser la formule $\log(a^n) = n\times \log(a)$.
\[x = \frac{\log(13.0)}{\log(0.55)}\]
\end{enumerate}
\end{solution}
\begin{exercise}[subtitle={Restaurant}]
Un \emph{food truck}, ouvert le midi et le soir, propose deux types de formules :
\setlength\parindent{10mm}
\begin{itemize}
\item la formule \emph{Burger} ;
\item la formule \emph{Wok}.
\end{itemize}
\setlength\parindent{0mm}
\medskip
Le gérant a remarqué que 9\,\% de ses ventes ont lieu le midi. Le quart des ventes du midi correspondent à la formule \emph{Burger}, alors que 11\,\% des ventes du soir correspondent à la formule \emph{Wok}.
Le gérant se constitue un fichier en notant, pour chaque vente, la formule choisie et le moment de cette vente (midi ou soir).
On prélève une fiche de façon équiprobable. On définit les quatre évènements suivants:
\begin{enumerate}
\item $M$ : \og la fiche correspond à une vente du midi\fg{} ;
\item $S$ : \og la fiche correspond à une vente du soir\fg {};
\item $W$ : \og la fiche correspond à une formule \emph{Wok} \fg{} ;
\item $B$ : \og la fiche correspond à une formule \emph{Burger} \fg.
\end{enumerate}
\setlength\parindent{0mm}
\medskip
\begin{enumerate}
\item Recopier puis compléter l'arbre pondéré
\begin{center}
\begin{tikzpicture}[sloped]
\node {.}
child {node {$M$}
child {node {$W$}
edge from parent
node[above] {...}
}
child {node {$B$}
edge from parent
node[above] {...}
}
edge from parent
node[above] {...}
}
child[missing] {}
child { node {$S$}
child {node {$W$}
edge from parent
node[above] {...}
}
child {node {$B$}
edge from parent
node[above] {...}
}
edge from parent
node[above] {...}
} ;
\end{tikzpicture}
\end{center}
\item Calculer la probabilité de l'évènement $M \cap W$. Interpréter ce résultat dans le contexte de l'exercice.
\item Montrer que la probabilité que la fiche choisie corresponde à une formule \emph{Burger} est égale à $0.8324$.
\item On a prélevé une fiche correspondant à la formule \emph{Burger}. Quelle est la probabilité, arrondie au millième, que la vente ait eu lieu le soir?
\end{enumerate}
\end{exercise}
\begin{solution}
\begin{enumerate}
\item
\begin{center}
\begin{tikzpicture}[sloped]
\node {.}
child {node {$M$}
child {node {$W$}
edge from parent
node[above] {$0.75$}
}
child {node {$B$}
edge from parent
node[above] {$0.25$}
}
edge from parent
node[above] {$0.09$}
}
child[missing] {}
child { node {$S$}
child {node {$W$}
edge from parent
node[above] {$0.11$}
}
child {node {$B$}
edge from parent
node[above] {$0.89$}
}
edge from parent
node[above] {$0.91$}
} ;
\end{tikzpicture}
\end{center}
\item On calcule la probabilité que la vente soit un wok et ait eu lieu à midi
\[ P(M\cap W) = P(M) \times P_M(W) = 0.09 \times 0.75 = 0.0675 \]
\item Probabilité que la vente soit un burger.
\[
P(B) = P(M\cap B) + P(S\cap B) = 0.09 \times 0.75 + 0.91 \times 0.11 = 0.8324
\]
\item On cherche à calculer la quantité $P_B(S)$. Pour cela on utilise la formule de Bayes
\[
P_B(S) = \frac{P(B\cap S)}{P(B)} = \frac{P_S(B) \times P(S)}{P(B)} = \frac{0.89\times 0.91}{0.8324} = 0.9729697260932244 \approx 0.973
\]
\end{enumerate}
\end{solution}
\begin{exercise}[subtitle={Continent plastique}]
\textit{Les quantités évoqués dans cette exercice sont générés au hasard et sont donc complètement farfelus.}
\medskip
Le \og continent de plastique\fg{} est la plus grande des plaques de déchets plastiques évoluant sur les océans. Elle occupe actuellement dans l'océan Pacifique une surface dont l'aire est évaluée à plus de $1,6$ million de km$^2$, entre Hawaï et la Californie.
En 2017, des scientifiques ont estimé qu'il y avait $4$ millions de tonnes de déchets plastiques qui était déversé chaque année dans les océans et que cette quantité augmentait de $19\n\%$ par chaque année.
On modélise l'évolution de la masse de ces déchets plastiques déversée chaque année, si rien n'est fait pour la réduire, par une suite géométrique $\left(u_n\right)$. L'arrondi au centième du terme $u_n$ représente la masse de ces déchets déversée chaque année, exprimée en million de tonnes, pour l'année $(2017 + n)$.
\medskip
\begin{enumerate}
\item Expliquer pourquoi la suite $u_n$ est géométrique?
\item Calculer $u_1$ et $u_2$.
\item Exprimer $u_n$ en fonction de $n$.
\item Au début de l'année 2017, il y avait $300$ millions de tonnes de déchets plastique. Calculer la quantité totale de déchets plastiques en 2030.
\item On souhaite déterminer en quelle année la masse totale de ces déchets plastiques aura pour la première fois augmenté de $50$\,\% par rapport à sa valeur de 2017.
\begin{enumerate}
\item Recopier et compléter l'algorithme ci-dessous pour que la variable $N$ contienne la réponse au problème posé.
\begin{center}
\begin{tabularx}{0.4\linewidth}{|X|}\hline
$N = 2017$\\
$U = 4$ \\
$S = 300 + U$ \\
while $S < 450$: \\
\hspace{1cm} $N = \ldots$\\
\hspace{1cm} $U = \ldots$\\
\hspace{1cm} $S = \ldots$\\
\hline
\end{tabularx}
\end{center}
\item Que contiennent les variables $S$, $U$ et $N$ après exécution de cet algorithme ?
Interpréter les résultats dans le contexte de l'exercice.
\end{enumerate}
\end{enumerate}
\end{exercise}
\begin{solution}
\begin{enumerate}
\item Une augmentation de $19\,\%$ revient à multiplier la quantité par $1.19$. La suite est donc bien géométrique. Son premier terme est $u_0 = 4$ et sa raison est $q = 1.19$
\item
\[
u_1 = u_0 * 1.19 = 4.76
\]
\[
u_2 = u_0 * 1.19^2 = 5.6644
\]
\item
\[
u_n = u_0 \times q^n = 4 \times 1.19^n
\]
\item On calcule la quantité totale déversée entre 2017 et 2030.
\[
\sum_{n = 0}^{13} u_n = u_0 \times \frac{1-q^{13}}{1-q} = 4 \times \frac{1 - 1.19^{13}}{1 - 1.19} = 180.98
\]
On en déduit la quantité totale de déchets en 2030
\[
300 + 180.98 = 480.98
\]
\item
\begin{enumerate}
\item ~
\begin{center}
\begin{tabularx}{0.4\linewidth}{|X|}\hline
$N \gets 2017$\\
$U \gets 4$ \\
$S \gets 300 + U$ \\
Tant que $S < 450$ \\
\hspace{1cm} $N \gets N + 1$\\
\hspace{1cm} $U \gets U * 1.19$\\
\hspace{1cm} $S \gets S + u$\\
Fin Tant que\\\hline
\end{tabularx}
\end{center}
\item \textit{Pas de correction automatisé}
\end{enumerate}
\end{enumerate}
\end{solution}
\end{document}
%%% Local Variables:
%%% mode: latex
%%% TeX-master: "master"
%%% End:

Binary file not shown.

View File

@ -20,7 +20,7 @@
\item De janvier à septembre, une quantité a augmenté de $\Var{t}\,\%$. Faire un schéma pour représenter la situation puis calculer le taux d'évolution moyen mensuel.
%- set valeur = randint(110, 150)
\item Une quantité augmente de $\Var{t}\,\%$ par ans. En 2020, elle est de \Var{valeur}\euro. Quelle était sa valeur en 2019? Faire un schéma pour représenter la situation.
\item Déterminer l'équation de la droite
\item Déterminer l'équation de la droite \\
%- set b = randint(-4, -1)
%- set denom = randint(2, 4)
%- set a = -2*b/denom
@ -192,7 +192,7 @@
\end{solution}
\begin{exercise}[subtitle={Continent plastique}]
\textit{Les quantités évoqués dans cette exercices sont générés au hasard et sont donc complètement farfelus.}
\textit{Les quantités évoqués dans cette exercice sont générés au hasard et sont donc complètement farfelus.}
\medskip
%- set u0 = randint(2, 20)
%- set t = round(0.1 + 0.2*random(), 2)
@ -216,14 +216,14 @@
\begin{center}
\begin{tabularx}{0.4\linewidth}{|X|}\hline
$N \gets 2017$\\
$U \gets \Var{u0}$ \\
$S \gets 300 + U$ \\
Tant que $S < 450$ \\
\hspace{1cm} $N \gets \ldots$\\
\hspace{1cm} $U \gets \ldots$\\
\hspace{1cm} $S \gets \ldots$\\
Fin Tant que\\\hline
$N = 2017$\\
$U = \Var{u0}$ \\
$S = 300 + U$ \\
while $S < 450$: \\
\hspace{1cm} $N = \ldots$\\
\hspace{1cm} $U = \ldots$\\
\hspace{1cm} $S = \ldots$\\
\hline
\end{tabularx}
\end{center}
\item Que contiennent les variables $S$, $U$ et $N$ après exécution de cet algorithme ?